Allowed Publications
Slot System
Featured Buckets
Featured Buckets Admin

A Veteran Presenting for Low Testosterone and Lower Urinary Tract Symptoms

Article Type
Changed
Thu, 12/22/2022 - 13:51

►Anish Bhatnagar, MD, Chief Medical Resident, Veterans Affairs Boston Healthcare System (VABHS) and Beth Israel Deaconess Medical Center (BIDMC): The patient noted erectile dysfunction starting 4 years ago, with accompanied decreased libido. However, until recently, he was able to achieve acceptable erectile capacity with medications. As part of his previous evaluations for erectile dysfunction, the patient had 2 total testosterone levels checked 6 months apart, both low at 150 ng/dL and 38.3 ng/dL (reference range, 220-892). The results of additional hormone studies are shown in the Table. Dr. Ananthakrishnan, can you help us interpret these laboratory results and tell us what tests you might order next?

►Sonia Ananthakrishnan, MD, Section of Endocrinology, Diabetes and Nutrition, Boston Medical Center (BMC) and Assistant Professor of Medicine, Boston University School of Medicine (BUSM): When patients present with signs of hypogonadism and an initial low morning testosterone levels, the next test should be a confirmatory repeat morning testosterone level as was done in this case. If this level is also low (for most assays < 300 ng/dL), further evaluation for primary vs secondary hypogonadism should be pursued with measurement of luteinizing hormone and follicle-stimulating hormone levels. Secondary hypogonadism should be suspected when these levels are low or inappropriately normal in the setting of a low testosterone level as in this patient. This patient does not appear to be on any medication or have reversible illnesses that we traditionally think of as possibly causing these hormone irregularities. Key examples include medications such as gonadotropin-releasing hormone analogs, glucocorticoids, and opioids, as well as conditions such as hyperprolactinemia, sleep apnea, diabetes mellitus, anorexia nervosa, or other chronic systemic illnesses, including cirrhosis or lung disease. In this setting, further evaluation of the patient’s anterior pituitary function should be undertaken. Initial screening tests showed mildly elevated prolactin and low normal thyroid-stimulating hormone levels, with a relatively normal free thyroxine. Given these abnormalities in the context of the patient’s total testosterone level < 150 ng/dL, magnetic resonance imaging (MRI) of the anterior pituitary is indicated, and what I would recommend next for evaluation of pituitary and/or hypothalamic tumor or infiltrative disease.1

fdp03910406_t.png

►Dr. Bhatnagar: An MRI of the brain showed a large 2.7-cm sellar mass, with suprasellar extension and mass effect on the optic chiasm and pituitary infundibulum, partial extension into the right sphenoid sinus, and invasion into the right cavernous sinus. These findings were consistent with a pituitary macroadenoma. The patient was subsequently evaluated by a neurosurgeon who felt that because of the extension and compression of the mass, the patient would benefit from surgical resection.

Given his lower urinary tract symptoms, a prostate-specific antigen level was checked and returned elevated at 11.5 ng/mL. In the setting of these abnormalities, the patient underwent MRI of the abdomen, which noted a new 5.6-cm enhancing mass in the upper pole of his solitary right kidney, highly concerning for new RCC. After a multidisciplinary discussion, urology scheduled the patient for partial right nephrectomy first, with plans for pituitary resection only if the patient had adequate recovery following the urologic procedure.

Dr. Rifkin, this patient went straight from imaging to presumed diagnosis to planned surgical intervention without a confirmatory biopsy. In a patient who already has chronic kidney disease stage 4, why would we not want to pursue biopsy prior to this invasive procedure on his solitary kidney? In addition, given his baseline advanced renal disease, why pursue partial nephrectomy to delay initiation of hemodialysis instead of total nephrectomy and beginning hemodialysis?

►Ian Rifkin, MBBCh, PhD, MSc, Chief, Renal Section, VABHS, Section of Nephrology, BMC, and Associate Professor of Medicine, BUSM: In most cases, imaging alone is used to make a presumptive diagnosis of benign vs malignant renal masses. In one study, RCC was identified by MRI with 85% sensitivity and 76% specificity.2 However, as imaging and biopsy techniques have advanced, there are progressing discussions regarding the utility of biopsy. That being said, there are a number of situations in which patients currently undergo biopsy, particularly when there is diagnostic uncertainty.3 In this patient, with a history of RCC and imaging findings concerning for RCC, biopsy is unnecessary given the high clinical suspicion.

Regarding the choice of partial vs total nephrectomy, there are 2 important distinctions to be made. The first is that though it was previously felt that early initiation of dialysis improves survival, newer studies suggest that early initiation based off of glomerular filtration rate (GFR) offers no survival benefits compared to delayed initiation.4 Second, though there is less clinical data to support this, there is a signal toward the use of partial nephrectomy decreasing mortality compared to radical nephrectomy in management of RCC.5 In this patient, partial nephrectomy may not only increase rates of survival, but also delay initiation of dialysis.

►Dr. Bhatnagar: Prior to undergoing partial right nephrectomy, a morning cortisol level was found to be 5.8 μg/dL with an associated corticotropin (ACTH) level of 26 pg/mL. Dr. Ananthakrishnan, how would you interpret these laboratory results and what might you recommend prior to surgery?

 

 



►Dr. Ananthakrishnan: In a healthy patient, surgery often results in a several-fold increase in the secretion of cortisol to balance the unique stressors surgery places on the body.6 This patient is at increased risk for complete or partial adrenal insufficiency in the setting of both his pituitary macroadenoma as well as his previous left nephrectomy, which could have affected his left adrenal gland as well. Thus, this patient may not be able to mount the appropriate cortisol response needed to counter the stresses of surgery. His cortisol level is abnormally low for a morning value, with a relatively normal ACTH reference range of 6 to 50 pg/mL. He may have some degree of adrenal insufficiency, and thus will benefit from perioperative steroids.

►Dr. Bhatnagar: The patient was started on hydrocortisone and underwent a successful laparoscopic partial right nephrectomy. During the procedure, an estimated 2.5 L of blood was lost, with transfusion of 3 units of packed red blood cells. A surgical drain was left in the peritoneum. Postoperatively, he developed hypotension, requiring vasopressors and prolonged continuation of stress dosing of hydrocortisone. Over the next 4 days, the patient was weaned off vasopressors, and his creatinine level was noted to increase from a baseline of 1.8 mg/dL to 4.4 mg/dL.

Dr. Rifkin, how do you think about renal recovery in the patient postnephrectomy, and should we be concerned with the dramatic rise in his creatinine level?

►Dr. Rifkin: Removal of renal mass will result in an initial reduction of GFR proportional to the amount of functional renal tissue removed. However, in as early as 1 week, the residual nephrons begin to compensate through various mechanisms, such as modulation of efferent and afferent arterioles and renal tissue growth by hypertrophy and hyperplasia.7 In the acute setting, it may be difficult to distinguish an acute renal injury vs physiological GFR reduction postnephron loss, but often the initially elevated creatinine level may normalize/stabilize over time. Other markers of kidney function should concomitantly be monitored, including urine output, electrolyte/acid-base status, and urine sediment examination. In this patient, although his creatinine level may be elevated over the first few days, if his urine output remains robust and the urine sediment examination is normal, my concern for permanent kidney injury would be lessened.

fdp03910406_clincal_takeaways.png


►Dr. Bhatnagar: During the first 4 postoperative days the patient produced approximately 1 L of urine per day with a stable creatinine level. It is over this same time that the hydrocortisone was discontinued given improving hemodynamics. However, throughout postoperative day 5, the patient’s creatinine level acutely rose to a peak of 5.8 mg/dL. In addition, his urine output dramatically dropped to < 5 mL per hour, with blood clots noted in his Foley catheter. Dr. Rifkin, what is your differential for causing this acute change in both his creatinine level and urine output this far out from his procedure, and what might you do to help further evaluate?

►Dr. Rifkin: The most common cause of acute kidney injury in hospitalized patients is acute tubular necrosis (ATN).8 However, in this patient, who was recovering well postoperatively, was hemodynamically stable with a robust urine output, and in whom no apparent cause for ATN could be identified, other diagnoses were more likely. Considering the abrupt onset of oligo-anuria, the most likely diagnosis was urinary tract obstruction, particularly given the frank blood and blood clots that were present in the urine. Additional possibilities might be a late surgical complication or infection. Surgical complications could range from direct damage to the renal parenchyma to urinary leakage into the peritoneum from the site of anastomosis or tissue injury. Infections introduced either intraoperatively or developed postoperatively could also cause this sudden drop in urine output, though one would expect more systemic symptoms with this. Given that this patient has a surgical drain in place in the peritoneum, I would recommend testing the creatinine level in the peritoneal fluid drainage. If it is comparable to serum levels, this would argue against a urine leak, as we would expect the level to be significantly elevated in a leak. In addition, he should have imaging of the urinary tract followed by procedures to decompress the presumed obstructed urinary tract. These procedures might include either cystoscopy with ureteral stent placement or percutaneous nephrostomy, depending on the result of the imaging.

►Dr. Bhatnagar: The creatinine level obtained from the surgical drain was roughly equivalent to the serum creatinine, decreasing suspicion for a urine leak as the cause of his findings. Cystoscopy with ureteral stent placement was performed with subsequent increase in both urine output and concomitant decrease in serum creatinine.

Around this time, the patient also began to note blurry vision. Evaluation revealed difficulty with visual field confrontation in the right lower quadrant, right eye ptosis, right eye impaired adduction, absent abduction and impaired upgaze, but intact downgaze. Diplopia was present with gaze in all directions. His constellation of physical examination findings were concerning for a pathologic lesion partially involving cranial nerves II and III, with definitive involvement of cranial nerve VI, but sparing of cranial nerve IV. Repeat MRI of the brain showed hemorrhage into the sellar mass, with ongoing mass effect on the optic chiasm and extension into the sinuses (eAppendix). These findings were consistent with pituitary apoplexy. Dr. Ananthakrishnan, can you tell us more about pituitary apoplexy?

fdp03910406_eappendix.png

►Dr. Ananthakrishnan: Pituitary apoplexy is a clinical syndrome resulting from acute hemorrhage or infarction of the pituitary gland. It typically occurs in patients with preexisting pituitary adenomas and is characterized by the onset of headache, fever, vomiting, meningismus, decreased consciousness, and sometimes death. In addition, given the location of the pituitary gland within the sella, rapid changes in size can result in compression of cranial nerves III, IV, and VI, as well as the optic chiasm, resulting in ophthalmoplegia and visual disturbances as seen in this patient.9

 

 

There are a multitude of causes of pituitary apoplexy, including alterations in coagulopathy, pituitary stimulation (eg, dynamic pituitary hormone testing), and both acute increases and decreases in blood flow.10 This patient likely had an ischemic event due to changes in vascular perfusion, spurred by both his blood loss intraoperatively and ongoing hematuria. Management of pituitary apoplexy is dependent on the patient’s hemodynamics, mass effect symptoms, electrolyte balances, and hormone dysfunction. The decision for conservative management vs surgical intervention should be made in consultation with both neurosurgery and endocrinology. Once the patient is hemodynamically stable, the next step in evaluating this patient should be repeating his hormone studies.

►Dr. Bhatnagar: An assessment of pituitary function was consistent with values obtained preoperatively. After multidisciplinary discussions, surgery was deferred, and hydrocortisone was reinitiated to reduce inflammation caused by bleeding into the mass. As the ophthalmoplegia improved, this was transitioned to dexamethasone.

Twelve days after admission, he was discharged to a subacute rehabilitation center, with improvement in his ophthalmoplegia and stabilization of his creatinine level and urine output.

References

1. Bhasin S, Cunningham GR, Hayes FJ, et al. Testosterone therapy in men with androgen deficiency syndromes: an Endocrine Society clinical practice guideline. J Clin Endocrinol Metab. 2010;95(6):2536-2559. doi:10.1210/jc.2009-2354

2. Kay FU, Canvasser NE, Xi Y, et al. Diagnostic performance and interreader agreement of a standardized MR imaging approach in the prediction of small renal mass histology. Radiology. 2018;287(2):543-553. doi:10.1148/radiol.2018171557

3. Sahni VA, Silverman SG. Biopsy of renal masses: when and why. Cancer Imaging. 2009;9(1):44-55. doi:10.1102/1470-7330.2009.0005

4. Cooper BA, Branley P, Bulfone L, et al. A randomized, controlled trial of early versus late initiation of dialysis. N Engl J Med. 2010;363(7):609-619. doi:10.1056/NEJMoa1000552

5. Kunath F, Schmidt S, Krabbe L-M, et al. Partial nephrectomy versus radical nephrectomy for clinical localised renal masses. Cochrane Database Syst Rev. 2017;5(5):CD012045. doi:10.1002/14651858.CD012045.pub2

6. Kehlet H, Binder C. Adrenocortical function and clinical course during and after surgery in unsupplemented glucocorticoid-treated patients. Br J Anaesth. 1973;45(10):1043-1048. doi:10.1093/bja/45.10.1043

7. Chapman D, Moore R, Klarenbach S, Braam B. Residual renal function after partial or radical nephrectomy for renal cell carcinoma. Can Urol Assoc J. 2010;4(5):337-343. doi:10.5489/cuaj.909

8. Rahman M, Shad F, Smith MC. Acute kidney injury: a guide to diagnosis and management. Am Fam Physician. 2012;86(7):631-639.

9. Ranabir S, Baruah MP. Pituitary apoplexy. Indian J Endocrinol Metab. 2011;15(suppl 3):S188-S196. doi:10.4103/2230-8210.84862

10. Glezer A, Bronstein MD. Pituitary apoplexy: pathophysiology, diagnosis and management. Arch Endocrinol Metab. 2015;59(3):259-264. doi:10.1590/2359-3997000000047

Article PDF
Author and Disclosure Information

Anish Bhatnagar, MDa,b; Sonia Ananthakrishan, MDc,d; Ian Rifkin, MBBCh, PhD, MScc,d,e; Anthony C. Breu, MDb,e
Correspondence:
Anthony Breu (anthony.breu@va.gov)

a Beth Israel Deaconess Medical Center, Boston, Massachusetts
b Harvard Medical School, Boston, Massachusetts
c Boston Medical Center, Massachusetts
d Boston University School of Medicine, Massachusetts
e Veterans Affairs Boston Healthcare System, West Roxbury, Massachusetts

Author disclosures

The authors report no actual or potential conflicts of interest or outside sources of funding with regard to this article.

Disclaimer

The opinions expressed herein are those of the authors and do not necessarily reflect those of Federal Practitioner , Frontline Medical Communications Inc., the US Government, or any of its agencies. This article may discuss unlabeled or investigational use of certain drugs. Please review the complete prescribing information for specific drugs or drug combinations—including indications, contraindications, warnings, and adverse effects—before administering pharmacologic therapy to patients.

Ethics and consent

Written patient consent was obtained prior to publication.

Issue
Federal Practitioner - 39(10)a
Publications
Topics
Page Number
406-409
Sections
Author and Disclosure Information

Anish Bhatnagar, MDa,b; Sonia Ananthakrishan, MDc,d; Ian Rifkin, MBBCh, PhD, MScc,d,e; Anthony C. Breu, MDb,e
Correspondence:
Anthony Breu (anthony.breu@va.gov)

a Beth Israel Deaconess Medical Center, Boston, Massachusetts
b Harvard Medical School, Boston, Massachusetts
c Boston Medical Center, Massachusetts
d Boston University School of Medicine, Massachusetts
e Veterans Affairs Boston Healthcare System, West Roxbury, Massachusetts

Author disclosures

The authors report no actual or potential conflicts of interest or outside sources of funding with regard to this article.

Disclaimer

The opinions expressed herein are those of the authors and do not necessarily reflect those of Federal Practitioner , Frontline Medical Communications Inc., the US Government, or any of its agencies. This article may discuss unlabeled or investigational use of certain drugs. Please review the complete prescribing information for specific drugs or drug combinations—including indications, contraindications, warnings, and adverse effects—before administering pharmacologic therapy to patients.

Ethics and consent

Written patient consent was obtained prior to publication.

Author and Disclosure Information

Anish Bhatnagar, MDa,b; Sonia Ananthakrishan, MDc,d; Ian Rifkin, MBBCh, PhD, MScc,d,e; Anthony C. Breu, MDb,e
Correspondence:
Anthony Breu (anthony.breu@va.gov)

a Beth Israel Deaconess Medical Center, Boston, Massachusetts
b Harvard Medical School, Boston, Massachusetts
c Boston Medical Center, Massachusetts
d Boston University School of Medicine, Massachusetts
e Veterans Affairs Boston Healthcare System, West Roxbury, Massachusetts

Author disclosures

The authors report no actual or potential conflicts of interest or outside sources of funding with regard to this article.

Disclaimer

The opinions expressed herein are those of the authors and do not necessarily reflect those of Federal Practitioner , Frontline Medical Communications Inc., the US Government, or any of its agencies. This article may discuss unlabeled or investigational use of certain drugs. Please review the complete prescribing information for specific drugs or drug combinations—including indications, contraindications, warnings, and adverse effects—before administering pharmacologic therapy to patients.

Ethics and consent

Written patient consent was obtained prior to publication.

Article PDF
Article PDF

►Anish Bhatnagar, MD, Chief Medical Resident, Veterans Affairs Boston Healthcare System (VABHS) and Beth Israel Deaconess Medical Center (BIDMC): The patient noted erectile dysfunction starting 4 years ago, with accompanied decreased libido. However, until recently, he was able to achieve acceptable erectile capacity with medications. As part of his previous evaluations for erectile dysfunction, the patient had 2 total testosterone levels checked 6 months apart, both low at 150 ng/dL and 38.3 ng/dL (reference range, 220-892). The results of additional hormone studies are shown in the Table. Dr. Ananthakrishnan, can you help us interpret these laboratory results and tell us what tests you might order next?

►Sonia Ananthakrishnan, MD, Section of Endocrinology, Diabetes and Nutrition, Boston Medical Center (BMC) and Assistant Professor of Medicine, Boston University School of Medicine (BUSM): When patients present with signs of hypogonadism and an initial low morning testosterone levels, the next test should be a confirmatory repeat morning testosterone level as was done in this case. If this level is also low (for most assays < 300 ng/dL), further evaluation for primary vs secondary hypogonadism should be pursued with measurement of luteinizing hormone and follicle-stimulating hormone levels. Secondary hypogonadism should be suspected when these levels are low or inappropriately normal in the setting of a low testosterone level as in this patient. This patient does not appear to be on any medication or have reversible illnesses that we traditionally think of as possibly causing these hormone irregularities. Key examples include medications such as gonadotropin-releasing hormone analogs, glucocorticoids, and opioids, as well as conditions such as hyperprolactinemia, sleep apnea, diabetes mellitus, anorexia nervosa, or other chronic systemic illnesses, including cirrhosis or lung disease. In this setting, further evaluation of the patient’s anterior pituitary function should be undertaken. Initial screening tests showed mildly elevated prolactin and low normal thyroid-stimulating hormone levels, with a relatively normal free thyroxine. Given these abnormalities in the context of the patient’s total testosterone level < 150 ng/dL, magnetic resonance imaging (MRI) of the anterior pituitary is indicated, and what I would recommend next for evaluation of pituitary and/or hypothalamic tumor or infiltrative disease.1

fdp03910406_t.png

►Dr. Bhatnagar: An MRI of the brain showed a large 2.7-cm sellar mass, with suprasellar extension and mass effect on the optic chiasm and pituitary infundibulum, partial extension into the right sphenoid sinus, and invasion into the right cavernous sinus. These findings were consistent with a pituitary macroadenoma. The patient was subsequently evaluated by a neurosurgeon who felt that because of the extension and compression of the mass, the patient would benefit from surgical resection.

Given his lower urinary tract symptoms, a prostate-specific antigen level was checked and returned elevated at 11.5 ng/mL. In the setting of these abnormalities, the patient underwent MRI of the abdomen, which noted a new 5.6-cm enhancing mass in the upper pole of his solitary right kidney, highly concerning for new RCC. After a multidisciplinary discussion, urology scheduled the patient for partial right nephrectomy first, with plans for pituitary resection only if the patient had adequate recovery following the urologic procedure.

Dr. Rifkin, this patient went straight from imaging to presumed diagnosis to planned surgical intervention without a confirmatory biopsy. In a patient who already has chronic kidney disease stage 4, why would we not want to pursue biopsy prior to this invasive procedure on his solitary kidney? In addition, given his baseline advanced renal disease, why pursue partial nephrectomy to delay initiation of hemodialysis instead of total nephrectomy and beginning hemodialysis?

►Ian Rifkin, MBBCh, PhD, MSc, Chief, Renal Section, VABHS, Section of Nephrology, BMC, and Associate Professor of Medicine, BUSM: In most cases, imaging alone is used to make a presumptive diagnosis of benign vs malignant renal masses. In one study, RCC was identified by MRI with 85% sensitivity and 76% specificity.2 However, as imaging and biopsy techniques have advanced, there are progressing discussions regarding the utility of biopsy. That being said, there are a number of situations in which patients currently undergo biopsy, particularly when there is diagnostic uncertainty.3 In this patient, with a history of RCC and imaging findings concerning for RCC, biopsy is unnecessary given the high clinical suspicion.

Regarding the choice of partial vs total nephrectomy, there are 2 important distinctions to be made. The first is that though it was previously felt that early initiation of dialysis improves survival, newer studies suggest that early initiation based off of glomerular filtration rate (GFR) offers no survival benefits compared to delayed initiation.4 Second, though there is less clinical data to support this, there is a signal toward the use of partial nephrectomy decreasing mortality compared to radical nephrectomy in management of RCC.5 In this patient, partial nephrectomy may not only increase rates of survival, but also delay initiation of dialysis.

►Dr. Bhatnagar: Prior to undergoing partial right nephrectomy, a morning cortisol level was found to be 5.8 μg/dL with an associated corticotropin (ACTH) level of 26 pg/mL. Dr. Ananthakrishnan, how would you interpret these laboratory results and what might you recommend prior to surgery?

 

 



►Dr. Ananthakrishnan: In a healthy patient, surgery often results in a several-fold increase in the secretion of cortisol to balance the unique stressors surgery places on the body.6 This patient is at increased risk for complete or partial adrenal insufficiency in the setting of both his pituitary macroadenoma as well as his previous left nephrectomy, which could have affected his left adrenal gland as well. Thus, this patient may not be able to mount the appropriate cortisol response needed to counter the stresses of surgery. His cortisol level is abnormally low for a morning value, with a relatively normal ACTH reference range of 6 to 50 pg/mL. He may have some degree of adrenal insufficiency, and thus will benefit from perioperative steroids.

►Dr. Bhatnagar: The patient was started on hydrocortisone and underwent a successful laparoscopic partial right nephrectomy. During the procedure, an estimated 2.5 L of blood was lost, with transfusion of 3 units of packed red blood cells. A surgical drain was left in the peritoneum. Postoperatively, he developed hypotension, requiring vasopressors and prolonged continuation of stress dosing of hydrocortisone. Over the next 4 days, the patient was weaned off vasopressors, and his creatinine level was noted to increase from a baseline of 1.8 mg/dL to 4.4 mg/dL.

Dr. Rifkin, how do you think about renal recovery in the patient postnephrectomy, and should we be concerned with the dramatic rise in his creatinine level?

►Dr. Rifkin: Removal of renal mass will result in an initial reduction of GFR proportional to the amount of functional renal tissue removed. However, in as early as 1 week, the residual nephrons begin to compensate through various mechanisms, such as modulation of efferent and afferent arterioles and renal tissue growth by hypertrophy and hyperplasia.7 In the acute setting, it may be difficult to distinguish an acute renal injury vs physiological GFR reduction postnephron loss, but often the initially elevated creatinine level may normalize/stabilize over time. Other markers of kidney function should concomitantly be monitored, including urine output, electrolyte/acid-base status, and urine sediment examination. In this patient, although his creatinine level may be elevated over the first few days, if his urine output remains robust and the urine sediment examination is normal, my concern for permanent kidney injury would be lessened.

fdp03910406_clincal_takeaways.png


►Dr. Bhatnagar: During the first 4 postoperative days the patient produced approximately 1 L of urine per day with a stable creatinine level. It is over this same time that the hydrocortisone was discontinued given improving hemodynamics. However, throughout postoperative day 5, the patient’s creatinine level acutely rose to a peak of 5.8 mg/dL. In addition, his urine output dramatically dropped to < 5 mL per hour, with blood clots noted in his Foley catheter. Dr. Rifkin, what is your differential for causing this acute change in both his creatinine level and urine output this far out from his procedure, and what might you do to help further evaluate?

►Dr. Rifkin: The most common cause of acute kidney injury in hospitalized patients is acute tubular necrosis (ATN).8 However, in this patient, who was recovering well postoperatively, was hemodynamically stable with a robust urine output, and in whom no apparent cause for ATN could be identified, other diagnoses were more likely. Considering the abrupt onset of oligo-anuria, the most likely diagnosis was urinary tract obstruction, particularly given the frank blood and blood clots that were present in the urine. Additional possibilities might be a late surgical complication or infection. Surgical complications could range from direct damage to the renal parenchyma to urinary leakage into the peritoneum from the site of anastomosis or tissue injury. Infections introduced either intraoperatively or developed postoperatively could also cause this sudden drop in urine output, though one would expect more systemic symptoms with this. Given that this patient has a surgical drain in place in the peritoneum, I would recommend testing the creatinine level in the peritoneal fluid drainage. If it is comparable to serum levels, this would argue against a urine leak, as we would expect the level to be significantly elevated in a leak. In addition, he should have imaging of the urinary tract followed by procedures to decompress the presumed obstructed urinary tract. These procedures might include either cystoscopy with ureteral stent placement or percutaneous nephrostomy, depending on the result of the imaging.

►Dr. Bhatnagar: The creatinine level obtained from the surgical drain was roughly equivalent to the serum creatinine, decreasing suspicion for a urine leak as the cause of his findings. Cystoscopy with ureteral stent placement was performed with subsequent increase in both urine output and concomitant decrease in serum creatinine.

Around this time, the patient also began to note blurry vision. Evaluation revealed difficulty with visual field confrontation in the right lower quadrant, right eye ptosis, right eye impaired adduction, absent abduction and impaired upgaze, but intact downgaze. Diplopia was present with gaze in all directions. His constellation of physical examination findings were concerning for a pathologic lesion partially involving cranial nerves II and III, with definitive involvement of cranial nerve VI, but sparing of cranial nerve IV. Repeat MRI of the brain showed hemorrhage into the sellar mass, with ongoing mass effect on the optic chiasm and extension into the sinuses (eAppendix). These findings were consistent with pituitary apoplexy. Dr. Ananthakrishnan, can you tell us more about pituitary apoplexy?

fdp03910406_eappendix.png

►Dr. Ananthakrishnan: Pituitary apoplexy is a clinical syndrome resulting from acute hemorrhage or infarction of the pituitary gland. It typically occurs in patients with preexisting pituitary adenomas and is characterized by the onset of headache, fever, vomiting, meningismus, decreased consciousness, and sometimes death. In addition, given the location of the pituitary gland within the sella, rapid changes in size can result in compression of cranial nerves III, IV, and VI, as well as the optic chiasm, resulting in ophthalmoplegia and visual disturbances as seen in this patient.9

 

 

There are a multitude of causes of pituitary apoplexy, including alterations in coagulopathy, pituitary stimulation (eg, dynamic pituitary hormone testing), and both acute increases and decreases in blood flow.10 This patient likely had an ischemic event due to changes in vascular perfusion, spurred by both his blood loss intraoperatively and ongoing hematuria. Management of pituitary apoplexy is dependent on the patient’s hemodynamics, mass effect symptoms, electrolyte balances, and hormone dysfunction. The decision for conservative management vs surgical intervention should be made in consultation with both neurosurgery and endocrinology. Once the patient is hemodynamically stable, the next step in evaluating this patient should be repeating his hormone studies.

►Dr. Bhatnagar: An assessment of pituitary function was consistent with values obtained preoperatively. After multidisciplinary discussions, surgery was deferred, and hydrocortisone was reinitiated to reduce inflammation caused by bleeding into the mass. As the ophthalmoplegia improved, this was transitioned to dexamethasone.

Twelve days after admission, he was discharged to a subacute rehabilitation center, with improvement in his ophthalmoplegia and stabilization of his creatinine level and urine output.

►Anish Bhatnagar, MD, Chief Medical Resident, Veterans Affairs Boston Healthcare System (VABHS) and Beth Israel Deaconess Medical Center (BIDMC): The patient noted erectile dysfunction starting 4 years ago, with accompanied decreased libido. However, until recently, he was able to achieve acceptable erectile capacity with medications. As part of his previous evaluations for erectile dysfunction, the patient had 2 total testosterone levels checked 6 months apart, both low at 150 ng/dL and 38.3 ng/dL (reference range, 220-892). The results of additional hormone studies are shown in the Table. Dr. Ananthakrishnan, can you help us interpret these laboratory results and tell us what tests you might order next?

►Sonia Ananthakrishnan, MD, Section of Endocrinology, Diabetes and Nutrition, Boston Medical Center (BMC) and Assistant Professor of Medicine, Boston University School of Medicine (BUSM): When patients present with signs of hypogonadism and an initial low morning testosterone levels, the next test should be a confirmatory repeat morning testosterone level as was done in this case. If this level is also low (for most assays < 300 ng/dL), further evaluation for primary vs secondary hypogonadism should be pursued with measurement of luteinizing hormone and follicle-stimulating hormone levels. Secondary hypogonadism should be suspected when these levels are low or inappropriately normal in the setting of a low testosterone level as in this patient. This patient does not appear to be on any medication or have reversible illnesses that we traditionally think of as possibly causing these hormone irregularities. Key examples include medications such as gonadotropin-releasing hormone analogs, glucocorticoids, and opioids, as well as conditions such as hyperprolactinemia, sleep apnea, diabetes mellitus, anorexia nervosa, or other chronic systemic illnesses, including cirrhosis or lung disease. In this setting, further evaluation of the patient’s anterior pituitary function should be undertaken. Initial screening tests showed mildly elevated prolactin and low normal thyroid-stimulating hormone levels, with a relatively normal free thyroxine. Given these abnormalities in the context of the patient’s total testosterone level < 150 ng/dL, magnetic resonance imaging (MRI) of the anterior pituitary is indicated, and what I would recommend next for evaluation of pituitary and/or hypothalamic tumor or infiltrative disease.1

fdp03910406_t.png

►Dr. Bhatnagar: An MRI of the brain showed a large 2.7-cm sellar mass, with suprasellar extension and mass effect on the optic chiasm and pituitary infundibulum, partial extension into the right sphenoid sinus, and invasion into the right cavernous sinus. These findings were consistent with a pituitary macroadenoma. The patient was subsequently evaluated by a neurosurgeon who felt that because of the extension and compression of the mass, the patient would benefit from surgical resection.

Given his lower urinary tract symptoms, a prostate-specific antigen level was checked and returned elevated at 11.5 ng/mL. In the setting of these abnormalities, the patient underwent MRI of the abdomen, which noted a new 5.6-cm enhancing mass in the upper pole of his solitary right kidney, highly concerning for new RCC. After a multidisciplinary discussion, urology scheduled the patient for partial right nephrectomy first, with plans for pituitary resection only if the patient had adequate recovery following the urologic procedure.

Dr. Rifkin, this patient went straight from imaging to presumed diagnosis to planned surgical intervention without a confirmatory biopsy. In a patient who already has chronic kidney disease stage 4, why would we not want to pursue biopsy prior to this invasive procedure on his solitary kidney? In addition, given his baseline advanced renal disease, why pursue partial nephrectomy to delay initiation of hemodialysis instead of total nephrectomy and beginning hemodialysis?

►Ian Rifkin, MBBCh, PhD, MSc, Chief, Renal Section, VABHS, Section of Nephrology, BMC, and Associate Professor of Medicine, BUSM: In most cases, imaging alone is used to make a presumptive diagnosis of benign vs malignant renal masses. In one study, RCC was identified by MRI with 85% sensitivity and 76% specificity.2 However, as imaging and biopsy techniques have advanced, there are progressing discussions regarding the utility of biopsy. That being said, there are a number of situations in which patients currently undergo biopsy, particularly when there is diagnostic uncertainty.3 In this patient, with a history of RCC and imaging findings concerning for RCC, biopsy is unnecessary given the high clinical suspicion.

Regarding the choice of partial vs total nephrectomy, there are 2 important distinctions to be made. The first is that though it was previously felt that early initiation of dialysis improves survival, newer studies suggest that early initiation based off of glomerular filtration rate (GFR) offers no survival benefits compared to delayed initiation.4 Second, though there is less clinical data to support this, there is a signal toward the use of partial nephrectomy decreasing mortality compared to radical nephrectomy in management of RCC.5 In this patient, partial nephrectomy may not only increase rates of survival, but also delay initiation of dialysis.

►Dr. Bhatnagar: Prior to undergoing partial right nephrectomy, a morning cortisol level was found to be 5.8 μg/dL with an associated corticotropin (ACTH) level of 26 pg/mL. Dr. Ananthakrishnan, how would you interpret these laboratory results and what might you recommend prior to surgery?

 

 



►Dr. Ananthakrishnan: In a healthy patient, surgery often results in a several-fold increase in the secretion of cortisol to balance the unique stressors surgery places on the body.6 This patient is at increased risk for complete or partial adrenal insufficiency in the setting of both his pituitary macroadenoma as well as his previous left nephrectomy, which could have affected his left adrenal gland as well. Thus, this patient may not be able to mount the appropriate cortisol response needed to counter the stresses of surgery. His cortisol level is abnormally low for a morning value, with a relatively normal ACTH reference range of 6 to 50 pg/mL. He may have some degree of adrenal insufficiency, and thus will benefit from perioperative steroids.

►Dr. Bhatnagar: The patient was started on hydrocortisone and underwent a successful laparoscopic partial right nephrectomy. During the procedure, an estimated 2.5 L of blood was lost, with transfusion of 3 units of packed red blood cells. A surgical drain was left in the peritoneum. Postoperatively, he developed hypotension, requiring vasopressors and prolonged continuation of stress dosing of hydrocortisone. Over the next 4 days, the patient was weaned off vasopressors, and his creatinine level was noted to increase from a baseline of 1.8 mg/dL to 4.4 mg/dL.

Dr. Rifkin, how do you think about renal recovery in the patient postnephrectomy, and should we be concerned with the dramatic rise in his creatinine level?

►Dr. Rifkin: Removal of renal mass will result in an initial reduction of GFR proportional to the amount of functional renal tissue removed. However, in as early as 1 week, the residual nephrons begin to compensate through various mechanisms, such as modulation of efferent and afferent arterioles and renal tissue growth by hypertrophy and hyperplasia.7 In the acute setting, it may be difficult to distinguish an acute renal injury vs physiological GFR reduction postnephron loss, but often the initially elevated creatinine level may normalize/stabilize over time. Other markers of kidney function should concomitantly be monitored, including urine output, electrolyte/acid-base status, and urine sediment examination. In this patient, although his creatinine level may be elevated over the first few days, if his urine output remains robust and the urine sediment examination is normal, my concern for permanent kidney injury would be lessened.

fdp03910406_clincal_takeaways.png


►Dr. Bhatnagar: During the first 4 postoperative days the patient produced approximately 1 L of urine per day with a stable creatinine level. It is over this same time that the hydrocortisone was discontinued given improving hemodynamics. However, throughout postoperative day 5, the patient’s creatinine level acutely rose to a peak of 5.8 mg/dL. In addition, his urine output dramatically dropped to < 5 mL per hour, with blood clots noted in his Foley catheter. Dr. Rifkin, what is your differential for causing this acute change in both his creatinine level and urine output this far out from his procedure, and what might you do to help further evaluate?

►Dr. Rifkin: The most common cause of acute kidney injury in hospitalized patients is acute tubular necrosis (ATN).8 However, in this patient, who was recovering well postoperatively, was hemodynamically stable with a robust urine output, and in whom no apparent cause for ATN could be identified, other diagnoses were more likely. Considering the abrupt onset of oligo-anuria, the most likely diagnosis was urinary tract obstruction, particularly given the frank blood and blood clots that were present in the urine. Additional possibilities might be a late surgical complication or infection. Surgical complications could range from direct damage to the renal parenchyma to urinary leakage into the peritoneum from the site of anastomosis or tissue injury. Infections introduced either intraoperatively or developed postoperatively could also cause this sudden drop in urine output, though one would expect more systemic symptoms with this. Given that this patient has a surgical drain in place in the peritoneum, I would recommend testing the creatinine level in the peritoneal fluid drainage. If it is comparable to serum levels, this would argue against a urine leak, as we would expect the level to be significantly elevated in a leak. In addition, he should have imaging of the urinary tract followed by procedures to decompress the presumed obstructed urinary tract. These procedures might include either cystoscopy with ureteral stent placement or percutaneous nephrostomy, depending on the result of the imaging.

►Dr. Bhatnagar: The creatinine level obtained from the surgical drain was roughly equivalent to the serum creatinine, decreasing suspicion for a urine leak as the cause of his findings. Cystoscopy with ureteral stent placement was performed with subsequent increase in both urine output and concomitant decrease in serum creatinine.

Around this time, the patient also began to note blurry vision. Evaluation revealed difficulty with visual field confrontation in the right lower quadrant, right eye ptosis, right eye impaired adduction, absent abduction and impaired upgaze, but intact downgaze. Diplopia was present with gaze in all directions. His constellation of physical examination findings were concerning for a pathologic lesion partially involving cranial nerves II and III, with definitive involvement of cranial nerve VI, but sparing of cranial nerve IV. Repeat MRI of the brain showed hemorrhage into the sellar mass, with ongoing mass effect on the optic chiasm and extension into the sinuses (eAppendix). These findings were consistent with pituitary apoplexy. Dr. Ananthakrishnan, can you tell us more about pituitary apoplexy?

fdp03910406_eappendix.png

►Dr. Ananthakrishnan: Pituitary apoplexy is a clinical syndrome resulting from acute hemorrhage or infarction of the pituitary gland. It typically occurs in patients with preexisting pituitary adenomas and is characterized by the onset of headache, fever, vomiting, meningismus, decreased consciousness, and sometimes death. In addition, given the location of the pituitary gland within the sella, rapid changes in size can result in compression of cranial nerves III, IV, and VI, as well as the optic chiasm, resulting in ophthalmoplegia and visual disturbances as seen in this patient.9

 

 

There are a multitude of causes of pituitary apoplexy, including alterations in coagulopathy, pituitary stimulation (eg, dynamic pituitary hormone testing), and both acute increases and decreases in blood flow.10 This patient likely had an ischemic event due to changes in vascular perfusion, spurred by both his blood loss intraoperatively and ongoing hematuria. Management of pituitary apoplexy is dependent on the patient’s hemodynamics, mass effect symptoms, electrolyte balances, and hormone dysfunction. The decision for conservative management vs surgical intervention should be made in consultation with both neurosurgery and endocrinology. Once the patient is hemodynamically stable, the next step in evaluating this patient should be repeating his hormone studies.

►Dr. Bhatnagar: An assessment of pituitary function was consistent with values obtained preoperatively. After multidisciplinary discussions, surgery was deferred, and hydrocortisone was reinitiated to reduce inflammation caused by bleeding into the mass. As the ophthalmoplegia improved, this was transitioned to dexamethasone.

Twelve days after admission, he was discharged to a subacute rehabilitation center, with improvement in his ophthalmoplegia and stabilization of his creatinine level and urine output.

References

1. Bhasin S, Cunningham GR, Hayes FJ, et al. Testosterone therapy in men with androgen deficiency syndromes: an Endocrine Society clinical practice guideline. J Clin Endocrinol Metab. 2010;95(6):2536-2559. doi:10.1210/jc.2009-2354

2. Kay FU, Canvasser NE, Xi Y, et al. Diagnostic performance and interreader agreement of a standardized MR imaging approach in the prediction of small renal mass histology. Radiology. 2018;287(2):543-553. doi:10.1148/radiol.2018171557

3. Sahni VA, Silverman SG. Biopsy of renal masses: when and why. Cancer Imaging. 2009;9(1):44-55. doi:10.1102/1470-7330.2009.0005

4. Cooper BA, Branley P, Bulfone L, et al. A randomized, controlled trial of early versus late initiation of dialysis. N Engl J Med. 2010;363(7):609-619. doi:10.1056/NEJMoa1000552

5. Kunath F, Schmidt S, Krabbe L-M, et al. Partial nephrectomy versus radical nephrectomy for clinical localised renal masses. Cochrane Database Syst Rev. 2017;5(5):CD012045. doi:10.1002/14651858.CD012045.pub2

6. Kehlet H, Binder C. Adrenocortical function and clinical course during and after surgery in unsupplemented glucocorticoid-treated patients. Br J Anaesth. 1973;45(10):1043-1048. doi:10.1093/bja/45.10.1043

7. Chapman D, Moore R, Klarenbach S, Braam B. Residual renal function after partial or radical nephrectomy for renal cell carcinoma. Can Urol Assoc J. 2010;4(5):337-343. doi:10.5489/cuaj.909

8. Rahman M, Shad F, Smith MC. Acute kidney injury: a guide to diagnosis and management. Am Fam Physician. 2012;86(7):631-639.

9. Ranabir S, Baruah MP. Pituitary apoplexy. Indian J Endocrinol Metab. 2011;15(suppl 3):S188-S196. doi:10.4103/2230-8210.84862

10. Glezer A, Bronstein MD. Pituitary apoplexy: pathophysiology, diagnosis and management. Arch Endocrinol Metab. 2015;59(3):259-264. doi:10.1590/2359-3997000000047

References

1. Bhasin S, Cunningham GR, Hayes FJ, et al. Testosterone therapy in men with androgen deficiency syndromes: an Endocrine Society clinical practice guideline. J Clin Endocrinol Metab. 2010;95(6):2536-2559. doi:10.1210/jc.2009-2354

2. Kay FU, Canvasser NE, Xi Y, et al. Diagnostic performance and interreader agreement of a standardized MR imaging approach in the prediction of small renal mass histology. Radiology. 2018;287(2):543-553. doi:10.1148/radiol.2018171557

3. Sahni VA, Silverman SG. Biopsy of renal masses: when and why. Cancer Imaging. 2009;9(1):44-55. doi:10.1102/1470-7330.2009.0005

4. Cooper BA, Branley P, Bulfone L, et al. A randomized, controlled trial of early versus late initiation of dialysis. N Engl J Med. 2010;363(7):609-619. doi:10.1056/NEJMoa1000552

5. Kunath F, Schmidt S, Krabbe L-M, et al. Partial nephrectomy versus radical nephrectomy for clinical localised renal masses. Cochrane Database Syst Rev. 2017;5(5):CD012045. doi:10.1002/14651858.CD012045.pub2

6. Kehlet H, Binder C. Adrenocortical function and clinical course during and after surgery in unsupplemented glucocorticoid-treated patients. Br J Anaesth. 1973;45(10):1043-1048. doi:10.1093/bja/45.10.1043

7. Chapman D, Moore R, Klarenbach S, Braam B. Residual renal function after partial or radical nephrectomy for renal cell carcinoma. Can Urol Assoc J. 2010;4(5):337-343. doi:10.5489/cuaj.909

8. Rahman M, Shad F, Smith MC. Acute kidney injury: a guide to diagnosis and management. Am Fam Physician. 2012;86(7):631-639.

9. Ranabir S, Baruah MP. Pituitary apoplexy. Indian J Endocrinol Metab. 2011;15(suppl 3):S188-S196. doi:10.4103/2230-8210.84862

10. Glezer A, Bronstein MD. Pituitary apoplexy: pathophysiology, diagnosis and management. Arch Endocrinol Metab. 2015;59(3):259-264. doi:10.1590/2359-3997000000047

Issue
Federal Practitioner - 39(10)a
Issue
Federal Practitioner - 39(10)a
Page Number
406-409
Page Number
406-409
Publications
Publications
Topics
Article Type
Sections
Teambase XML
<?xml version="1.0" encoding="UTF-8"?>
<!--$RCSfile: InCopy_agile.xsl,v $ $Revision: 1.35 $-->
<!--$RCSfile: drupal.xsl,v $ $Revision: 1.7 $-->
<root generator="drupal.xsl" gversion="1.7"> <header> <fileName>1022 FED Forum</fileName> <TBEID>0C02AAC9.SIG</TBEID> <TBUniqueIdentifier>NJ_0C02AAC9</TBUniqueIdentifier> <newsOrJournal>Journal</newsOrJournal> <publisherName>Frontline Medical Communications Inc.</publisherName> <storyname/> <articleType>1</articleType> <TBLocation>Copyfitting-FED</TBLocation> <QCDate/> <firstPublished>20221012T153451</firstPublished> <LastPublished>20221012T153451</LastPublished> <pubStatus qcode="stat:"/> <embargoDate/> <killDate/> <CMSDate>20221012T153450</CMSDate> <articleSource/> <facebookInfo/> <meetingNumber/> <byline/> <bylineText> Anish Bhatnagar, MD a,b ; Sonia Ananthakrishan, MD c,d ; Ian Rifkin, MBBCh, PhD, MSc c, d,e ; Anthony C. Breu, MD b,e </bylineText> <bylineFull/> <bylineTitleText/> <USOrGlobal/> <wireDocType/> <newsDocType/> <journalDocType/> <linkLabel/> <pageRange/> <citation/> <quizID/> <indexIssueDate/> <itemClass qcode="ninat:text"/> <provider qcode="provider:"> <name/> <rightsInfo> <copyrightHolder> <name/> </copyrightHolder> <copyrightNotice/> </rightsInfo> </provider> <abstract/> <metaDescription>Case Presentation: A 76-year-old patient presented with decreased libido, repeated low testosterone levels, and lower urinary tract symptoms. He had a history o</metaDescription> <articlePDF/> <teaserImage/> <title>A Veteran Presenting for Low Testosterone and Lower Urinary Tract Symptoms</title> <deck/> <eyebrow>VA Boston Medical Forum</eyebrow> <disclaimer/> <AuthorList/> <articleURL/> <doi/> <pubMedID/> <publishXMLStatus/> <publishXMLVersion>1</publishXMLVersion> <useEISSN>0</useEISSN> <urgency/> <pubPubdateYear/> <pubPubdateMonth/> <pubPubdateDay/> <pubVolume/> <pubNumber/> <wireChannels/> <primaryCMSID/> <CMSIDs> <CMSID>3639</CMSID> </CMSIDs> <keywords/> <seeAlsos/> <publications_g> <publicationData> <publicationCode>FED</publicationCode> <pubIssueName>October 2022</pubIssueName> <pubArticleType>Feature Articles | 3639</pubArticleType> <pubTopics/> <pubCategories/> <pubSections/> <journalTitle>Fed Pract</journalTitle> <journalFullTitle>Federal Practitioner</journalFullTitle> <copyrightStatement>Copyright 2017 Frontline Medical Communications Inc., Parsippany, NJ, USA. All rights reserved.</copyrightStatement> </publicationData> </publications_g> <publications> <term canonical="true">16</term> </publications> <sections> <term canonical="true">58388</term> </sections> <topics> <term canonical="true">27442</term> </topics> <links/> </header> <itemSet> <newsItem> <itemMeta> <itemRole>Main</itemRole> <itemClass>text</itemClass> <title>A Veteran Presenting for Low Testosterone and Lower Urinary Tract Symptoms</title> <deck/> </itemMeta> <itemContent> <p><strong>Case Presentation: </strong><i>A 76-year-old patient presented with decreased libido, repeated low testosterone levels, and lower urinary tract symptoms. He had a history of a left nephrectomy approximately 17 years prior for presumed renal cell carcinoma (RCC) subsequently complicated by chronic kidney disease stage 4, benign prostatic hypertrophy, erectile dysfunction, and a 50-pack-year history of smoking. His medications included tadalafil, tamsulosin, oxybutynin, and losartan. <br/><br/></i><strong></strong><strong>Anish Bhatnagar, MD, Chief Medical Resident, Veterans Affairs Boston Healthcare System (VABHS) and Beth Israel Deaconess Medical Center (BIDMC): </strong>The patient noted erectile dysfunction starting 4 years ago, with accompanied decreased libido. However, until recently, he was able to achieve acceptable erectile capacity with medications. As part of his previous evaluations for erectile dysfunction, the patient had 2 total testosterone levels checked 6 months apart, both low at 150 ng/dL and 38.3 ng/dL (reference range, 220-892). The results of additional hormone studies are shown in the Table. Dr. Ananthakrishnan, can you help us interpret these laboratory results and tell us what tests you might order next? </p> <p><strong></strong><strong>Sonia Ananthakrishnan, MD, Section of Endocrinology, Diabetes and Nutrition, Boston Medical Center (BMC) and Assistant Professor of Medicine, Boston University School of Medicine (BUSM):</strong><b> </b>When patients present with signs of hypogonadism and an initial low morning testosterone levels, the next test should be a confirmatory repeat morning testosterone level as was done in this case. If this level is also low (for most assays &lt; 300 ng/dL), further evaluation for primary vs secondary hypogonadism should be pursued with measurement of luteinizing hormone and follicle-stimulating hormone levels. Secondary hypogonadism should be suspected when these levels are low or inappropriately normal in the setting of a low testosterone level as in this patient. This patient does not appear to be on any medication or have reversible illnesses that we traditionally think of as possibly causing these hormone irregularities. Key examples include medications such as gonadotropin-releasing hormone analogs, glucocorticoids, and opioids, as well as conditions such as hyperprolactinemia, sleep apnea, diabetes mellitus, anorexia nervosa, or other chronic systemic illnesses, including cirrhosis or lung disease. In this setting, further evaluation of the patient’s anterior pituitary function should be undertaken. Initial screening tests showed mildly elevated prolactin and low normal thyroid-stimulating hormone levels, with a relatively normal free thyroxine. Given these abnormalities in the context of the patient’s total testosterone level &lt; 150 ng/dL, magnetic resonance imaging (MRI) of the anterior pituitary is indicated, and what I would recommend next for evaluation of pituitary and/or hypothalamic tumor or infiltrative disease.<sup>1</sup><strong></strong><strong>Dr. Bhatnagar: </strong>An MRI of the brain showed a large 2.7-cm sellar mass, with suprasellar extension and mass effect on the optic chiasm and pituitary infundibulum, partial extension into the right sphenoid sinus, and invasion into the right cavernous sinus. These findings were consistent with a pituitary macroadenoma. The patient was subsequently evaluated by a neurosurgeon who felt that because of the extension and compression of the mass, the patient would benefit from surgical resection. </p> <p>Given his lower urinary tract symptoms, a prostate-specific antigen level was checked and returned elevated at 11.5 ng/mL. In the setting of these abnormalities, the patient underwent MRI of the abdomen, which noted a new 5.6-cm enhancing mass in the upper pole of his solitary right kidney, highly concerning for new RCC. After a multidisciplinary discussion, urology scheduled the patient for partial right nephrectomy first, with plans for pituitary resection only if the patient had adequate recovery following the urologic procedure.<br/><br/>Dr. Rifkin, this patient went straight from imaging to presumed diagnosis to planned surgical intervention without a confirmatory biopsy. In a patient who already has chronic kidney disease stage 4, why would we not want to pursue biopsy prior to this invasive procedure on his solitary kidney? In addition, given his baseline advanced renal disease, why pursue partial nephrectomy to delay initiation of hemodialysis instead of total nephrectomy and beginning hemodialysis?</p> <p><strong></strong><strong>Ian Rifkin, MBBCh, PhD, MSc, Chief, Renal Section, VABHS, Section of Nephrology, BMC, and Associate Professor of Medicine, BUSM:</strong><b> </b>In most cases, imaging alone is used to make a presumptive diagnosis of benign vs malignant renal masses. In one study, RCC was identified by MRI with 85% sensitivity and 76% specificity.<sup>2</sup> However, as imaging and biopsy techniques have advanced, there are progressing discussions regarding the utility of biopsy. That being said, there are a number of situations in which patients currently undergo biopsy, particularly when there is diagnostic uncertainty.<sup>3 </sup>In this patient, with a history of RCC and imaging findings concerning for RCC, biopsy is unnecessary given the high clinical suspicion. </p> <p>Regarding the choice of partial vs total nephrectomy, there are 2 important distinctions to be made. The first is that though it was previously felt that early initiation of dialysis improves survival, newer studies suggest that early initiation based off of glomerular filtration rate (GFR) offers no survival benefits compared to delayed initiation.<sup>4</sup> Second, though there is less clinical data to support this, there is a signal toward the use of partial nephrectomy decreasing mortality compared to radical nephrectomy in management of RCC.<sup>5</sup> In this patient, partial nephrectomy may not only increase rates of survival, but also delay initiation of dialysis. </p> <p><strong></strong><strong>Dr. Bhatnagar:</strong><b> </b>Prior to undergoing partial right nephrectomy, a morning cortisol level was found to be 5.8 μg/dL with an associated corticotropin (ACTH) level of 26 pg/mL. Dr. Ananthakrishnan, how would you interpret these laboratory results and what might you recommend prior to surgery? <br/><br/><strong></strong><strong>Dr. Ananthakrishnan:</strong> In a healthy patient, surgery often results in a several-fold increase in the secretion of cortisol to balance the unique stressors surgery places on the body.<sup>6</sup> This patient is at increased risk for complete or partial adrenal insufficiency in the setting of both his pituitary macroadenoma as well as his previous left nephrectomy, which could have affected his left adrenal gland as well. Thus, this patient may not be able to mount the appropriate cortisol response needed to counter the stresses of surgery. His cortisol level is abnormally low for a morning value, with a relatively normal ACTH reference range of 6 to 50 pg/mL. He may have some degree of adrenal insufficiency, and thus will benefit from perioperative steroids.<strong></strong><strong>Dr. Bhatnagar: </strong>The patient was started on hydrocortisone and underwent a successful laparoscopic partial right nephrectomy. During the procedure, an estimated 2.5 L of blood was lost, with transfusion of 3 units of packed red blood cells. A surgical drain was left in the peritoneum. Postoperatively, he developed hypotension, requiring vasopressors and prolonged continuation of stress dosing of hydrocortisone. Over the next 4 days, the patient was weaned off vasopressors, and his creatinine level was noted to increase from a baseline of 1.8 mg/dL to 4.4 mg/dL. </p> <p>Dr. Rifkin, how do you think about renal recovery in the patient postnephrectomy, and should we be concerned with the dramatic rise in his creatinine level? </p> <p><strong></strong><strong>Dr. Rifkin:</strong> Removal of renal mass will result in an initial reduction of GFR proportional to the amount of functional renal tissue removed. However, in as early as 1 week, the residual nephrons begin to compensate through various mechanisms, such as modulation of efferent and afferent arterioles and renal tissue growth by hypertrophy and hyperplasia.<sup>7</sup> In the acute setting, it may be difficult to distinguish an acute renal injury vs physiological GFR reduction postnephron loss, but often the initially elevated creatinine level may normalize/stabilize over time. Other markers of kidney function should concomitantly be monitored, including urine output, electrolyte/acid-base status, and urine sediment examination. In this patient, although his creatinine level may be elevated over the first few days, if his urine output remains robust and the urine sediment examination is normal, my concern for permanent kidney injury would be lessened. <br/><br/><strong></strong><strong>Dr. Bhatnagar:</strong><b> </b>During the first 4 postoperative days the patient produced approximately 1 L of urine per day with a stable creatinine level. It is over this same time that the hydrocortisone was discontinued given improving hemodynamics. However, throughout postoperative day 5, the patient’s creatinine level acutely rose to a peak of 5.8 mg/dL. In addition, his urine output dramatically dropped to &lt; 5 mL per hour, with blood clots noted in his Foley catheter. Dr. Rifkin, what is your differential for causing this acute change in both his creatinine level and urine output this far out from his procedure, and what might you do to help further evaluate? <strong></strong><strong>Dr. Rifkin:</strong><b> </b>The most common cause of acute kidney injury in hospitalized patients is acute tubular necrosis (ATN).<sup>8</sup> However, in this patient, who was recovering well postoperatively, was hemodynamically stable with a robust urine output, and in whom no apparent cause for ATN could be identified, other diagnoses were more likely. Considering the abrupt onset of oligo-anuria, the most likely diagnosis was urinary tract obstruction, particularly given the frank blood and blood clots that were present in the urine. Additional possibilities might be a late surgical complication or infection. Surgical complications could range from direct damage to the renal parenchyma to urinary leakage into the peritoneum from the site of anastomosis or tissue injury. Infections introduced either intraoperatively or developed postoperatively could also cause this sudden drop in urine output, though one would expect more systemic symptoms with this. Given that this patient has a surgical drain in place in the peritoneum, I would recommend testing the creatinine level in the peritoneal fluid drainage. If it is comparable to serum levels, this would argue against a urine leak, as we would expect the level to be significantly elevated in a leak. In addition, he should have imaging of the urinary tract followed by procedures to decompress the presumed obstructed urinary tract. These procedures might include either cystoscopy with ureteral stent placement or percutaneous nephrostomy, depending on the result of the imaging.<br/><br/><strong></strong><strong>Dr. Bhatnagar:</strong><b> </b>The creatinine level obtained from the surgical drain was roughly equivalent to the serum creatinine, decreasing suspicion for a urine leak as the cause of his findings. Cystoscopy with ureteral stent placement was performed with subsequent increase in both urine output and concomitant decrease in serum creatinine.</p> <p>Around this time, the patient also began to note blurry vision. Evaluation revealed difficulty with visual field confrontation in the right lower quadrant, right eye ptosis, right eye impaired adduction, absent abduction and impaired upgaze, but intact downgaze. Diplopia was present with gaze in all directions. His constellation of physical examination findings were concerning for a pathologic lesion partially involving cranial nerves II and III, with definitive involvement of cranial nerve VI, but sparing of cranial nerve IV. Repeat MRI of the brain showed hemorrhage into the sellar mass, with ongoing mass effect on the optic chiasm and extension into the sinuses (eAppendix, available at doi:10.12788/fp.0326). These findings were consistent with pituitary apoplexy. Dr. Ananthakrishnan, can you tell us more about pituitary apoplexy? </p> <p><strong></strong><strong>Dr. Ananthakrishnan: </strong>Pituitary apoplexy is a clinical syndrome resulting from acute hemorrhage or infarction of the pituitary gland. It typically occurs in patients with preexisting pituitary adenomas and is characterized by the onset of headache, fever, vomiting, meningismus, decreased consciousness, and sometimes death. In addition, given the location of the pituitary gland within the sella, rapid changes in size can result in compression of cranial nerves III, IV, and VI, as well as the optic chiasm, resulting in ophthalmoplegia and visual disturbances as seen in this patient.<sup>9</sup> </p> <p>There are a multitude of causes of pituitary apoplexy, including alterations in coagulopathy, pituitary stimulation (eg, dynamic pituitary hormone testing), and both acute increases and decreases in blood flow.<sup>10 </sup>This patient likely had an ischemic event due to changes in vascular perfusion, spurred by both his blood loss intraoperatively and ongoing hematuria. Management of pituitary apoplexy is dependent on the patient’s hemodynamics, mass effect symptoms, electrolyte balances, and hormone dysfunction. The decision for conservative management vs surgical intervention should be made in consultation with both neurosurgery and endocrinology. Once the patient is hemodynamically stable, the next step in evaluating this patient should be repeating his hormone studies. </p> <p><strong></strong><strong>Dr. Bhatnagar:</strong> An assessment of pituitary function was consistent with values obtained preoperatively. After multidisciplinary discussions, surgery was deferred, and hydrocortisone was reinitiated to reduce inflammation caused by bleeding into the mass. As the ophthalmoplegia improved, this was transitioned to dexamethasone. </p> <p>Twelve days after admission, he was discharged to a subacute rehabilitation center, with improvement in his ophthalmoplegia and stabilization of his creatinine level and urine output. </p> <h3>Author affiliations</h3> <p> <em> <sup> a </sup> Beth Israel Deaconess Medical Center, Boston, Massachusetts <br/><br/> <sup> b </sup> Harvard Medical School, Boston, Massachusetts<br/><br/> <sup> c </sup> Boston Medical Center, Massachusetts<br/><br/> <sup> d </sup> Boston University School of Medicine, Massachusetts<br/><br/> <sup> e </sup> Veterans Affairs Boston Healthcare System, West Roxbury, Massachusetts </em> </p> <h3>Author disclosures</h3> <p> <em> The authors report no actual or potential conflicts of interest or outside sources of funding with regard to this article. </em> </p> <h3>Disclaimer</h3> <p> <em> The opinions expressed herein are those of the authors and do not necessarily reflect those of <i>Federal Practitioner</i> , Frontline Medical Communications Inc., the US Government, or any of its agencies. This article may discuss unlabeled or investigational use of certain drugs. Please review the complete prescribing information for specific drugs or drug combinations—including indications, contraindications, warnings, and adverse effects—before administering pharmacologic therapy to patients. </em> </p> <h3>Ethics and consent</h3> <p> <em> Written patient consent was obtained prior to publication. </em> </p> <h3>References</h3> <p class="reference"> 1. Bhasin S, Cunningham GR, Hayes FJ, et al. Testosterone therapy in men with androgen deficiency syndromes: an Endocrine Society clinical practice guideline. <i>J Clin Endocrinol Metab. </i>2010;95(6):2536-2559. doi:10.1210/jc.2009-2354<br/><br/> 2. Kay FU, Canvasser NE, Xi Y, et al. Diagnostic performance and interreader agreement of a standardized MR imaging approach in the prediction of small renal mass histology. <i>Radiology.</i> 2018;287(2):543-553. doi:10.1148/radiol.2018171557<br/><br/> 3. Sahni VA, Silverman SG. Biopsy of renal masses: when and why.<i> Cancer Imaging</i>. 2009;9(1):44-55. doi:10.1102/1470-7330.2009.0005<br/><br/> 4. Cooper BA, Branley P, Bulfone L, et al. A randomized, controlled trial of early versus late initiation of dialysis. <i>N Engl J Med. </i>2010;363(7):609-619. doi:10.1056/NEJMoa1000552<br/><br/> 5. Kunath F, Schmidt S, Krabbe L-M, et al. Partial nephrectomy versus radical nephrectomy for clinical localised renal masses. <i>Cochrane Database Syst Rev.</i> 2017;5(5):CD012045. doi:10.1002/14651858.CD012045.pub2<br/><br/> 6. Kehlet H, Binder C. Adrenocortical function and clinical course during and after surgery in unsupplemented glucocorticoid-treated patients. <i>Br J Anaesth. </i>1973;45(10):1043-1048. doi:10.1093/bja/45.10.1043<br/><br/> 7. Chapman D, Moore R, Klarenbach S, Braam B. Residual renal function after partial or radical nephrectomy for renal cell carcinoma. <i>Can Urol Assoc J. </i>2010;4(5):337-343. doi:10.5489/cuaj.909<br/><br/> 8. Rahman M, Shad F, Smith MC. Acute kidney injury: a guide to diagnosis and management. <i>Am Fam Physician.</i> 2012;86(7):631-639.<br/><br/> 9. Ranabir S, Baruah MP. Pituitary apoplexy.<i> Indian J Endocrinol Metab. </i>2011;15(suppl 3):S188-S196. doi:10.4103/2230-8210.84862<br/><br/>10. Glezer A, Bronstein MD. Pituitary apoplexy: pathophysiology, diagnosis and management. <i>Arch Endocrinol Metab. </i>2015;59(3):259-264. doi:10.1590/2359-3997000000047</p> </itemContent> </newsItem> </itemSet></root>
Disallow All Ads
Content Gating
No Gating (article Unlocked/Free)
Alternative CME
Disqus Comments
Default
Use ProPublica
Hide sidebar & use full width
render the right sidebar.
Conference Recap Checkbox
Not Conference Recap
Clinical Edge
Display the Slideshow in this Article
Medscape Article
Display survey writer
Reuters content
Disable Inline Native ads
WebMD Article
Article PDF Media

A Veteran With Recurrent, Painful Knee Effusion

Article Type
Changed
Wed, 07/13/2022 - 12:14

Case Presentation: A 39-year-old Air Force veteran was admitted to the US Department of Veterans Affairs Boston Healthcare System (VABHS) for evaluation of recurrent, painful right knee effusions. On presentation, his vital signs were stable, and the examination was significant for a right knee with a large effusion and tenderness to palpation without erythema or warmth. His white blood cell count was 12.0 cells/L with an erythrocyte sedimentation rate of 23 mm/h and C-reactive protein of 11.87 mg/L. He was in remission from alcohol use but had relapsed on alcohol in the past day to treat the pain. He had a history of IV drug use but was in remission. He was previously active and enjoyed long hikes. Nine months prior to presentation, he developed his first large right knee effusion associated with pain. He reported no antecedent trauma. At that time, he presented to another hospital and underwent arthrocentesis with orthopedic surgery, but this did not lead to a diagnosis, and the effusion reaccumulated within 24 hours. Four months later, he received a corticosteroid injection that provided only minor, temporary relief. He received 5 additional arthrocenteses over 9 months, all without definitive diagnosis and with rapid reaccumulation of the fluid. His most recent arthrocentesis was 3 weeks before admission.

►Lauren E. Merz, MD, MSc, Chief Medical Resident, VABHS: Dr. Jindal, what is your approach and differential diagnosis for joint effusions in hospitalized patients?

►Shivani Jindal, MD, MPH, Hospitalist, VABHS, Instructor in Medicine, Boston University School of Medicine (BUSM): A thorough history and physical examination are important. I specifically ask about chronicity, pain, and trauma. A medical history of potential infectious exposures and the history of the present illness are also important, such as the risk of sexually transmitted infections, exposure to Lyme disease or other viral illnesses. Gonococcal arthritis is one of the most common causes of nontraumatic monoarthritis in young adults but can also present as a migratory polyarthritis.1

It sounds like he was quite active and liked to hike so a history of tick exposure is important to ascertain. I would also ask about eye inflammation and back pain to assess possible ankylosing spondyarthritis. Other inflammatory etiologies, such as gout are common, but it would be surprising to miss this diagnosis on repeated arthocenteses. A physical examination can confirm monoarthritis over polyarthritis and assess for signs of inflammatory arthritis (eg, warmth and erythema). The most important etiology to assess for and rule out in a person admitted to the hospital is septic arthritis. The severe pain, mild leukocytosis, and mildly elevated inflammatory markers could be consistent with this diagnosis but are nonspecific. However, the chronicity of this patient’s presentation and hemodynamic stability make septic arthritis less likely overall and a more indolent infection or other inflammatory process more likely.

►Dr. Merz: The patient’s medical history included posttraumatic stress disorder (PTSD) and antisocial personality disorder with multiple prior suicide attempts. He also had a history of opioid use disorder (OUD) with prior overdose and alcohol use disorder (AUD). Given his stated preference to avoid opioids and normal liver function and liver chemistry testing, the initial treatment was with acetaminophen. After this failed to provide satisfactory pain control, IV hydromorphone was added.

 

 

Dr. Jindal, how do you approach pain control in the hospital for musculoskeletal issues like this?

►Dr. Jindal: Typically, nonsteroidal anti-inflammatory medications (NSAIDs) are most effective for musculoskeletal pain, often in the form of ketorolac or ibuprofen. However, we are often limited in our NSAID use by kidney disease, gastritis, or cardiovascular disease. Selective COX-2 inhibitors (eg, celecoxib) have the advantage of a lower risk of gastrointestinal bleeding. Topical formulations (eg, diclofenac) may also come with a lower adverse effect profile. Corticosteroids are also an option but come with their own adverse effect profile. This patient does not have any of these comorbidities. Adjuvant therapies such as lidocaine patches or capsaicin cream can also provide relief. Gabapentin or pregabalin are indicated for any component of neuropathic pain. Opioids can be helpful for acute musculoskeletal pain, but there is no long-term benefit in chronic musculoskeletal pain.2 The experience of pain is also multifactorial so ensuring that anxiety and insomnia are addressed is key.

►Dr. Merz: On hospital day 1, the patient asked to leave to consume alcohol to ease unremitting pain. He also expressed suicidal ideation and discharge was therefore felt to be unsafe. He was reluctant to engage with psychiatry and became physically combative while attempting to leave the hospital, necessitating the use of sedating medications and physical restraints.

Dr. Shahal, what factors led to the decision to place an involuntary hold, and how do you balance patient autonomy and patient safety?

►Dr. Talya Shahal, MD, Consult-Liaison Psychiatry Service, VABHS, Instructor in Psychiatry, Harvard Medical School: This is a delicate balance that requires constant reassessment. The patient initially presented to the emergency department with suicidal ideation, stating he was not able to tolerate the pain and thus resumed alcohol consumption after a period of nonuse. He had multiple risk factors for suicide, including 9 prior suicide attempts with the latest less than a year before presentation, active substance use with alcohol and other recreational drugs, PTSD, pain, veteran status, male sex, single status, and a history of trauma.3,4 He was also displaying impulsivity and limited insight, did not engage in his psychiatric assessment, and attempted to assault staff. As such, his suicide risk was assessed to be high at the time of the evaluation, which led to the decision to place an involuntary hold. However, we reevaluate this decision at least daily in order to reassess the risk and ensure that the balance between patient safety and autonomy are maintained.

►Dr. Merz: The involuntary hold was removed within 48 hours as the patient remained calm and engaged with the primary and consulting teams. He requested escalating doses of opioids as he felt the short-acting IV medications were not providing sustained relief. However, he was also noted to be walking outside of the hospital without assistance, and he repeatedly declined nonopioid pain modalities as well as buprenorphine/naloxone. The chronic pain service was consulted but was unable to see the patient as he was frequently outside of his room.

 

 

Dr. Shahal, how do you address OUD, pain, and stigma in the hospital?

►Dr. Shahal: It is important to remember that patients with substance use disorder (SUD) and mental illness frequently have physical causes for pain and are often undertreated.5 Patients with SUD may also have higher tolerance for opioids and may need higher doses to treat the pain.5 Modalities like buprenorphine/naloxone can be effective to treat OUD and pain, but these usually cannot be initiated while the patient is on short-acting opioids as this would precipitate withdrawal.6 However, withdrawal can be managed while inpatient, and this can be a good time to start these medications as practitioners can aggressively help with symptom control. Proactively addressing mental health concerns, particularly anxiety, AUD, insomnia, PTSD, and depression, can also have a direct impact on the perception of pain and assist with better control.2 In addition, nonpharmacologic options, such as meditation, deep breathing, and even acupuncture and Reiki can be helpful and of course less harmful to treat pain.2

► Dr. Merz: An X-ray of the knee showed no acute fracture or joint space narrowing. Magnetic resonance imaging confirmed a large knee effusion with no evidence of ligament injury. Synovial fluid showed turbid, yellow fluid with 14,110 nucleated cells (84% segmented cells and 4000 RBCs). Gram stain was negative, culture had no growth, and there were no crystals. Anticyclic citrullinated peptide (anti-CCP), rheumatoid factor, HIV testing, and HLA-B27 were negative.

Dr. Serrao, what do these studies tell us about the joint effusion and the possible diagnoses?

► Dr. Richard Serrao, MD, Infectious Disease, VABHS, Clinical Associate Professor in Medicine, BUSM: I would expect the white blood cell (WBC) count to be > 50,000 cells with > 75% polymorphonuclear cells and a positive Gram stain if this was a bacterial infection resulting in septic arthritis.7 This patient’s studies are not consistent with this diagnosis nor is the chronicity of his presentation. There are 2 important bacteria that can present with inflammatory arthritis and less pronounced findings on arthrocentesis: Borrelia burgdorferi (the bacteria causing Lyme arthritis) and Neisseria gonorrhea. Lyme arthritis could be consistent with this relapsing remitting presentation as you expect a WBC count between 3000 and 100,000 cells with a mean value between 10,000 and 25,000 cells, > 50% polymorphonuclear leukocytes, and negative Gram stains.8 Gonococcal infections often do not have marked elevations in the WBC count and the Gram stain can be variable, but you still expect the WBC count to be > 30,000 cells.7 Inflammatory causes such as gout or autoimmune conditions such as lupus often have a WBC count between 2000 and 100,000 with a negative Gram stain, which could be consistent with this patient’s presentation.7 However, the lack of crystals rules out gout and the negative anti-CCP, rheumatoid factor, and HLA-B27 make rheumatologic diseases less likely.

►Dr. Merz: The patient received a phone call from another hospital where an arthrocentesis had been performed 3 weeks before. The results included a positive polymerase chain reaction (PCR) test for Lyme disease in the synovial fluid. A subsequent serum Lyme screen was positive for 1 of 3 immunoglobulin (Ig) M bands and 10 of 10 IgG bands.

 

 

Dr. Serrao, how does Lyme arthritis typically present, and are there aspects of this case that make you suspect the diagnosis? Does the serum Lyme test give us any additional information?

►Dr. Serrao: Lyme arthritis is a late manifestation of Lyme disease. Patients typically have persistent or intermittent arthritis, and large joints are more commonly impacted than small joints. Monoarthritis of the knee is the most common, but oligoarthritis is possible as well. The swelling usually begins abruptly, lasts for weeks to months, and effusions typically recur quickly after aspiration. These findings are consistent with the patient’s clinical history.

For diagnostics, the IgG Western blot is positive if 5 of the 10 bands are positive.9 This patient far exceeds the IgG band number to diagnose Lyme disease. All patients with Lyme arthritis will have positive IgG serologies since Lyme arthritis is a late manifestation of the infection. IgM reactivity may be present, but are not necessary to diagnose Lyme arthritis.10 Synovial fluid is often not analyzed for antibody responses as they are susceptible to false positive results, but synovial PCR testing like this patient had detects approximately 70% of patients with untreated Lyme arthritis.11 However, PCR positivity does not necessarily equate with active infection. Serologic testing for Lyme disease by enzyme-linked immunosorbent assay and Western blot as well as careful history and the exclusion of other diagnoses are usually sufficient to make the diagnosis.

► Dr. Merz: On further history the patient reported that 5 years prior he found a tick on his skin with a bull’s-eye rash. He was treated with 28 days of doxycycline at that time. He did not recall any tick bites or rashes in the years since.

Dr. Serrao, is it surprising that he developed Lyme arthritis 5 years after exposure and after being treated appropriately? What is the typical treatment approach for a patient like this?

►Dr. Serrao: It is atypical to develop Lyme arthritis 5 years after reported treatment of what appeared to be early localized disease, namely, erythema migrans. This stage is usually cured with 10 days of treatment alone (he received 28 days) and is generally abortive of subsequent stages, including Lyme arthritis. Furthermore, the patient reported no symptoms of arthritis until recently since that time. Therefore, one can argue that the excessively long span of time from treatment to these first episodes of arthritis suggests the patient could have been reinfected. When available, comparing the types and number of Western blot bands (eg, new and/or more bands on subsequent serologic testing) can support a reinfection diagnosis. A delayed postinfectious inflammatory process from excessive proinflammatory immune responses that block wound repair resulting in proliferative synovitis is also possible.12 This is defined as the postinfectious, postantibiotic, or antibiotic-refractory Lyme arthritis, a diagnosis of exclusion more apparent only after patients receive appropriate antibiotic courses for the possibility of untreated Lyme as an active infection.12

Given the inherent diagnostic uncertainty between an active infection and posttreatment Lyme arthritis syndromes, it is best to approach most cases of Lyme arthritis as an active infection first especially if not yet treated with antibiotics. Diagnosis of postinflammatory processes should be considered if symptoms persist after appropriate antibiotics, and then short-term use of disease-modifying antirheumatic drugs, rather than further courses of antibiotics, is recommended.

 

 

► Dr. Merz: The patient was initiated on doxycycline with the plan to transition to ceftriaxone if there was no response. One day after diagnosis and treatment initiation and in the setting of continued pain, the patient again asked to leave the hospital to drink alcohol. After eloping and becoming intoxicated with alcohol, he returned to his room. He remained concerned about his continued pain and lack of adequate pain control. At the time, he was receiving hydromorphone, ketorolac, lorazepam, gabapentin, and quetiapine.

Dr. Serrao, do you expect this degree of pain from Lyme arthritis?

► Dr. Serrao: Lyme arthritis is typically less painful than other forms of infectious or inflammatory arthritis. Pain is usually caused by the pressure from the acute accumulation and reaccumulation of fluid. In this case, the rapid accumulation of fluid that this patient experienced as well as relief with arthrocentesis suggests that the size and acuity of the effusion was causing great discomfort. Repeated arthrocentesis can prove to be a preventative strategy to minimize synovial herniation.

►Dr. Merz: Dr. Shahal, how do you balance the patient subjectively telling you that they are in pain with objective signs that they may be tolerating the pain like walking around unassisted? Is there anything else that could have been done to prevent this adverse outcome?

►Dr. Shahal: This is one of the hardest pieces of pain management. We want to practice beneficence by believing our patients and addressing their discomfort, but we also want to practice nonmaleficence by avoiding inappropriate long-term pain treatments like opioids that have significant harm as well as avoiding exacerbating this patient’s underlying SUD. An agent like buprenorphine/naloxone could have been an excellent fit to treat pain and SUD, but the patient’s lack of interest and the frequent use of short-acting opioids were major barriers. A chronic pain consult early on is helpful in cases like this as well, but they were unable to see him since he was often out of his room. Repeated arthrocentesis may also have helped the pain. Treatment of anxiety and insomnia with medications like hydroxyzine, trazodone, melatonin, gabapentin, or buspirone as well as interventions like sleep hygiene protocols or spiritual care may have helped somewhat as well.

We know that there is a vicious cycle between pain and poorly controlled mood symptoms. Many of our veterans have PTSD, anxiety, and SUD that are exacerbated by hospitalization and pain. Maintaining optimal communication between the patient and the practitioners, using trauma-informed care, understanding the patient’s goals of care, setting expectations and limits, and attempting to address the patient’s needs while attempting to minimize stigma might be helpful. However, despite optimal care, sometimes these events cannot be avoided.

►Dr. Merz: The patient was ultimately transferred to an inpatient psychiatric unit where a taper plan for the short-acting opioids was implemented. He was psychiatrically stabilized and discharged a few days later off opioids and on doxycycline. On follow-up a few weeks later, his pain had markedly improved, and the effusion was significantly reduced in size. His mood and impulsivity had stabilized. He continues to follow-up in the infectious disease clinic.

 

fdp03907315_takeaways.png

References

1. Siva C, Velazquez C, Mody A, Brasington R. Diagnosing acute monoarthritis in adults: a practical approach for the family physician. Am Fam Physician. 2003;68(1):83-90.

2. Qaseem A, McLean RM, O’Gurek D, et al. Nonpharmacologic and pharmacologic management of acute pain from non-low back, musculoskeletal injuries in adults: a clinical guideline from the American College of Physicians and American Academy of Family Physicians. Ann Intern Med. 2020;173(9):739-748. doi:10.7326/M19-3602

3. Silverman MM, Berman AL. Suicide risk assessment and risk formulation part I: a focus on suicide ideation in assessing suicide risk. Suicide Life Threat Behav. 2014;44(4):420-431. doi:10.1111/sltb.12065

4. Berman AL, Silverman MM. Suicide risk assessment and risk formulation part II: Suicide risk formulation and the determination of levels of risk. Suicide Life Threat Behav. 2014;44(4):432-443. doi:10.1111/sltb.12067

5. Quinlan J, Cox F. Acute pain management in patients with drug dependence syndrome. Pain Rep. 2017;2(4):e611. Published 2017 Jul 27. doi:10.1097/PR9.0000000000000611

6. Chou R, Wagner J, Ahmed AY, et al. Treatments for Acute Pain: A Systematic Review. Agency for Healthcare Research and Quality; 2020. https://www.ncbi.nlm.nih.gov/books/NBK566506/

7. Seidman AJ, Limaiem F. Synovial fluid analysis. In: StatPearls [Internet]. Treasure Island (FL): StatPearls Publishing; 2022. Updated May 8, 2022. https://www.ncbi.nlm.nih.gov/books/NBK537114

8. Arvikar SL, Steere AC. Diagnosis and treatment of Lyme arthritis. Infect Dis Clin North Am. 2015;29(2):269-280. doi:10.1016/j.idc.2015.02.004

9. Centers for Disease Control and Prevention. Recommendations for test performance and interpretation from the Second National Conference on Serologic Diagnosis of Lyme Disease. JAMA. 1995;274(12):937.

10. Craft JE, Grodzicki RL, Steere AC. Antibody response in Lyme disease: evaluation of diagnostic tests. J Infect Dis. 1984;149(5):789-795. doi:10.1093/infdis/149.5.789

11. Nocton JJ, Dressler F, Rutledge BJ, Rys PN, Persing DH, Steere AC. Detection of Borrelia burgdorferi DNA by polymerase chain reaction in synovial fluid from patients with Lyme arthritis. N Engl J Med. 1994;330(4):229-234. doi:10.1056/NEJM199401273300401

12. Steere AC. Posttreatment Lyme disease syndromes: distinct pathogenesis caused by maladaptive host responses. J Clin Invest. 2020;130(5):2148-2151. doi:10.1172/JCI138062

Article PDF
Author and Disclosure Information

Lauren E. Merz, MD, MSca,b; Shivani Jindal, MD, MPHb,c; Talya Shahal, MDb,d; Richard Serrao, MDb,c; and Anthony C. Breu, MDb,d
Correspondence: Anthony Breu (anthony.breu@va.gov)

a Brigham and Women’s Hospital, Boston, Massachusetts
b Veterans Affairs Boston Healthcare System, West Roxbury, Massachusetts
c Boston University School of Medicine, Massachusetts
d Harvard Medical School, Boston, Massachusetts

Author disclosures

The authors report no actual or potential conflicts of interest or outside sources of funding with regard to this article.

Disclaimer

The opinions expressed herein are those of the authors and do not necessarily reflect those of Federal Practitioner , Frontline Medical Communications Inc., the US Government, or any of its agencies. This article may discuss unlabeled or investigational use of certain drugs. Please review the complete prescribing information for specific drugs or drug combinations—including indications, contraindications, warnings, and adverse effects—before administering pharmacologic therapy to patients.

Ethics and consent

Informed consent was obtained from the patient reported in this case report.

Issue
Federal Practitioner - 39(7)a
Publications
Topics
Page Number
315-319
Sections
Author and Disclosure Information

Lauren E. Merz, MD, MSca,b; Shivani Jindal, MD, MPHb,c; Talya Shahal, MDb,d; Richard Serrao, MDb,c; and Anthony C. Breu, MDb,d
Correspondence: Anthony Breu (anthony.breu@va.gov)

a Brigham and Women’s Hospital, Boston, Massachusetts
b Veterans Affairs Boston Healthcare System, West Roxbury, Massachusetts
c Boston University School of Medicine, Massachusetts
d Harvard Medical School, Boston, Massachusetts

Author disclosures

The authors report no actual or potential conflicts of interest or outside sources of funding with regard to this article.

Disclaimer

The opinions expressed herein are those of the authors and do not necessarily reflect those of Federal Practitioner , Frontline Medical Communications Inc., the US Government, or any of its agencies. This article may discuss unlabeled or investigational use of certain drugs. Please review the complete prescribing information for specific drugs or drug combinations—including indications, contraindications, warnings, and adverse effects—before administering pharmacologic therapy to patients.

Ethics and consent

Informed consent was obtained from the patient reported in this case report.

Author and Disclosure Information

Lauren E. Merz, MD, MSca,b; Shivani Jindal, MD, MPHb,c; Talya Shahal, MDb,d; Richard Serrao, MDb,c; and Anthony C. Breu, MDb,d
Correspondence: Anthony Breu (anthony.breu@va.gov)

a Brigham and Women’s Hospital, Boston, Massachusetts
b Veterans Affairs Boston Healthcare System, West Roxbury, Massachusetts
c Boston University School of Medicine, Massachusetts
d Harvard Medical School, Boston, Massachusetts

Author disclosures

The authors report no actual or potential conflicts of interest or outside sources of funding with regard to this article.

Disclaimer

The opinions expressed herein are those of the authors and do not necessarily reflect those of Federal Practitioner , Frontline Medical Communications Inc., the US Government, or any of its agencies. This article may discuss unlabeled or investigational use of certain drugs. Please review the complete prescribing information for specific drugs or drug combinations—including indications, contraindications, warnings, and adverse effects—before administering pharmacologic therapy to patients.

Ethics and consent

Informed consent was obtained from the patient reported in this case report.

Article PDF
Article PDF

Case Presentation: A 39-year-old Air Force veteran was admitted to the US Department of Veterans Affairs Boston Healthcare System (VABHS) for evaluation of recurrent, painful right knee effusions. On presentation, his vital signs were stable, and the examination was significant for a right knee with a large effusion and tenderness to palpation without erythema or warmth. His white blood cell count was 12.0 cells/L with an erythrocyte sedimentation rate of 23 mm/h and C-reactive protein of 11.87 mg/L. He was in remission from alcohol use but had relapsed on alcohol in the past day to treat the pain. He had a history of IV drug use but was in remission. He was previously active and enjoyed long hikes. Nine months prior to presentation, he developed his first large right knee effusion associated with pain. He reported no antecedent trauma. At that time, he presented to another hospital and underwent arthrocentesis with orthopedic surgery, but this did not lead to a diagnosis, and the effusion reaccumulated within 24 hours. Four months later, he received a corticosteroid injection that provided only minor, temporary relief. He received 5 additional arthrocenteses over 9 months, all without definitive diagnosis and with rapid reaccumulation of the fluid. His most recent arthrocentesis was 3 weeks before admission.

►Lauren E. Merz, MD, MSc, Chief Medical Resident, VABHS: Dr. Jindal, what is your approach and differential diagnosis for joint effusions in hospitalized patients?

►Shivani Jindal, MD, MPH, Hospitalist, VABHS, Instructor in Medicine, Boston University School of Medicine (BUSM): A thorough history and physical examination are important. I specifically ask about chronicity, pain, and trauma. A medical history of potential infectious exposures and the history of the present illness are also important, such as the risk of sexually transmitted infections, exposure to Lyme disease or other viral illnesses. Gonococcal arthritis is one of the most common causes of nontraumatic monoarthritis in young adults but can also present as a migratory polyarthritis.1

It sounds like he was quite active and liked to hike so a history of tick exposure is important to ascertain. I would also ask about eye inflammation and back pain to assess possible ankylosing spondyarthritis. Other inflammatory etiologies, such as gout are common, but it would be surprising to miss this diagnosis on repeated arthocenteses. A physical examination can confirm monoarthritis over polyarthritis and assess for signs of inflammatory arthritis (eg, warmth and erythema). The most important etiology to assess for and rule out in a person admitted to the hospital is septic arthritis. The severe pain, mild leukocytosis, and mildly elevated inflammatory markers could be consistent with this diagnosis but are nonspecific. However, the chronicity of this patient’s presentation and hemodynamic stability make septic arthritis less likely overall and a more indolent infection or other inflammatory process more likely.

►Dr. Merz: The patient’s medical history included posttraumatic stress disorder (PTSD) and antisocial personality disorder with multiple prior suicide attempts. He also had a history of opioid use disorder (OUD) with prior overdose and alcohol use disorder (AUD). Given his stated preference to avoid opioids and normal liver function and liver chemistry testing, the initial treatment was with acetaminophen. After this failed to provide satisfactory pain control, IV hydromorphone was added.

 

 

Dr. Jindal, how do you approach pain control in the hospital for musculoskeletal issues like this?

►Dr. Jindal: Typically, nonsteroidal anti-inflammatory medications (NSAIDs) are most effective for musculoskeletal pain, often in the form of ketorolac or ibuprofen. However, we are often limited in our NSAID use by kidney disease, gastritis, or cardiovascular disease. Selective COX-2 inhibitors (eg, celecoxib) have the advantage of a lower risk of gastrointestinal bleeding. Topical formulations (eg, diclofenac) may also come with a lower adverse effect profile. Corticosteroids are also an option but come with their own adverse effect profile. This patient does not have any of these comorbidities. Adjuvant therapies such as lidocaine patches or capsaicin cream can also provide relief. Gabapentin or pregabalin are indicated for any component of neuropathic pain. Opioids can be helpful for acute musculoskeletal pain, but there is no long-term benefit in chronic musculoskeletal pain.2 The experience of pain is also multifactorial so ensuring that anxiety and insomnia are addressed is key.

►Dr. Merz: On hospital day 1, the patient asked to leave to consume alcohol to ease unremitting pain. He also expressed suicidal ideation and discharge was therefore felt to be unsafe. He was reluctant to engage with psychiatry and became physically combative while attempting to leave the hospital, necessitating the use of sedating medications and physical restraints.

Dr. Shahal, what factors led to the decision to place an involuntary hold, and how do you balance patient autonomy and patient safety?

►Dr. Talya Shahal, MD, Consult-Liaison Psychiatry Service, VABHS, Instructor in Psychiatry, Harvard Medical School: This is a delicate balance that requires constant reassessment. The patient initially presented to the emergency department with suicidal ideation, stating he was not able to tolerate the pain and thus resumed alcohol consumption after a period of nonuse. He had multiple risk factors for suicide, including 9 prior suicide attempts with the latest less than a year before presentation, active substance use with alcohol and other recreational drugs, PTSD, pain, veteran status, male sex, single status, and a history of trauma.3,4 He was also displaying impulsivity and limited insight, did not engage in his psychiatric assessment, and attempted to assault staff. As such, his suicide risk was assessed to be high at the time of the evaluation, which led to the decision to place an involuntary hold. However, we reevaluate this decision at least daily in order to reassess the risk and ensure that the balance between patient safety and autonomy are maintained.

►Dr. Merz: The involuntary hold was removed within 48 hours as the patient remained calm and engaged with the primary and consulting teams. He requested escalating doses of opioids as he felt the short-acting IV medications were not providing sustained relief. However, he was also noted to be walking outside of the hospital without assistance, and he repeatedly declined nonopioid pain modalities as well as buprenorphine/naloxone. The chronic pain service was consulted but was unable to see the patient as he was frequently outside of his room.

 

 

Dr. Shahal, how do you address OUD, pain, and stigma in the hospital?

►Dr. Shahal: It is important to remember that patients with substance use disorder (SUD) and mental illness frequently have physical causes for pain and are often undertreated.5 Patients with SUD may also have higher tolerance for opioids and may need higher doses to treat the pain.5 Modalities like buprenorphine/naloxone can be effective to treat OUD and pain, but these usually cannot be initiated while the patient is on short-acting opioids as this would precipitate withdrawal.6 However, withdrawal can be managed while inpatient, and this can be a good time to start these medications as practitioners can aggressively help with symptom control. Proactively addressing mental health concerns, particularly anxiety, AUD, insomnia, PTSD, and depression, can also have a direct impact on the perception of pain and assist with better control.2 In addition, nonpharmacologic options, such as meditation, deep breathing, and even acupuncture and Reiki can be helpful and of course less harmful to treat pain.2

► Dr. Merz: An X-ray of the knee showed no acute fracture or joint space narrowing. Magnetic resonance imaging confirmed a large knee effusion with no evidence of ligament injury. Synovial fluid showed turbid, yellow fluid with 14,110 nucleated cells (84% segmented cells and 4000 RBCs). Gram stain was negative, culture had no growth, and there were no crystals. Anticyclic citrullinated peptide (anti-CCP), rheumatoid factor, HIV testing, and HLA-B27 were negative.

Dr. Serrao, what do these studies tell us about the joint effusion and the possible diagnoses?

► Dr. Richard Serrao, MD, Infectious Disease, VABHS, Clinical Associate Professor in Medicine, BUSM: I would expect the white blood cell (WBC) count to be > 50,000 cells with > 75% polymorphonuclear cells and a positive Gram stain if this was a bacterial infection resulting in septic arthritis.7 This patient’s studies are not consistent with this diagnosis nor is the chronicity of his presentation. There are 2 important bacteria that can present with inflammatory arthritis and less pronounced findings on arthrocentesis: Borrelia burgdorferi (the bacteria causing Lyme arthritis) and Neisseria gonorrhea. Lyme arthritis could be consistent with this relapsing remitting presentation as you expect a WBC count between 3000 and 100,000 cells with a mean value between 10,000 and 25,000 cells, > 50% polymorphonuclear leukocytes, and negative Gram stains.8 Gonococcal infections often do not have marked elevations in the WBC count and the Gram stain can be variable, but you still expect the WBC count to be > 30,000 cells.7 Inflammatory causes such as gout or autoimmune conditions such as lupus often have a WBC count between 2000 and 100,000 with a negative Gram stain, which could be consistent with this patient’s presentation.7 However, the lack of crystals rules out gout and the negative anti-CCP, rheumatoid factor, and HLA-B27 make rheumatologic diseases less likely.

►Dr. Merz: The patient received a phone call from another hospital where an arthrocentesis had been performed 3 weeks before. The results included a positive polymerase chain reaction (PCR) test for Lyme disease in the synovial fluid. A subsequent serum Lyme screen was positive for 1 of 3 immunoglobulin (Ig) M bands and 10 of 10 IgG bands.

 

 

Dr. Serrao, how does Lyme arthritis typically present, and are there aspects of this case that make you suspect the diagnosis? Does the serum Lyme test give us any additional information?

►Dr. Serrao: Lyme arthritis is a late manifestation of Lyme disease. Patients typically have persistent or intermittent arthritis, and large joints are more commonly impacted than small joints. Monoarthritis of the knee is the most common, but oligoarthritis is possible as well. The swelling usually begins abruptly, lasts for weeks to months, and effusions typically recur quickly after aspiration. These findings are consistent with the patient’s clinical history.

For diagnostics, the IgG Western blot is positive if 5 of the 10 bands are positive.9 This patient far exceeds the IgG band number to diagnose Lyme disease. All patients with Lyme arthritis will have positive IgG serologies since Lyme arthritis is a late manifestation of the infection. IgM reactivity may be present, but are not necessary to diagnose Lyme arthritis.10 Synovial fluid is often not analyzed for antibody responses as they are susceptible to false positive results, but synovial PCR testing like this patient had detects approximately 70% of patients with untreated Lyme arthritis.11 However, PCR positivity does not necessarily equate with active infection. Serologic testing for Lyme disease by enzyme-linked immunosorbent assay and Western blot as well as careful history and the exclusion of other diagnoses are usually sufficient to make the diagnosis.

► Dr. Merz: On further history the patient reported that 5 years prior he found a tick on his skin with a bull’s-eye rash. He was treated with 28 days of doxycycline at that time. He did not recall any tick bites or rashes in the years since.

Dr. Serrao, is it surprising that he developed Lyme arthritis 5 years after exposure and after being treated appropriately? What is the typical treatment approach for a patient like this?

►Dr. Serrao: It is atypical to develop Lyme arthritis 5 years after reported treatment of what appeared to be early localized disease, namely, erythema migrans. This stage is usually cured with 10 days of treatment alone (he received 28 days) and is generally abortive of subsequent stages, including Lyme arthritis. Furthermore, the patient reported no symptoms of arthritis until recently since that time. Therefore, one can argue that the excessively long span of time from treatment to these first episodes of arthritis suggests the patient could have been reinfected. When available, comparing the types and number of Western blot bands (eg, new and/or more bands on subsequent serologic testing) can support a reinfection diagnosis. A delayed postinfectious inflammatory process from excessive proinflammatory immune responses that block wound repair resulting in proliferative synovitis is also possible.12 This is defined as the postinfectious, postantibiotic, or antibiotic-refractory Lyme arthritis, a diagnosis of exclusion more apparent only after patients receive appropriate antibiotic courses for the possibility of untreated Lyme as an active infection.12

Given the inherent diagnostic uncertainty between an active infection and posttreatment Lyme arthritis syndromes, it is best to approach most cases of Lyme arthritis as an active infection first especially if not yet treated with antibiotics. Diagnosis of postinflammatory processes should be considered if symptoms persist after appropriate antibiotics, and then short-term use of disease-modifying antirheumatic drugs, rather than further courses of antibiotics, is recommended.

 

 

► Dr. Merz: The patient was initiated on doxycycline with the plan to transition to ceftriaxone if there was no response. One day after diagnosis and treatment initiation and in the setting of continued pain, the patient again asked to leave the hospital to drink alcohol. After eloping and becoming intoxicated with alcohol, he returned to his room. He remained concerned about his continued pain and lack of adequate pain control. At the time, he was receiving hydromorphone, ketorolac, lorazepam, gabapentin, and quetiapine.

Dr. Serrao, do you expect this degree of pain from Lyme arthritis?

► Dr. Serrao: Lyme arthritis is typically less painful than other forms of infectious or inflammatory arthritis. Pain is usually caused by the pressure from the acute accumulation and reaccumulation of fluid. In this case, the rapid accumulation of fluid that this patient experienced as well as relief with arthrocentesis suggests that the size and acuity of the effusion was causing great discomfort. Repeated arthrocentesis can prove to be a preventative strategy to minimize synovial herniation.

►Dr. Merz: Dr. Shahal, how do you balance the patient subjectively telling you that they are in pain with objective signs that they may be tolerating the pain like walking around unassisted? Is there anything else that could have been done to prevent this adverse outcome?

►Dr. Shahal: This is one of the hardest pieces of pain management. We want to practice beneficence by believing our patients and addressing their discomfort, but we also want to practice nonmaleficence by avoiding inappropriate long-term pain treatments like opioids that have significant harm as well as avoiding exacerbating this patient’s underlying SUD. An agent like buprenorphine/naloxone could have been an excellent fit to treat pain and SUD, but the patient’s lack of interest and the frequent use of short-acting opioids were major barriers. A chronic pain consult early on is helpful in cases like this as well, but they were unable to see him since he was often out of his room. Repeated arthrocentesis may also have helped the pain. Treatment of anxiety and insomnia with medications like hydroxyzine, trazodone, melatonin, gabapentin, or buspirone as well as interventions like sleep hygiene protocols or spiritual care may have helped somewhat as well.

We know that there is a vicious cycle between pain and poorly controlled mood symptoms. Many of our veterans have PTSD, anxiety, and SUD that are exacerbated by hospitalization and pain. Maintaining optimal communication between the patient and the practitioners, using trauma-informed care, understanding the patient’s goals of care, setting expectations and limits, and attempting to address the patient’s needs while attempting to minimize stigma might be helpful. However, despite optimal care, sometimes these events cannot be avoided.

►Dr. Merz: The patient was ultimately transferred to an inpatient psychiatric unit where a taper plan for the short-acting opioids was implemented. He was psychiatrically stabilized and discharged a few days later off opioids and on doxycycline. On follow-up a few weeks later, his pain had markedly improved, and the effusion was significantly reduced in size. His mood and impulsivity had stabilized. He continues to follow-up in the infectious disease clinic.

 

fdp03907315_takeaways.png

Case Presentation: A 39-year-old Air Force veteran was admitted to the US Department of Veterans Affairs Boston Healthcare System (VABHS) for evaluation of recurrent, painful right knee effusions. On presentation, his vital signs were stable, and the examination was significant for a right knee with a large effusion and tenderness to palpation without erythema or warmth. His white blood cell count was 12.0 cells/L with an erythrocyte sedimentation rate of 23 mm/h and C-reactive protein of 11.87 mg/L. He was in remission from alcohol use but had relapsed on alcohol in the past day to treat the pain. He had a history of IV drug use but was in remission. He was previously active and enjoyed long hikes. Nine months prior to presentation, he developed his first large right knee effusion associated with pain. He reported no antecedent trauma. At that time, he presented to another hospital and underwent arthrocentesis with orthopedic surgery, but this did not lead to a diagnosis, and the effusion reaccumulated within 24 hours. Four months later, he received a corticosteroid injection that provided only minor, temporary relief. He received 5 additional arthrocenteses over 9 months, all without definitive diagnosis and with rapid reaccumulation of the fluid. His most recent arthrocentesis was 3 weeks before admission.

►Lauren E. Merz, MD, MSc, Chief Medical Resident, VABHS: Dr. Jindal, what is your approach and differential diagnosis for joint effusions in hospitalized patients?

►Shivani Jindal, MD, MPH, Hospitalist, VABHS, Instructor in Medicine, Boston University School of Medicine (BUSM): A thorough history and physical examination are important. I specifically ask about chronicity, pain, and trauma. A medical history of potential infectious exposures and the history of the present illness are also important, such as the risk of sexually transmitted infections, exposure to Lyme disease or other viral illnesses. Gonococcal arthritis is one of the most common causes of nontraumatic monoarthritis in young adults but can also present as a migratory polyarthritis.1

It sounds like he was quite active and liked to hike so a history of tick exposure is important to ascertain. I would also ask about eye inflammation and back pain to assess possible ankylosing spondyarthritis. Other inflammatory etiologies, such as gout are common, but it would be surprising to miss this diagnosis on repeated arthocenteses. A physical examination can confirm monoarthritis over polyarthritis and assess for signs of inflammatory arthritis (eg, warmth and erythema). The most important etiology to assess for and rule out in a person admitted to the hospital is septic arthritis. The severe pain, mild leukocytosis, and mildly elevated inflammatory markers could be consistent with this diagnosis but are nonspecific. However, the chronicity of this patient’s presentation and hemodynamic stability make septic arthritis less likely overall and a more indolent infection or other inflammatory process more likely.

►Dr. Merz: The patient’s medical history included posttraumatic stress disorder (PTSD) and antisocial personality disorder with multiple prior suicide attempts. He also had a history of opioid use disorder (OUD) with prior overdose and alcohol use disorder (AUD). Given his stated preference to avoid opioids and normal liver function and liver chemistry testing, the initial treatment was with acetaminophen. After this failed to provide satisfactory pain control, IV hydromorphone was added.

 

 

Dr. Jindal, how do you approach pain control in the hospital for musculoskeletal issues like this?

►Dr. Jindal: Typically, nonsteroidal anti-inflammatory medications (NSAIDs) are most effective for musculoskeletal pain, often in the form of ketorolac or ibuprofen. However, we are often limited in our NSAID use by kidney disease, gastritis, or cardiovascular disease. Selective COX-2 inhibitors (eg, celecoxib) have the advantage of a lower risk of gastrointestinal bleeding. Topical formulations (eg, diclofenac) may also come with a lower adverse effect profile. Corticosteroids are also an option but come with their own adverse effect profile. This patient does not have any of these comorbidities. Adjuvant therapies such as lidocaine patches or capsaicin cream can also provide relief. Gabapentin or pregabalin are indicated for any component of neuropathic pain. Opioids can be helpful for acute musculoskeletal pain, but there is no long-term benefit in chronic musculoskeletal pain.2 The experience of pain is also multifactorial so ensuring that anxiety and insomnia are addressed is key.

►Dr. Merz: On hospital day 1, the patient asked to leave to consume alcohol to ease unremitting pain. He also expressed suicidal ideation and discharge was therefore felt to be unsafe. He was reluctant to engage with psychiatry and became physically combative while attempting to leave the hospital, necessitating the use of sedating medications and physical restraints.

Dr. Shahal, what factors led to the decision to place an involuntary hold, and how do you balance patient autonomy and patient safety?

►Dr. Talya Shahal, MD, Consult-Liaison Psychiatry Service, VABHS, Instructor in Psychiatry, Harvard Medical School: This is a delicate balance that requires constant reassessment. The patient initially presented to the emergency department with suicidal ideation, stating he was not able to tolerate the pain and thus resumed alcohol consumption after a period of nonuse. He had multiple risk factors for suicide, including 9 prior suicide attempts with the latest less than a year before presentation, active substance use with alcohol and other recreational drugs, PTSD, pain, veteran status, male sex, single status, and a history of trauma.3,4 He was also displaying impulsivity and limited insight, did not engage in his psychiatric assessment, and attempted to assault staff. As such, his suicide risk was assessed to be high at the time of the evaluation, which led to the decision to place an involuntary hold. However, we reevaluate this decision at least daily in order to reassess the risk and ensure that the balance between patient safety and autonomy are maintained.

►Dr. Merz: The involuntary hold was removed within 48 hours as the patient remained calm and engaged with the primary and consulting teams. He requested escalating doses of opioids as he felt the short-acting IV medications were not providing sustained relief. However, he was also noted to be walking outside of the hospital without assistance, and he repeatedly declined nonopioid pain modalities as well as buprenorphine/naloxone. The chronic pain service was consulted but was unable to see the patient as he was frequently outside of his room.

 

 

Dr. Shahal, how do you address OUD, pain, and stigma in the hospital?

►Dr. Shahal: It is important to remember that patients with substance use disorder (SUD) and mental illness frequently have physical causes for pain and are often undertreated.5 Patients with SUD may also have higher tolerance for opioids and may need higher doses to treat the pain.5 Modalities like buprenorphine/naloxone can be effective to treat OUD and pain, but these usually cannot be initiated while the patient is on short-acting opioids as this would precipitate withdrawal.6 However, withdrawal can be managed while inpatient, and this can be a good time to start these medications as practitioners can aggressively help with symptom control. Proactively addressing mental health concerns, particularly anxiety, AUD, insomnia, PTSD, and depression, can also have a direct impact on the perception of pain and assist with better control.2 In addition, nonpharmacologic options, such as meditation, deep breathing, and even acupuncture and Reiki can be helpful and of course less harmful to treat pain.2

► Dr. Merz: An X-ray of the knee showed no acute fracture or joint space narrowing. Magnetic resonance imaging confirmed a large knee effusion with no evidence of ligament injury. Synovial fluid showed turbid, yellow fluid with 14,110 nucleated cells (84% segmented cells and 4000 RBCs). Gram stain was negative, culture had no growth, and there were no crystals. Anticyclic citrullinated peptide (anti-CCP), rheumatoid factor, HIV testing, and HLA-B27 were negative.

Dr. Serrao, what do these studies tell us about the joint effusion and the possible diagnoses?

► Dr. Richard Serrao, MD, Infectious Disease, VABHS, Clinical Associate Professor in Medicine, BUSM: I would expect the white blood cell (WBC) count to be > 50,000 cells with > 75% polymorphonuclear cells and a positive Gram stain if this was a bacterial infection resulting in septic arthritis.7 This patient’s studies are not consistent with this diagnosis nor is the chronicity of his presentation. There are 2 important bacteria that can present with inflammatory arthritis and less pronounced findings on arthrocentesis: Borrelia burgdorferi (the bacteria causing Lyme arthritis) and Neisseria gonorrhea. Lyme arthritis could be consistent with this relapsing remitting presentation as you expect a WBC count between 3000 and 100,000 cells with a mean value between 10,000 and 25,000 cells, > 50% polymorphonuclear leukocytes, and negative Gram stains.8 Gonococcal infections often do not have marked elevations in the WBC count and the Gram stain can be variable, but you still expect the WBC count to be > 30,000 cells.7 Inflammatory causes such as gout or autoimmune conditions such as lupus often have a WBC count between 2000 and 100,000 with a negative Gram stain, which could be consistent with this patient’s presentation.7 However, the lack of crystals rules out gout and the negative anti-CCP, rheumatoid factor, and HLA-B27 make rheumatologic diseases less likely.

►Dr. Merz: The patient received a phone call from another hospital where an arthrocentesis had been performed 3 weeks before. The results included a positive polymerase chain reaction (PCR) test for Lyme disease in the synovial fluid. A subsequent serum Lyme screen was positive for 1 of 3 immunoglobulin (Ig) M bands and 10 of 10 IgG bands.

 

 

Dr. Serrao, how does Lyme arthritis typically present, and are there aspects of this case that make you suspect the diagnosis? Does the serum Lyme test give us any additional information?

►Dr. Serrao: Lyme arthritis is a late manifestation of Lyme disease. Patients typically have persistent or intermittent arthritis, and large joints are more commonly impacted than small joints. Monoarthritis of the knee is the most common, but oligoarthritis is possible as well. The swelling usually begins abruptly, lasts for weeks to months, and effusions typically recur quickly after aspiration. These findings are consistent with the patient’s clinical history.

For diagnostics, the IgG Western blot is positive if 5 of the 10 bands are positive.9 This patient far exceeds the IgG band number to diagnose Lyme disease. All patients with Lyme arthritis will have positive IgG serologies since Lyme arthritis is a late manifestation of the infection. IgM reactivity may be present, but are not necessary to diagnose Lyme arthritis.10 Synovial fluid is often not analyzed for antibody responses as they are susceptible to false positive results, but synovial PCR testing like this patient had detects approximately 70% of patients with untreated Lyme arthritis.11 However, PCR positivity does not necessarily equate with active infection. Serologic testing for Lyme disease by enzyme-linked immunosorbent assay and Western blot as well as careful history and the exclusion of other diagnoses are usually sufficient to make the diagnosis.

► Dr. Merz: On further history the patient reported that 5 years prior he found a tick on his skin with a bull’s-eye rash. He was treated with 28 days of doxycycline at that time. He did not recall any tick bites or rashes in the years since.

Dr. Serrao, is it surprising that he developed Lyme arthritis 5 years after exposure and after being treated appropriately? What is the typical treatment approach for a patient like this?

►Dr. Serrao: It is atypical to develop Lyme arthritis 5 years after reported treatment of what appeared to be early localized disease, namely, erythema migrans. This stage is usually cured with 10 days of treatment alone (he received 28 days) and is generally abortive of subsequent stages, including Lyme arthritis. Furthermore, the patient reported no symptoms of arthritis until recently since that time. Therefore, one can argue that the excessively long span of time from treatment to these first episodes of arthritis suggests the patient could have been reinfected. When available, comparing the types and number of Western blot bands (eg, new and/or more bands on subsequent serologic testing) can support a reinfection diagnosis. A delayed postinfectious inflammatory process from excessive proinflammatory immune responses that block wound repair resulting in proliferative synovitis is also possible.12 This is defined as the postinfectious, postantibiotic, or antibiotic-refractory Lyme arthritis, a diagnosis of exclusion more apparent only after patients receive appropriate antibiotic courses for the possibility of untreated Lyme as an active infection.12

Given the inherent diagnostic uncertainty between an active infection and posttreatment Lyme arthritis syndromes, it is best to approach most cases of Lyme arthritis as an active infection first especially if not yet treated with antibiotics. Diagnosis of postinflammatory processes should be considered if symptoms persist after appropriate antibiotics, and then short-term use of disease-modifying antirheumatic drugs, rather than further courses of antibiotics, is recommended.

 

 

► Dr. Merz: The patient was initiated on doxycycline with the plan to transition to ceftriaxone if there was no response. One day after diagnosis and treatment initiation and in the setting of continued pain, the patient again asked to leave the hospital to drink alcohol. After eloping and becoming intoxicated with alcohol, he returned to his room. He remained concerned about his continued pain and lack of adequate pain control. At the time, he was receiving hydromorphone, ketorolac, lorazepam, gabapentin, and quetiapine.

Dr. Serrao, do you expect this degree of pain from Lyme arthritis?

► Dr. Serrao: Lyme arthritis is typically less painful than other forms of infectious or inflammatory arthritis. Pain is usually caused by the pressure from the acute accumulation and reaccumulation of fluid. In this case, the rapid accumulation of fluid that this patient experienced as well as relief with arthrocentesis suggests that the size and acuity of the effusion was causing great discomfort. Repeated arthrocentesis can prove to be a preventative strategy to minimize synovial herniation.

►Dr. Merz: Dr. Shahal, how do you balance the patient subjectively telling you that they are in pain with objective signs that they may be tolerating the pain like walking around unassisted? Is there anything else that could have been done to prevent this adverse outcome?

►Dr. Shahal: This is one of the hardest pieces of pain management. We want to practice beneficence by believing our patients and addressing their discomfort, but we also want to practice nonmaleficence by avoiding inappropriate long-term pain treatments like opioids that have significant harm as well as avoiding exacerbating this patient’s underlying SUD. An agent like buprenorphine/naloxone could have been an excellent fit to treat pain and SUD, but the patient’s lack of interest and the frequent use of short-acting opioids were major barriers. A chronic pain consult early on is helpful in cases like this as well, but they were unable to see him since he was often out of his room. Repeated arthrocentesis may also have helped the pain. Treatment of anxiety and insomnia with medications like hydroxyzine, trazodone, melatonin, gabapentin, or buspirone as well as interventions like sleep hygiene protocols or spiritual care may have helped somewhat as well.

We know that there is a vicious cycle between pain and poorly controlled mood symptoms. Many of our veterans have PTSD, anxiety, and SUD that are exacerbated by hospitalization and pain. Maintaining optimal communication between the patient and the practitioners, using trauma-informed care, understanding the patient’s goals of care, setting expectations and limits, and attempting to address the patient’s needs while attempting to minimize stigma might be helpful. However, despite optimal care, sometimes these events cannot be avoided.

►Dr. Merz: The patient was ultimately transferred to an inpatient psychiatric unit where a taper plan for the short-acting opioids was implemented. He was psychiatrically stabilized and discharged a few days later off opioids and on doxycycline. On follow-up a few weeks later, his pain had markedly improved, and the effusion was significantly reduced in size. His mood and impulsivity had stabilized. He continues to follow-up in the infectious disease clinic.

 

fdp03907315_takeaways.png

References

1. Siva C, Velazquez C, Mody A, Brasington R. Diagnosing acute monoarthritis in adults: a practical approach for the family physician. Am Fam Physician. 2003;68(1):83-90.

2. Qaseem A, McLean RM, O’Gurek D, et al. Nonpharmacologic and pharmacologic management of acute pain from non-low back, musculoskeletal injuries in adults: a clinical guideline from the American College of Physicians and American Academy of Family Physicians. Ann Intern Med. 2020;173(9):739-748. doi:10.7326/M19-3602

3. Silverman MM, Berman AL. Suicide risk assessment and risk formulation part I: a focus on suicide ideation in assessing suicide risk. Suicide Life Threat Behav. 2014;44(4):420-431. doi:10.1111/sltb.12065

4. Berman AL, Silverman MM. Suicide risk assessment and risk formulation part II: Suicide risk formulation and the determination of levels of risk. Suicide Life Threat Behav. 2014;44(4):432-443. doi:10.1111/sltb.12067

5. Quinlan J, Cox F. Acute pain management in patients with drug dependence syndrome. Pain Rep. 2017;2(4):e611. Published 2017 Jul 27. doi:10.1097/PR9.0000000000000611

6. Chou R, Wagner J, Ahmed AY, et al. Treatments for Acute Pain: A Systematic Review. Agency for Healthcare Research and Quality; 2020. https://www.ncbi.nlm.nih.gov/books/NBK566506/

7. Seidman AJ, Limaiem F. Synovial fluid analysis. In: StatPearls [Internet]. Treasure Island (FL): StatPearls Publishing; 2022. Updated May 8, 2022. https://www.ncbi.nlm.nih.gov/books/NBK537114

8. Arvikar SL, Steere AC. Diagnosis and treatment of Lyme arthritis. Infect Dis Clin North Am. 2015;29(2):269-280. doi:10.1016/j.idc.2015.02.004

9. Centers for Disease Control and Prevention. Recommendations for test performance and interpretation from the Second National Conference on Serologic Diagnosis of Lyme Disease. JAMA. 1995;274(12):937.

10. Craft JE, Grodzicki RL, Steere AC. Antibody response in Lyme disease: evaluation of diagnostic tests. J Infect Dis. 1984;149(5):789-795. doi:10.1093/infdis/149.5.789

11. Nocton JJ, Dressler F, Rutledge BJ, Rys PN, Persing DH, Steere AC. Detection of Borrelia burgdorferi DNA by polymerase chain reaction in synovial fluid from patients with Lyme arthritis. N Engl J Med. 1994;330(4):229-234. doi:10.1056/NEJM199401273300401

12. Steere AC. Posttreatment Lyme disease syndromes: distinct pathogenesis caused by maladaptive host responses. J Clin Invest. 2020;130(5):2148-2151. doi:10.1172/JCI138062

References

1. Siva C, Velazquez C, Mody A, Brasington R. Diagnosing acute monoarthritis in adults: a practical approach for the family physician. Am Fam Physician. 2003;68(1):83-90.

2. Qaseem A, McLean RM, O’Gurek D, et al. Nonpharmacologic and pharmacologic management of acute pain from non-low back, musculoskeletal injuries in adults: a clinical guideline from the American College of Physicians and American Academy of Family Physicians. Ann Intern Med. 2020;173(9):739-748. doi:10.7326/M19-3602

3. Silverman MM, Berman AL. Suicide risk assessment and risk formulation part I: a focus on suicide ideation in assessing suicide risk. Suicide Life Threat Behav. 2014;44(4):420-431. doi:10.1111/sltb.12065

4. Berman AL, Silverman MM. Suicide risk assessment and risk formulation part II: Suicide risk formulation and the determination of levels of risk. Suicide Life Threat Behav. 2014;44(4):432-443. doi:10.1111/sltb.12067

5. Quinlan J, Cox F. Acute pain management in patients with drug dependence syndrome. Pain Rep. 2017;2(4):e611. Published 2017 Jul 27. doi:10.1097/PR9.0000000000000611

6. Chou R, Wagner J, Ahmed AY, et al. Treatments for Acute Pain: A Systematic Review. Agency for Healthcare Research and Quality; 2020. https://www.ncbi.nlm.nih.gov/books/NBK566506/

7. Seidman AJ, Limaiem F. Synovial fluid analysis. In: StatPearls [Internet]. Treasure Island (FL): StatPearls Publishing; 2022. Updated May 8, 2022. https://www.ncbi.nlm.nih.gov/books/NBK537114

8. Arvikar SL, Steere AC. Diagnosis and treatment of Lyme arthritis. Infect Dis Clin North Am. 2015;29(2):269-280. doi:10.1016/j.idc.2015.02.004

9. Centers for Disease Control and Prevention. Recommendations for test performance and interpretation from the Second National Conference on Serologic Diagnosis of Lyme Disease. JAMA. 1995;274(12):937.

10. Craft JE, Grodzicki RL, Steere AC. Antibody response in Lyme disease: evaluation of diagnostic tests. J Infect Dis. 1984;149(5):789-795. doi:10.1093/infdis/149.5.789

11. Nocton JJ, Dressler F, Rutledge BJ, Rys PN, Persing DH, Steere AC. Detection of Borrelia burgdorferi DNA by polymerase chain reaction in synovial fluid from patients with Lyme arthritis. N Engl J Med. 1994;330(4):229-234. doi:10.1056/NEJM199401273300401

12. Steere AC. Posttreatment Lyme disease syndromes: distinct pathogenesis caused by maladaptive host responses. J Clin Invest. 2020;130(5):2148-2151. doi:10.1172/JCI138062

Issue
Federal Practitioner - 39(7)a
Issue
Federal Practitioner - 39(7)a
Page Number
315-319
Page Number
315-319
Publications
Publications
Topics
Article Type
Sections
Teambase XML
<?xml version="1.0" encoding="UTF-8"?>
<!--$RCSfile: InCopy_agile.xsl,v $ $Revision: 1.35 $-->
<!--$RCSfile: drupal.xsl,v $ $Revision: 1.7 $-->
<root generator="drupal.xsl" gversion="1.7"> <header> <fileName>0722 FED Forum</fileName> <TBEID>0C029DEF.SIG</TBEID> <TBUniqueIdentifier>NJ_0C029DEF</TBUniqueIdentifier> <newsOrJournal>Journal</newsOrJournal> <publisherName>Frontline Medical Communications Inc.</publisherName> <storyname/> <articleType>1</articleType> <TBLocation>Copyfitting-FED</TBLocation> <QCDate/> <firstPublished>20220701T135510</firstPublished> <LastPublished>20220701T135510</LastPublished> <pubStatus qcode="stat:"/> <embargoDate/> <killDate/> <CMSDate>20220701T135510</CMSDate> <articleSource/> <facebookInfo/> <meetingNumber/> <byline/> <bylineText>Lauren E. Merz, MD, MSca,b; Shivani Jindal, MD, MPHb,c; Talya Shahal, MDb,d; Richard Serrao, MDb,c; and Anthony C. Breu, MDb,d</bylineText> <bylineFull/> <bylineTitleText/> <USOrGlobal/> <wireDocType/> <newsDocType/> <journalDocType/> <linkLabel/> <pageRange/> <citation/> <quizID/> <indexIssueDate/> <itemClass qcode="ninat:text"/> <provider qcode="provider:"> <name/> <rightsInfo> <copyrightHolder> <name/> </copyrightHolder> <copyrightNotice/> </rightsInfo> </provider> <abstract/> <metaDescription>Case Presentation: A 39-year-old Air Force veteran was admitted to the US Department of Veterans Affairs Boston Healthcare System (VABHS) for evaluation of recu</metaDescription> <articlePDF/> <teaserImage/> <title>A Veteran With Recurrent, Painful Knee Effusion</title> <deck/> <eyebrow>VA Boston Medical Forum</eyebrow> <disclaimer/> <AuthorList/> <articleURL/> <doi/> <pubMedID/> <publishXMLStatus/> <publishXMLVersion>1</publishXMLVersion> <useEISSN>0</useEISSN> <urgency/> <pubPubdateYear/> <pubPubdateMonth/> <pubPubdateDay/> <pubVolume/> <pubNumber/> <wireChannels/> <primaryCMSID/> <CMSIDs> <CMSID>2847</CMSID> <CMSID>2885</CMSID> <CMSID>2901</CMSID> <CMSID>3731</CMSID> </CMSIDs> <keywords/> <seeAlsos/> <publications_g> <publicationData> <publicationCode>FED</publicationCode> <pubIssueName>July 2022</pubIssueName> <pubArticleType>Departments | 3731</pubArticleType> <pubTopics> <pubTopic>Allergy and Immunology | 2847</pubTopic> <pubTopic>Pain | 2885</pubTopic> <pubTopic>Substance Use Disorder | 2901</pubTopic> </pubTopics> <pubCategories/> <pubSections/> <journalTitle>Fed Pract</journalTitle> <journalFullTitle>Federal Practitioner</journalFullTitle> <copyrightStatement>Copyright 2017 Frontline Medical Communications Inc., Parsippany, NJ, USA. All rights reserved.</copyrightStatement> </publicationData> </publications_g> <publications> <term canonical="true">16</term> </publications> <sections> <term canonical="true">58388</term> </sections> <topics> <term canonical="true">268</term> <term>264</term> </topics> <links/> </header> <itemSet> <newsItem> <itemMeta> <itemRole>Main</itemRole> <itemClass>text</itemClass> <title>A Veteran With Recurrent, Painful Knee Effusion</title> <deck/> </itemMeta> <itemContent> <p><b>Case Presentation:</b><b> </b><i>A 39-year-old Air Force veteran was admitted to the US Department of Veterans Affairs Boston Healthcare System (VABHS) for evaluation of recurrent, painful right knee effusions. On presentation, his vital signs were stable, and the examination was significant for a right knee with a large effusion and tenderness to palpation without erythema or warmth. His white blood cell count was 12.0 cells/L with an erythrocyte sedimentation rate of 23 mm/h and C-reactive protein of 11.87 mg/L. He was in remission from alcohol use but had relapsed on alcohol in the past day to treat the pain. He had a history of IV drug use but was in remission. He was previously active and enjoyed long hikes. Nine months prior to presentation, he developed his first large right knee effusion associated with pain. He reported no antecedent trauma. At that time, he presented to another hospital and underwent arthrocentesis with orthopedic surgery, but this did not lead to a diagnosis, and the effusion reaccumulated within 24 hours. Four months later, he received a corticosteroid injection that provided only minor, temporary relief. He received 5 additional arthrocenteses over 9 months, all without definitive diagnosis and with rapid reaccumulation of the fluid. His most recent arthrocentesis was 3 weeks before admission</i><i>. <br/><br/></i><strong></strong><strong> </strong><strong>Lauren E. Merz, MD, MSc, Chief Medical Resident, VABHS:</strong><b> </b><i> </i>Dr. Jindal, what is your approach and differential diagnosis for joint effusions in hospitalized patients? <br/><br/><strong></strong><strong> </strong><strong>Shivani Jindal, MD, MPH, Hospitalist, VABHS, Instructor in Medicine, Boston University School of Medicine (BUSM):</strong><b><i> </i></b>A thorough history and physical examination are important. I specifically ask about chronicity, pain, and trauma. A medical history of potential infectious exposures and the history of the present illness are also important, such as the risk of sexually transmitted infections, exposure to Lyme disease or other viral illnesses. Gonococcal arthritis is one of the most common causes of nontraumatic monoarthritis in young adults but can also present as a migratory polyarthritis.<sup>1</sup> </p> <p>It sounds like he was quite active and liked to hike so a history of tick exposure is important to ascertain. I would also ask about eye inflammation and back pain to assess possible ankylosing spondyarthritis. Other inflammatory etiologies, such as gout are common, but it would be surprising to miss this diagnosis on repeated arthocenteses. A physical examination can confirm monoarthritis over polyarthritis and assess for signs of inflammatory arthritis (eg, warmth and erythema). The most important etiology to assess for and rule out in a person admitted to the hospital is septic arthritis. The severe pain, mild leukocytosis, and mildly elevated inflammatory markers could be consistent with this diagnosis but are nonspecific. However, the chronicity of this patient’s presentation and hemodynamic stability make septic arthritis less likely overall and a more indolent infection or other inflammatory process more likely. </p> <p><strong></strong><strong> </strong><strong>Dr. Merz:</strong><b> </b>The patient’s medical history included posttraumatic stress disorder (PTSD) and antisocial personality disorder with multiple prior suicide attempts. He also had a history of opioid use disorder (OUD) with prior overdose and alcohol use disorder (AUD). Given his stated preference to avoid opioids and normal liver function and liver chemistry testing, the initial treatment was with acetaminophen. After this failed to provide satisfactory pain control, IV hydromorphone was added. </p> <p>Dr. Jindal, how do you approach pain control in the hospital for musculoskeletal issues like this? </p> <p><strong></strong><strong> </strong><strong>Dr. Jindal:</strong><b> </b>Typically, nonsteroidal anti-inflammatory medications (NSAIDs) are most effective for musculoskeletal pain, often in the form of ketorolac or ibuprofen. However, we are often limited in our NSAID use by kidney disease, gastritis, or cardiovascular disease. <hl name="1"/>Selective COX-2 inhibitors (eg, celecoxib) have the advantage of a lower risk of gastrointestinal bleeding. Topical formulations (eg, diclofenac) may also come with a lower adverse effect profile. Corticosteroids are also an option but come with their own adverse effect profile. This patient does not have any of these comorbidities. Adjuvant therapies such as lidocaine patches or capsaicin cream can also provide relief. Gabapentin or pregabalin are indicated for any component of neuropathic pain. Opioids can be helpful for acute musculoskeletal pain, but there is no long-term benefit in chronic musculoskeletal pain.<sup>2</sup> The experience of pain is also multifactorial so ensuring that anxiety and insomnia are addressed is key. <br/><br/><strong></strong><strong> </strong><strong>Dr. Merz:</strong> On hospital day 1, the patient asked to leave to consume alcohol to ease unremitting pain. He also expressed suicidal ideation and discharge was therefore felt to be unsafe. He<b> </b>was reluctant to engage with psychiatry and became physically combative while attempting to leave the hospital, necessitating the use of sedating medications and physical restraints. </p> <p>Dr. Shahal, what factors led to the decision to place an involuntary hold, and how do you balance patient autonomy and patient safety? </p> <p><strong></strong><strong> </strong><strong>Dr. Talya Shahal, MD, Consult-Liaison Psychiatry Service, VABHS, Instructor in Psychiatry, Harvard Medical School:</strong><b> </b>This is a delicate balance that requires constant reassessment. The patient initially presented to the emergency department with suicidal ideation, stating he was not able to tolerate the pain and thus resumed alcohol consumption after a period of nonuse. He had multiple risk factors for suicide, including 9 prior suicide attempts with the latest less than a year before presentation, active substance use with alcohol and other recreational drugs, PTSD, pain, veteran status, male sex, single status, and a history of trauma.<sup>3,4</sup> He was also displaying impulsivity and limited insight, did not engage in his psychiatric assessment, and attempted to assault staff. As such, his suicide risk was assessed to be high at the time of the evaluation, which led to the decision to place an involuntary hold. However, we reevaluate this decision at least daily in order to reassess the risk and ensure that the balance between patient safety and autonomy are maintained. <br/><br/><strong></strong><strong> </strong><strong>Dr. Merz:</strong><b> </b>The involuntary hold was removed within 48 hours as the patient remained calm and engaged with the primary and consulting teams. He requested escalating doses of opioids as he felt the short-acting IV medications were not providing sustained relief. However, he was also noted to be walking outside of the hospital without assistance, and he repeatedly declined nonopioid pain modalities as well as buprenorphine/naloxone. The chronic pain service was consulted but was unable to see the patient as he was frequently outside of his room. </p> <p>Dr. Shahal, how do you address OUD, pain, and stigma in the hospital? </p> <p><strong></strong><strong> </strong><strong>Dr. Shahal</strong><b>: </b>It is important to remember that patients with substance use disorder (SUD) and mental illness frequently have physical causes for pain and are often undertreated.<sup>5</sup> Patients with SUD may also have higher tolerance for opioids and may need higher doses to treat the pain.<sup>5</sup> Modalities like buprenorphine/naloxone can be effective to treat OUD and pain, but these usually cannot be initiated while the patient is on short-acting opioids as this would precipitate withdrawal.<sup>6</sup> However, withdrawal can be managed while inpatient, and this can be a good time to start these medications as practitioners can aggressively help with symptom control. Proactively addressing mental health concerns, particularly anxiety, AUD, insomnia, PTSD, and depression, can also have a direct impact on the perception of pain and assist with better control.<sup>2</sup> In addition, nonpharmacologic options, such as meditation, deep breathing, and even acupuncture and Reiki can be helpful and of course less harmful to treat pain.<sup>2</sup> <br/><br/><strong></strong><strong> </strong><strong>Dr. Merz: </strong>An<b> </b>X-ray of the knee showed no acute fracture or joint space narrowing. Magnetic resonance imaging confirmed a large knee effusion with no evidence of ligament injury. Synovial fluid showed turbid, yellow fluid with 14,110 nucleated cells (84% segmented cells and 4000 RBCs). Gram stain was negative, culture had no growth, and there were no crystals. Anticyclic citrullinated peptide (anti-CCP), rheumatoid factor, HIV testing, and HLA-B27 were negative. </p> <p>Dr. Serrao, what do these studies tell us about the joint effusion and the possible diagnoses? </p> <p><strong></strong><strong> </strong><strong>Dr. Richard Serrao, MD, Infectious Disease, VABHS, Clinical Associate Professor in Medicine, BUSM: </strong>I would expect the white blood cell (WBC) count to be &gt; 50,000 cells with &gt; 75% polymorphonuclear cells and a positive Gram stain if this was a bacterial infection resulting in septic arthritis.<sup>7</sup> This patient’s studies are not consistent with this diagnosis nor is the chronicity of his presentation. There are 2 important bacteria that can present with inflammatory arthritis and less pronounced findings on arthrocentesis: <i>Borrelia burgdorferi </i>(the bacteria causing Lyme arthritis) and <i>Neisseria gonorrhea. </i>Lyme arthritis could be consistent with this relapsing remitting presentation as you expect a WBC count between 3000 and 100,000 cells with a mean value between 10,000 and 25,000 cells, &gt; 50% polymorphonuclear leukocytes, and negative Gram stains.<sup>8</sup> Gonococcal infections often do not have marked elevations in the WBC count and the Gram stain can be variable, but you still expect the WBC count to be &gt; 30,000 cells.<sup>7 </sup>Inflammatory causes such as gout or autoimmune conditions such as lupus often have a WBC count between 2000 and 100,000 with a negative Gram stain, which could be consistent with this patient’s presentation.<sup>7</sup> However, the lack of crystals rules out gout and the negative anti-CCP, rheumatoid factor, and HLA-B27 make rheumatologic diseases less likely. <br/><br/><strong></strong><strong> </strong><strong>Dr. Merz: </strong>The patient received a phone call from another hospital where an arthrocentesis had been performed 3 weeks before. The results included a positive polymerase chain reaction (PCR) test for Lyme disease in the synovial fluid. A subsequent serum Lyme screen was positive for 1 of 3 immunoglobulin (Ig) M bands and 10 of 10 IgG bands. </p> <p>Dr. Serrao, how does Lyme arthritis typically present, and are there aspects of this case that make you suspect the diagnosis? Does the serum Lyme test give us any additional information? </p> <p><strong></strong><strong> </strong><strong>Dr. Serrao:</strong><b> </b>Lyme arthritis is a late manifestation of Lyme disease. Patients typically have persistent or intermittent arthritis, and large joints are more commonly impacted than small joints. Monoarthritis of the knee is the most common, but oligoarthritis is possible as well. The swelling usually begins abruptly, lasts for weeks to months, and effusions typically recur quickly after aspiration. These findings are consistent with the patient’s clinical history. </p> <p>For diagnostics, the IgG Western blot is positive if 5 of the 10 bands are positive.<sup>9</sup> This patient far exceeds the IgG band number to diagnose Lyme disease. All patients with Lyme arthritis will have positive IgG serologies since Lyme arthritis is a late manifestation of the infection. IgM reactivity may be present, but are not necessary to diagnose Lyme arthritis.<sup>10</sup> Synovial fluid is often not analyzed for antibody responses as they are susceptible to false positive results, but synovial PCR testing like this patient had detects approximately 70% of patients with untreated Lyme arthritis.<sup>11</sup> However, PCR positivity does not necessarily equate with active infection. Serologic testing for Lyme disease by enzyme-linked immunosorbent assay and Western blot as well as careful history and the exclusion of other diagnoses are usually sufficient to make the diagnosis. </p> <p><strong></strong><strong> </strong><strong>Dr. Merz:</strong><b> </b>On further history the patient reported that 5 years prior he found a tick on his skin with a bull’s-eye rash. He was treated with 28 days of doxycycline at that time. He did not recall any tick bites or rashes in the years since. </p> <p>Dr. Serrao, is it surprising that he developed Lyme arthritis 5 years after exposure and after being treated appropriately? What is the typical treatment approach for a patient like this?</p> <p><strong></strong><strong> </strong><strong>Dr. Serrao: </strong>It is atypical to develop Lyme arthritis 5 years after reported treatment of what appeared to be early localized disease, namely, erythema migrans. This stage is usually cured with 10 days of treatment alone (he received 28 days) and is generally abortive of subsequent stages, including Lyme arthritis. Furthermore, the patient reported no symptoms of arthritis until recently since that time. Therefore, one can argue that the excessively long span of time from treatment to these first episodes of arthritis suggests the patient could have been reinfected. When available, comparing the types and number of Western blot bands (eg, new and/or more bands on subsequent serologic testing) can support a reinfection diagnosis. A delayed postinfectious inflammatory process from excessive proinflammatory immune responses that block wound repair resulting in proliferative synovitis is also possible.<sup>12</sup> This is defined as the postinfectious, postantibiotic, or antibiotic-refractory Lyme arthritis, a diagnosis of exclusion more apparent only after patients receive appropriate antibiotic courses for the possibility of untreated Lyme as an active infection.<sup>12</sup></p> <p>Given the inherent diagnostic uncertainty between an active infection and posttreatment Lyme arthritis syndromes, it is best to approach most cases of Lyme arthritis as an active infection first especially if not yet treated with antibiotics. Diagnosis of postinflammatory processes should be considered if symptoms persist after appropriate antibiotics, and then short-term use of disease-modifying antirheumatic drugs, rather than further courses of antibiotics, is recommended.</p> <p><strong></strong><strong> </strong><strong>Dr. Merz: </strong>The patient was initiated on doxycycline with the plan to transition to ceftriaxone if there was no response. One day after diagnosis and treatment initiation and in the setting of continued pain, the patient again asked to leave the hospital to drink alcohol. After eloping and becoming intoxicated with alcohol, he returned to his room. He remained concerned about his continued pain and lack of adequate pain control. At the time, he was receiving hydromorphone, ketorolac, lorazepam, gabapentin, and quetiapine. </p> <p>Dr. Serrao, do you expect this degree of pain from Lyme arthritis? </p> <p><strong></strong><strong> </strong><strong>Dr. Serrao:</strong><b> </b>Lyme arthritis is typically less painful than other forms of infectious or inflammatory arthritis. Pain is usually caused by the pressure from the acute accumulation and reaccumulation of fluid. In this case, the rapid accumulation of fluid that this patient experienced as well as relief with arthrocentesis suggests that the size and acuity of the effusion was causing great discomfort. Repeated arthrocentesis can prove to be a preventative strategy to minimize synovial herniation.<br/><br/><strong></strong><strong> </strong><strong>Dr. Merz: </strong>Dr. Shahal, how do you balance the patient subjectively telling you that they are in pain with objective signs that they may be tolerating the pain like walking around unassisted? Is there anything else that could have been done to prevent this adverse outcome?<br/><br/><strong></strong><strong> </strong><strong>Dr. Shahal:</strong><b> </b>This is one of the hardest pieces of pain management. We want to practice beneficence by believing our patients and addressing their discomfort, but we also want to practice nonmaleficence by avoiding inappropriate long-term pain treatments like opioids that have significant harm as well as avoiding exacerbating this patient’s underlying SUD. An agent like buprenorphine/naloxone could have been an excellent fit to treat pain and SUD, but the patient’s lack of interest and the frequent use of short-acting opioids were major barriers. A chronic pain consult early on is helpful in cases like this as well, but they were unable to see him since he was often out of his room. Repeated arthrocentesis may also have helped the pain. Treatment of anxiety and insomnia with medications like hydroxyzine, trazodone, melatonin, gabapentin, or buspirone as well as interventions like sleep hygiene protocols or spiritual care may have helped somewhat as well. </p> <p>We know that there is a vicious cycle between pain and poorly controlled mood symptoms. Many of our veterans have PTSD, anxiety, and SUD that are exacerbated by hospitalization and pain. Maintaining optimal communication between the patient and the practitioners, using trauma-informed care, understanding the patient’s goals of care, setting expectations and limits, and attempting to address the patient’s needs while attempting to minimize stigma might be helpful. However, despite optimal care, sometimes these events cannot be avoided. </p> <p><strong></strong><strong> </strong><strong>Dr. Merz:</strong><b> </b>The patient was ultimately transferred to an inpatient psychiatric unit where a taper plan for the short-acting opioids was implemented. He was psychiatrically stabilized and discharged a few days later off opioids and on doxycycline. On follow-up a few weeks later, his pain had markedly improved, and the effusion was significantly reduced in size. His mood and impulsivity had stabilized. He continues to follow-up in the infectious disease clinic. </p> <h3>Author affiliations</h3> <p> <em> <sup>a</sup> Brigham and Women’s Hospital, Boston, Massachusetts<br/><br/> <sup>b</sup> Veterans Affairs Boston Healthcare System, West Roxbury, Massachusetts<br/><br/> <sup>c</sup> Boston University School of Medicine, Massachusetts<br/><br/> <sup>d</sup> Harvard Medical School, Boston, Massachusetts </em> </p> <h3>Author disclosures</h3> <p> <em> The authors report no actual or potential conflicts of interest or outside sources of funding with regard to this article. </em> </p> <h3>Disclaimer</h3> <p> <em> The opinions expressed herein are those of the authors and do not necessarily reflect those of <i>Federal Practitioner</i> , Frontline Medical Communications Inc., the US Government, or any of its agencies. This article may discuss unlabeled or investigational use of certain drugs. Please review the complete prescribing information for specific drugs or drug combinations—including indications, contraindications, warnings, and adverse effects—before administering pharmacologic therapy to patients. </em> </p> <h3>Ethics and consent</h3> <p> <em> Informed consent was obtained from the patient reported in this case report. </em> </p> <h3>References</h3> <p class="reference"> 1. Siva C, Velazquez C, Mody A, Brasington R. Diagnosing acute monoarthritis in adults: a practical approach for the family physician. <i>Am Fam Physician.</i> 2003;68(1):83-90.<br/><br/> 2. Qaseem A, McLean RM, O’Gurek D, et al. Nonpharmacologic and pharmacologic management of acute pain from non-low back, musculoskeletal injuries in adults: a clinical guideline from the American College of Physicians and American Academy of Family Physicians. <i>Ann Intern Med. </i>2020;173(9):739-748. doi:10.7326/M19-3602<br/><br/> 3. Silverman MM, Berman AL. Suicide risk assessment and risk formulation part I: a focus on suicide ideation in assessing suicide risk. <i>Suicide Life Threat Behav.</i> 2014;44(4):420-431. doi:10.1111/sltb.12065<br/><br/> 4. Berman AL, Silverman MM. Suicide risk assessment and risk formulation part II: Suicide risk formulation and the determination of levels of risk. <i>Suicide Life Threat Behav. </i>2014;44(4):432-443. doi:10.1111/sltb.12067<br/><br/> 5. Quinlan J, Cox F. Acute pain management in patients with drug dependence syndrome. <i>Pain Rep.</i> 2017;2(4):e611. Published 2017 Jul 27. doi:10.1097/PR9.0000000000000611<br/><br/> 6. Chou R, Wagner J, Ahmed AY, et al. Treatments for Acute Pain: A Systematic Review. Agency for Healthcare Research and Quality; 2020. https://www.ncbi.nlm.nih.gov/books/NBK566506/<br/><br/> 7. Seidman AJ, Limaiem F. Synovial fluid analysis. In: StatPearls [Internet]. Treasure Island (FL): StatPearls Publishing; 2022. Updated May 8, 2022. https://www.ncbi.nlm.nih.gov/books/NBK537114<br/><br/> 8. Arvikar SL, Steere AC. Diagnosis and treatment of Lyme arthritis. <i>Infect Dis Clin North Am.</i> 2015;29(2):269-280. doi:10.1016/j.idc.2015.02.004<br/><br/> 9. Centers for Disease Control and Prevention. Recommendations for test performance and interpretation from the Second National Conference on Serologic Diagnosis of Lyme Disease. <i>JAMA.</i> 1995;274(12):937.<br/><br/>10. Craft JE, Grodzicki RL, Steere AC. Antibody response in Lyme disease: evaluation of diagnostic tests. <i>J Infect Dis. </i>1984;149(5):789-795. doi:10.1093/infdis/149.5.789<br/><br/>11. Nocton JJ, Dressler F, Rutledge BJ, Rys PN, Persing DH, Steere AC. Detection of Borrelia burgdorferi DNA by polymerase chain reaction in synovial fluid from patients with Lyme arthritis. <i>N Engl J Med.</i> 1994;330(4):229-234. doi:10.1056/NEJM199401273300401<br/><br/>12. Steere AC. Posttreatment Lyme disease syndromes: distinct pathogenesis caused by maladaptive host responses. <i>J Clin Invest.</i> 2020;130(5):2148-2151. doi:10.1172/JCI138062</p> </itemContent> </newsItem> </itemSet></root>
Disallow All Ads
Content Gating
No Gating (article Unlocked/Free)
Alternative CME
Disqus Comments
Default
Use ProPublica
Hide sidebar & use full width
render the right sidebar.
Conference Recap Checkbox
Not Conference Recap
Clinical Edge
Display the Slideshow in this Article
Medscape Article
Display survey writer
Reuters content
Disable Inline Native ads
WebMD Article
Article PDF Media

A Veteran Presenting With Shortness of Breath, Cough, and Leukocytosis

Article Type
Changed
Tue, 06/22/2021 - 11:49

Case Presentation: A 62-year-old male presented with shortness of breath and a cough productive of green sputum. He had a history of hyperlipidemia, posttraumatic stress disorder, bipolar disorder, obstructive sleep apnea, and a 50 pack-year history of smoking. His medications included prazosin, melatonin, lithium, and gabapentin. He also had a significant exposure history including asbestos and chemical paints following his leave from the military. At the initial evaluation, laboratory work revealed a leukocytosis with white blood cell (WBC) count 20 k/cm3and otherwise normal transaminases, albumin, and electrolytes. A chest X-ray revealed a new left hilar mass.

►Manisha Apte, MD, Chief Medical Resident, VA Boston Healthcare System (VABHS) and Boston Medical Center (BMC): To work up his new left hilar mass, a computed tomography (CT) of the chest was ordered (Figure 1), which revealed an apical left lower lobe mass extending into the left hilum encasing part of the ascending aorta. Enlarged mediastinal subcentimeter paratracheal and superior mediastinal lymph nodes also were identified and the pattern raised the concern for lymphangitic carcinomatosis. Dr. Fine, what do you make of the CT findings?

fdp03806e46_f1.png



► Alan Fine, MD, Section of Pulmonary and Critical Care, VABHS and Professor of Medicine, Boston University School of Medicine: This mass had irregular edges with septal thickening, which may be why there was a concern for lymphangitic spread. There were no clear tissue planes to see if this process was invading the mediastinum. The mass was irregular, a single lesion, and proximal, making it consistent with a lung cancer. In fact, with his history of smoking, asbestos exposure, the numbers 1 to 10 diagnoses were lung cancer. The lack of demarcation of tissue planes supports this. There are some infections, classically actinomycosis, that do cross and invade anatomical barriers.1 But this looked like a primary lung cancer.

► Dr. Apte: The patient was referred to a pulmonologist where an additional history of night sweats and weight loss were noted. Dr. Fine, we have a patient with a newly identified lung mass, and while we have reason to suspect malignancy as you have already noted, there are many other etiologies to consider, including infections (histoplasmosis, cryptococcosis, bacterial abscess), inflammatory processes (sarcoidosis, rheumatoid nodule) and vasculitis (granulomatosis with polyangiitis). What should be the next step taken to make a diagnosis?


►Dr. Fine: For cancer specifically, we would like to both stage and make a diagnosis with one procedure. That’s part of the utility of a positron emission tomography (PET) scan: We can see lymph node involvement and stage the cancer. We must consider the patient’s comorbidities and the location of the lesion (ie, is it amenable to needle biopsy?). In this case, there are enlarged mediastinal lymph nodes, so one could perform a bronchoscopy with endobronchial ultrasound, which is a relatively noninvasive way to sample the lymph nodes to ideally stage and make a diagnosis as safely as possible. If we are considering infection, needle aspiration is not as sensitive.2

► Dr. Apte: The patient underwent a PET CT, which redemonstrated the lung mass with a loss of aortic fat plane suspicious for aortic involvement as well as lymph nodes in levels 7 and 8 that were concerning for malignancy. Subsequent bronchoscopy with biopsy and endobronchial ultrasound did not show evidence of malignancy; washing and brushing from the mass and lymph node specimens did not identify malignant cells. Benign respiratory mucosa with mild chronic inflammation was noted. Dr. Fine, given the nonspecific findings on the PET scan, negative findings on our bronchoscopy, and a negative biopsy, should we be satisfied that we have ruled out cancer?


► Dr. Fine: No, bronchoscopy has its limitations. It’s highly sensitive to the diagnosis of malignancy if you can see an endobronchial lesion, but we did not see one here. You can only go so far with the scope, and it’s not uncommon for us not to be able to make the diagnosis with bronchoscopy. Malignancy is still the most likely diagnosis, and we need to work this up further. I would perform another biopsy.


►Dr. Apte: Four weeks later, the patient presented with continued shortness of breath, fatigue, and fever. A repeat chest CT showed an opacity suggestive of pneumonia. Given the continued concern for cancer a CT-guided needle biopsy was performed and was once again negative for malignancy. The decision was made to pursue a video-assisted thorascopic surgery (VATS). Following the VATS, the patient developed rigors, fever, and tachycardia with new atrial fibrillation. While being evaluated hypercalcemia was identified, with further workup revealing a low parathyroid hormone (PTH) and low PTH-related peptide. Dr. Fine, the presence of hypercalcemia and a new arrythmia raised the possibility of sarcoidosis. Could this be sarcoidosis?


►Dr. Fine: Sarcoidosis is one of the great masqueraders in medicine. There is a type of sarcoidosis called nodular sarcoidosis where you see masslike distribution in the lung, but generally there are multiple masses and so this presentation would be atypical.3 There is also a phenomenon called sarcoidal reactions usually in the presence of cancer. Again, one tends to see multiple tiny lesions in the lung. It is certainly on the differential, but I would consider it to be less likely than cancer. It is also relatively common to develop atrial fibrillation after manipulation from a lung surgery.4 The other possibility I am concerned about is whether the mass is invading the mediastinum and involving the pericardium.


►Dr. Apte: Results from the VATS biopsy once again returned negative for malignancy and instead showed signs of focal micro-abscesses, atypical pneumocytes, and prominent neutrophils. A diagnosis of acute fibrinous organizing pneumonia (AFOP) was offered. Dr. Fine, what is AFOP?


►Dr. Fine: This is the first case of AFOP I had seen and probably the first case many in our department have seen. This is a relatively new entity with limited reported cases in the literature and is a pathological diagnosis originally recorded in autopsies from patients at the National Institutes of Health.5,6 Given the complexity of the lesion, the diagnosis is difficult to make. Most commonly, AFOP is associated with other systemic entities, most commonly hematologic malignancies like lymphomas and leukemias. It has also been associated with vasculitis and certain drugs. The mechanism is poorly understood, and although pneumonia is a part of the term, this just implies there is inflammation of the lung (ie, pneumonitis).


► Dr. Apte: Given the association of AFOP with underlying hematologic malignancies, an emphasis was placed on another finding: the patient’s increasing WBC count. The total WBC count had been 20 k/cm3 at the time of his lung mass discovery but had increased to > 40 k/cm3 with a differential of neutrophils > 80%. Flow cytometry was negative, and his peripheral smear was read as normal. Dr. Gilbert, what might explain this patient’s leukocytosis?

►Gary Gilbert, MD, Section of Hematology and Oncology, VABHS and Associate Professor of Medicine, Harvard Medical School (HMS): This patient had an elevated WBC for 4 months. Initially, the cause was likely lithium as this is known to cause a leukocytosis.7 More recently, the total WBC had increased and there were a couple of other abnormalities: A consistently elevated absolute monocyte count and a markedly elevated mature neutrophil count. These findings are consistent with a leukemoid reaction (ie, a WBC count > 50,000/µL from causes other than leukemia). The question becomes what is this a leukemoid reaction in response to? Once we have excluded a lung malignancy (a well-known common cause of a leukemoid reaction) we must consider a clonal myeloproliferative disorder. This is particularly true because many things that cause a leukemoid reaction (eg, lobar pneumonia) do not cause a persistently elevated neutrophil count. That this patient does have a persistently elevated neutrophil count suggests something abnormal about the neutrophils themselves.

► Dr. Apte: A bone marrow biopsy was performed. Dr. Gilbert, can you comment on this patient’s bone marrow biopsy and whether a myeloproliferative disorder may have played a role in the marked leukocytosis?

► Dr. Gilbert: The bone marrow biopsy was hyperplastic with myeloid predominance and normal maturation in all lineages. A deep sequencing analysis demonstrated the absence of chromosomal abnormalities or genetic mutations that are associated with myeloproliferative disorders. This excludes the possibility of a myeloproliferative disorder.


► Dr. Apte: The patient was started on 60 mg of prednisone daily, which led to marked improvement in his symptoms. He was discharged in stable condition but presented again with abdominal pain. A complete blood count once again showed increased WBC and new thrombocytosis. A CT angiogram (CTA) showed the prior lung mass with new signs of central necrosis. In the abdomen, new splenic and renal infarct were identified, along with signs of multiple arterial thrombi in the abdomen and internal and external iliac vessel wall thickening. These findings were read as concerning for a medium vessel vasculitis. Dr. Kaur, what are some of the imaging findings you would expect to see in vasculitis, and what about this patient’s CT is consistent with a medium vessel vasculitis?


► Maneet Kaur, MD, Section of Rheumatology, VABHS: Vasculitis is inflammation of the vessel wall that can lead to vascular injury and activation of the coagulation cascade. Sometimes these findings can be seen on imaging with evidence of stenosis, microaneurysms, and thrombosis distal to the stenosis. The nomenclature of vasculitis is not simple and has been revised many times. Medium-vessel vasculitis does not just affect the medium vessels (eg, visceral arteries) but can overlap with distal large vessels and smaller cutaneous vessels.

 

 

The first thing that comes to mind in this case is polyarteritis nodosa (PAN), an immune complex-mediated medium vessel disease that can involve large and small vessels in muscle, nerve, and skin. It can also present with masses.

►Dr. Apte: To address the arterial thrombi seen on CTA, arterial-brachial indices were obtained and showed bilateral occlusive disease in his distal extremities; findings that could be explained by vasculitis. His VATS biopsy pathology was reviewed for signs of vasculitis. Dr. Huang, can you review these slides for us please?

fdp03806e46_box.png

►Qin Huang, MD, Department of Pathology and Laboratory Medicine, VABHS and Assistant Professor of Pathology, HMS: This patient had a history of smoking, and there are many black pigment-laden macrophages present in the lung tissue. There were areas of hemorrhage and fibrin deposition and an overall picture of organizing pneumonia. At a lower power, you can see neutrophils everywhere, some in the form of micro-abscesses. The arterial walls did not show signs of vasculitis (Figure 2). Based on the clinical information and radiology findings, we suspected an acute infection-related pneumonia or a primary lung malignancy causing obstruction pneumonia. We suggested a rebiopsy of the lung mass to rule out a primary lung malignancy.

fdp03806e46_f2.png

► Dr. Apte: Given his CT findings, a serologic rheumatologic workup including antineutrophil cytoplasmic antibody, antinuclear antibody, and rheumatoid factors were sent and returned negative. The location of arterial wall inflammation on imaging made it unamenable for biopsy. The patient began to experience bilateral temporal pain, which raised the concern for a large vessel vasculitis, specifically giant cell arteritis. Bilateral temporal artery biopsies were obtained and were not suggestive of vasculitis. Dr. Kaur, we still do not have any serologic or biopsy confirmation to support a diagnosis of vasculitis. Can we still call this a vasculitis?

►Dr. Kaur: Few things can cause the picture that was seen radiographically. A few noninflammatory causes like fibromuscular dysplasia can cause both large and small vessel stenosis, but the elevations in erythrocyte sedimentation rate and C-reactive protein along with response to steroids makes these diagnoses unlikely. Sometimes we must make a clinical diagnosis for vasculitis based on the clinical picture, and I would feel comfortable treating this patient for vasculitis.

With that said, I remain concerned that this patient also has a malignancy. His WBC increased to > 70 k/cm3 and his calcium to > 13 mg/dL. These findings are hard to explain by vasculitis alone. There are cancer-associated vasculitis, and I suspect this is the explanation here.8 His temporal pain was pointing to large vessel involvement, so he could have an undifferentiated vasculitis.

► Dr. Apte: A decision was made to empirically treat with tocilizumab, an IL-6 receptor antagonist, for an undifferentiated autoimmune disease, in addition to tapering steroids. The patient underwent a second VATS, which again revealed AFOP but no signs of malignancy. Unfortunately, he developed multiple complications over the subsequent weeks and passed away. An autopsy was requested by family members and pathology from his lung mass was reviewed. (Figure 3). Dr. Huang, can you review these slides for us?


fdp03806e46_f3.png


► Dr. Huang: The left lung mass at autopsy shows nests, poorly formed clusters, and individuals of malignant neoplastic nonkeratinizing squamous cells embedded in a desmoplastic stroma in the mass center, consistent with poorly differentiated squamous cell carcinoma, and a circumscribed area of residual subacute organizing pneumonia with abscess, granulomatous changes, and early fibrosis at the periphery of this mass.

►Dr. Apte: Based on autopsy findings, the final diagnosis was poorly differentiated squamous cell carcinoma associated with subacute organizing pneumonia and medium vessel vasculitis, which presented with a severe leukocytosis ultimately thought to be a leukemoid reaction from his lung cancer.

References

1. Valour F, Sénéchal A, Dupieux C, et al. Actinomycosis: etiology, clinical features, diagnosis, treatment, and management. Infect Drug Resist. 2014;7:183-197. Published 2014 Jul 5. doi:10.2147/IDR.S39601

2. de Bazelaire C, Coffin A, Cohen-Zarade S, et al. CT-guided biopsies in lung infections in patients with haematological malignancies. Diagn Interv Imaging. 2013;94(2):202-215. doi:10.1016/j.diii.2012.12.008

3. Sweidan AJ, Singh NK, Stein A, Tanios M. Nodular sarcoidosis masquerading as cancer. Clin Med Insights Circ Respir Pulm Med. 2017;11:1179548417703123. Published 2017 Apr 12. doi:10.1177/1179548417703123

4. Bagheri R, Yousefi Y, Rezai R, Azemonfar V, Keshtan FG. Atrial fibrillation after lung surgery: incidence, underlying factors, and predictors. Kardiochir Torakochirurgia Pol. 2019;16(2):53-56. doi:10.5114/kitp.2019.86355

5. Lu J, Yin Q, Zha Y, et al. Acute fibrinous and organizing pneumonia: two case reports and literature review. BMC Pulm Med. 2019;19(1):141. Published 2019 Aug 5. doi:10.1186/s12890-019-0861-3

6. Beasley MB, Franks TJ, Galvin JR, Gochuico B, Travis WD. Acute fibrinous and organizing pneumonia: a histological pattern of lung injury and possible variant of diffuse alveolar damage. Arch Pathol Lab Med. 2002;126(9):1064-1070. doi:10.5858/2002-126-1064-AFAOP

7. Murphy DL, Goodwin FK, Bunney WE Jr. Leukocytosis during lithium treatment. Am J Psychiatry. 1971;127(11):1559-1561. doi:10.1176/ajp.127.11.1559

8. Fain O, Hamidou M, Cacoub P, et al. Vasculitides associated with malignancies: analysis of sixty patients. Arthritis Rheum. 2007;57(8):1473-1480. doi:10.1002/art.23085

Article PDF
Author and Disclosure Information

Correspondence: Anthony Breu (anthony.breu@va.gov)

Author disclosures
The authors report no actual or potential conflicts of interest with regard to this article.

Disclaimer
The opinions expressed herein are those of the authors and do not necessarily reflect those of Federal Practitioner , Frontline Medical Communications Inc., the US Government, or any of its agencies. This article may discuss unlabeled or investigational use of certain drugs. Please review the complete prescribing information for specific drugs or drug combinations—including indications, contraindications, warnings, and adverse effects—before administering pharmacologic therapy to patients.

Issue
Federal Practitioner - 38(6)a
Publications
Topics
Page Number
e46-e50
Sections
Author and Disclosure Information

Correspondence: Anthony Breu (anthony.breu@va.gov)

Author disclosures
The authors report no actual or potential conflicts of interest with regard to this article.

Disclaimer
The opinions expressed herein are those of the authors and do not necessarily reflect those of Federal Practitioner , Frontline Medical Communications Inc., the US Government, or any of its agencies. This article may discuss unlabeled or investigational use of certain drugs. Please review the complete prescribing information for specific drugs or drug combinations—including indications, contraindications, warnings, and adverse effects—before administering pharmacologic therapy to patients.

Author and Disclosure Information

Correspondence: Anthony Breu (anthony.breu@va.gov)

Author disclosures
The authors report no actual or potential conflicts of interest with regard to this article.

Disclaimer
The opinions expressed herein are those of the authors and do not necessarily reflect those of Federal Practitioner , Frontline Medical Communications Inc., the US Government, or any of its agencies. This article may discuss unlabeled or investigational use of certain drugs. Please review the complete prescribing information for specific drugs or drug combinations—including indications, contraindications, warnings, and adverse effects—before administering pharmacologic therapy to patients.

Article PDF
Article PDF

Case Presentation: A 62-year-old male presented with shortness of breath and a cough productive of green sputum. He had a history of hyperlipidemia, posttraumatic stress disorder, bipolar disorder, obstructive sleep apnea, and a 50 pack-year history of smoking. His medications included prazosin, melatonin, lithium, and gabapentin. He also had a significant exposure history including asbestos and chemical paints following his leave from the military. At the initial evaluation, laboratory work revealed a leukocytosis with white blood cell (WBC) count 20 k/cm3and otherwise normal transaminases, albumin, and electrolytes. A chest X-ray revealed a new left hilar mass.

►Manisha Apte, MD, Chief Medical Resident, VA Boston Healthcare System (VABHS) and Boston Medical Center (BMC): To work up his new left hilar mass, a computed tomography (CT) of the chest was ordered (Figure 1), which revealed an apical left lower lobe mass extending into the left hilum encasing part of the ascending aorta. Enlarged mediastinal subcentimeter paratracheal and superior mediastinal lymph nodes also were identified and the pattern raised the concern for lymphangitic carcinomatosis. Dr. Fine, what do you make of the CT findings?

fdp03806e46_f1.png



► Alan Fine, MD, Section of Pulmonary and Critical Care, VABHS and Professor of Medicine, Boston University School of Medicine: This mass had irregular edges with septal thickening, which may be why there was a concern for lymphangitic spread. There were no clear tissue planes to see if this process was invading the mediastinum. The mass was irregular, a single lesion, and proximal, making it consistent with a lung cancer. In fact, with his history of smoking, asbestos exposure, the numbers 1 to 10 diagnoses were lung cancer. The lack of demarcation of tissue planes supports this. There are some infections, classically actinomycosis, that do cross and invade anatomical barriers.1 But this looked like a primary lung cancer.

► Dr. Apte: The patient was referred to a pulmonologist where an additional history of night sweats and weight loss were noted. Dr. Fine, we have a patient with a newly identified lung mass, and while we have reason to suspect malignancy as you have already noted, there are many other etiologies to consider, including infections (histoplasmosis, cryptococcosis, bacterial abscess), inflammatory processes (sarcoidosis, rheumatoid nodule) and vasculitis (granulomatosis with polyangiitis). What should be the next step taken to make a diagnosis?


►Dr. Fine: For cancer specifically, we would like to both stage and make a diagnosis with one procedure. That’s part of the utility of a positron emission tomography (PET) scan: We can see lymph node involvement and stage the cancer. We must consider the patient’s comorbidities and the location of the lesion (ie, is it amenable to needle biopsy?). In this case, there are enlarged mediastinal lymph nodes, so one could perform a bronchoscopy with endobronchial ultrasound, which is a relatively noninvasive way to sample the lymph nodes to ideally stage and make a diagnosis as safely as possible. If we are considering infection, needle aspiration is not as sensitive.2

► Dr. Apte: The patient underwent a PET CT, which redemonstrated the lung mass with a loss of aortic fat plane suspicious for aortic involvement as well as lymph nodes in levels 7 and 8 that were concerning for malignancy. Subsequent bronchoscopy with biopsy and endobronchial ultrasound did not show evidence of malignancy; washing and brushing from the mass and lymph node specimens did not identify malignant cells. Benign respiratory mucosa with mild chronic inflammation was noted. Dr. Fine, given the nonspecific findings on the PET scan, negative findings on our bronchoscopy, and a negative biopsy, should we be satisfied that we have ruled out cancer?


► Dr. Fine: No, bronchoscopy has its limitations. It’s highly sensitive to the diagnosis of malignancy if you can see an endobronchial lesion, but we did not see one here. You can only go so far with the scope, and it’s not uncommon for us not to be able to make the diagnosis with bronchoscopy. Malignancy is still the most likely diagnosis, and we need to work this up further. I would perform another biopsy.


►Dr. Apte: Four weeks later, the patient presented with continued shortness of breath, fatigue, and fever. A repeat chest CT showed an opacity suggestive of pneumonia. Given the continued concern for cancer a CT-guided needle biopsy was performed and was once again negative for malignancy. The decision was made to pursue a video-assisted thorascopic surgery (VATS). Following the VATS, the patient developed rigors, fever, and tachycardia with new atrial fibrillation. While being evaluated hypercalcemia was identified, with further workup revealing a low parathyroid hormone (PTH) and low PTH-related peptide. Dr. Fine, the presence of hypercalcemia and a new arrythmia raised the possibility of sarcoidosis. Could this be sarcoidosis?


►Dr. Fine: Sarcoidosis is one of the great masqueraders in medicine. There is a type of sarcoidosis called nodular sarcoidosis where you see masslike distribution in the lung, but generally there are multiple masses and so this presentation would be atypical.3 There is also a phenomenon called sarcoidal reactions usually in the presence of cancer. Again, one tends to see multiple tiny lesions in the lung. It is certainly on the differential, but I would consider it to be less likely than cancer. It is also relatively common to develop atrial fibrillation after manipulation from a lung surgery.4 The other possibility I am concerned about is whether the mass is invading the mediastinum and involving the pericardium.


►Dr. Apte: Results from the VATS biopsy once again returned negative for malignancy and instead showed signs of focal micro-abscesses, atypical pneumocytes, and prominent neutrophils. A diagnosis of acute fibrinous organizing pneumonia (AFOP) was offered. Dr. Fine, what is AFOP?


►Dr. Fine: This is the first case of AFOP I had seen and probably the first case many in our department have seen. This is a relatively new entity with limited reported cases in the literature and is a pathological diagnosis originally recorded in autopsies from patients at the National Institutes of Health.5,6 Given the complexity of the lesion, the diagnosis is difficult to make. Most commonly, AFOP is associated with other systemic entities, most commonly hematologic malignancies like lymphomas and leukemias. It has also been associated with vasculitis and certain drugs. The mechanism is poorly understood, and although pneumonia is a part of the term, this just implies there is inflammation of the lung (ie, pneumonitis).


► Dr. Apte: Given the association of AFOP with underlying hematologic malignancies, an emphasis was placed on another finding: the patient’s increasing WBC count. The total WBC count had been 20 k/cm3 at the time of his lung mass discovery but had increased to > 40 k/cm3 with a differential of neutrophils > 80%. Flow cytometry was negative, and his peripheral smear was read as normal. Dr. Gilbert, what might explain this patient’s leukocytosis?

►Gary Gilbert, MD, Section of Hematology and Oncology, VABHS and Associate Professor of Medicine, Harvard Medical School (HMS): This patient had an elevated WBC for 4 months. Initially, the cause was likely lithium as this is known to cause a leukocytosis.7 More recently, the total WBC had increased and there were a couple of other abnormalities: A consistently elevated absolute monocyte count and a markedly elevated mature neutrophil count. These findings are consistent with a leukemoid reaction (ie, a WBC count > 50,000/µL from causes other than leukemia). The question becomes what is this a leukemoid reaction in response to? Once we have excluded a lung malignancy (a well-known common cause of a leukemoid reaction) we must consider a clonal myeloproliferative disorder. This is particularly true because many things that cause a leukemoid reaction (eg, lobar pneumonia) do not cause a persistently elevated neutrophil count. That this patient does have a persistently elevated neutrophil count suggests something abnormal about the neutrophils themselves.

► Dr. Apte: A bone marrow biopsy was performed. Dr. Gilbert, can you comment on this patient’s bone marrow biopsy and whether a myeloproliferative disorder may have played a role in the marked leukocytosis?

► Dr. Gilbert: The bone marrow biopsy was hyperplastic with myeloid predominance and normal maturation in all lineages. A deep sequencing analysis demonstrated the absence of chromosomal abnormalities or genetic mutations that are associated with myeloproliferative disorders. This excludes the possibility of a myeloproliferative disorder.


► Dr. Apte: The patient was started on 60 mg of prednisone daily, which led to marked improvement in his symptoms. He was discharged in stable condition but presented again with abdominal pain. A complete blood count once again showed increased WBC and new thrombocytosis. A CT angiogram (CTA) showed the prior lung mass with new signs of central necrosis. In the abdomen, new splenic and renal infarct were identified, along with signs of multiple arterial thrombi in the abdomen and internal and external iliac vessel wall thickening. These findings were read as concerning for a medium vessel vasculitis. Dr. Kaur, what are some of the imaging findings you would expect to see in vasculitis, and what about this patient’s CT is consistent with a medium vessel vasculitis?


► Maneet Kaur, MD, Section of Rheumatology, VABHS: Vasculitis is inflammation of the vessel wall that can lead to vascular injury and activation of the coagulation cascade. Sometimes these findings can be seen on imaging with evidence of stenosis, microaneurysms, and thrombosis distal to the stenosis. The nomenclature of vasculitis is not simple and has been revised many times. Medium-vessel vasculitis does not just affect the medium vessels (eg, visceral arteries) but can overlap with distal large vessels and smaller cutaneous vessels.

 

 

The first thing that comes to mind in this case is polyarteritis nodosa (PAN), an immune complex-mediated medium vessel disease that can involve large and small vessels in muscle, nerve, and skin. It can also present with masses.

►Dr. Apte: To address the arterial thrombi seen on CTA, arterial-brachial indices were obtained and showed bilateral occlusive disease in his distal extremities; findings that could be explained by vasculitis. His VATS biopsy pathology was reviewed for signs of vasculitis. Dr. Huang, can you review these slides for us please?

fdp03806e46_box.png

►Qin Huang, MD, Department of Pathology and Laboratory Medicine, VABHS and Assistant Professor of Pathology, HMS: This patient had a history of smoking, and there are many black pigment-laden macrophages present in the lung tissue. There were areas of hemorrhage and fibrin deposition and an overall picture of organizing pneumonia. At a lower power, you can see neutrophils everywhere, some in the form of micro-abscesses. The arterial walls did not show signs of vasculitis (Figure 2). Based on the clinical information and radiology findings, we suspected an acute infection-related pneumonia or a primary lung malignancy causing obstruction pneumonia. We suggested a rebiopsy of the lung mass to rule out a primary lung malignancy.

fdp03806e46_f2.png

► Dr. Apte: Given his CT findings, a serologic rheumatologic workup including antineutrophil cytoplasmic antibody, antinuclear antibody, and rheumatoid factors were sent and returned negative. The location of arterial wall inflammation on imaging made it unamenable for biopsy. The patient began to experience bilateral temporal pain, which raised the concern for a large vessel vasculitis, specifically giant cell arteritis. Bilateral temporal artery biopsies were obtained and were not suggestive of vasculitis. Dr. Kaur, we still do not have any serologic or biopsy confirmation to support a diagnosis of vasculitis. Can we still call this a vasculitis?

►Dr. Kaur: Few things can cause the picture that was seen radiographically. A few noninflammatory causes like fibromuscular dysplasia can cause both large and small vessel stenosis, but the elevations in erythrocyte sedimentation rate and C-reactive protein along with response to steroids makes these diagnoses unlikely. Sometimes we must make a clinical diagnosis for vasculitis based on the clinical picture, and I would feel comfortable treating this patient for vasculitis.

With that said, I remain concerned that this patient also has a malignancy. His WBC increased to > 70 k/cm3 and his calcium to > 13 mg/dL. These findings are hard to explain by vasculitis alone. There are cancer-associated vasculitis, and I suspect this is the explanation here.8 His temporal pain was pointing to large vessel involvement, so he could have an undifferentiated vasculitis.

► Dr. Apte: A decision was made to empirically treat with tocilizumab, an IL-6 receptor antagonist, for an undifferentiated autoimmune disease, in addition to tapering steroids. The patient underwent a second VATS, which again revealed AFOP but no signs of malignancy. Unfortunately, he developed multiple complications over the subsequent weeks and passed away. An autopsy was requested by family members and pathology from his lung mass was reviewed. (Figure 3). Dr. Huang, can you review these slides for us?


fdp03806e46_f3.png


► Dr. Huang: The left lung mass at autopsy shows nests, poorly formed clusters, and individuals of malignant neoplastic nonkeratinizing squamous cells embedded in a desmoplastic stroma in the mass center, consistent with poorly differentiated squamous cell carcinoma, and a circumscribed area of residual subacute organizing pneumonia with abscess, granulomatous changes, and early fibrosis at the periphery of this mass.

►Dr. Apte: Based on autopsy findings, the final diagnosis was poorly differentiated squamous cell carcinoma associated with subacute organizing pneumonia and medium vessel vasculitis, which presented with a severe leukocytosis ultimately thought to be a leukemoid reaction from his lung cancer.

Case Presentation: A 62-year-old male presented with shortness of breath and a cough productive of green sputum. He had a history of hyperlipidemia, posttraumatic stress disorder, bipolar disorder, obstructive sleep apnea, and a 50 pack-year history of smoking. His medications included prazosin, melatonin, lithium, and gabapentin. He also had a significant exposure history including asbestos and chemical paints following his leave from the military. At the initial evaluation, laboratory work revealed a leukocytosis with white blood cell (WBC) count 20 k/cm3and otherwise normal transaminases, albumin, and electrolytes. A chest X-ray revealed a new left hilar mass.

►Manisha Apte, MD, Chief Medical Resident, VA Boston Healthcare System (VABHS) and Boston Medical Center (BMC): To work up his new left hilar mass, a computed tomography (CT) of the chest was ordered (Figure 1), which revealed an apical left lower lobe mass extending into the left hilum encasing part of the ascending aorta. Enlarged mediastinal subcentimeter paratracheal and superior mediastinal lymph nodes also were identified and the pattern raised the concern for lymphangitic carcinomatosis. Dr. Fine, what do you make of the CT findings?

fdp03806e46_f1.png



► Alan Fine, MD, Section of Pulmonary and Critical Care, VABHS and Professor of Medicine, Boston University School of Medicine: This mass had irregular edges with septal thickening, which may be why there was a concern for lymphangitic spread. There were no clear tissue planes to see if this process was invading the mediastinum. The mass was irregular, a single lesion, and proximal, making it consistent with a lung cancer. In fact, with his history of smoking, asbestos exposure, the numbers 1 to 10 diagnoses were lung cancer. The lack of demarcation of tissue planes supports this. There are some infections, classically actinomycosis, that do cross and invade anatomical barriers.1 But this looked like a primary lung cancer.

► Dr. Apte: The patient was referred to a pulmonologist where an additional history of night sweats and weight loss were noted. Dr. Fine, we have a patient with a newly identified lung mass, and while we have reason to suspect malignancy as you have already noted, there are many other etiologies to consider, including infections (histoplasmosis, cryptococcosis, bacterial abscess), inflammatory processes (sarcoidosis, rheumatoid nodule) and vasculitis (granulomatosis with polyangiitis). What should be the next step taken to make a diagnosis?


►Dr. Fine: For cancer specifically, we would like to both stage and make a diagnosis with one procedure. That’s part of the utility of a positron emission tomography (PET) scan: We can see lymph node involvement and stage the cancer. We must consider the patient’s comorbidities and the location of the lesion (ie, is it amenable to needle biopsy?). In this case, there are enlarged mediastinal lymph nodes, so one could perform a bronchoscopy with endobronchial ultrasound, which is a relatively noninvasive way to sample the lymph nodes to ideally stage and make a diagnosis as safely as possible. If we are considering infection, needle aspiration is not as sensitive.2

► Dr. Apte: The patient underwent a PET CT, which redemonstrated the lung mass with a loss of aortic fat plane suspicious for aortic involvement as well as lymph nodes in levels 7 and 8 that were concerning for malignancy. Subsequent bronchoscopy with biopsy and endobronchial ultrasound did not show evidence of malignancy; washing and brushing from the mass and lymph node specimens did not identify malignant cells. Benign respiratory mucosa with mild chronic inflammation was noted. Dr. Fine, given the nonspecific findings on the PET scan, negative findings on our bronchoscopy, and a negative biopsy, should we be satisfied that we have ruled out cancer?


► Dr. Fine: No, bronchoscopy has its limitations. It’s highly sensitive to the diagnosis of malignancy if you can see an endobronchial lesion, but we did not see one here. You can only go so far with the scope, and it’s not uncommon for us not to be able to make the diagnosis with bronchoscopy. Malignancy is still the most likely diagnosis, and we need to work this up further. I would perform another biopsy.


►Dr. Apte: Four weeks later, the patient presented with continued shortness of breath, fatigue, and fever. A repeat chest CT showed an opacity suggestive of pneumonia. Given the continued concern for cancer a CT-guided needle biopsy was performed and was once again negative for malignancy. The decision was made to pursue a video-assisted thorascopic surgery (VATS). Following the VATS, the patient developed rigors, fever, and tachycardia with new atrial fibrillation. While being evaluated hypercalcemia was identified, with further workup revealing a low parathyroid hormone (PTH) and low PTH-related peptide. Dr. Fine, the presence of hypercalcemia and a new arrythmia raised the possibility of sarcoidosis. Could this be sarcoidosis?


►Dr. Fine: Sarcoidosis is one of the great masqueraders in medicine. There is a type of sarcoidosis called nodular sarcoidosis where you see masslike distribution in the lung, but generally there are multiple masses and so this presentation would be atypical.3 There is also a phenomenon called sarcoidal reactions usually in the presence of cancer. Again, one tends to see multiple tiny lesions in the lung. It is certainly on the differential, but I would consider it to be less likely than cancer. It is also relatively common to develop atrial fibrillation after manipulation from a lung surgery.4 The other possibility I am concerned about is whether the mass is invading the mediastinum and involving the pericardium.


►Dr. Apte: Results from the VATS biopsy once again returned negative for malignancy and instead showed signs of focal micro-abscesses, atypical pneumocytes, and prominent neutrophils. A diagnosis of acute fibrinous organizing pneumonia (AFOP) was offered. Dr. Fine, what is AFOP?


►Dr. Fine: This is the first case of AFOP I had seen and probably the first case many in our department have seen. This is a relatively new entity with limited reported cases in the literature and is a pathological diagnosis originally recorded in autopsies from patients at the National Institutes of Health.5,6 Given the complexity of the lesion, the diagnosis is difficult to make. Most commonly, AFOP is associated with other systemic entities, most commonly hematologic malignancies like lymphomas and leukemias. It has also been associated with vasculitis and certain drugs. The mechanism is poorly understood, and although pneumonia is a part of the term, this just implies there is inflammation of the lung (ie, pneumonitis).


► Dr. Apte: Given the association of AFOP with underlying hematologic malignancies, an emphasis was placed on another finding: the patient’s increasing WBC count. The total WBC count had been 20 k/cm3 at the time of his lung mass discovery but had increased to > 40 k/cm3 with a differential of neutrophils > 80%. Flow cytometry was negative, and his peripheral smear was read as normal. Dr. Gilbert, what might explain this patient’s leukocytosis?

►Gary Gilbert, MD, Section of Hematology and Oncology, VABHS and Associate Professor of Medicine, Harvard Medical School (HMS): This patient had an elevated WBC for 4 months. Initially, the cause was likely lithium as this is known to cause a leukocytosis.7 More recently, the total WBC had increased and there were a couple of other abnormalities: A consistently elevated absolute monocyte count and a markedly elevated mature neutrophil count. These findings are consistent with a leukemoid reaction (ie, a WBC count > 50,000/µL from causes other than leukemia). The question becomes what is this a leukemoid reaction in response to? Once we have excluded a lung malignancy (a well-known common cause of a leukemoid reaction) we must consider a clonal myeloproliferative disorder. This is particularly true because many things that cause a leukemoid reaction (eg, lobar pneumonia) do not cause a persistently elevated neutrophil count. That this patient does have a persistently elevated neutrophil count suggests something abnormal about the neutrophils themselves.

► Dr. Apte: A bone marrow biopsy was performed. Dr. Gilbert, can you comment on this patient’s bone marrow biopsy and whether a myeloproliferative disorder may have played a role in the marked leukocytosis?

► Dr. Gilbert: The bone marrow biopsy was hyperplastic with myeloid predominance and normal maturation in all lineages. A deep sequencing analysis demonstrated the absence of chromosomal abnormalities or genetic mutations that are associated with myeloproliferative disorders. This excludes the possibility of a myeloproliferative disorder.


► Dr. Apte: The patient was started on 60 mg of prednisone daily, which led to marked improvement in his symptoms. He was discharged in stable condition but presented again with abdominal pain. A complete blood count once again showed increased WBC and new thrombocytosis. A CT angiogram (CTA) showed the prior lung mass with new signs of central necrosis. In the abdomen, new splenic and renal infarct were identified, along with signs of multiple arterial thrombi in the abdomen and internal and external iliac vessel wall thickening. These findings were read as concerning for a medium vessel vasculitis. Dr. Kaur, what are some of the imaging findings you would expect to see in vasculitis, and what about this patient’s CT is consistent with a medium vessel vasculitis?


► Maneet Kaur, MD, Section of Rheumatology, VABHS: Vasculitis is inflammation of the vessel wall that can lead to vascular injury and activation of the coagulation cascade. Sometimes these findings can be seen on imaging with evidence of stenosis, microaneurysms, and thrombosis distal to the stenosis. The nomenclature of vasculitis is not simple and has been revised many times. Medium-vessel vasculitis does not just affect the medium vessels (eg, visceral arteries) but can overlap with distal large vessels and smaller cutaneous vessels.

 

 

The first thing that comes to mind in this case is polyarteritis nodosa (PAN), an immune complex-mediated medium vessel disease that can involve large and small vessels in muscle, nerve, and skin. It can also present with masses.

►Dr. Apte: To address the arterial thrombi seen on CTA, arterial-brachial indices were obtained and showed bilateral occlusive disease in his distal extremities; findings that could be explained by vasculitis. His VATS biopsy pathology was reviewed for signs of vasculitis. Dr. Huang, can you review these slides for us please?

fdp03806e46_box.png

►Qin Huang, MD, Department of Pathology and Laboratory Medicine, VABHS and Assistant Professor of Pathology, HMS: This patient had a history of smoking, and there are many black pigment-laden macrophages present in the lung tissue. There were areas of hemorrhage and fibrin deposition and an overall picture of organizing pneumonia. At a lower power, you can see neutrophils everywhere, some in the form of micro-abscesses. The arterial walls did not show signs of vasculitis (Figure 2). Based on the clinical information and radiology findings, we suspected an acute infection-related pneumonia or a primary lung malignancy causing obstruction pneumonia. We suggested a rebiopsy of the lung mass to rule out a primary lung malignancy.

fdp03806e46_f2.png

► Dr. Apte: Given his CT findings, a serologic rheumatologic workup including antineutrophil cytoplasmic antibody, antinuclear antibody, and rheumatoid factors were sent and returned negative. The location of arterial wall inflammation on imaging made it unamenable for biopsy. The patient began to experience bilateral temporal pain, which raised the concern for a large vessel vasculitis, specifically giant cell arteritis. Bilateral temporal artery biopsies were obtained and were not suggestive of vasculitis. Dr. Kaur, we still do not have any serologic or biopsy confirmation to support a diagnosis of vasculitis. Can we still call this a vasculitis?

►Dr. Kaur: Few things can cause the picture that was seen radiographically. A few noninflammatory causes like fibromuscular dysplasia can cause both large and small vessel stenosis, but the elevations in erythrocyte sedimentation rate and C-reactive protein along with response to steroids makes these diagnoses unlikely. Sometimes we must make a clinical diagnosis for vasculitis based on the clinical picture, and I would feel comfortable treating this patient for vasculitis.

With that said, I remain concerned that this patient also has a malignancy. His WBC increased to > 70 k/cm3 and his calcium to > 13 mg/dL. These findings are hard to explain by vasculitis alone. There are cancer-associated vasculitis, and I suspect this is the explanation here.8 His temporal pain was pointing to large vessel involvement, so he could have an undifferentiated vasculitis.

► Dr. Apte: A decision was made to empirically treat with tocilizumab, an IL-6 receptor antagonist, for an undifferentiated autoimmune disease, in addition to tapering steroids. The patient underwent a second VATS, which again revealed AFOP but no signs of malignancy. Unfortunately, he developed multiple complications over the subsequent weeks and passed away. An autopsy was requested by family members and pathology from his lung mass was reviewed. (Figure 3). Dr. Huang, can you review these slides for us?


fdp03806e46_f3.png


► Dr. Huang: The left lung mass at autopsy shows nests, poorly formed clusters, and individuals of malignant neoplastic nonkeratinizing squamous cells embedded in a desmoplastic stroma in the mass center, consistent with poorly differentiated squamous cell carcinoma, and a circumscribed area of residual subacute organizing pneumonia with abscess, granulomatous changes, and early fibrosis at the periphery of this mass.

►Dr. Apte: Based on autopsy findings, the final diagnosis was poorly differentiated squamous cell carcinoma associated with subacute organizing pneumonia and medium vessel vasculitis, which presented with a severe leukocytosis ultimately thought to be a leukemoid reaction from his lung cancer.

References

1. Valour F, Sénéchal A, Dupieux C, et al. Actinomycosis: etiology, clinical features, diagnosis, treatment, and management. Infect Drug Resist. 2014;7:183-197. Published 2014 Jul 5. doi:10.2147/IDR.S39601

2. de Bazelaire C, Coffin A, Cohen-Zarade S, et al. CT-guided biopsies in lung infections in patients with haematological malignancies. Diagn Interv Imaging. 2013;94(2):202-215. doi:10.1016/j.diii.2012.12.008

3. Sweidan AJ, Singh NK, Stein A, Tanios M. Nodular sarcoidosis masquerading as cancer. Clin Med Insights Circ Respir Pulm Med. 2017;11:1179548417703123. Published 2017 Apr 12. doi:10.1177/1179548417703123

4. Bagheri R, Yousefi Y, Rezai R, Azemonfar V, Keshtan FG. Atrial fibrillation after lung surgery: incidence, underlying factors, and predictors. Kardiochir Torakochirurgia Pol. 2019;16(2):53-56. doi:10.5114/kitp.2019.86355

5. Lu J, Yin Q, Zha Y, et al. Acute fibrinous and organizing pneumonia: two case reports and literature review. BMC Pulm Med. 2019;19(1):141. Published 2019 Aug 5. doi:10.1186/s12890-019-0861-3

6. Beasley MB, Franks TJ, Galvin JR, Gochuico B, Travis WD. Acute fibrinous and organizing pneumonia: a histological pattern of lung injury and possible variant of diffuse alveolar damage. Arch Pathol Lab Med. 2002;126(9):1064-1070. doi:10.5858/2002-126-1064-AFAOP

7. Murphy DL, Goodwin FK, Bunney WE Jr. Leukocytosis during lithium treatment. Am J Psychiatry. 1971;127(11):1559-1561. doi:10.1176/ajp.127.11.1559

8. Fain O, Hamidou M, Cacoub P, et al. Vasculitides associated with malignancies: analysis of sixty patients. Arthritis Rheum. 2007;57(8):1473-1480. doi:10.1002/art.23085

References

1. Valour F, Sénéchal A, Dupieux C, et al. Actinomycosis: etiology, clinical features, diagnosis, treatment, and management. Infect Drug Resist. 2014;7:183-197. Published 2014 Jul 5. doi:10.2147/IDR.S39601

2. de Bazelaire C, Coffin A, Cohen-Zarade S, et al. CT-guided biopsies in lung infections in patients with haematological malignancies. Diagn Interv Imaging. 2013;94(2):202-215. doi:10.1016/j.diii.2012.12.008

3. Sweidan AJ, Singh NK, Stein A, Tanios M. Nodular sarcoidosis masquerading as cancer. Clin Med Insights Circ Respir Pulm Med. 2017;11:1179548417703123. Published 2017 Apr 12. doi:10.1177/1179548417703123

4. Bagheri R, Yousefi Y, Rezai R, Azemonfar V, Keshtan FG. Atrial fibrillation after lung surgery: incidence, underlying factors, and predictors. Kardiochir Torakochirurgia Pol. 2019;16(2):53-56. doi:10.5114/kitp.2019.86355

5. Lu J, Yin Q, Zha Y, et al. Acute fibrinous and organizing pneumonia: two case reports and literature review. BMC Pulm Med. 2019;19(1):141. Published 2019 Aug 5. doi:10.1186/s12890-019-0861-3

6. Beasley MB, Franks TJ, Galvin JR, Gochuico B, Travis WD. Acute fibrinous and organizing pneumonia: a histological pattern of lung injury and possible variant of diffuse alveolar damage. Arch Pathol Lab Med. 2002;126(9):1064-1070. doi:10.5858/2002-126-1064-AFAOP

7. Murphy DL, Goodwin FK, Bunney WE Jr. Leukocytosis during lithium treatment. Am J Psychiatry. 1971;127(11):1559-1561. doi:10.1176/ajp.127.11.1559

8. Fain O, Hamidou M, Cacoub P, et al. Vasculitides associated with malignancies: analysis of sixty patients. Arthritis Rheum. 2007;57(8):1473-1480. doi:10.1002/art.23085

Issue
Federal Practitioner - 38(6)a
Issue
Federal Practitioner - 38(6)a
Page Number
e46-e50
Page Number
e46-e50
Publications
Publications
Topics
Article Type
Sections
Disallow All Ads
Content Gating
No Gating (article Unlocked/Free)
Alternative CME
Disqus Comments
Default
Use ProPublica
Hide sidebar & use full width
render the right sidebar.
Conference Recap Checkbox
Not Conference Recap
Clinical Edge
Display the Slideshow in this Article
Medscape Article
Display survey writer
Reuters content
Disable Inline Native ads
WebMD Article
Article PDF Media

A Veteran Presenting With Chronic Progressive Dyspnea on Exertion

Article Type
Changed
Wed, 06/16/2021 - 11:54

Case Presentation: A 45-year-old US Coast Guard veteran with a medical history of asthma and chronic back pain was referred to the VA Boston Healthcare System (VABHS) for evaluation of progressive, unexplained dyspnea. Two years prior to presentation, the patient was an avid outdoorsman and highly active. At the time of his initial primary care physician (PCP) evaluation he reported dyspnea on exertion, and symptoms consistent with an upper respiratory tract infection (URTI) and a recent tick bite with an associated rash. He was treated with intranasal fluticasone and a course of antibiotics. His URTI symptoms and rash improved; however the dyspnea persisted and progressed over the ensuing winter and he was referred for pulmonary function testing. Additional history included a 20 pack-year history of smoking (resolved 10 years prior to the first VABHS clinical encounter) and a family history of premature coronary artery disease (CAD) in his father and 2 paternal uncles. He lived in northern New England where he previously worked as a cemetery groundskeeper.

►Kristopher Clark, MD, Chief Medical Resident, VABHS and Boston University/Boston Medical Center: Dr. Goldstein, how do you approach a patient who presents with progressive dyspnea?

►Ronald Goldstein, MD, Chief of Pulmonary and Critical Care VABHS: The evaluation of dyspnea is a common problem for pulmonary physicians. The sensation of dyspnea may originate from a wide variety of etiologies that involve pulmonary and cardiovascular disorders, neuromuscular impairment, deconditioning, or psychological issues. It is important to characterize the temporal pattern, severity, progression, relation to exertion or other triggers, the smoking history, environmental and occupational exposures to pulmonary toxins, associated symptoms, and the history of pulmonary problems.1

The physical examination may help to identify an airway or parenchymal disorder. Wheezing on chest examination would point to an obstructive defect and crackles to a possible restrictive problem, including pulmonary fibrosis. A cardiac examination should be performed to assess for evidence of heart failure, valvular heart disease, or the presence of loud P2 suggestive of pulmonary hypertension (PH). Laboratory studies, including complete blood counts are indicated.

A more complete pulmonary evaluation usually involves pulmonary function tests (PFTs), oximetry with exertion, and chest imaging. Additional cardiac testing might include electrocardiogram (ECG) and cardiac echocardiogram, followed by an exercise study, if needed. A B-natriuretic peptide determination could be considered if there is concern for congestive heart failure.2

►Dr. Clark: The initial physical examination was normal and laboratory tests were unrevealing. Given his history of asthma, he underwent spirometry testing (Table 1).

fdp03711522_t1.png

Dr. Goldstein, aside from unexplained dyspnea, what are other indications for spirometry and when should we consider ordering a full PFT, including lung volumes and diffusion capacity? Can you interpret this patient’s spirometry results?
 

►Dr. Goldstein: Spirometry is indicated to evaluate for a suspected obstructive defect. The test is usually performed with and without a bronchodilator to assess airway reactivity. A change in > 12% and > 200 mL suggests acute bronchodilator responsiveness. Periodic spirometry determinations are useful to assess the effect of medications or progression of disease. A reduction in forced vital capacity (FVC) may suggest a restrictive component. This possibility requires measure of lung volumes.

 

 

A full set of PFTs (ie, spirometry plus assessment of lung volumes and diffusion capacity) is required to evaluate the abnormalities associated with chronic obstructive pulmonary disease (COPD), interstitial diseases, vascular abnormalities (particularly PH), as well as for certain preoperative assessments. The single breath diffusing capacity for carbon monoxide is a measure of the overall capillary alveolar surface area of the lung. It is decreased in emphysema and interstitial disease as well as pulmonary vascular disorders. It would be particularly useful in this case as the spirometry studies were normal.

In this case, the normal FVC renders a significant restrictive disorder unlikely and his normal forced expiratory volume (FEV1) and FEV1/FVC make a significant obstructive disorder unlikely. He did not show any bronchodilator response; however, this finding does not exclude the presence of underlying asthma or reactive airway disease as patients often will not show a bronchodilator response at time of testing if they are not experiencing active bronchospasm or constriction. Further provocative testing with a methacholine challenge could be used to assess for reactive airway disease.

►Dr. Clark: The patient continued to have dyspnea when he returned to his PCP. Given his family history of premature CAD, an ECG was obtained that showed normal sinus rhythm at a rate of 70 beats per minute. A cardiology consult was placed, and he was referred for cardiac stress testing.

Dr. Maron, there are many forms of cardiac stress tests. In this case, the patient is referred for a stress test due his dyspnea. Does that symptom help you decide which test to order? How often does dyspnea present as an anginal equivalent in the absence of other cardiovascular symptoms or known cardiovascular disease?

►Bradley Maron, MD, Codirector, Pulmonary Vascular Disease Center, VABHS: In this case, stress testing should include a functional (ie, exercise) assessment if possible. Exercise capacity is a critical determinant of prognosis across the spectrum of cardiovascular disease and in a young person can be particularly informative on global health status. Furthermore, the chief complaint from this patient is dyspnea on exertion, and therefore, exercise testing is likely to be needed to reproduce or provoke the main symptom in this case. Estimates for dyspnea as a presenting symptom for ischemic heart disease vary but may be as high as 25%.3 It should be noted that cardiopulmonary exercise testing is useful for evaluating patients with unexplained dyspnea, as exercise hypoxemia, blunted decrease in VD/VT (ventilatory dead space/tidal volume), and evidence of a pulmonary mechanical limit to physical activity can inform the differential diagnosis.

►Dr. Clark: The patient underwent exercise treadmill testing and was able reach the target heart rate (> 85% age-predicted maximal heart rate) and achieve 11 metabolic equivalents. He had no chest pain or diagnostic ECG changes. The report made no mention of whether he experienced dyspnea during the test and was read as negative for exercise-induced ischemia.

He was seen by a cardiologist who noted an increased intensity S2 heart sound on examination without any other cardiopulmonary findings. It was noted that his symptoms occurred when tamping the ground or starting to walk up a hill but resolved with rest. It was also noted that his symptoms did not occur with gradual increased activity such as that performed during an exercise tolerance test. A 2-view chest X-ray was obtained and read as normal. Given the data from this evaluation thus far, the patient was told that his symptoms were most likely a result of his asthma exacerbated by dirt and dust exposure. Continued use of albuterol inhaler therapy was recommended, and no further diagnostic assessment was pursued.

Approximately 11 months later, the patient presented again to his PCP and reported progressive dyspnea. He had delayed seeking further care as he started to “feel like my symptoms were possibly in my head” given his prior negative workup. His symptoms had escalated drastically to the point where he felt short of breath with minimal exertion in addition to feeling sweaty, dizzy, fatigued, and having near-syncope when standing.

He was referred for a transthoracic echocardiogram (TTE) that revealed a left ventricular ejection fraction (LVEF) of 55 to 60% with diastolic relaxation abnormality and a normal-sized left atrium. The TTE also showed (qualitatively) a moderately dilated right ventricle with reduced systolic function, moderately severe tricuspid regurgitation, and severe elevation (> 60 mm Hg) in estimated right ventricular systolic pressure.

Dr. Maron, can you comment on how these findings may explain the patient’s symptoms? What differential diagnoses would you now consider?
 

 

 

►Dr. Maron: These echocardiography results exclude left ventricular systolic dysfunction or primary left-sided valvular disease at rest as a cause of the patient’s symptoms. In light of the patient’s prior normal stress test, high grade coronary disease in the absence of LV systolic dysfunction on echocardiography also seems unlikely. Estimated pulmonary artery systolic pressure > 60 mm Hg by echocardiography is highly suggestive of PH, but in and of itself does not diagnose PH nor inform pulmonary artery wedge pressure or pulmonary vascular resistance. Along with a direct measurement of pulmonary artery (PA) pressure, these data are needed to establish, classify, and prognosticate PH clinically.

►Dr. Clark: The patient was referred to a pulmonologist. His examination included bibasilar crackles and an enhanced P2 heart sound. A comprehensive pulmonary history was obtained, which noted his smoking history, possible asbestos exposure while serving in the Coast Guard, nighttime snoring without witnessed apnea events, and no personal or family history of thromboembolism or connective tissue disease.

Dr. Goldstein, is there anything in this patient’s history that could explain his symptoms and echocardiograph findings? Which tests would you order next?
 

►Dr. Goldstein: PH may be secondary to a wide variety of disorders including left heart disease (Group 2), advanced COPD, interstitial fibrosis, obstructive sleep apnea (OSA), or other lung diseases (Group 3), thromboembolic disorders (Group 4), and other systemic diseases such as sarcoidosis (Group 5). Group 1 is pulmonary arterial hypertension. (Table 2).

fdp03711522_t2.png

A right heart catheterization should be done to confirm the PA pressures estimated by echocardiogram. As to a cause, clinically he does not have heart failure. The limited smoking history and spirometry data do not support advanced COPD. He was noted to have crackles on physical examination suggesting an interstitial disorder. To assess the extent of interstitial disease, we would obtain a noncontrast computed tomography (CT) of the chest. The history of snoring suggesting the possibility of OSA indicating the need for overnight oximetry as significant nocturnal hypoxemia is a possible contributing cause to PH. A polysomnogram would be required to fully evaluate a sleep disturbance. The possible asbestos exposure is not likely a contributing factor as asbestosis requires significant exposure. We would obtain a ventilation/perfusion (V/Q) scan to rule out chronic thromboembolic disease. Targeted tests for causes of Group 5 disease should also be done.
 

►Dr. Clark: The impression from his pulmonologist was that the patient has severe PH, though the specific etiology was not yet known. Dr. Maron, can you review for us the pathophysiology behind PH and describe how the disease is classified?

►Dr. Maron: Elevated mean pulmonary artery pressure (> 20 mm Hg) diagnosed by supine right heart catheterization is the sine qua non of PH.4 However, this alone does not inform pathophysiology. As Dr. Goldstein noted, elevated PA pressure may be due to left heart disease, primary parenchymal lung disease/sleep-disordered breathing, in situ thrombotic remodeling of pulmonary arterioles following prior luminal pulmonary embolism, or in the setting of various specific predisposing conditions, such as sickle cell disease and sarcoidosis among others.5

 

 

Alternatively, pulmonary arterial hypertension (PAH) is suspected in patients with no identifiable cause of PH, pulmonary artery wedge pressure 15 mm Hg and pulmonary vascular resistance of 3.0 Wood units.6 Importantly, PAH is not synonymous with PH but is a circumspect PH disease subgroup. In turn, PAH may be idiopathic, hereditary, or associated with other select, predisposing disorders, namely systemic sclerosis. In PAH, the interplay between genetic and molecular factors results in effacement of distal pulmonary arterioles due to plexigenic, fibrotic, and/or concentric hypertrophic remodeling. Increased vascular resistance promotes early right ventricular dilation and impaired systolic function. As a result, patients with PAH are at particularly elevated risk for cor pulmonale.
 

►Dr. Clark: Overnight oximetry revealed baseline oxygen saturation of 94%, an oxygen nadir of 84% with a total of 7 minutes with oxygen < 90%. On a 6-minute walk test, the patient had a max heart rate of 116 and oxygen nadir of 93%. Chest CT with and without contrast showed no evidence of pulmonary emboli but noted mild emphysematous changes. A V/Q revealed no evidence of acute or chronic pulmonary thromboembolic disease. Coronary catheterization showed normal coronary anatomy without significant CAD. A right heart catheterization showed findings consistent with severe PH with normal left-sided filling pressures (Table 3).

fdp03711522_t3.png

The patient returned a normal antinuclear antibody, C-reactive protein, HIV, and liver function panel. Based on these findings, a presumptive diagnosis of group 1 PH (idiopathic PAH) was made. Given the severity of his right heart dysfunction, he was transferred to the cardiac care unit and initiated on epoprostenol.

Dr. Maron, can you review the different treatment options for idiopathic PAH and explain why epoprostenol was chosen for this patient?
 

►Dr. Maron: There are 14 US Food and Drug Administration-approved drug therapies for patients with PAH, which all target either nitric oxide signaling, endothelin receptors, or the prostacyclin pathway. In the current era, treatment-naïve patients with PAH are generally initiated on calcium channel antagonist therapy if there is evidence of vasoreactivity during right heart catheterization (following nitric oxide administration), dual therapy most often with an endothelin receptor antagonist and phosphodiesterase inhibitor, or parenteral prostacyclin therapy. Since < 5% of patients will demonstrate vasoreactivity, the decision at point of care in incident patients with PAH often focuses on dual oral therapy or initiation of parenteral prostacyclin therapy. In this case, the patient reported presyncope with minimal physical activity (eg, bending over or walking up stairs) and severely decreased functional status (ie, New York Heart Association Functional [NYHA] Class III – IV), and he had a cardiac index within the range of cardiogenic shock (< 2.0 L/min/m2). Collectively, this clinical profile is considered particularly high risk, therefore, a recommendation for parenteral continuous prostacyclin therapy was made.

Dr. Clark: The patient tolerated epoprostenol and reported improvement in his symptoms. He had a tunneled line catheter placed for continuous epoprostenol infusion. He was discharged home and scheduled for outpatient follow-up in a PH clinic. At 4 months following discharge, he was reporting steady clinical and functional improvement as well as improvement in his dyspnea. A second therapy (oral phosphodiesterase type-V inhibitor) was initiated and tolerated well. Overall, he reported resolution of presyncope, NYHA Functional Class II symptoms, and the absence of important drug effects.

References

1.. Manning HL, Schwartzstein RM. Pathophysiology of dyspnea. N Engl J Med. 1995;333(23):1547-1553. doi:10.1056/NEJM199512073332307

2. Parshall MB, Schwartzstein RM, Adams L, et al. An official American Thoracic Society statement: update on the mechanisms, assessment, and management of dyspnea. Am J Respir Crit Care Med. 2012;185(4):435-452. doi:10.1164/rccm.201111-2042ST

3. Phibbs B, Holmes RW, Lowe CR. Transient myocardial ischemia: the significance of dyspnea. Am J Med Sci. 1968;256(4):210-221. doi:10.1097/00000441-196810000-00002

4. Maron BA, Hess E, Maddox TM, et al. Association of borderline pulmonary hypertension with mortality and hospitalization in a large patient cohort: insights from the veterans affairs clinical assessment, reporting, and tracking program. Circulation. 2016;133(13):1240-1248. doi:10.1161/CIRCULATIONAHA.115.020207

5. Simonneau G, Montani D, Celermajer DS, et al. Haemodynamic definitions and updated clinical classification of pulmonary hypertension. Eur Respir J. 2019;53(1):1801913. Published 2019 Jan 24. doi:10.1183/13993003.01913-2018

6. Maron BA, Galiè N. Diagnosis, Treatment, and Clinical Management of Pulmonary Arterial Hypertension in the Contemporary Era: A Review. JAMA Cardiol. 2016;1(9):1056-1065. doi:10.1001/jamacardio.2016.4471

Article PDF
Author and Disclosure Information

Correspondence: Anthony Breu (anthony.breu@va.gov)

Author disclosures
The authors report no actual or potential conflicts of interest with regard to this article.

Disclaimer
The opinions expressed herein are those of the authors and do not necessarily reflect those of Federal Practitioner, Frontline Medical Communications Inc., the US Government, or any of its agencies.

Issue
Federal Practitioner - 37(11)a
Publications
Topics
Page Number
522-526
Sections
Author and Disclosure Information

Correspondence: Anthony Breu (anthony.breu@va.gov)

Author disclosures
The authors report no actual or potential conflicts of interest with regard to this article.

Disclaimer
The opinions expressed herein are those of the authors and do not necessarily reflect those of Federal Practitioner, Frontline Medical Communications Inc., the US Government, or any of its agencies.

Author and Disclosure Information

Correspondence: Anthony Breu (anthony.breu@va.gov)

Author disclosures
The authors report no actual or potential conflicts of interest with regard to this article.

Disclaimer
The opinions expressed herein are those of the authors and do not necessarily reflect those of Federal Practitioner, Frontline Medical Communications Inc., the US Government, or any of its agencies.

Article PDF
Article PDF

Case Presentation: A 45-year-old US Coast Guard veteran with a medical history of asthma and chronic back pain was referred to the VA Boston Healthcare System (VABHS) for evaluation of progressive, unexplained dyspnea. Two years prior to presentation, the patient was an avid outdoorsman and highly active. At the time of his initial primary care physician (PCP) evaluation he reported dyspnea on exertion, and symptoms consistent with an upper respiratory tract infection (URTI) and a recent tick bite with an associated rash. He was treated with intranasal fluticasone and a course of antibiotics. His URTI symptoms and rash improved; however the dyspnea persisted and progressed over the ensuing winter and he was referred for pulmonary function testing. Additional history included a 20 pack-year history of smoking (resolved 10 years prior to the first VABHS clinical encounter) and a family history of premature coronary artery disease (CAD) in his father and 2 paternal uncles. He lived in northern New England where he previously worked as a cemetery groundskeeper.

►Kristopher Clark, MD, Chief Medical Resident, VABHS and Boston University/Boston Medical Center: Dr. Goldstein, how do you approach a patient who presents with progressive dyspnea?

►Ronald Goldstein, MD, Chief of Pulmonary and Critical Care VABHS: The evaluation of dyspnea is a common problem for pulmonary physicians. The sensation of dyspnea may originate from a wide variety of etiologies that involve pulmonary and cardiovascular disorders, neuromuscular impairment, deconditioning, or psychological issues. It is important to characterize the temporal pattern, severity, progression, relation to exertion or other triggers, the smoking history, environmental and occupational exposures to pulmonary toxins, associated symptoms, and the history of pulmonary problems.1

The physical examination may help to identify an airway or parenchymal disorder. Wheezing on chest examination would point to an obstructive defect and crackles to a possible restrictive problem, including pulmonary fibrosis. A cardiac examination should be performed to assess for evidence of heart failure, valvular heart disease, or the presence of loud P2 suggestive of pulmonary hypertension (PH). Laboratory studies, including complete blood counts are indicated.

A more complete pulmonary evaluation usually involves pulmonary function tests (PFTs), oximetry with exertion, and chest imaging. Additional cardiac testing might include electrocardiogram (ECG) and cardiac echocardiogram, followed by an exercise study, if needed. A B-natriuretic peptide determination could be considered if there is concern for congestive heart failure.2

►Dr. Clark: The initial physical examination was normal and laboratory tests were unrevealing. Given his history of asthma, he underwent spirometry testing (Table 1).

fdp03711522_t1.png

Dr. Goldstein, aside from unexplained dyspnea, what are other indications for spirometry and when should we consider ordering a full PFT, including lung volumes and diffusion capacity? Can you interpret this patient’s spirometry results?
 

►Dr. Goldstein: Spirometry is indicated to evaluate for a suspected obstructive defect. The test is usually performed with and without a bronchodilator to assess airway reactivity. A change in > 12% and > 200 mL suggests acute bronchodilator responsiveness. Periodic spirometry determinations are useful to assess the effect of medications or progression of disease. A reduction in forced vital capacity (FVC) may suggest a restrictive component. This possibility requires measure of lung volumes.

 

 

A full set of PFTs (ie, spirometry plus assessment of lung volumes and diffusion capacity) is required to evaluate the abnormalities associated with chronic obstructive pulmonary disease (COPD), interstitial diseases, vascular abnormalities (particularly PH), as well as for certain preoperative assessments. The single breath diffusing capacity for carbon monoxide is a measure of the overall capillary alveolar surface area of the lung. It is decreased in emphysema and interstitial disease as well as pulmonary vascular disorders. It would be particularly useful in this case as the spirometry studies were normal.

In this case, the normal FVC renders a significant restrictive disorder unlikely and his normal forced expiratory volume (FEV1) and FEV1/FVC make a significant obstructive disorder unlikely. He did not show any bronchodilator response; however, this finding does not exclude the presence of underlying asthma or reactive airway disease as patients often will not show a bronchodilator response at time of testing if they are not experiencing active bronchospasm or constriction. Further provocative testing with a methacholine challenge could be used to assess for reactive airway disease.

►Dr. Clark: The patient continued to have dyspnea when he returned to his PCP. Given his family history of premature CAD, an ECG was obtained that showed normal sinus rhythm at a rate of 70 beats per minute. A cardiology consult was placed, and he was referred for cardiac stress testing.

Dr. Maron, there are many forms of cardiac stress tests. In this case, the patient is referred for a stress test due his dyspnea. Does that symptom help you decide which test to order? How often does dyspnea present as an anginal equivalent in the absence of other cardiovascular symptoms or known cardiovascular disease?

►Bradley Maron, MD, Codirector, Pulmonary Vascular Disease Center, VABHS: In this case, stress testing should include a functional (ie, exercise) assessment if possible. Exercise capacity is a critical determinant of prognosis across the spectrum of cardiovascular disease and in a young person can be particularly informative on global health status. Furthermore, the chief complaint from this patient is dyspnea on exertion, and therefore, exercise testing is likely to be needed to reproduce or provoke the main symptom in this case. Estimates for dyspnea as a presenting symptom for ischemic heart disease vary but may be as high as 25%.3 It should be noted that cardiopulmonary exercise testing is useful for evaluating patients with unexplained dyspnea, as exercise hypoxemia, blunted decrease in VD/VT (ventilatory dead space/tidal volume), and evidence of a pulmonary mechanical limit to physical activity can inform the differential diagnosis.

►Dr. Clark: The patient underwent exercise treadmill testing and was able reach the target heart rate (> 85% age-predicted maximal heart rate) and achieve 11 metabolic equivalents. He had no chest pain or diagnostic ECG changes. The report made no mention of whether he experienced dyspnea during the test and was read as negative for exercise-induced ischemia.

He was seen by a cardiologist who noted an increased intensity S2 heart sound on examination without any other cardiopulmonary findings. It was noted that his symptoms occurred when tamping the ground or starting to walk up a hill but resolved with rest. It was also noted that his symptoms did not occur with gradual increased activity such as that performed during an exercise tolerance test. A 2-view chest X-ray was obtained and read as normal. Given the data from this evaluation thus far, the patient was told that his symptoms were most likely a result of his asthma exacerbated by dirt and dust exposure. Continued use of albuterol inhaler therapy was recommended, and no further diagnostic assessment was pursued.

Approximately 11 months later, the patient presented again to his PCP and reported progressive dyspnea. He had delayed seeking further care as he started to “feel like my symptoms were possibly in my head” given his prior negative workup. His symptoms had escalated drastically to the point where he felt short of breath with minimal exertion in addition to feeling sweaty, dizzy, fatigued, and having near-syncope when standing.

He was referred for a transthoracic echocardiogram (TTE) that revealed a left ventricular ejection fraction (LVEF) of 55 to 60% with diastolic relaxation abnormality and a normal-sized left atrium. The TTE also showed (qualitatively) a moderately dilated right ventricle with reduced systolic function, moderately severe tricuspid regurgitation, and severe elevation (> 60 mm Hg) in estimated right ventricular systolic pressure.

Dr. Maron, can you comment on how these findings may explain the patient’s symptoms? What differential diagnoses would you now consider?
 

 

 

►Dr. Maron: These echocardiography results exclude left ventricular systolic dysfunction or primary left-sided valvular disease at rest as a cause of the patient’s symptoms. In light of the patient’s prior normal stress test, high grade coronary disease in the absence of LV systolic dysfunction on echocardiography also seems unlikely. Estimated pulmonary artery systolic pressure > 60 mm Hg by echocardiography is highly suggestive of PH, but in and of itself does not diagnose PH nor inform pulmonary artery wedge pressure or pulmonary vascular resistance. Along with a direct measurement of pulmonary artery (PA) pressure, these data are needed to establish, classify, and prognosticate PH clinically.

►Dr. Clark: The patient was referred to a pulmonologist. His examination included bibasilar crackles and an enhanced P2 heart sound. A comprehensive pulmonary history was obtained, which noted his smoking history, possible asbestos exposure while serving in the Coast Guard, nighttime snoring without witnessed apnea events, and no personal or family history of thromboembolism or connective tissue disease.

Dr. Goldstein, is there anything in this patient’s history that could explain his symptoms and echocardiograph findings? Which tests would you order next?
 

►Dr. Goldstein: PH may be secondary to a wide variety of disorders including left heart disease (Group 2), advanced COPD, interstitial fibrosis, obstructive sleep apnea (OSA), or other lung diseases (Group 3), thromboembolic disorders (Group 4), and other systemic diseases such as sarcoidosis (Group 5). Group 1 is pulmonary arterial hypertension. (Table 2).

fdp03711522_t2.png

A right heart catheterization should be done to confirm the PA pressures estimated by echocardiogram. As to a cause, clinically he does not have heart failure. The limited smoking history and spirometry data do not support advanced COPD. He was noted to have crackles on physical examination suggesting an interstitial disorder. To assess the extent of interstitial disease, we would obtain a noncontrast computed tomography (CT) of the chest. The history of snoring suggesting the possibility of OSA indicating the need for overnight oximetry as significant nocturnal hypoxemia is a possible contributing cause to PH. A polysomnogram would be required to fully evaluate a sleep disturbance. The possible asbestos exposure is not likely a contributing factor as asbestosis requires significant exposure. We would obtain a ventilation/perfusion (V/Q) scan to rule out chronic thromboembolic disease. Targeted tests for causes of Group 5 disease should also be done.
 

►Dr. Clark: The impression from his pulmonologist was that the patient has severe PH, though the specific etiology was not yet known. Dr. Maron, can you review for us the pathophysiology behind PH and describe how the disease is classified?

►Dr. Maron: Elevated mean pulmonary artery pressure (> 20 mm Hg) diagnosed by supine right heart catheterization is the sine qua non of PH.4 However, this alone does not inform pathophysiology. As Dr. Goldstein noted, elevated PA pressure may be due to left heart disease, primary parenchymal lung disease/sleep-disordered breathing, in situ thrombotic remodeling of pulmonary arterioles following prior luminal pulmonary embolism, or in the setting of various specific predisposing conditions, such as sickle cell disease and sarcoidosis among others.5

 

 

Alternatively, pulmonary arterial hypertension (PAH) is suspected in patients with no identifiable cause of PH, pulmonary artery wedge pressure 15 mm Hg and pulmonary vascular resistance of 3.0 Wood units.6 Importantly, PAH is not synonymous with PH but is a circumspect PH disease subgroup. In turn, PAH may be idiopathic, hereditary, or associated with other select, predisposing disorders, namely systemic sclerosis. In PAH, the interplay between genetic and molecular factors results in effacement of distal pulmonary arterioles due to plexigenic, fibrotic, and/or concentric hypertrophic remodeling. Increased vascular resistance promotes early right ventricular dilation and impaired systolic function. As a result, patients with PAH are at particularly elevated risk for cor pulmonale.
 

►Dr. Clark: Overnight oximetry revealed baseline oxygen saturation of 94%, an oxygen nadir of 84% with a total of 7 minutes with oxygen < 90%. On a 6-minute walk test, the patient had a max heart rate of 116 and oxygen nadir of 93%. Chest CT with and without contrast showed no evidence of pulmonary emboli but noted mild emphysematous changes. A V/Q revealed no evidence of acute or chronic pulmonary thromboembolic disease. Coronary catheterization showed normal coronary anatomy without significant CAD. A right heart catheterization showed findings consistent with severe PH with normal left-sided filling pressures (Table 3).

fdp03711522_t3.png

The patient returned a normal antinuclear antibody, C-reactive protein, HIV, and liver function panel. Based on these findings, a presumptive diagnosis of group 1 PH (idiopathic PAH) was made. Given the severity of his right heart dysfunction, he was transferred to the cardiac care unit and initiated on epoprostenol.

Dr. Maron, can you review the different treatment options for idiopathic PAH and explain why epoprostenol was chosen for this patient?
 

►Dr. Maron: There are 14 US Food and Drug Administration-approved drug therapies for patients with PAH, which all target either nitric oxide signaling, endothelin receptors, or the prostacyclin pathway. In the current era, treatment-naïve patients with PAH are generally initiated on calcium channel antagonist therapy if there is evidence of vasoreactivity during right heart catheterization (following nitric oxide administration), dual therapy most often with an endothelin receptor antagonist and phosphodiesterase inhibitor, or parenteral prostacyclin therapy. Since < 5% of patients will demonstrate vasoreactivity, the decision at point of care in incident patients with PAH often focuses on dual oral therapy or initiation of parenteral prostacyclin therapy. In this case, the patient reported presyncope with minimal physical activity (eg, bending over or walking up stairs) and severely decreased functional status (ie, New York Heart Association Functional [NYHA] Class III – IV), and he had a cardiac index within the range of cardiogenic shock (< 2.0 L/min/m2). Collectively, this clinical profile is considered particularly high risk, therefore, a recommendation for parenteral continuous prostacyclin therapy was made.

Dr. Clark: The patient tolerated epoprostenol and reported improvement in his symptoms. He had a tunneled line catheter placed for continuous epoprostenol infusion. He was discharged home and scheduled for outpatient follow-up in a PH clinic. At 4 months following discharge, he was reporting steady clinical and functional improvement as well as improvement in his dyspnea. A second therapy (oral phosphodiesterase type-V inhibitor) was initiated and tolerated well. Overall, he reported resolution of presyncope, NYHA Functional Class II symptoms, and the absence of important drug effects.

Case Presentation: A 45-year-old US Coast Guard veteran with a medical history of asthma and chronic back pain was referred to the VA Boston Healthcare System (VABHS) for evaluation of progressive, unexplained dyspnea. Two years prior to presentation, the patient was an avid outdoorsman and highly active. At the time of his initial primary care physician (PCP) evaluation he reported dyspnea on exertion, and symptoms consistent with an upper respiratory tract infection (URTI) and a recent tick bite with an associated rash. He was treated with intranasal fluticasone and a course of antibiotics. His URTI symptoms and rash improved; however the dyspnea persisted and progressed over the ensuing winter and he was referred for pulmonary function testing. Additional history included a 20 pack-year history of smoking (resolved 10 years prior to the first VABHS clinical encounter) and a family history of premature coronary artery disease (CAD) in his father and 2 paternal uncles. He lived in northern New England where he previously worked as a cemetery groundskeeper.

►Kristopher Clark, MD, Chief Medical Resident, VABHS and Boston University/Boston Medical Center: Dr. Goldstein, how do you approach a patient who presents with progressive dyspnea?

►Ronald Goldstein, MD, Chief of Pulmonary and Critical Care VABHS: The evaluation of dyspnea is a common problem for pulmonary physicians. The sensation of dyspnea may originate from a wide variety of etiologies that involve pulmonary and cardiovascular disorders, neuromuscular impairment, deconditioning, or psychological issues. It is important to characterize the temporal pattern, severity, progression, relation to exertion or other triggers, the smoking history, environmental and occupational exposures to pulmonary toxins, associated symptoms, and the history of pulmonary problems.1

The physical examination may help to identify an airway or parenchymal disorder. Wheezing on chest examination would point to an obstructive defect and crackles to a possible restrictive problem, including pulmonary fibrosis. A cardiac examination should be performed to assess for evidence of heart failure, valvular heart disease, or the presence of loud P2 suggestive of pulmonary hypertension (PH). Laboratory studies, including complete blood counts are indicated.

A more complete pulmonary evaluation usually involves pulmonary function tests (PFTs), oximetry with exertion, and chest imaging. Additional cardiac testing might include electrocardiogram (ECG) and cardiac echocardiogram, followed by an exercise study, if needed. A B-natriuretic peptide determination could be considered if there is concern for congestive heart failure.2

►Dr. Clark: The initial physical examination was normal and laboratory tests were unrevealing. Given his history of asthma, he underwent spirometry testing (Table 1).

fdp03711522_t1.png

Dr. Goldstein, aside from unexplained dyspnea, what are other indications for spirometry and when should we consider ordering a full PFT, including lung volumes and diffusion capacity? Can you interpret this patient’s spirometry results?
 

►Dr. Goldstein: Spirometry is indicated to evaluate for a suspected obstructive defect. The test is usually performed with and without a bronchodilator to assess airway reactivity. A change in > 12% and > 200 mL suggests acute bronchodilator responsiveness. Periodic spirometry determinations are useful to assess the effect of medications or progression of disease. A reduction in forced vital capacity (FVC) may suggest a restrictive component. This possibility requires measure of lung volumes.

 

 

A full set of PFTs (ie, spirometry plus assessment of lung volumes and diffusion capacity) is required to evaluate the abnormalities associated with chronic obstructive pulmonary disease (COPD), interstitial diseases, vascular abnormalities (particularly PH), as well as for certain preoperative assessments. The single breath diffusing capacity for carbon monoxide is a measure of the overall capillary alveolar surface area of the lung. It is decreased in emphysema and interstitial disease as well as pulmonary vascular disorders. It would be particularly useful in this case as the spirometry studies were normal.

In this case, the normal FVC renders a significant restrictive disorder unlikely and his normal forced expiratory volume (FEV1) and FEV1/FVC make a significant obstructive disorder unlikely. He did not show any bronchodilator response; however, this finding does not exclude the presence of underlying asthma or reactive airway disease as patients often will not show a bronchodilator response at time of testing if they are not experiencing active bronchospasm or constriction. Further provocative testing with a methacholine challenge could be used to assess for reactive airway disease.

►Dr. Clark: The patient continued to have dyspnea when he returned to his PCP. Given his family history of premature CAD, an ECG was obtained that showed normal sinus rhythm at a rate of 70 beats per minute. A cardiology consult was placed, and he was referred for cardiac stress testing.

Dr. Maron, there are many forms of cardiac stress tests. In this case, the patient is referred for a stress test due his dyspnea. Does that symptom help you decide which test to order? How often does dyspnea present as an anginal equivalent in the absence of other cardiovascular symptoms or known cardiovascular disease?

►Bradley Maron, MD, Codirector, Pulmonary Vascular Disease Center, VABHS: In this case, stress testing should include a functional (ie, exercise) assessment if possible. Exercise capacity is a critical determinant of prognosis across the spectrum of cardiovascular disease and in a young person can be particularly informative on global health status. Furthermore, the chief complaint from this patient is dyspnea on exertion, and therefore, exercise testing is likely to be needed to reproduce or provoke the main symptom in this case. Estimates for dyspnea as a presenting symptom for ischemic heart disease vary but may be as high as 25%.3 It should be noted that cardiopulmonary exercise testing is useful for evaluating patients with unexplained dyspnea, as exercise hypoxemia, blunted decrease in VD/VT (ventilatory dead space/tidal volume), and evidence of a pulmonary mechanical limit to physical activity can inform the differential diagnosis.

►Dr. Clark: The patient underwent exercise treadmill testing and was able reach the target heart rate (> 85% age-predicted maximal heart rate) and achieve 11 metabolic equivalents. He had no chest pain or diagnostic ECG changes. The report made no mention of whether he experienced dyspnea during the test and was read as negative for exercise-induced ischemia.

He was seen by a cardiologist who noted an increased intensity S2 heart sound on examination without any other cardiopulmonary findings. It was noted that his symptoms occurred when tamping the ground or starting to walk up a hill but resolved with rest. It was also noted that his symptoms did not occur with gradual increased activity such as that performed during an exercise tolerance test. A 2-view chest X-ray was obtained and read as normal. Given the data from this evaluation thus far, the patient was told that his symptoms were most likely a result of his asthma exacerbated by dirt and dust exposure. Continued use of albuterol inhaler therapy was recommended, and no further diagnostic assessment was pursued.

Approximately 11 months later, the patient presented again to his PCP and reported progressive dyspnea. He had delayed seeking further care as he started to “feel like my symptoms were possibly in my head” given his prior negative workup. His symptoms had escalated drastically to the point where he felt short of breath with minimal exertion in addition to feeling sweaty, dizzy, fatigued, and having near-syncope when standing.

He was referred for a transthoracic echocardiogram (TTE) that revealed a left ventricular ejection fraction (LVEF) of 55 to 60% with diastolic relaxation abnormality and a normal-sized left atrium. The TTE also showed (qualitatively) a moderately dilated right ventricle with reduced systolic function, moderately severe tricuspid regurgitation, and severe elevation (> 60 mm Hg) in estimated right ventricular systolic pressure.

Dr. Maron, can you comment on how these findings may explain the patient’s symptoms? What differential diagnoses would you now consider?
 

 

 

►Dr. Maron: These echocardiography results exclude left ventricular systolic dysfunction or primary left-sided valvular disease at rest as a cause of the patient’s symptoms. In light of the patient’s prior normal stress test, high grade coronary disease in the absence of LV systolic dysfunction on echocardiography also seems unlikely. Estimated pulmonary artery systolic pressure > 60 mm Hg by echocardiography is highly suggestive of PH, but in and of itself does not diagnose PH nor inform pulmonary artery wedge pressure or pulmonary vascular resistance. Along with a direct measurement of pulmonary artery (PA) pressure, these data are needed to establish, classify, and prognosticate PH clinically.

►Dr. Clark: The patient was referred to a pulmonologist. His examination included bibasilar crackles and an enhanced P2 heart sound. A comprehensive pulmonary history was obtained, which noted his smoking history, possible asbestos exposure while serving in the Coast Guard, nighttime snoring without witnessed apnea events, and no personal or family history of thromboembolism or connective tissue disease.

Dr. Goldstein, is there anything in this patient’s history that could explain his symptoms and echocardiograph findings? Which tests would you order next?
 

►Dr. Goldstein: PH may be secondary to a wide variety of disorders including left heart disease (Group 2), advanced COPD, interstitial fibrosis, obstructive sleep apnea (OSA), or other lung diseases (Group 3), thromboembolic disorders (Group 4), and other systemic diseases such as sarcoidosis (Group 5). Group 1 is pulmonary arterial hypertension. (Table 2).

fdp03711522_t2.png

A right heart catheterization should be done to confirm the PA pressures estimated by echocardiogram. As to a cause, clinically he does not have heart failure. The limited smoking history and spirometry data do not support advanced COPD. He was noted to have crackles on physical examination suggesting an interstitial disorder. To assess the extent of interstitial disease, we would obtain a noncontrast computed tomography (CT) of the chest. The history of snoring suggesting the possibility of OSA indicating the need for overnight oximetry as significant nocturnal hypoxemia is a possible contributing cause to PH. A polysomnogram would be required to fully evaluate a sleep disturbance. The possible asbestos exposure is not likely a contributing factor as asbestosis requires significant exposure. We would obtain a ventilation/perfusion (V/Q) scan to rule out chronic thromboembolic disease. Targeted tests for causes of Group 5 disease should also be done.
 

►Dr. Clark: The impression from his pulmonologist was that the patient has severe PH, though the specific etiology was not yet known. Dr. Maron, can you review for us the pathophysiology behind PH and describe how the disease is classified?

►Dr. Maron: Elevated mean pulmonary artery pressure (> 20 mm Hg) diagnosed by supine right heart catheterization is the sine qua non of PH.4 However, this alone does not inform pathophysiology. As Dr. Goldstein noted, elevated PA pressure may be due to left heart disease, primary parenchymal lung disease/sleep-disordered breathing, in situ thrombotic remodeling of pulmonary arterioles following prior luminal pulmonary embolism, or in the setting of various specific predisposing conditions, such as sickle cell disease and sarcoidosis among others.5

 

 

Alternatively, pulmonary arterial hypertension (PAH) is suspected in patients with no identifiable cause of PH, pulmonary artery wedge pressure 15 mm Hg and pulmonary vascular resistance of 3.0 Wood units.6 Importantly, PAH is not synonymous with PH but is a circumspect PH disease subgroup. In turn, PAH may be idiopathic, hereditary, or associated with other select, predisposing disorders, namely systemic sclerosis. In PAH, the interplay between genetic and molecular factors results in effacement of distal pulmonary arterioles due to plexigenic, fibrotic, and/or concentric hypertrophic remodeling. Increased vascular resistance promotes early right ventricular dilation and impaired systolic function. As a result, patients with PAH are at particularly elevated risk for cor pulmonale.
 

►Dr. Clark: Overnight oximetry revealed baseline oxygen saturation of 94%, an oxygen nadir of 84% with a total of 7 minutes with oxygen < 90%. On a 6-minute walk test, the patient had a max heart rate of 116 and oxygen nadir of 93%. Chest CT with and without contrast showed no evidence of pulmonary emboli but noted mild emphysematous changes. A V/Q revealed no evidence of acute or chronic pulmonary thromboembolic disease. Coronary catheterization showed normal coronary anatomy without significant CAD. A right heart catheterization showed findings consistent with severe PH with normal left-sided filling pressures (Table 3).

fdp03711522_t3.png

The patient returned a normal antinuclear antibody, C-reactive protein, HIV, and liver function panel. Based on these findings, a presumptive diagnosis of group 1 PH (idiopathic PAH) was made. Given the severity of his right heart dysfunction, he was transferred to the cardiac care unit and initiated on epoprostenol.

Dr. Maron, can you review the different treatment options for idiopathic PAH and explain why epoprostenol was chosen for this patient?
 

►Dr. Maron: There are 14 US Food and Drug Administration-approved drug therapies for patients with PAH, which all target either nitric oxide signaling, endothelin receptors, or the prostacyclin pathway. In the current era, treatment-naïve patients with PAH are generally initiated on calcium channel antagonist therapy if there is evidence of vasoreactivity during right heart catheterization (following nitric oxide administration), dual therapy most often with an endothelin receptor antagonist and phosphodiesterase inhibitor, or parenteral prostacyclin therapy. Since < 5% of patients will demonstrate vasoreactivity, the decision at point of care in incident patients with PAH often focuses on dual oral therapy or initiation of parenteral prostacyclin therapy. In this case, the patient reported presyncope with minimal physical activity (eg, bending over or walking up stairs) and severely decreased functional status (ie, New York Heart Association Functional [NYHA] Class III – IV), and he had a cardiac index within the range of cardiogenic shock (< 2.0 L/min/m2). Collectively, this clinical profile is considered particularly high risk, therefore, a recommendation for parenteral continuous prostacyclin therapy was made.

Dr. Clark: The patient tolerated epoprostenol and reported improvement in his symptoms. He had a tunneled line catheter placed for continuous epoprostenol infusion. He was discharged home and scheduled for outpatient follow-up in a PH clinic. At 4 months following discharge, he was reporting steady clinical and functional improvement as well as improvement in his dyspnea. A second therapy (oral phosphodiesterase type-V inhibitor) was initiated and tolerated well. Overall, he reported resolution of presyncope, NYHA Functional Class II symptoms, and the absence of important drug effects.

References

1.. Manning HL, Schwartzstein RM. Pathophysiology of dyspnea. N Engl J Med. 1995;333(23):1547-1553. doi:10.1056/NEJM199512073332307

2. Parshall MB, Schwartzstein RM, Adams L, et al. An official American Thoracic Society statement: update on the mechanisms, assessment, and management of dyspnea. Am J Respir Crit Care Med. 2012;185(4):435-452. doi:10.1164/rccm.201111-2042ST

3. Phibbs B, Holmes RW, Lowe CR. Transient myocardial ischemia: the significance of dyspnea. Am J Med Sci. 1968;256(4):210-221. doi:10.1097/00000441-196810000-00002

4. Maron BA, Hess E, Maddox TM, et al. Association of borderline pulmonary hypertension with mortality and hospitalization in a large patient cohort: insights from the veterans affairs clinical assessment, reporting, and tracking program. Circulation. 2016;133(13):1240-1248. doi:10.1161/CIRCULATIONAHA.115.020207

5. Simonneau G, Montani D, Celermajer DS, et al. Haemodynamic definitions and updated clinical classification of pulmonary hypertension. Eur Respir J. 2019;53(1):1801913. Published 2019 Jan 24. doi:10.1183/13993003.01913-2018

6. Maron BA, Galiè N. Diagnosis, Treatment, and Clinical Management of Pulmonary Arterial Hypertension in the Contemporary Era: A Review. JAMA Cardiol. 2016;1(9):1056-1065. doi:10.1001/jamacardio.2016.4471

References

1.. Manning HL, Schwartzstein RM. Pathophysiology of dyspnea. N Engl J Med. 1995;333(23):1547-1553. doi:10.1056/NEJM199512073332307

2. Parshall MB, Schwartzstein RM, Adams L, et al. An official American Thoracic Society statement: update on the mechanisms, assessment, and management of dyspnea. Am J Respir Crit Care Med. 2012;185(4):435-452. doi:10.1164/rccm.201111-2042ST

3. Phibbs B, Holmes RW, Lowe CR. Transient myocardial ischemia: the significance of dyspnea. Am J Med Sci. 1968;256(4):210-221. doi:10.1097/00000441-196810000-00002

4. Maron BA, Hess E, Maddox TM, et al. Association of borderline pulmonary hypertension with mortality and hospitalization in a large patient cohort: insights from the veterans affairs clinical assessment, reporting, and tracking program. Circulation. 2016;133(13):1240-1248. doi:10.1161/CIRCULATIONAHA.115.020207

5. Simonneau G, Montani D, Celermajer DS, et al. Haemodynamic definitions and updated clinical classification of pulmonary hypertension. Eur Respir J. 2019;53(1):1801913. Published 2019 Jan 24. doi:10.1183/13993003.01913-2018

6. Maron BA, Galiè N. Diagnosis, Treatment, and Clinical Management of Pulmonary Arterial Hypertension in the Contemporary Era: A Review. JAMA Cardiol. 2016;1(9):1056-1065. doi:10.1001/jamacardio.2016.4471

Issue
Federal Practitioner - 37(11)a
Issue
Federal Practitioner - 37(11)a
Page Number
522-526
Page Number
522-526
Publications
Publications
Topics
Article Type
Sections
Disallow All Ads
Content Gating
No Gating (article Unlocked/Free)
Alternative CME
Disqus Comments
Default
Use ProPublica
Hide sidebar & use full width
render the right sidebar.
Conference Recap Checkbox
Not Conference Recap
Clinical Edge
Display the Slideshow in this Article
Medscape Article
Display survey writer
Reuters content
Disable Inline Native ads
WebMD Article
Article PDF Media

A Veteran Presenting With Altered Mental Status and Clonus

Article Type
Changed
Fri, 04/10/2020 - 10:13

►Zachary Reese, MD, Chief Medical Resident, VABHS and Beth Israel Deaconess Medical Center (BIDMC):Dr. Weller, the differential diagnosis for altered mental status is quite broad. How does the presence of clonus change or focus your approach to altered mental status?

►Jason Weller, MD, Instructor of Neurology, Boston Medical Center (BMC) and VABHS:The presence of clonus does not significantly narrow the differential. It does, however, suggest a central component to the patient’s altered mental status. Specifically, it implies that the underlying process, whether systemic or neurologic, interferes with central nervous system (CNS) control of the neuromuscular system.1 The differential is still quite broad and includes metabolic derangements (eg, uremia, electrolyte disturbances, hypercarbia, and thyroid dysfunction), medication toxicity from olanzapine or duloxetine, and vascular processes (eg, CNS vasculitis). Infectious etiologies, both within the CNS and systemically, can cause encephalopathy, as can autoimmune processes, such as immune-mediated encephalitis. Finally, primary neurologic conditions such as myoclonic epilepsy can be considered. Given the patient’s medical history, serotonin syndrome must be considered.

fdp03704165_t.png

Dr. Reese: Given the concern for serotonin syndrome, the admitting medical team discontinued the patient’s duloxetine. Dr. Weller, what is the pathophysiology of serotonin syndrome, and how is it diagnosed?

Dr. Weller: Serotonin is ubiquitous throughout the body and brain. Serotonin syndrome is caused by excess endogenous or exogenous serotonin, and this is usually caused by a variety of medications. The symptoms range from tachycardia, agitation, and diaphoresis to sustained clonus, hyperthermia, and shock.2,3 The extent of serotonin syndrome is typically thought to reflect the degree of serotonergic activity.4

Serotonin syndrome is a clinical diagnosis. While there are no tests that can confirm the diagnosis, the Hunter criteria can be used to assist with making the diagnosis.5 Per the Hunter criteria, a patient can be diagnosed with serotonin syndrome if they have taken a serotonergic agent and have at least 1 of the following: spontaneous clonus, inducible or ocular clonus with agitation or diaphoresis, tremor and hyperreflexia, or hypertonia with fever and clonus. This patient had taken duloxetine and had inducible clonus and diaphoresis, thus suggesting a diagnosis of serotonin syndrome.

Dr. Reese: Aside from selective serotonin reuptake inhibitors (SSRIs), are there other medications that we typically prescribe that can cause serotonin syndrome?

Dr. Weller: In addition to SSRIs and serotonin-norepinephrine reuptake inhibitors (SNRIs), other commonly prescribed medications that can cause serotonin syndrome are 5-HT3 antagonists (eg, ondansetron), 5-HT agonists (eg, triptans), and opioids (eg, fentanyl and tramadol). There are also case reports of atypical antipsychotics (eg, olanzapine) causing serotonin syndrome because of their antagonism of the 5-HT2 and 5-HT3 receptors.2 Additionally, linezolid is commonly overlooked as a cause of serotonin syndrome given its action as a monoamine oxidase inhibitor.4 In this patient, it would be prudent to discontinue olanzapine and duloxetine.

Dr. Reese: Duloxetine, olanzapine, and buprenorphine/naloxone were discontinuedgiven concern for serotonin syndrome. Although there are not strong data that buprenorphine/ naloxone can cause serotonin syndrome, the team discontinued the medication in case it might be contributing to the patient’s encephalopathy, while closely monitoring the patient for withdrawal. There was a rapid improvement in the patient’s symptoms over the 24 hours after discontinuation of the 3 medications.

As part of the initial workup, the patient received a computed tomography (CT) scan of his chest to follow up pulmonary nodules identified 16 months prior. The CT scan showed interval growth of the pulmonary nodules in the right lower lobe to 2 cm with extension into the major fissure, which was concerning for malignancy. Plans were made for an outpatient positron emission tomography (PET) scan after hospital discharge.

Dr. Schlechter and Dr. Rangachari, what factors can help us determine whether or not further workup of a malignancy should occur before discharge or can be deferred to the outpatient setting?

Benjamin Schlechter, MD, Instructor in Medicine, BIDMC; and Deepa Rangachari, MD, Assistant Professor of Medicine, BIDMC: Key considerations in this domain include rapidity of growth and any threat to critical end-organ function (ie, brain, heart, lungs, kidney, and liver). If the malignancy is bulky and/or rapidly progressing to the point that the patient has significant symptoms burden and/or end-organ dysfunction, then initiating the evaluation as an inpatient may be necessary. For suspected intrathoracic malignancies, considering whether this may be a high-grade process (ie, small cell lung cancer) is often a vital branch point. Key considerations in this regard are the following: Is it a bulky central tumor? Is there evidence of widespread metastatic disease, an obstructing mass, and/or tumor lysis? One final and critical aspect to consider is whether there are any patient- specific barriers to timely and reliable outpatient follow-up. If there is no evidence of rapid progression, bulky disease with threatened end-organ involvement, and/or issues with timely and reliable follow-up, then outpatient evaluation is often the best approach to ensure a comprehensive and well-coordinated effort on the patient’s behalf.

Dr. Reese: Buprenorphine/naloxone was restarted without return of the symptoms. The patient was discharged home with an outpatient PET scan scheduled the following week. Unfortunately, the patient was unable to keep this appointment. Three weeks after hospital discharge, the patient presented again to the emergency department with gradually worsening altered mental status, confusion, visual hallucinations, and myoclonic jerking of the arms and legs. Medication adherence was confirmed by the patient’s wife, resulting in a low concern for serotonin syndrome. Physical examination revealed confusion, dysarthria, diffuse, arrhythmic, myoclonic jerking in all extremities, asterixis in the upper extremities, and hyperreflexia.

A CT scan of the brain did not reveal an intracranial process. A spot electroencephalograph (EEG) and magnetic resonance image (MRI) of the brain were obtained. Dr. Weller, what is the utility of spot EEG vs 24-hour EEG? When might we choose one over the other?

Dr. Weller: If a patient is persistently altered, then a spot EEG would be sufficient to capture a seizure if that is what is causing the patient’s altered mental status. However, if the patient’s mental status is waxing and waning, then that may warrant a 24-hour EEG because the patient may need to be monitored for longer periods to capture an event that is causing intermittent alterations in mental status.6 Additionally, patients who are acutely ill may require long-term monitoring for the purpose of treatment and outcome management.

Dr. Reese: The spot EEG showed nearly continuous generalized slowing indicative of a diffuse encephalopathy. The MRI of the brain showed scattered, nonspecific periventricular T2 hyperintense foci, suggestive of advanced chronic microvascular ischemic changes.

A PET CT was obtained and revealed mildly fluorodeoxyglucose (FDG)-avid, enlarging nodules within the right lower lobe, which was suspicious for malignancy. There were no other areas of FDG avidity on the PET scan. Valproic acid was initiated for treatment of myoclonus with transition to clonazepam when no improvement was seen. After starting clonazepam, the patient’s condition stabilized.

Dr. Weller, given the additional history, how has your differential diagnosis changed?

Dr. Weller: Given the patient’s laboratory findings, we can be quite sure that there is not a contributing metabolic process. The findings suggestive of metastatic cancer, along with the profound neurologic changes, are most concerning for a paraneoplastic syndrome. I would suggest biopsy and consideration of a lumbar puncture. One can also send serum markers, including a paraneoplastic antibody panel.

Dr. Reese: Biopsy of the mass in his right lower lobe revealed squamous cell lung cancer. Dr. Schlechter and Dr. Rangachari, do you have a framework for the different forms of lung cancer?

Dr. Schlechter/Dr. Rangachari: The 2 broad categories of lung cancer are small cell and non-small cell (NSCLC). Small cell lung cancer has a tight association with tobacco exposure and is often clinically defined by rapid, bulky progression (ie, weeks to months).7,8 NSCLCs are also commonly seen in those with tobacco exposure, though not always. The main subgroups in this category are adenocarcinoma and squamous cell carcinoma. These cancers often evolve at a slower pace (ie, months to years).8 While small cell lung cancers are highgrade tumors and exquisitely sensitive to chemotherapy and radiation, NSCLCs tend to be less responsive to such therapies. The staging evaluation for either entity is the same and consists of defining localized vs metastatic disease.

Dr. Reese: Because this patient had an MRI and PET scan that were both negative for metastatic disease, can we assume that this patient had stage I NSCLC?

Dr. Schlechter/Dr. Rangachari: Not necessarily. While PET and MRI brain are exceptionally helpful in detecting distant metastases, they may over- or underestimate intrathoracic lymph node involvement by as much as 20%.9 As such, dedicated lymph node staging—either via bronchoscopy (endobronchial ultrasound) or surgically (mediastinoscopy) is indicated as lymph node involvement can significantly alter the stage, prognosis, and optimal therapeutic approach.10,11

Dr. Reese: After this diagnosis was made, the teams caring for this patient attributed his altered mental status to a paraneoplastic syndrome. What is a paraneoplastic syndrome, and how does a paraneoplastic syndrome from malignancy present? Does its presence worsen a patient’s prognosis?

fdp03704165_box.png

Dr. Schlechter/Dr. Rangachari: A paraneoplastic syndrome is defined by an immunologic response to the cancer that ends up erroneously targeting self-antigens. Paraneoplastic syndromes are associated with a broad array of clinical findings—from endocrinopathy to encephalopathy—and certain neoplasms are more commonly associated with these syndromes than others (eg, small cell lung cancer and thymoma). Further, severity and onset of a paraneoplastic syndrome does not correlate with the burden of visible disease—and the syndrome may predate the cancer diagnosis by months to years.11 While treatment of the cancer affords the best hope of resolving the paraneoplastic syndrome, the cancer and the paraneoplastic process may have a discordant trajectory, with the paraneoplastic syndrome persisting even after the cancer is maximally treated. Although one might assume that paraneoplastic syndromes portend worse outcomes, in some cases, a presentation with the paraneoplastic syndrome may afford sooner detection of an otherwise occult/asymptomatic malignancy.

Dr. Reese: The following week, the serum paraneoplastic antibody panel that tested for anti-Yo antibody, anti-Ri antibody,and anti-Hu antibody came back negative. Dr. Weller, what does this mean? Since we have yet to obtain a lumbar puncture, might his symptoms still be caused by a paraneoplastic syndrome?

Dr. Weller: The negative serum test just means that he does not have antibodies to those 3 antibodies. There are now over 30 different paraneoplastic antibodies that have been discovered, and there are always more that are being discovered. So this negative test result does not exclude a paraneoplastic syndrome in the appropriate clinical context.12 Furthermore, the sensitivity and specificity for certain antibodies are different based upon source fluid, and cerebrospinal fluid testing would provide more diagnostic clarity. A negative test for paraneoplastic syndrome, by itself, would similarly not exclude a paraneoplastic syndrome. Often, empiric treatment is the best diagnostic option for paraneoplastic and autoimmune encephalopathies.

Dr. Reese: The following week, the patient was discharged to rehabilitation with clonazepam for his symptoms and a scheduled follow-up. Given the patient’s frailty and medical comorbidities, thoracic surgery recommended consultation with radiation oncology. Dr. Schlechter and Dr. Rangachari, when do we decide to use radiation vs chemotherapy for someone with lung cancer?

Dr. Schlechter/Dr. Rangachari: Patients with early stage, nonmetastatic NSCLC may not always be candidates for surgical resection on the basis of pulmonary function, other medical comorbidities (as in this case), anatomic considerations, and/or patient preference. In these cases, if there is lung-limited disease without lymph node involvement (ie, stage I/II NSCLC) and the patient is not felt to be an operative candidate, then alternatives to surgery include either radiation or ablation.13,14 As we care for an aging and comorbid population, evolving evidence suggests that well-selected patients with early stage disease undergoing these nonoperative approaches have roughly equivalent outcomes to those undergoing conventional surgical resection.13 In such cases, multidisciplinary consultation with a team having dedicated expertise in these various operative and nonoperative modalities is essential.

Dr. Reese: The patient followed up with radiation oncology for consideration of radiation treatment, but his simulation CT scan showed some ground-glass opacity that were concerning for inflammation vs infection. The patient’s case was discussed at the multidisciplinary tumor board, and it was determined to treat him with antibiotics for a possible pneumonia before proceeding with radiation therapy. After he completed antibiotic treatment, he underwent 10 fractions of radiation treatment, which he tolerated well.

References

1. Kojovic M, Cordivari C, Bhatia K. Myoclonic disorders: a practical approach for diagnosis and treatment. Ther Adv Neurol Disord. 2011;4(1):47-62.

2. Volpi-Abadie J, Kaye AM, Kaye AD. Serotonin syndrome. Ochsner J. 2013;13(4):533-540.

3. Arora B, Kannikeswaran N. The serotonin syndrome-the need for physician’s awareness. Int J Emerg Med. 2010;3(4):373-377.

4. Boyer EW, Shannon M. The serotonin syndrome [published correction appears in N Engl J Med. 2007;356(23):2437 and N Engl J Med. 2009;361(17):1714]. N Engl J Med.
2005;352(11):1112-1120.

5. Dunkley EJC, Isbister GK, Sibbritt D, Dawson AH, Whyte IM. The Hunter Serotonin Toxicity Criteria: simple and accurate diagnostic decision rules for serotonin toxicity. QJM.
2003;96(9):635-642.

6. Nordli DR Jr. Usefulness of video-EEG monitoring. Epilepsia. 2006;47(suppl 1):26-30.

7. Ettinger DS, Aisner J. Changing face of small-cell lung cancer: real and artifact. J Clin Oncol. 2006;24(28):4526-4527.

8. Travis WD, Brambilla E, Nicholson AG, et al. The 2015 World Health Organization classification of lung tumors: impact of genetic, clinical and radiologic advances since the 2004 classification. J Thorac Oncol. 2015;10(9):1243-1260.

9. Cerfolio RJ, Bryant AS, Ojha B, Eloubeidi M. Improving the inaccuracies of clinical staging of patients with NSCLC: a prospective trial. Ann Thorac Surg. 2005;80(4):1207-1214.

10. El-Osta H, Jani P, Mansour A, Rascoe P, Jafri S. Endobronchial ultrasound for nodal staging of patients with non-smallcell lung cancer with radiologically normal mediastinum. A meta-analysis. Ann Am Thorac Soc. 2018;15(7):864-874.

11. Darnell RB, Posner JB. Paraneoplastic syndromes involving the nervous system. N Engl J Med. 2003;349(16):1543-1554.

12. McKeon A. Autoimmune Encephalopathies and Dementias. Continuum (Minneap Minn). 2016;22(2 Dementia): 538-558.

13. Molina JR, Yang P, Cassivi SD, Schild SE, Adjei AA. Nonsmall cell lung cancer: epidemiology, risk factors, treatment, and survivorship. Mayo Clin Proc. 2008;83(5):584-594.

14. Ettinger DS, Aisner DL, Wood DE, et al. NCCN Guidelines Insights: Non-Small Cell Lung Cancer, Version 5.2018. J Natl Compr Canc Netw. 2018;16(7):807-821.

Article PDF
Author and Disclosure Information

Zachary Reese is Chief Medical Resident at Veterans Affairs Boston Healthcare System (VABHS) in Massachusetts, and Beth Israel Deaconess Medical Center (BIDMC). Jason Weller is an Instructor of Neurology at Boston Medical Center and VABHS. Benjamin Schlechter is an Instructor in Medicine, and Deepa Rangachari is an Assistant Professor of Medicine, both at BIDMC. Anthony Breu is a Hospitalist and the Director of Resident Education at VA Boston Healthcare System (VABHS) and an Assistant Professor of Medicine at Harvard University in Massachusetts. He supervises the VA Boston Medical Forum chief resident case conferences. All patients or their surrogate decision makers understand and have signed appropriate patient release forms. This article has received an abbreviated peer review.

Author disclosures
The authors report no actual or potential conflicts of interest with regard to this article.

Disclaimer
The opinions expressed herein are those of the authors and do not necessarily reflect those of Federal Practitioner, Frontline Medical Communications Inc., the US Government, or any of its agencies. This article may discuss unlabeled or investigational use of certain drugs. Please review the complete prescribing information for specific drugs or drug combinations—including indications, contraindications, warnings, and adverse effects—before administering pharmacologic therapy to patients.

Issue
Federal Practitioner - 37(4)a
Publications
Topics
Page Number
165-169
Sections
Author and Disclosure Information

Zachary Reese is Chief Medical Resident at Veterans Affairs Boston Healthcare System (VABHS) in Massachusetts, and Beth Israel Deaconess Medical Center (BIDMC). Jason Weller is an Instructor of Neurology at Boston Medical Center and VABHS. Benjamin Schlechter is an Instructor in Medicine, and Deepa Rangachari is an Assistant Professor of Medicine, both at BIDMC. Anthony Breu is a Hospitalist and the Director of Resident Education at VA Boston Healthcare System (VABHS) and an Assistant Professor of Medicine at Harvard University in Massachusetts. He supervises the VA Boston Medical Forum chief resident case conferences. All patients or their surrogate decision makers understand and have signed appropriate patient release forms. This article has received an abbreviated peer review.

Author disclosures
The authors report no actual or potential conflicts of interest with regard to this article.

Disclaimer
The opinions expressed herein are those of the authors and do not necessarily reflect those of Federal Practitioner, Frontline Medical Communications Inc., the US Government, or any of its agencies. This article may discuss unlabeled or investigational use of certain drugs. Please review the complete prescribing information for specific drugs or drug combinations—including indications, contraindications, warnings, and adverse effects—before administering pharmacologic therapy to patients.

Author and Disclosure Information

Zachary Reese is Chief Medical Resident at Veterans Affairs Boston Healthcare System (VABHS) in Massachusetts, and Beth Israel Deaconess Medical Center (BIDMC). Jason Weller is an Instructor of Neurology at Boston Medical Center and VABHS. Benjamin Schlechter is an Instructor in Medicine, and Deepa Rangachari is an Assistant Professor of Medicine, both at BIDMC. Anthony Breu is a Hospitalist and the Director of Resident Education at VA Boston Healthcare System (VABHS) and an Assistant Professor of Medicine at Harvard University in Massachusetts. He supervises the VA Boston Medical Forum chief resident case conferences. All patients or their surrogate decision makers understand and have signed appropriate patient release forms. This article has received an abbreviated peer review.

Author disclosures
The authors report no actual or potential conflicts of interest with regard to this article.

Disclaimer
The opinions expressed herein are those of the authors and do not necessarily reflect those of Federal Practitioner, Frontline Medical Communications Inc., the US Government, or any of its agencies. This article may discuss unlabeled or investigational use of certain drugs. Please review the complete prescribing information for specific drugs or drug combinations—including indications, contraindications, warnings, and adverse effects—before administering pharmacologic therapy to patients.

Article PDF
Article PDF
Related Articles

►Zachary Reese, MD, Chief Medical Resident, VABHS and Beth Israel Deaconess Medical Center (BIDMC):Dr. Weller, the differential diagnosis for altered mental status is quite broad. How does the presence of clonus change or focus your approach to altered mental status?

►Jason Weller, MD, Instructor of Neurology, Boston Medical Center (BMC) and VABHS:The presence of clonus does not significantly narrow the differential. It does, however, suggest a central component to the patient’s altered mental status. Specifically, it implies that the underlying process, whether systemic or neurologic, interferes with central nervous system (CNS) control of the neuromuscular system.1 The differential is still quite broad and includes metabolic derangements (eg, uremia, electrolyte disturbances, hypercarbia, and thyroid dysfunction), medication toxicity from olanzapine or duloxetine, and vascular processes (eg, CNS vasculitis). Infectious etiologies, both within the CNS and systemically, can cause encephalopathy, as can autoimmune processes, such as immune-mediated encephalitis. Finally, primary neurologic conditions such as myoclonic epilepsy can be considered. Given the patient’s medical history, serotonin syndrome must be considered.

fdp03704165_t.png

Dr. Reese: Given the concern for serotonin syndrome, the admitting medical team discontinued the patient’s duloxetine. Dr. Weller, what is the pathophysiology of serotonin syndrome, and how is it diagnosed?

Dr. Weller: Serotonin is ubiquitous throughout the body and brain. Serotonin syndrome is caused by excess endogenous or exogenous serotonin, and this is usually caused by a variety of medications. The symptoms range from tachycardia, agitation, and diaphoresis to sustained clonus, hyperthermia, and shock.2,3 The extent of serotonin syndrome is typically thought to reflect the degree of serotonergic activity.4

Serotonin syndrome is a clinical diagnosis. While there are no tests that can confirm the diagnosis, the Hunter criteria can be used to assist with making the diagnosis.5 Per the Hunter criteria, a patient can be diagnosed with serotonin syndrome if they have taken a serotonergic agent and have at least 1 of the following: spontaneous clonus, inducible or ocular clonus with agitation or diaphoresis, tremor and hyperreflexia, or hypertonia with fever and clonus. This patient had taken duloxetine and had inducible clonus and diaphoresis, thus suggesting a diagnosis of serotonin syndrome.

Dr. Reese: Aside from selective serotonin reuptake inhibitors (SSRIs), are there other medications that we typically prescribe that can cause serotonin syndrome?

Dr. Weller: In addition to SSRIs and serotonin-norepinephrine reuptake inhibitors (SNRIs), other commonly prescribed medications that can cause serotonin syndrome are 5-HT3 antagonists (eg, ondansetron), 5-HT agonists (eg, triptans), and opioids (eg, fentanyl and tramadol). There are also case reports of atypical antipsychotics (eg, olanzapine) causing serotonin syndrome because of their antagonism of the 5-HT2 and 5-HT3 receptors.2 Additionally, linezolid is commonly overlooked as a cause of serotonin syndrome given its action as a monoamine oxidase inhibitor.4 In this patient, it would be prudent to discontinue olanzapine and duloxetine.

Dr. Reese: Duloxetine, olanzapine, and buprenorphine/naloxone were discontinuedgiven concern for serotonin syndrome. Although there are not strong data that buprenorphine/ naloxone can cause serotonin syndrome, the team discontinued the medication in case it might be contributing to the patient’s encephalopathy, while closely monitoring the patient for withdrawal. There was a rapid improvement in the patient’s symptoms over the 24 hours after discontinuation of the 3 medications.

As part of the initial workup, the patient received a computed tomography (CT) scan of his chest to follow up pulmonary nodules identified 16 months prior. The CT scan showed interval growth of the pulmonary nodules in the right lower lobe to 2 cm with extension into the major fissure, which was concerning for malignancy. Plans were made for an outpatient positron emission tomography (PET) scan after hospital discharge.

Dr. Schlechter and Dr. Rangachari, what factors can help us determine whether or not further workup of a malignancy should occur before discharge or can be deferred to the outpatient setting?

Benjamin Schlechter, MD, Instructor in Medicine, BIDMC; and Deepa Rangachari, MD, Assistant Professor of Medicine, BIDMC: Key considerations in this domain include rapidity of growth and any threat to critical end-organ function (ie, brain, heart, lungs, kidney, and liver). If the malignancy is bulky and/or rapidly progressing to the point that the patient has significant symptoms burden and/or end-organ dysfunction, then initiating the evaluation as an inpatient may be necessary. For suspected intrathoracic malignancies, considering whether this may be a high-grade process (ie, small cell lung cancer) is often a vital branch point. Key considerations in this regard are the following: Is it a bulky central tumor? Is there evidence of widespread metastatic disease, an obstructing mass, and/or tumor lysis? One final and critical aspect to consider is whether there are any patient- specific barriers to timely and reliable outpatient follow-up. If there is no evidence of rapid progression, bulky disease with threatened end-organ involvement, and/or issues with timely and reliable follow-up, then outpatient evaluation is often the best approach to ensure a comprehensive and well-coordinated effort on the patient’s behalf.

Dr. Reese: Buprenorphine/naloxone was restarted without return of the symptoms. The patient was discharged home with an outpatient PET scan scheduled the following week. Unfortunately, the patient was unable to keep this appointment. Three weeks after hospital discharge, the patient presented again to the emergency department with gradually worsening altered mental status, confusion, visual hallucinations, and myoclonic jerking of the arms and legs. Medication adherence was confirmed by the patient’s wife, resulting in a low concern for serotonin syndrome. Physical examination revealed confusion, dysarthria, diffuse, arrhythmic, myoclonic jerking in all extremities, asterixis in the upper extremities, and hyperreflexia.

A CT scan of the brain did not reveal an intracranial process. A spot electroencephalograph (EEG) and magnetic resonance image (MRI) of the brain were obtained. Dr. Weller, what is the utility of spot EEG vs 24-hour EEG? When might we choose one over the other?

Dr. Weller: If a patient is persistently altered, then a spot EEG would be sufficient to capture a seizure if that is what is causing the patient’s altered mental status. However, if the patient’s mental status is waxing and waning, then that may warrant a 24-hour EEG because the patient may need to be monitored for longer periods to capture an event that is causing intermittent alterations in mental status.6 Additionally, patients who are acutely ill may require long-term monitoring for the purpose of treatment and outcome management.

Dr. Reese: The spot EEG showed nearly continuous generalized slowing indicative of a diffuse encephalopathy. The MRI of the brain showed scattered, nonspecific periventricular T2 hyperintense foci, suggestive of advanced chronic microvascular ischemic changes.

A PET CT was obtained and revealed mildly fluorodeoxyglucose (FDG)-avid, enlarging nodules within the right lower lobe, which was suspicious for malignancy. There were no other areas of FDG avidity on the PET scan. Valproic acid was initiated for treatment of myoclonus with transition to clonazepam when no improvement was seen. After starting clonazepam, the patient’s condition stabilized.

Dr. Weller, given the additional history, how has your differential diagnosis changed?

Dr. Weller: Given the patient’s laboratory findings, we can be quite sure that there is not a contributing metabolic process. The findings suggestive of metastatic cancer, along with the profound neurologic changes, are most concerning for a paraneoplastic syndrome. I would suggest biopsy and consideration of a lumbar puncture. One can also send serum markers, including a paraneoplastic antibody panel.

Dr. Reese: Biopsy of the mass in his right lower lobe revealed squamous cell lung cancer. Dr. Schlechter and Dr. Rangachari, do you have a framework for the different forms of lung cancer?

Dr. Schlechter/Dr. Rangachari: The 2 broad categories of lung cancer are small cell and non-small cell (NSCLC). Small cell lung cancer has a tight association with tobacco exposure and is often clinically defined by rapid, bulky progression (ie, weeks to months).7,8 NSCLCs are also commonly seen in those with tobacco exposure, though not always. The main subgroups in this category are adenocarcinoma and squamous cell carcinoma. These cancers often evolve at a slower pace (ie, months to years).8 While small cell lung cancers are highgrade tumors and exquisitely sensitive to chemotherapy and radiation, NSCLCs tend to be less responsive to such therapies. The staging evaluation for either entity is the same and consists of defining localized vs metastatic disease.

Dr. Reese: Because this patient had an MRI and PET scan that were both negative for metastatic disease, can we assume that this patient had stage I NSCLC?

Dr. Schlechter/Dr. Rangachari: Not necessarily. While PET and MRI brain are exceptionally helpful in detecting distant metastases, they may over- or underestimate intrathoracic lymph node involvement by as much as 20%.9 As such, dedicated lymph node staging—either via bronchoscopy (endobronchial ultrasound) or surgically (mediastinoscopy) is indicated as lymph node involvement can significantly alter the stage, prognosis, and optimal therapeutic approach.10,11

Dr. Reese: After this diagnosis was made, the teams caring for this patient attributed his altered mental status to a paraneoplastic syndrome. What is a paraneoplastic syndrome, and how does a paraneoplastic syndrome from malignancy present? Does its presence worsen a patient’s prognosis?

fdp03704165_box.png

Dr. Schlechter/Dr. Rangachari: A paraneoplastic syndrome is defined by an immunologic response to the cancer that ends up erroneously targeting self-antigens. Paraneoplastic syndromes are associated with a broad array of clinical findings—from endocrinopathy to encephalopathy—and certain neoplasms are more commonly associated with these syndromes than others (eg, small cell lung cancer and thymoma). Further, severity and onset of a paraneoplastic syndrome does not correlate with the burden of visible disease—and the syndrome may predate the cancer diagnosis by months to years.11 While treatment of the cancer affords the best hope of resolving the paraneoplastic syndrome, the cancer and the paraneoplastic process may have a discordant trajectory, with the paraneoplastic syndrome persisting even after the cancer is maximally treated. Although one might assume that paraneoplastic syndromes portend worse outcomes, in some cases, a presentation with the paraneoplastic syndrome may afford sooner detection of an otherwise occult/asymptomatic malignancy.

Dr. Reese: The following week, the serum paraneoplastic antibody panel that tested for anti-Yo antibody, anti-Ri antibody,and anti-Hu antibody came back negative. Dr. Weller, what does this mean? Since we have yet to obtain a lumbar puncture, might his symptoms still be caused by a paraneoplastic syndrome?

Dr. Weller: The negative serum test just means that he does not have antibodies to those 3 antibodies. There are now over 30 different paraneoplastic antibodies that have been discovered, and there are always more that are being discovered. So this negative test result does not exclude a paraneoplastic syndrome in the appropriate clinical context.12 Furthermore, the sensitivity and specificity for certain antibodies are different based upon source fluid, and cerebrospinal fluid testing would provide more diagnostic clarity. A negative test for paraneoplastic syndrome, by itself, would similarly not exclude a paraneoplastic syndrome. Often, empiric treatment is the best diagnostic option for paraneoplastic and autoimmune encephalopathies.

Dr. Reese: The following week, the patient was discharged to rehabilitation with clonazepam for his symptoms and a scheduled follow-up. Given the patient’s frailty and medical comorbidities, thoracic surgery recommended consultation with radiation oncology. Dr. Schlechter and Dr. Rangachari, when do we decide to use radiation vs chemotherapy for someone with lung cancer?

Dr. Schlechter/Dr. Rangachari: Patients with early stage, nonmetastatic NSCLC may not always be candidates for surgical resection on the basis of pulmonary function, other medical comorbidities (as in this case), anatomic considerations, and/or patient preference. In these cases, if there is lung-limited disease without lymph node involvement (ie, stage I/II NSCLC) and the patient is not felt to be an operative candidate, then alternatives to surgery include either radiation or ablation.13,14 As we care for an aging and comorbid population, evolving evidence suggests that well-selected patients with early stage disease undergoing these nonoperative approaches have roughly equivalent outcomes to those undergoing conventional surgical resection.13 In such cases, multidisciplinary consultation with a team having dedicated expertise in these various operative and nonoperative modalities is essential.

Dr. Reese: The patient followed up with radiation oncology for consideration of radiation treatment, but his simulation CT scan showed some ground-glass opacity that were concerning for inflammation vs infection. The patient’s case was discussed at the multidisciplinary tumor board, and it was determined to treat him with antibiotics for a possible pneumonia before proceeding with radiation therapy. After he completed antibiotic treatment, he underwent 10 fractions of radiation treatment, which he tolerated well.

►Zachary Reese, MD, Chief Medical Resident, VABHS and Beth Israel Deaconess Medical Center (BIDMC):Dr. Weller, the differential diagnosis for altered mental status is quite broad. How does the presence of clonus change or focus your approach to altered mental status?

►Jason Weller, MD, Instructor of Neurology, Boston Medical Center (BMC) and VABHS:The presence of clonus does not significantly narrow the differential. It does, however, suggest a central component to the patient’s altered mental status. Specifically, it implies that the underlying process, whether systemic or neurologic, interferes with central nervous system (CNS) control of the neuromuscular system.1 The differential is still quite broad and includes metabolic derangements (eg, uremia, electrolyte disturbances, hypercarbia, and thyroid dysfunction), medication toxicity from olanzapine or duloxetine, and vascular processes (eg, CNS vasculitis). Infectious etiologies, both within the CNS and systemically, can cause encephalopathy, as can autoimmune processes, such as immune-mediated encephalitis. Finally, primary neurologic conditions such as myoclonic epilepsy can be considered. Given the patient’s medical history, serotonin syndrome must be considered.

fdp03704165_t.png

Dr. Reese: Given the concern for serotonin syndrome, the admitting medical team discontinued the patient’s duloxetine. Dr. Weller, what is the pathophysiology of serotonin syndrome, and how is it diagnosed?

Dr. Weller: Serotonin is ubiquitous throughout the body and brain. Serotonin syndrome is caused by excess endogenous or exogenous serotonin, and this is usually caused by a variety of medications. The symptoms range from tachycardia, agitation, and diaphoresis to sustained clonus, hyperthermia, and shock.2,3 The extent of serotonin syndrome is typically thought to reflect the degree of serotonergic activity.4

Serotonin syndrome is a clinical diagnosis. While there are no tests that can confirm the diagnosis, the Hunter criteria can be used to assist with making the diagnosis.5 Per the Hunter criteria, a patient can be diagnosed with serotonin syndrome if they have taken a serotonergic agent and have at least 1 of the following: spontaneous clonus, inducible or ocular clonus with agitation or diaphoresis, tremor and hyperreflexia, or hypertonia with fever and clonus. This patient had taken duloxetine and had inducible clonus and diaphoresis, thus suggesting a diagnosis of serotonin syndrome.

Dr. Reese: Aside from selective serotonin reuptake inhibitors (SSRIs), are there other medications that we typically prescribe that can cause serotonin syndrome?

Dr. Weller: In addition to SSRIs and serotonin-norepinephrine reuptake inhibitors (SNRIs), other commonly prescribed medications that can cause serotonin syndrome are 5-HT3 antagonists (eg, ondansetron), 5-HT agonists (eg, triptans), and opioids (eg, fentanyl and tramadol). There are also case reports of atypical antipsychotics (eg, olanzapine) causing serotonin syndrome because of their antagonism of the 5-HT2 and 5-HT3 receptors.2 Additionally, linezolid is commonly overlooked as a cause of serotonin syndrome given its action as a monoamine oxidase inhibitor.4 In this patient, it would be prudent to discontinue olanzapine and duloxetine.

Dr. Reese: Duloxetine, olanzapine, and buprenorphine/naloxone were discontinuedgiven concern for serotonin syndrome. Although there are not strong data that buprenorphine/ naloxone can cause serotonin syndrome, the team discontinued the medication in case it might be contributing to the patient’s encephalopathy, while closely monitoring the patient for withdrawal. There was a rapid improvement in the patient’s symptoms over the 24 hours after discontinuation of the 3 medications.

As part of the initial workup, the patient received a computed tomography (CT) scan of his chest to follow up pulmonary nodules identified 16 months prior. The CT scan showed interval growth of the pulmonary nodules in the right lower lobe to 2 cm with extension into the major fissure, which was concerning for malignancy. Plans were made for an outpatient positron emission tomography (PET) scan after hospital discharge.

Dr. Schlechter and Dr. Rangachari, what factors can help us determine whether or not further workup of a malignancy should occur before discharge or can be deferred to the outpatient setting?

Benjamin Schlechter, MD, Instructor in Medicine, BIDMC; and Deepa Rangachari, MD, Assistant Professor of Medicine, BIDMC: Key considerations in this domain include rapidity of growth and any threat to critical end-organ function (ie, brain, heart, lungs, kidney, and liver). If the malignancy is bulky and/or rapidly progressing to the point that the patient has significant symptoms burden and/or end-organ dysfunction, then initiating the evaluation as an inpatient may be necessary. For suspected intrathoracic malignancies, considering whether this may be a high-grade process (ie, small cell lung cancer) is often a vital branch point. Key considerations in this regard are the following: Is it a bulky central tumor? Is there evidence of widespread metastatic disease, an obstructing mass, and/or tumor lysis? One final and critical aspect to consider is whether there are any patient- specific barriers to timely and reliable outpatient follow-up. If there is no evidence of rapid progression, bulky disease with threatened end-organ involvement, and/or issues with timely and reliable follow-up, then outpatient evaluation is often the best approach to ensure a comprehensive and well-coordinated effort on the patient’s behalf.

Dr. Reese: Buprenorphine/naloxone was restarted without return of the symptoms. The patient was discharged home with an outpatient PET scan scheduled the following week. Unfortunately, the patient was unable to keep this appointment. Three weeks after hospital discharge, the patient presented again to the emergency department with gradually worsening altered mental status, confusion, visual hallucinations, and myoclonic jerking of the arms and legs. Medication adherence was confirmed by the patient’s wife, resulting in a low concern for serotonin syndrome. Physical examination revealed confusion, dysarthria, diffuse, arrhythmic, myoclonic jerking in all extremities, asterixis in the upper extremities, and hyperreflexia.

A CT scan of the brain did not reveal an intracranial process. A spot electroencephalograph (EEG) and magnetic resonance image (MRI) of the brain were obtained. Dr. Weller, what is the utility of spot EEG vs 24-hour EEG? When might we choose one over the other?

Dr. Weller: If a patient is persistently altered, then a spot EEG would be sufficient to capture a seizure if that is what is causing the patient’s altered mental status. However, if the patient’s mental status is waxing and waning, then that may warrant a 24-hour EEG because the patient may need to be monitored for longer periods to capture an event that is causing intermittent alterations in mental status.6 Additionally, patients who are acutely ill may require long-term monitoring for the purpose of treatment and outcome management.

Dr. Reese: The spot EEG showed nearly continuous generalized slowing indicative of a diffuse encephalopathy. The MRI of the brain showed scattered, nonspecific periventricular T2 hyperintense foci, suggestive of advanced chronic microvascular ischemic changes.

A PET CT was obtained and revealed mildly fluorodeoxyglucose (FDG)-avid, enlarging nodules within the right lower lobe, which was suspicious for malignancy. There were no other areas of FDG avidity on the PET scan. Valproic acid was initiated for treatment of myoclonus with transition to clonazepam when no improvement was seen. After starting clonazepam, the patient’s condition stabilized.

Dr. Weller, given the additional history, how has your differential diagnosis changed?

Dr. Weller: Given the patient’s laboratory findings, we can be quite sure that there is not a contributing metabolic process. The findings suggestive of metastatic cancer, along with the profound neurologic changes, are most concerning for a paraneoplastic syndrome. I would suggest biopsy and consideration of a lumbar puncture. One can also send serum markers, including a paraneoplastic antibody panel.

Dr. Reese: Biopsy of the mass in his right lower lobe revealed squamous cell lung cancer. Dr. Schlechter and Dr. Rangachari, do you have a framework for the different forms of lung cancer?

Dr. Schlechter/Dr. Rangachari: The 2 broad categories of lung cancer are small cell and non-small cell (NSCLC). Small cell lung cancer has a tight association with tobacco exposure and is often clinically defined by rapid, bulky progression (ie, weeks to months).7,8 NSCLCs are also commonly seen in those with tobacco exposure, though not always. The main subgroups in this category are adenocarcinoma and squamous cell carcinoma. These cancers often evolve at a slower pace (ie, months to years).8 While small cell lung cancers are highgrade tumors and exquisitely sensitive to chemotherapy and radiation, NSCLCs tend to be less responsive to such therapies. The staging evaluation for either entity is the same and consists of defining localized vs metastatic disease.

Dr. Reese: Because this patient had an MRI and PET scan that were both negative for metastatic disease, can we assume that this patient had stage I NSCLC?

Dr. Schlechter/Dr. Rangachari: Not necessarily. While PET and MRI brain are exceptionally helpful in detecting distant metastases, they may over- or underestimate intrathoracic lymph node involvement by as much as 20%.9 As such, dedicated lymph node staging—either via bronchoscopy (endobronchial ultrasound) or surgically (mediastinoscopy) is indicated as lymph node involvement can significantly alter the stage, prognosis, and optimal therapeutic approach.10,11

Dr. Reese: After this diagnosis was made, the teams caring for this patient attributed his altered mental status to a paraneoplastic syndrome. What is a paraneoplastic syndrome, and how does a paraneoplastic syndrome from malignancy present? Does its presence worsen a patient’s prognosis?

fdp03704165_box.png

Dr. Schlechter/Dr. Rangachari: A paraneoplastic syndrome is defined by an immunologic response to the cancer that ends up erroneously targeting self-antigens. Paraneoplastic syndromes are associated with a broad array of clinical findings—from endocrinopathy to encephalopathy—and certain neoplasms are more commonly associated with these syndromes than others (eg, small cell lung cancer and thymoma). Further, severity and onset of a paraneoplastic syndrome does not correlate with the burden of visible disease—and the syndrome may predate the cancer diagnosis by months to years.11 While treatment of the cancer affords the best hope of resolving the paraneoplastic syndrome, the cancer and the paraneoplastic process may have a discordant trajectory, with the paraneoplastic syndrome persisting even after the cancer is maximally treated. Although one might assume that paraneoplastic syndromes portend worse outcomes, in some cases, a presentation with the paraneoplastic syndrome may afford sooner detection of an otherwise occult/asymptomatic malignancy.

Dr. Reese: The following week, the serum paraneoplastic antibody panel that tested for anti-Yo antibody, anti-Ri antibody,and anti-Hu antibody came back negative. Dr. Weller, what does this mean? Since we have yet to obtain a lumbar puncture, might his symptoms still be caused by a paraneoplastic syndrome?

Dr. Weller: The negative serum test just means that he does not have antibodies to those 3 antibodies. There are now over 30 different paraneoplastic antibodies that have been discovered, and there are always more that are being discovered. So this negative test result does not exclude a paraneoplastic syndrome in the appropriate clinical context.12 Furthermore, the sensitivity and specificity for certain antibodies are different based upon source fluid, and cerebrospinal fluid testing would provide more diagnostic clarity. A negative test for paraneoplastic syndrome, by itself, would similarly not exclude a paraneoplastic syndrome. Often, empiric treatment is the best diagnostic option for paraneoplastic and autoimmune encephalopathies.

Dr. Reese: The following week, the patient was discharged to rehabilitation with clonazepam for his symptoms and a scheduled follow-up. Given the patient’s frailty and medical comorbidities, thoracic surgery recommended consultation with radiation oncology. Dr. Schlechter and Dr. Rangachari, when do we decide to use radiation vs chemotherapy for someone with lung cancer?

Dr. Schlechter/Dr. Rangachari: Patients with early stage, nonmetastatic NSCLC may not always be candidates for surgical resection on the basis of pulmonary function, other medical comorbidities (as in this case), anatomic considerations, and/or patient preference. In these cases, if there is lung-limited disease without lymph node involvement (ie, stage I/II NSCLC) and the patient is not felt to be an operative candidate, then alternatives to surgery include either radiation or ablation.13,14 As we care for an aging and comorbid population, evolving evidence suggests that well-selected patients with early stage disease undergoing these nonoperative approaches have roughly equivalent outcomes to those undergoing conventional surgical resection.13 In such cases, multidisciplinary consultation with a team having dedicated expertise in these various operative and nonoperative modalities is essential.

Dr. Reese: The patient followed up with radiation oncology for consideration of radiation treatment, but his simulation CT scan showed some ground-glass opacity that were concerning for inflammation vs infection. The patient’s case was discussed at the multidisciplinary tumor board, and it was determined to treat him with antibiotics for a possible pneumonia before proceeding with radiation therapy. After he completed antibiotic treatment, he underwent 10 fractions of radiation treatment, which he tolerated well.

References

1. Kojovic M, Cordivari C, Bhatia K. Myoclonic disorders: a practical approach for diagnosis and treatment. Ther Adv Neurol Disord. 2011;4(1):47-62.

2. Volpi-Abadie J, Kaye AM, Kaye AD. Serotonin syndrome. Ochsner J. 2013;13(4):533-540.

3. Arora B, Kannikeswaran N. The serotonin syndrome-the need for physician’s awareness. Int J Emerg Med. 2010;3(4):373-377.

4. Boyer EW, Shannon M. The serotonin syndrome [published correction appears in N Engl J Med. 2007;356(23):2437 and N Engl J Med. 2009;361(17):1714]. N Engl J Med.
2005;352(11):1112-1120.

5. Dunkley EJC, Isbister GK, Sibbritt D, Dawson AH, Whyte IM. The Hunter Serotonin Toxicity Criteria: simple and accurate diagnostic decision rules for serotonin toxicity. QJM.
2003;96(9):635-642.

6. Nordli DR Jr. Usefulness of video-EEG monitoring. Epilepsia. 2006;47(suppl 1):26-30.

7. Ettinger DS, Aisner J. Changing face of small-cell lung cancer: real and artifact. J Clin Oncol. 2006;24(28):4526-4527.

8. Travis WD, Brambilla E, Nicholson AG, et al. The 2015 World Health Organization classification of lung tumors: impact of genetic, clinical and radiologic advances since the 2004 classification. J Thorac Oncol. 2015;10(9):1243-1260.

9. Cerfolio RJ, Bryant AS, Ojha B, Eloubeidi M. Improving the inaccuracies of clinical staging of patients with NSCLC: a prospective trial. Ann Thorac Surg. 2005;80(4):1207-1214.

10. El-Osta H, Jani P, Mansour A, Rascoe P, Jafri S. Endobronchial ultrasound for nodal staging of patients with non-smallcell lung cancer with radiologically normal mediastinum. A meta-analysis. Ann Am Thorac Soc. 2018;15(7):864-874.

11. Darnell RB, Posner JB. Paraneoplastic syndromes involving the nervous system. N Engl J Med. 2003;349(16):1543-1554.

12. McKeon A. Autoimmune Encephalopathies and Dementias. Continuum (Minneap Minn). 2016;22(2 Dementia): 538-558.

13. Molina JR, Yang P, Cassivi SD, Schild SE, Adjei AA. Nonsmall cell lung cancer: epidemiology, risk factors, treatment, and survivorship. Mayo Clin Proc. 2008;83(5):584-594.

14. Ettinger DS, Aisner DL, Wood DE, et al. NCCN Guidelines Insights: Non-Small Cell Lung Cancer, Version 5.2018. J Natl Compr Canc Netw. 2018;16(7):807-821.

References

1. Kojovic M, Cordivari C, Bhatia K. Myoclonic disorders: a practical approach for diagnosis and treatment. Ther Adv Neurol Disord. 2011;4(1):47-62.

2. Volpi-Abadie J, Kaye AM, Kaye AD. Serotonin syndrome. Ochsner J. 2013;13(4):533-540.

3. Arora B, Kannikeswaran N. The serotonin syndrome-the need for physician’s awareness. Int J Emerg Med. 2010;3(4):373-377.

4. Boyer EW, Shannon M. The serotonin syndrome [published correction appears in N Engl J Med. 2007;356(23):2437 and N Engl J Med. 2009;361(17):1714]. N Engl J Med.
2005;352(11):1112-1120.

5. Dunkley EJC, Isbister GK, Sibbritt D, Dawson AH, Whyte IM. The Hunter Serotonin Toxicity Criteria: simple and accurate diagnostic decision rules for serotonin toxicity. QJM.
2003;96(9):635-642.

6. Nordli DR Jr. Usefulness of video-EEG monitoring. Epilepsia. 2006;47(suppl 1):26-30.

7. Ettinger DS, Aisner J. Changing face of small-cell lung cancer: real and artifact. J Clin Oncol. 2006;24(28):4526-4527.

8. Travis WD, Brambilla E, Nicholson AG, et al. The 2015 World Health Organization classification of lung tumors: impact of genetic, clinical and radiologic advances since the 2004 classification. J Thorac Oncol. 2015;10(9):1243-1260.

9. Cerfolio RJ, Bryant AS, Ojha B, Eloubeidi M. Improving the inaccuracies of clinical staging of patients with NSCLC: a prospective trial. Ann Thorac Surg. 2005;80(4):1207-1214.

10. El-Osta H, Jani P, Mansour A, Rascoe P, Jafri S. Endobronchial ultrasound for nodal staging of patients with non-smallcell lung cancer with radiologically normal mediastinum. A meta-analysis. Ann Am Thorac Soc. 2018;15(7):864-874.

11. Darnell RB, Posner JB. Paraneoplastic syndromes involving the nervous system. N Engl J Med. 2003;349(16):1543-1554.

12. McKeon A. Autoimmune Encephalopathies and Dementias. Continuum (Minneap Minn). 2016;22(2 Dementia): 538-558.

13. Molina JR, Yang P, Cassivi SD, Schild SE, Adjei AA. Nonsmall cell lung cancer: epidemiology, risk factors, treatment, and survivorship. Mayo Clin Proc. 2008;83(5):584-594.

14. Ettinger DS, Aisner DL, Wood DE, et al. NCCN Guidelines Insights: Non-Small Cell Lung Cancer, Version 5.2018. J Natl Compr Canc Netw. 2018;16(7):807-821.

Issue
Federal Practitioner - 37(4)a
Issue
Federal Practitioner - 37(4)a
Page Number
165-169
Page Number
165-169
Publications
Publications
Topics
Article Type
Sections
Disallow All Ads
Content Gating
No Gating (article Unlocked/Free)
Alternative CME
Disqus Comments
Default
Gate On Date
Wed, 04/08/2020 - 10:15
Un-Gate On Date
Wed, 04/08/2020 - 10:15
Use ProPublica
CFC Schedule Remove Status
Wed, 04/08/2020 - 10:15
Hide sidebar & use full width
render the right sidebar.
Article PDF Media

A Veteran With a Solitary Pulmonary Nodule

Article Type
Changed
Wed, 01/08/2020 - 14:56

Case Presentation. A 69-year-old veteran presented with an intermittent, waxing and waning cough. He had never smoked and had no family history of lung cancer. His primary care physician ordered a chest radiograph, which revealed a nodular opacity within the lingula concerning for a parenchymal nodule. Further characterization with a chest computed tomography (CT) demonstrated a 1.4-cm left upper lobe subpleural nodule with small satellite nodules (Figure 1). Given these imaging findings, the patient was referred to the pulmonary clinic.

fed03701038_f1.png


►Lauren Kearney, MD, Medical Resident, VA Boston Healthcare System (VABHS) and Boston Medical Center. What is the differential diagnosis of a solitary pulmonary nodule? What characteristics of the nodule do you consider to differentiate these diagnoses?

►Renda Wiener, MD, Pulmonary and Critical Care, VABHS, and Assistant Professor of Medicine, Boston University School of Medicine. Pulmonary nodules are well-defined lesions < 3 cm in diameter that are surrounded by lung parenchyma. Although cancer is a possibility (including primary lung cancers, metastatic cancers, or carcinoid tumors), most small nodules do not turn out to be malignant.1 Benign etiologies include infections, benign tumors, vascular malformations, and inflammatory conditions. Infectious causes of nodules are often granulomatous in nature, including fungi, Mycobacterium tuberculosis, and nontuberculous mycobacteria. Benign tumors are most commonly hamartomas, and these may be clearly distinguished based on imaging characteristics. Pulmonary arteriovenous malformations, hematomas, and infarcts may present as nodules as well. Inflammatory causes of nodules are important and relatively common, including granulomatosis with polyangiitis, rheumatoid arthritis, sarcoidosis, amyloidosis, and rounded atelectasis. 

To distinguish benign from malignant etiologies, we look for several features of pulmonary nodules on imaging. Larger size, irregular borders, and upper lobe location all increase the likelihood of cancer, whereas solid attenuation and calcification make cancer less likely. One of the most reassuring findings that suggests a benign etiology is lack of growth over a period of surveillance; after 2 years without growth we typically consider a nodule benign.1 And of course, we also consider the patient’s symptoms and risk factors: weight loss, hemoptysis, a history of cigarette smoking or asbestos exposure, or family history of cancer all increase the likelihood of malignancy.

Dr. Kearney. Given that the differential diagnosis is so broad, how do you think about the next step in evaluating a pulmonary nodule? How do you approach shared decision making with the patient? 

►Dr. Wiener. The characteristics of the patient, the nodule, and the circumstances in which the nodule were discovered are all important to consider. Incidental pulmonary nodules are often found on chest imaging. The imaging characteristics of the nodule are important, as are the patient’s risk factors. A similarly appearing nodule can have very different implications if the patient is a never-smoker exposed to endemic fungi, or a long-time smoker enrolled in a lung cancer screening program. Consultation with a pulmonologist is often appropriate.

It’s important to note that we lack high-quality evidence on the optimal strategy to evaluate pulmonary nodules, and there is no single “right answer“ for all patients. For patients with a low risk of malignancy (< 5%-10%)—which comprises the majority of the incidental nodules discovered—we typically favor serial CT surveillance of the nodule over a period of a few years, whereas for patients at high risk of malignancy (> 65%), we favor early surgical resection if the patient is able to tolerate that. For patients with an intermediate risk of malignancy (~5%-65%), we might consider serial CT surveillance, positron emission tomography (PET) scan, or biopsy.1 The American College of Chest Physicians guidelines for pulmonary nodule evaluation recommend discussing with patients the different options and the trade-offs of these options in a shared decision-making process.1

 

 

►Dr. Kearney. The patient’s pulmonologist laid out options, including monitoring with serial CT scans, obtaining a PET scan, performing CT-guided needle biopsy, or referring for surgical excision. In this case, the patient elected to undergo CT-guided needle biopsy. Dr. Huang, can you discuss the pathology results?

►Qin Huang, MD, Pathology and Laboratory Medicine, VABHS, and Assistant Professor of Pathology, Harvard Medical School (HMS). The microscopic examination of the needle biopsy of the lung mass revealed rare clusters of atypical cells with crushed cells adjacent to an extensive area of necrosis with scarring. The atypical cells were suspicious for carcinoma. The Gomori methenamine silver (GMS) and periodic acid-Schiff (PAS) stains were negative for common bacterial and fungal microorganisms.

►Dr. Kearney. The tumor board, pulmonologist, and patient decide to move forward with video-assisted excisional biopsy with lymphadenectomy. Dr. Huang, can you interpret the pathology?

►Dr. Huang. Figure 2 showed an hemotoxylin and eosin (H&E)-stained lung resection tissue section with multiple caseating necrotic granulomas. No foreign bodies were identified. There was no evidence of malignancy. The GMS stain revealed a fungal microorganism oval with morphology typical of histoplasma capsulatum (Figure 3).

fed03701038_f2f3.png


►Dr. Kearney. What are some of the different ways histoplasmosis can present? Which of these diagnoses fits this patient’s presentation?

►Judy Strymish, MD, Infectious Disease, VABHS, and Assistant Professor of Medicine, HMS. Most patients who inhale histoplasmosis spores develop asymptomatic or self-limited infection that is usually not detected. Patients at risk of symptomatic and clinically relevant disease include those who are immunocompromised, at extremes of ages, or exposed to larger inoculums. Acute pulmonary histoplasmosis can present with cough, shortness of breath, fever, chills, and less commonly, rheumatologic complaints such as erythema nodosum or erythema multiforme. Imaging often shows patchy infiltrates and enlarged mediastinal and hilar lymphadenopathy. Patients can go on to develop subacute or chronic pulmonary disease with focal opacities and mediastinal and hilar lymphadenopathy. Those with chronic disease can have cavitary lesions similar to patients with tuberculosis. Progressive disseminated histoplasmosis can develop in immunocompromised patients and disseminate through the reticuloendothelial system to other organs with the gastrointestinal tract, central nervous system, and adrenal glands.2

Pulmonary nodules are common incidental finding on chest imaging of patients who reside in histoplasmosis endemic regions, and they are often hard to differentiate from malignancies. There are 3 mediastinal manifestations: adenitis, granuloma, and fibrosis. Usually the syndromes are subclinical, but occasionally the nodes cause symptoms by impinging on other structures.2

This patient had a solitary pulmonary nodule with none of the associated features mentioned above. Pathology showed caseating granuloma and confirmed histoplasmosis.

Dr. Kearney. Given the diagnosis of solitary histoplasmoma, how should this patient be managed?

►Dr. Strymish. The optimal therapy for histoplasmosis depends on the patient’s clinical syndrome. Most infections are self-limited and require no therapy. However, patients who are immunocompromised, exposed to large inoculum, and have progressive disease require antifungal treatment, usually with itraconazole for mild-to-moderate disease and a combination of azole therapy and amphotericin B with extensive disease. Patients with few solitary pulmonary nodules do not benefit from antifungal therapy as the nodule could represent quiescent disease that is unlikely to have clinical impact; in this case, the treatment would be higher risk than the nodule.3

Dr. Kearney. While the discussion of the diagnosis is interesting, it is also important to acknowledge what the patient went through to arrive at this, an essentially benign diagnosis: 8 months, multiple imaging studies, and 2 invasive diagnostic procedures. Further, the patient had to grapple with the possibility of a diagnosis of cancer. Dr. Wiener, can you talk about the challenges in communicating with patients about pulmonary nodules when cancer is on the differential? What are some of the harms patients face and how can clinicians work to mitigate these harms?

fed03701038_box.png

►Dr. Wiener. My colleague Dr. Christopher Slatore of the Portland VA Medical Center and I studied communication about pulmonary nodules in a series of surveys and qualitative studies of patients with pulmonary nodules and the clinicians who take care of them. We found that there seems to be a disconnect between patients’ perceptions of pulmonary nodules and their clinicians, often due to inadequate communication about the nodule. Many clinicians indicated that they do not tell patients about the chance that a nodule may be cancer, because the clinicians know that cancer is unlikely (< 5% of incidentally detected pulmonary nodules turn out to be malignant), and they do not want to alarm patients unnecessarily. However, we found that patients almost immediately wondered about cancer when they learned about their pulmonary nodule, and without hearing explicitly from their clinician that cancer was unlikely, patients tended to overestimate the likelihood of a malignant nodule. Moreover, patients often were not told much about the evaluation plan for the nodule or the rationale for CT surveillance of small nodules instead of biopsy. This uncertainty about the risk of cancer and the plan for evaluating the nodule was difficult for some patients to live with; we found that about one-quarter of patients with a small pulmonary nodule experienced mild-moderate distress during the period of radiographic surveillance. Reassuringly, high-quality patient-clinician communication was associated with lower distress and higher adherence to pulmonary nodule evaluation.4

►Dr. Kearney. The patient was educated about his diagnosis of solitary histoplasmoma. Given that the patient was otherwise well appearing with no complicating factors, he was not treated with antifungal therapy. After an 8-month-long workup, the patient was relieved to receive a diagnosis that excluded cancer and did not require any further treatment. His case provides a good example of how to proceed in the workup of a solitary pulmonary nodule and on the importance of communication and shared decision making with our patients.

References

1. Gould MK, Donington J, Lynch WR, et al. Evaluation of individuals with pulmonary nodules: when is it lung cancer? Diagnosis and management of lung cancer, 3rd ed: American College of Chest Physicians evidence-based clinical practice guidelines. Chest. 2013;143(suppl 5):e93S-e120S.

2. Azar MM, Hage CA. Clinical perspectives in the diagnosis and management of histoplasmosis. Clin Chest Med. 2017;38(3):403-415.

3. Wheat LJ, Freifeld A, Kleiman MB, et al. Clinical practice guidelines for the management of patients with histoplasmosis: 2007 update by the Infectious Diseases Society of America. Clin Infect Dis. 2007;45(7):807-825.

4. Slatore CG, Wiener RS. Pulmonary nodules: a small problem for many, severe distress for some, and how to communicate about it. Chest. 2018;153(4):1004-1015.

Article PDF
Author and Disclosure Information

Anthony Breu is a Hospitalist and the Director of Resident Education at VA Boston Healthcare System (VABHS) and an Assistant Professor of Medicine at Harvard University. He supervises the VA Boston Medical Forum chief resident case conferences. Lauren Kearney is a Medical Resident at the VABHS and Boston Medical Center. Lakshmana Swamy is a Resident at Boston University School of Medicine. Max Brock is a Cardiology Fellow at Stritch School of Medicine in Maywood, Illinois. Renda Wiener is a Pulmonary and Critical Care Physician at VABHS and Assistant Professor of Medicine at the Boston University School of Medicine. Qin Huang is a Physician of Pathology and Laboratory Medicine at VABHS and Assistant Professor of Pathology at Harvard Medical School. Judy Strymish is a Infectious Disease Physician at VABHS and Assistant Professor of Medicine at Harvard Medical School.

Author disclosures
The authors report no actual or potential conflicts of interest with regard to this article.

Disclaimer
All patients or their surrogate decision makers understand and have signed appropriate patient release forms. This article has received an abbreviated peer review.

The opinions expressed herein are those of the authors and do not necessarily reflect those of Federal Practitioner, Frontline Medical Communications Inc., the US Government, or any of its agencies. This article may discuss unlabeled or investigational use of certain drugs. Please review the complete prescribing information for specific drugs or drug combinations—including indications, contraindications, warnings, and adverse effects—before administering pharmacologic therapy to patients.

Issue
Federal Practitioner - 37(1)a
Publications
Topics
Page Number
38-41
Sections
Author and Disclosure Information

Anthony Breu is a Hospitalist and the Director of Resident Education at VA Boston Healthcare System (VABHS) and an Assistant Professor of Medicine at Harvard University. He supervises the VA Boston Medical Forum chief resident case conferences. Lauren Kearney is a Medical Resident at the VABHS and Boston Medical Center. Lakshmana Swamy is a Resident at Boston University School of Medicine. Max Brock is a Cardiology Fellow at Stritch School of Medicine in Maywood, Illinois. Renda Wiener is a Pulmonary and Critical Care Physician at VABHS and Assistant Professor of Medicine at the Boston University School of Medicine. Qin Huang is a Physician of Pathology and Laboratory Medicine at VABHS and Assistant Professor of Pathology at Harvard Medical School. Judy Strymish is a Infectious Disease Physician at VABHS and Assistant Professor of Medicine at Harvard Medical School.

Author disclosures
The authors report no actual or potential conflicts of interest with regard to this article.

Disclaimer
All patients or their surrogate decision makers understand and have signed appropriate patient release forms. This article has received an abbreviated peer review.

The opinions expressed herein are those of the authors and do not necessarily reflect those of Federal Practitioner, Frontline Medical Communications Inc., the US Government, or any of its agencies. This article may discuss unlabeled or investigational use of certain drugs. Please review the complete prescribing information for specific drugs or drug combinations—including indications, contraindications, warnings, and adverse effects—before administering pharmacologic therapy to patients.

Author and Disclosure Information

Anthony Breu is a Hospitalist and the Director of Resident Education at VA Boston Healthcare System (VABHS) and an Assistant Professor of Medicine at Harvard University. He supervises the VA Boston Medical Forum chief resident case conferences. Lauren Kearney is a Medical Resident at the VABHS and Boston Medical Center. Lakshmana Swamy is a Resident at Boston University School of Medicine. Max Brock is a Cardiology Fellow at Stritch School of Medicine in Maywood, Illinois. Renda Wiener is a Pulmonary and Critical Care Physician at VABHS and Assistant Professor of Medicine at the Boston University School of Medicine. Qin Huang is a Physician of Pathology and Laboratory Medicine at VABHS and Assistant Professor of Pathology at Harvard Medical School. Judy Strymish is a Infectious Disease Physician at VABHS and Assistant Professor of Medicine at Harvard Medical School.

Author disclosures
The authors report no actual or potential conflicts of interest with regard to this article.

Disclaimer
All patients or their surrogate decision makers understand and have signed appropriate patient release forms. This article has received an abbreviated peer review.

The opinions expressed herein are those of the authors and do not necessarily reflect those of Federal Practitioner, Frontline Medical Communications Inc., the US Government, or any of its agencies. This article may discuss unlabeled or investigational use of certain drugs. Please review the complete prescribing information for specific drugs or drug combinations—including indications, contraindications, warnings, and adverse effects—before administering pharmacologic therapy to patients.

Article PDF
Article PDF
Related Articles

Case Presentation. A 69-year-old veteran presented with an intermittent, waxing and waning cough. He had never smoked and had no family history of lung cancer. His primary care physician ordered a chest radiograph, which revealed a nodular opacity within the lingula concerning for a parenchymal nodule. Further characterization with a chest computed tomography (CT) demonstrated a 1.4-cm left upper lobe subpleural nodule with small satellite nodules (Figure 1). Given these imaging findings, the patient was referred to the pulmonary clinic.

fed03701038_f1.png


►Lauren Kearney, MD, Medical Resident, VA Boston Healthcare System (VABHS) and Boston Medical Center. What is the differential diagnosis of a solitary pulmonary nodule? What characteristics of the nodule do you consider to differentiate these diagnoses?

►Renda Wiener, MD, Pulmonary and Critical Care, VABHS, and Assistant Professor of Medicine, Boston University School of Medicine. Pulmonary nodules are well-defined lesions < 3 cm in diameter that are surrounded by lung parenchyma. Although cancer is a possibility (including primary lung cancers, metastatic cancers, or carcinoid tumors), most small nodules do not turn out to be malignant.1 Benign etiologies include infections, benign tumors, vascular malformations, and inflammatory conditions. Infectious causes of nodules are often granulomatous in nature, including fungi, Mycobacterium tuberculosis, and nontuberculous mycobacteria. Benign tumors are most commonly hamartomas, and these may be clearly distinguished based on imaging characteristics. Pulmonary arteriovenous malformations, hematomas, and infarcts may present as nodules as well. Inflammatory causes of nodules are important and relatively common, including granulomatosis with polyangiitis, rheumatoid arthritis, sarcoidosis, amyloidosis, and rounded atelectasis. 

To distinguish benign from malignant etiologies, we look for several features of pulmonary nodules on imaging. Larger size, irregular borders, and upper lobe location all increase the likelihood of cancer, whereas solid attenuation and calcification make cancer less likely. One of the most reassuring findings that suggests a benign etiology is lack of growth over a period of surveillance; after 2 years without growth we typically consider a nodule benign.1 And of course, we also consider the patient’s symptoms and risk factors: weight loss, hemoptysis, a history of cigarette smoking or asbestos exposure, or family history of cancer all increase the likelihood of malignancy.

Dr. Kearney. Given that the differential diagnosis is so broad, how do you think about the next step in evaluating a pulmonary nodule? How do you approach shared decision making with the patient? 

►Dr. Wiener. The characteristics of the patient, the nodule, and the circumstances in which the nodule were discovered are all important to consider. Incidental pulmonary nodules are often found on chest imaging. The imaging characteristics of the nodule are important, as are the patient’s risk factors. A similarly appearing nodule can have very different implications if the patient is a never-smoker exposed to endemic fungi, or a long-time smoker enrolled in a lung cancer screening program. Consultation with a pulmonologist is often appropriate.

It’s important to note that we lack high-quality evidence on the optimal strategy to evaluate pulmonary nodules, and there is no single “right answer“ for all patients. For patients with a low risk of malignancy (< 5%-10%)—which comprises the majority of the incidental nodules discovered—we typically favor serial CT surveillance of the nodule over a period of a few years, whereas for patients at high risk of malignancy (> 65%), we favor early surgical resection if the patient is able to tolerate that. For patients with an intermediate risk of malignancy (~5%-65%), we might consider serial CT surveillance, positron emission tomography (PET) scan, or biopsy.1 The American College of Chest Physicians guidelines for pulmonary nodule evaluation recommend discussing with patients the different options and the trade-offs of these options in a shared decision-making process.1

 

 

►Dr. Kearney. The patient’s pulmonologist laid out options, including monitoring with serial CT scans, obtaining a PET scan, performing CT-guided needle biopsy, or referring for surgical excision. In this case, the patient elected to undergo CT-guided needle biopsy. Dr. Huang, can you discuss the pathology results?

►Qin Huang, MD, Pathology and Laboratory Medicine, VABHS, and Assistant Professor of Pathology, Harvard Medical School (HMS). The microscopic examination of the needle biopsy of the lung mass revealed rare clusters of atypical cells with crushed cells adjacent to an extensive area of necrosis with scarring. The atypical cells were suspicious for carcinoma. The Gomori methenamine silver (GMS) and periodic acid-Schiff (PAS) stains were negative for common bacterial and fungal microorganisms.

►Dr. Kearney. The tumor board, pulmonologist, and patient decide to move forward with video-assisted excisional biopsy with lymphadenectomy. Dr. Huang, can you interpret the pathology?

►Dr. Huang. Figure 2 showed an hemotoxylin and eosin (H&E)-stained lung resection tissue section with multiple caseating necrotic granulomas. No foreign bodies were identified. There was no evidence of malignancy. The GMS stain revealed a fungal microorganism oval with morphology typical of histoplasma capsulatum (Figure 3).

fed03701038_f2f3.png


►Dr. Kearney. What are some of the different ways histoplasmosis can present? Which of these diagnoses fits this patient’s presentation?

►Judy Strymish, MD, Infectious Disease, VABHS, and Assistant Professor of Medicine, HMS. Most patients who inhale histoplasmosis spores develop asymptomatic or self-limited infection that is usually not detected. Patients at risk of symptomatic and clinically relevant disease include those who are immunocompromised, at extremes of ages, or exposed to larger inoculums. Acute pulmonary histoplasmosis can present with cough, shortness of breath, fever, chills, and less commonly, rheumatologic complaints such as erythema nodosum or erythema multiforme. Imaging often shows patchy infiltrates and enlarged mediastinal and hilar lymphadenopathy. Patients can go on to develop subacute or chronic pulmonary disease with focal opacities and mediastinal and hilar lymphadenopathy. Those with chronic disease can have cavitary lesions similar to patients with tuberculosis. Progressive disseminated histoplasmosis can develop in immunocompromised patients and disseminate through the reticuloendothelial system to other organs with the gastrointestinal tract, central nervous system, and adrenal glands.2

Pulmonary nodules are common incidental finding on chest imaging of patients who reside in histoplasmosis endemic regions, and they are often hard to differentiate from malignancies. There are 3 mediastinal manifestations: adenitis, granuloma, and fibrosis. Usually the syndromes are subclinical, but occasionally the nodes cause symptoms by impinging on other structures.2

This patient had a solitary pulmonary nodule with none of the associated features mentioned above. Pathology showed caseating granuloma and confirmed histoplasmosis.

Dr. Kearney. Given the diagnosis of solitary histoplasmoma, how should this patient be managed?

►Dr. Strymish. The optimal therapy for histoplasmosis depends on the patient’s clinical syndrome. Most infections are self-limited and require no therapy. However, patients who are immunocompromised, exposed to large inoculum, and have progressive disease require antifungal treatment, usually with itraconazole for mild-to-moderate disease and a combination of azole therapy and amphotericin B with extensive disease. Patients with few solitary pulmonary nodules do not benefit from antifungal therapy as the nodule could represent quiescent disease that is unlikely to have clinical impact; in this case, the treatment would be higher risk than the nodule.3

Dr. Kearney. While the discussion of the diagnosis is interesting, it is also important to acknowledge what the patient went through to arrive at this, an essentially benign diagnosis: 8 months, multiple imaging studies, and 2 invasive diagnostic procedures. Further, the patient had to grapple with the possibility of a diagnosis of cancer. Dr. Wiener, can you talk about the challenges in communicating with patients about pulmonary nodules when cancer is on the differential? What are some of the harms patients face and how can clinicians work to mitigate these harms?

fed03701038_box.png

►Dr. Wiener. My colleague Dr. Christopher Slatore of the Portland VA Medical Center and I studied communication about pulmonary nodules in a series of surveys and qualitative studies of patients with pulmonary nodules and the clinicians who take care of them. We found that there seems to be a disconnect between patients’ perceptions of pulmonary nodules and their clinicians, often due to inadequate communication about the nodule. Many clinicians indicated that they do not tell patients about the chance that a nodule may be cancer, because the clinicians know that cancer is unlikely (< 5% of incidentally detected pulmonary nodules turn out to be malignant), and they do not want to alarm patients unnecessarily. However, we found that patients almost immediately wondered about cancer when they learned about their pulmonary nodule, and without hearing explicitly from their clinician that cancer was unlikely, patients tended to overestimate the likelihood of a malignant nodule. Moreover, patients often were not told much about the evaluation plan for the nodule or the rationale for CT surveillance of small nodules instead of biopsy. This uncertainty about the risk of cancer and the plan for evaluating the nodule was difficult for some patients to live with; we found that about one-quarter of patients with a small pulmonary nodule experienced mild-moderate distress during the period of radiographic surveillance. Reassuringly, high-quality patient-clinician communication was associated with lower distress and higher adherence to pulmonary nodule evaluation.4

►Dr. Kearney. The patient was educated about his diagnosis of solitary histoplasmoma. Given that the patient was otherwise well appearing with no complicating factors, he was not treated with antifungal therapy. After an 8-month-long workup, the patient was relieved to receive a diagnosis that excluded cancer and did not require any further treatment. His case provides a good example of how to proceed in the workup of a solitary pulmonary nodule and on the importance of communication and shared decision making with our patients.

Case Presentation. A 69-year-old veteran presented with an intermittent, waxing and waning cough. He had never smoked and had no family history of lung cancer. His primary care physician ordered a chest radiograph, which revealed a nodular opacity within the lingula concerning for a parenchymal nodule. Further characterization with a chest computed tomography (CT) demonstrated a 1.4-cm left upper lobe subpleural nodule with small satellite nodules (Figure 1). Given these imaging findings, the patient was referred to the pulmonary clinic.

fed03701038_f1.png


►Lauren Kearney, MD, Medical Resident, VA Boston Healthcare System (VABHS) and Boston Medical Center. What is the differential diagnosis of a solitary pulmonary nodule? What characteristics of the nodule do you consider to differentiate these diagnoses?

►Renda Wiener, MD, Pulmonary and Critical Care, VABHS, and Assistant Professor of Medicine, Boston University School of Medicine. Pulmonary nodules are well-defined lesions < 3 cm in diameter that are surrounded by lung parenchyma. Although cancer is a possibility (including primary lung cancers, metastatic cancers, or carcinoid tumors), most small nodules do not turn out to be malignant.1 Benign etiologies include infections, benign tumors, vascular malformations, and inflammatory conditions. Infectious causes of nodules are often granulomatous in nature, including fungi, Mycobacterium tuberculosis, and nontuberculous mycobacteria. Benign tumors are most commonly hamartomas, and these may be clearly distinguished based on imaging characteristics. Pulmonary arteriovenous malformations, hematomas, and infarcts may present as nodules as well. Inflammatory causes of nodules are important and relatively common, including granulomatosis with polyangiitis, rheumatoid arthritis, sarcoidosis, amyloidosis, and rounded atelectasis. 

To distinguish benign from malignant etiologies, we look for several features of pulmonary nodules on imaging. Larger size, irregular borders, and upper lobe location all increase the likelihood of cancer, whereas solid attenuation and calcification make cancer less likely. One of the most reassuring findings that suggests a benign etiology is lack of growth over a period of surveillance; after 2 years without growth we typically consider a nodule benign.1 And of course, we also consider the patient’s symptoms and risk factors: weight loss, hemoptysis, a history of cigarette smoking or asbestos exposure, or family history of cancer all increase the likelihood of malignancy.

Dr. Kearney. Given that the differential diagnosis is so broad, how do you think about the next step in evaluating a pulmonary nodule? How do you approach shared decision making with the patient? 

►Dr. Wiener. The characteristics of the patient, the nodule, and the circumstances in which the nodule were discovered are all important to consider. Incidental pulmonary nodules are often found on chest imaging. The imaging characteristics of the nodule are important, as are the patient’s risk factors. A similarly appearing nodule can have very different implications if the patient is a never-smoker exposed to endemic fungi, or a long-time smoker enrolled in a lung cancer screening program. Consultation with a pulmonologist is often appropriate.

It’s important to note that we lack high-quality evidence on the optimal strategy to evaluate pulmonary nodules, and there is no single “right answer“ for all patients. For patients with a low risk of malignancy (< 5%-10%)—which comprises the majority of the incidental nodules discovered—we typically favor serial CT surveillance of the nodule over a period of a few years, whereas for patients at high risk of malignancy (> 65%), we favor early surgical resection if the patient is able to tolerate that. For patients with an intermediate risk of malignancy (~5%-65%), we might consider serial CT surveillance, positron emission tomography (PET) scan, or biopsy.1 The American College of Chest Physicians guidelines for pulmonary nodule evaluation recommend discussing with patients the different options and the trade-offs of these options in a shared decision-making process.1

 

 

►Dr. Kearney. The patient’s pulmonologist laid out options, including monitoring with serial CT scans, obtaining a PET scan, performing CT-guided needle biopsy, or referring for surgical excision. In this case, the patient elected to undergo CT-guided needle biopsy. Dr. Huang, can you discuss the pathology results?

►Qin Huang, MD, Pathology and Laboratory Medicine, VABHS, and Assistant Professor of Pathology, Harvard Medical School (HMS). The microscopic examination of the needle biopsy of the lung mass revealed rare clusters of atypical cells with crushed cells adjacent to an extensive area of necrosis with scarring. The atypical cells were suspicious for carcinoma. The Gomori methenamine silver (GMS) and periodic acid-Schiff (PAS) stains were negative for common bacterial and fungal microorganisms.

►Dr. Kearney. The tumor board, pulmonologist, and patient decide to move forward with video-assisted excisional biopsy with lymphadenectomy. Dr. Huang, can you interpret the pathology?

►Dr. Huang. Figure 2 showed an hemotoxylin and eosin (H&E)-stained lung resection tissue section with multiple caseating necrotic granulomas. No foreign bodies were identified. There was no evidence of malignancy. The GMS stain revealed a fungal microorganism oval with morphology typical of histoplasma capsulatum (Figure 3).

fed03701038_f2f3.png


►Dr. Kearney. What are some of the different ways histoplasmosis can present? Which of these diagnoses fits this patient’s presentation?

►Judy Strymish, MD, Infectious Disease, VABHS, and Assistant Professor of Medicine, HMS. Most patients who inhale histoplasmosis spores develop asymptomatic or self-limited infection that is usually not detected. Patients at risk of symptomatic and clinically relevant disease include those who are immunocompromised, at extremes of ages, or exposed to larger inoculums. Acute pulmonary histoplasmosis can present with cough, shortness of breath, fever, chills, and less commonly, rheumatologic complaints such as erythema nodosum or erythema multiforme. Imaging often shows patchy infiltrates and enlarged mediastinal and hilar lymphadenopathy. Patients can go on to develop subacute or chronic pulmonary disease with focal opacities and mediastinal and hilar lymphadenopathy. Those with chronic disease can have cavitary lesions similar to patients with tuberculosis. Progressive disseminated histoplasmosis can develop in immunocompromised patients and disseminate through the reticuloendothelial system to other organs with the gastrointestinal tract, central nervous system, and adrenal glands.2

Pulmonary nodules are common incidental finding on chest imaging of patients who reside in histoplasmosis endemic regions, and they are often hard to differentiate from malignancies. There are 3 mediastinal manifestations: adenitis, granuloma, and fibrosis. Usually the syndromes are subclinical, but occasionally the nodes cause symptoms by impinging on other structures.2

This patient had a solitary pulmonary nodule with none of the associated features mentioned above. Pathology showed caseating granuloma and confirmed histoplasmosis.

Dr. Kearney. Given the diagnosis of solitary histoplasmoma, how should this patient be managed?

►Dr. Strymish. The optimal therapy for histoplasmosis depends on the patient’s clinical syndrome. Most infections are self-limited and require no therapy. However, patients who are immunocompromised, exposed to large inoculum, and have progressive disease require antifungal treatment, usually with itraconazole for mild-to-moderate disease and a combination of azole therapy and amphotericin B with extensive disease. Patients with few solitary pulmonary nodules do not benefit from antifungal therapy as the nodule could represent quiescent disease that is unlikely to have clinical impact; in this case, the treatment would be higher risk than the nodule.3

Dr. Kearney. While the discussion of the diagnosis is interesting, it is also important to acknowledge what the patient went through to arrive at this, an essentially benign diagnosis: 8 months, multiple imaging studies, and 2 invasive diagnostic procedures. Further, the patient had to grapple with the possibility of a diagnosis of cancer. Dr. Wiener, can you talk about the challenges in communicating with patients about pulmonary nodules when cancer is on the differential? What are some of the harms patients face and how can clinicians work to mitigate these harms?

fed03701038_box.png

►Dr. Wiener. My colleague Dr. Christopher Slatore of the Portland VA Medical Center and I studied communication about pulmonary nodules in a series of surveys and qualitative studies of patients with pulmonary nodules and the clinicians who take care of them. We found that there seems to be a disconnect between patients’ perceptions of pulmonary nodules and their clinicians, often due to inadequate communication about the nodule. Many clinicians indicated that they do not tell patients about the chance that a nodule may be cancer, because the clinicians know that cancer is unlikely (< 5% of incidentally detected pulmonary nodules turn out to be malignant), and they do not want to alarm patients unnecessarily. However, we found that patients almost immediately wondered about cancer when they learned about their pulmonary nodule, and without hearing explicitly from their clinician that cancer was unlikely, patients tended to overestimate the likelihood of a malignant nodule. Moreover, patients often were not told much about the evaluation plan for the nodule or the rationale for CT surveillance of small nodules instead of biopsy. This uncertainty about the risk of cancer and the plan for evaluating the nodule was difficult for some patients to live with; we found that about one-quarter of patients with a small pulmonary nodule experienced mild-moderate distress during the period of radiographic surveillance. Reassuringly, high-quality patient-clinician communication was associated with lower distress and higher adherence to pulmonary nodule evaluation.4

►Dr. Kearney. The patient was educated about his diagnosis of solitary histoplasmoma. Given that the patient was otherwise well appearing with no complicating factors, he was not treated with antifungal therapy. After an 8-month-long workup, the patient was relieved to receive a diagnosis that excluded cancer and did not require any further treatment. His case provides a good example of how to proceed in the workup of a solitary pulmonary nodule and on the importance of communication and shared decision making with our patients.

References

1. Gould MK, Donington J, Lynch WR, et al. Evaluation of individuals with pulmonary nodules: when is it lung cancer? Diagnosis and management of lung cancer, 3rd ed: American College of Chest Physicians evidence-based clinical practice guidelines. Chest. 2013;143(suppl 5):e93S-e120S.

2. Azar MM, Hage CA. Clinical perspectives in the diagnosis and management of histoplasmosis. Clin Chest Med. 2017;38(3):403-415.

3. Wheat LJ, Freifeld A, Kleiman MB, et al. Clinical practice guidelines for the management of patients with histoplasmosis: 2007 update by the Infectious Diseases Society of America. Clin Infect Dis. 2007;45(7):807-825.

4. Slatore CG, Wiener RS. Pulmonary nodules: a small problem for many, severe distress for some, and how to communicate about it. Chest. 2018;153(4):1004-1015.

References

1. Gould MK, Donington J, Lynch WR, et al. Evaluation of individuals with pulmonary nodules: when is it lung cancer? Diagnosis and management of lung cancer, 3rd ed: American College of Chest Physicians evidence-based clinical practice guidelines. Chest. 2013;143(suppl 5):e93S-e120S.

2. Azar MM, Hage CA. Clinical perspectives in the diagnosis and management of histoplasmosis. Clin Chest Med. 2017;38(3):403-415.

3. Wheat LJ, Freifeld A, Kleiman MB, et al. Clinical practice guidelines for the management of patients with histoplasmosis: 2007 update by the Infectious Diseases Society of America. Clin Infect Dis. 2007;45(7):807-825.

4. Slatore CG, Wiener RS. Pulmonary nodules: a small problem for many, severe distress for some, and how to communicate about it. Chest. 2018;153(4):1004-1015.

Issue
Federal Practitioner - 37(1)a
Issue
Federal Practitioner - 37(1)a
Page Number
38-41
Page Number
38-41
Publications
Publications
Topics
Article Type
Sections
Disallow All Ads
Content Gating
No Gating (article Unlocked/Free)
Alternative CME
Disqus Comments
Default
Use ProPublica
Hide sidebar & use full width
render the right sidebar.
Article PDF Media

A Veteran Presenting With Leg Swelling, Dyspnea, and Proteinuria

Article Type
Changed
Wed, 10/02/2019 - 10:15

*This article has been corrected to include a missing author.

 

Case Presentation. A 63-year-old male with well-controlled HIV (CD4 count 757, undetectable viral load), epilepsy, and hypertension presented to the VA Boston Healthcare System (VABHS) emergency department with 1 week of bilateral leg swelling and exertional shortness of breath. He reported having no fever, cough, chest pain, pain with inspiration and orthopnea. There was no personal or family history of pulmonary embolism. He reported weight gain but was unable to quantify how much. He also reported flare up of chronic knee pain, without swelling for which he had taken up to 4 tablets of naproxen daily for several weeks. His physical examination was notable for a heart rate of 105 beats per minute and bilateral pitting edema to his knees. Laboratory testing revealed a creatinine level of 2.5 mg/dL, which was increased from a baseline of 1.0 mg/dL (Table 1), and a urine protein-to-creatinine ratio of 7.8 mg/mg (Table 2). A renal ultrasound showed normal-sized kidneys without hydronephrosis or obstructing renal calculi. The patient was admitted for further workup of his dyspnea and acute kidney injury.

fed03609420_t.png

Jonathan Li, MD, Chief Medical Resident, VABHS and Beth Israel Deaconess Medical Center (BIDMC). Dr. William, based on the degree of proteinuria and edema, a diagnosis of nephrotic syndrome was made. How is nephrotic syndrome defined, and how is it distinguished from glomerulonephritis?

Jeffrey William, MD, Nephrologist, BIDMC, Assistant Professor of Medicine, Harvard Medical School. The pathophysiology of nephrotic disease and glomerulonephritis are quite distinct, resulting in symptoms and systemic manifestations that only slightly overlap. Glomerulonephritis is characterized by inflammation of the endothelial cells of the trilayered glomerular capillary, with a resulting active urine sediment with red blood cells, white blood cells, and casts. Nephrotic syndrome mostly affects the visceral epithelial cells of the glomerular capillary, commonly referred to as podocytes, and hence, the urine sediment in nephrotic disease is often inactive. Patients with nephrotic syndrome have nephrotic-range proteinuria (excretion of > 3.5 g per 24 h or a spot urine protein-creatinine ratio > 3.5 g in the steady state) and both hypoalbuminemia (< 3 g/dL) and peripheral edema. Lipiduria and hyperlipidemia are common findings in nephrotic syndrome but are not required for a clinical diagnosis.1 In contrast, glomerulonephritis is defined by a constellation of findings that include renal insufficiency (often indicated by an elevation in blood urea nitrogen and creatinine), hypertension, hematuria, and subnephrotic range proteinuria. In practice, patients may fulfill criteria of both nephrotic and nephritic syndromes, but the preponderance of clinical evidence often points one way or the other. In this case, nephrotic syndrome was diagnosed based on the urine protein-to-creatinine ratio of 7.8 mg/mg, hypoalbuminemia, and edema.

Dr. Li. What would be your first-line workup for evaluation of the etiology of this patient’s nephrotic syndrome?

Dr. William. Rather than memorizing a list of etiologies of nephrotic syndrome, it is essential to consider the pathophysiology of heavy proteinuria. Though the glomerular filtration barrier is extremely complex and defects in any component can cause proteinuria, disruption of the podocyte is often involved. Common disease processes that chiefly target the podocyte include minimal change disease, primary focal and segmental glomerulosclerosis (FSGS), and membranous nephropathy, all by differing mechanisms. Minimal change disease and idiopathic/primary FSGS are increasingly thought to be at differing points on a spectrum of the same disease.2 Secondary FSGS, on the other hand, is a progressive disease, commonly resulting from longstanding hypertension, diabetes mellitus, and obesity in adults. Membranous nephropathy can also be either primary or secondary. Primary membranous nephropathy is chiefly caused by a circulating IgG4 antibody to the podocyte membrane antigen PLA2R (M-type phospholipase A2 receptor), whereas secondary membranous nephropathy can be caused by a variety of systemic etiologies, including autoimmune disease (eg, systemic lupus erythematosus), certain malignancies, chronic infections (eg, hepatitis B and C), and many medications, including nonsteroidal anti-inflammatory drugs (NSAIDs).3-5 Paraprotein deposition diseases can also cause glomerular damage leading to nephrotic-range proteinuria.

 

 

Given these potential diagnoses, a careful history should be taken to assess exposures and recent medication use. Urine sediment evaluation is essential in the evaluation of nephrotic syndrome to determine if there is an underlying nephritic process. Select serologies may be sent to look for autoimmune disease, such as systemic lupus erythematosus and common viral exposures like hepatitis B or C. Serum and urine protein electrophoreses would be appropriate initial tests of suspected paraprotein-related diseases. Other serologies, such as antineutrophil cytoplasmic antibodies or antiglomerular basement membrane antibodies, would not necessarily be indicated here given the lack of hematuria and presence of nephrotic-range proteinuria.

Dr. Li. The initial evaluation was notable for an erythrocyte sedimentation rate > 120 (mm/h) and a weakly positive antinuclear antibody (ANA) titer of 1:40. The remainder of his initial workup did not reveal an etiology for his nephrotic syndrome (Table 3).

fed03609420_t2t3.png

Dr. William, is there a role for starting urgent empiric steroids in nephrotic syndrome while workup is ongoing? If so, do the severity of proteinuria and/or symptoms play a role or is this determination based on something else?

Dr. William. Edema is a primary symptom of nephrotic syndrome and can often be managed with diuretics alone. If a clear medication-mediated cause is suspected, discontinuation of this agent may result in spontaneous improvement without steroid treatment. However,in cases where an etiology is unclear and there are serious thrombotic complications requiring anticoagulation, and a renal biopsy is deemed to be too risky, then empiric steroid therapy may be necessary. Children with new-onset nephrotic syndrome are presumed to have minimal change disease, given its prevalence in this patient population, and are often given empiric steroids without obtaining a renal biopsy. However, in the adult population, a renal biopsy can typically be performed quickly and safely, with pathology results interpreted within days. In this patient, since a diagnosis was unclear and there was no contraindication to renal biopsy, a biopsy should be obtained before consideration of steroids.

Dr. Li. Steroids were deferred in anticipation of renal biopsy, which showed stage I membranous nephropathy, suggestive of membranous lupus nephritis Class V. The deposits were strongly reactive for immunoglobuline G (IgG), IgA, and complement 1q (C1q), showed co-dominant staining for IgG1, IgG2, and IgG3, and were weakly positive for the PLA2 receptor. Focal intimal arteritis in a small interlobular vessel was seen.

Dr. William, the pathology returned suggestive of lupus nephritis. Does the overall clinical picture fit with lupus nephritis?

Dr. William. Given the history and a rather low ANA, the diagnosis of lupus nephritis seems unlikely. The lack of IgG4 and PLA2R staining in the biopsy suggests that this membranous pattern on the biopsy is likely to be secondary to a systemic etiology, but further investigation should be pursued.

Dr. Li. The patient was discharged after the biopsy with a planned outpatient nephrology follow-up to discuss results and treatment. He was prescribed an oral diuretic, and his symptoms improved. Several days after discharge, he developed blurry vision and was evaluated in the Ophthalmology clinic. On fundoscopy, he was found to have acute papillitis, a form of optic neuritis. As part of initial evaluation of infectious etiologies of papillitis, ophthalmology recommended testing for syphilis.

 

 

Dr. Strymish, when we are considering secondary syphilis, what is the recommended approach to diagnostic testing?

Judith Strymish, MD, Infectious Diseases, BIDMC, Assistant Professor of Medicine, Harvard Medical School. The diagnosis of syphilis is usually made through serologic testing of blood specimens. Methods that detect the spirochete directly like dark-field smears are not readily available. Serologic tests include treponemal tests (eg, Treponema pallidum particle agglutination assay [TPPA]) and nontreponemal tests (eg, rapid plasma reagin [RPR]). One needs a confirmatory test because either test is associated with false positives. Either test can be done first. Most laboratories, including those at VABHS are now performing treponemal tests first as these have become more cost-effective.6 The TPPA treponemal test was found to have a lower false negative rate in primary syphilis compared with that of nontreponemal tests.7 Nontreponemal tests can be followed for response to therapy. If a patient has a history of treated syphilis, a nontreponemal test should be sent, since the treponemal test will remain positive for life.

If there is clinical concern for neurosyphilis, cerebrospinal fluid fluorescent (CSF) treponemal antibody needs to be sampled and sent for the nontreponemal venereal disease research laboratory (VDRL) test. The VDRL is highly specific for neurosyphilis but not as sensitive. Cerebrospinal fluid fluorescent treponemal antibody (CSF FTA) may also be sent; it is very sensitive but not very specific for neurosyphilis.

Dr. Li. An RPR returned positive at 1:512 (was negative 14 months prior on a routine screening test), with positive reflex TPPA (Table 4). A diagnosis of secondary syphilis was made. Dr. Strymish, at this point, what additional testing and treatment is necessary?

fed03609420_t4.png

Dr. Strymish. With papillitis and a very high RPR, we need to assume that he has ophthalmic syphilis. This can occur in any stage of syphilis, but his eye findings and high RPR are consistent with secondary syphilis. Ophthalmic syphilis has been on the upswing, even more than is expected with recent increases in syphilis cases.8 Ophthalmic syphilis is considered a form of neurosyphilis. A lumbar puncture and treatment for neurosyphilis is recommended.9,10

Dr. Li. A lumbar puncture was performed, and his CSF was VDRL positive. This confirmed a diagnosis of neurosyphilis (Table 4). The patient was treated for neurosyphilis with IV penicillin. The patient shared that he had episodes of unprotected oral sexual activity within the past year and approximately 1 year ago, he came in close contact (but no sexual activity) with a person who had a rash consistent with syphilis.Dr. William, syphilis would be a potential unifying diagnosis of his renal and ophthalmologic manifestations. Is syphilis known to cause membranous nephropathy?

Dr. William. Though it is uncommon, the nephrotic syndrome is a well-described complication of secondary syphilis.11,12 Syphilis has been shown to cause nephrotic syndrome in a variety of ways. Case reports abound linking syphilis to minimal change disease and other glomerular diseases.13,14 A case report from 1993 shows a membranous pattern of glomerular disease similar to this case.15 As a form of secondary membranous nephropathy, the immunofluorescence pattern can demonstrate staining similar to the “full house” seen in lupus nephritis (IgA, IgM, and C1q, in addition to IgG and C3).16 This explains the initial interpretation of this patient’s biopsy, as lupus nephritis would be a much more common etiology of secondary membranous nephropathy than is acute syphilis with this immunofluorescence pattern. However, the data in this case are highly suggestive of a causal relationship between secondary syphilis and membranous nephropathy.

Dr. Li. Dr. Strymish, how should this patient be screened for syphilis reinfection, and at what intervals would you recommend?

Dr. Strymish. He will need follow-up testing to make sure that his syphilis is effectively treated. If CSF pleocytosis was present initially, a CSF examination should be repeated every 6 months until the cell count is normal. He will also need follow-up for normalization of his RPR. Persons with HIV infection and primary or secondary syphilis should be evaluated clinically and serologically for treatment failure at 3, 6, 9, 12, and 24 months after therapy according to US Centers for Disease Control and Prevention guidelines.9

 

 

His treponemal test for syphilis will likely stay positive for life. His RPR should decrease significantly with effective treatment. It makes sense to screen with RPR alone as long as he continues to have risk factors for acquiring syphilis. Routine syphilis testing is recommended for pregnant women, sexually active men who have sex with men, sexually active persons with HIV, and persons taking PrEP (pre-exposure prophylaxis) for HIV prevention. He should be screened at least yearly for syphilis.

fed03609420_box.png

Dr. Li. Over the next several months, the patient’s creatinine normalized and his proteinuria resolved. His vision recovered, and he has had no further ophthalmologic complications.

Dr. William, what is his long-term renal prognosis? Do you expect that his acute episode of membranous nephropathy will have permanent effects on his renal function?

Dr. William. His rapid response to therapy for neurosyphilis provides evidence for this etiology of his renal dysfunction and glomerulonephritis. His long-term prognosis is quite good if the syphilis is the only reason for him to have renal disease. The renal damage is often reversible in these cases. However, given his prior extensive NSAID exposure and history of hypertension, he may be at higher risk for chronic kidney disease than an otherwise healthy patient, especially after an episode of acute kidney injury. Therefore, his renal function should continue to be monitored as an outpatient.

Acknowledgments

The authors thank this veteran for sharing his story and allowing us to learn from this unusual case for the benefit of our future patients.

References

1. Rennke H, Denker BM. Renal Pathophysiology: The Essentials. 6th ed. Philadelphia: Lippincott Williams & Wilkins; 2014.

2. Maas RJ, Deegens JK, Smeets B, Moeller MJ, Wetzels JF. Minimal change disease and idiopathic FSGS: manifestations of the same disease. Nat Rev Nephrol. 2016;12(12):768-776.

3. Beck LH Jr, Bonegio RG, Lambeau G, et al. M-type phospholipase A2 receptor as target antigen in idiopathic membranous nephropathy. N Engl J Med. 2009;361(1):11-21.

4. Rennke HG. Secondary membranoproliferative glomerulonephritis. Kidney Int. 1995;47(2):643-656.

5. Nawaz FA, Larsen CP, Troxell ML. Membranous nephropathy and nonsteroidal anti-inflammatory agents. Am J Kidney Dis. 2013;62(5):1012-1017.

6. Pillay A. Centers for Disease Control and Prevention Syphilis Summit—Diagnostics and laboratory issues. Sex Transm Dis. 2018;45(9S)(suppl 1):S13-S16. 

7. Levett PN, Fonseca K, Tsang RS, et al. Canadian Public Health Laboratory Network laboratory guidelines for the use of serological tests (excluding point-of-care tests) for the diagnosis of syphilis in Canada. Can J Infect Dis Med Microbiol. 2015;26(suppl A):6A-12A. 

8. Oliver SE, Aubin M, Atwell L, et al. Ocular syphilis—eight jurisdictions, United States, 2014-2015. MMWR Morb Mortal Wkly Rep. 2016;65(43):1185-1188.

9. Workowski KA, Bolan GA. Sexually transmitted diseases treatment guidelines, 2015. MMWR Recommendations and Reports 2015;64(RR3):1-137. [Erratum in MMWR Recomm Rep. 2015;64(33):924.]

10. US Centers for Disease Control and Prevention. Clinical advisory: ocular syphilis in the United States. https://www.cdc.gov/std/syphilis/clinicaladvisoryos2015.htm. Updated March 24, 2016. Accessed August 12, 2019.

11. Braunstein GD, Lewis EJ, Galvanek EG, Hamilton A, Bell WR. The nephrotic syndrome associated with secondary syphilis: an immune deposit disease. Am J Med. 1970;48:643-648.1.

12. Handoko ML, Duijvestein M, Scheepstra CG, de Fijter CW. Syphilis: a reversible cause of nephrotic syndrome. BMJ Case Rep. 2013;2013:pii:bcr2012008279

13. Krane NK, Espenan P, Walker PD, Bergman SM, Wallin JD. Renal disease and syphilis: a report of nephrotic syndrome with minimal change disease. Am J Kidney Dis. 1987;9(2):176-179.

14. Bhorade MS, Carag HB, Lee HJ, Potter EV, Dunea G. Nephropathy of secondary syphilis: a clinical and pathological spectrum. JAMA. 1971;216(7):1159-1166.

15. Hunte W, al-Ghraoui F, Cohen RJ. Secondary syphilis and the nephrotic syndrome. J Am Soc Nephrol. 1993;3(7):1351-1355.

16. Gamble CN, Reardan JB. Immunopathogenesis of syphilitic glomerulonephritis. Elution of antitreponemal antibody from glomerular immune-complex deposits. N Engl J Med. 1975;292(9):449-454.

Article PDF
Author and Disclosure Information

Madeline DiLorenzo is a Resident in the Department of Internal Medicine at Boston University Medical Center in Massachusetts. Anthony Breu is a Hospitalist and the Director of Resident Education at VA Boston Healthcare System and an Assistant Professor of Medicine at Harvard University in Massachusetts. He supervises the VA Boston Medical Forum chief resident case conferences. All patients or their surrogate decision makers understand and have signed appropriate patient release forms. This article has received an abbreviated peer review.
Correspondence: Anthony Breu (anthony.breu@va.gov)

Author disclosures
The authors report no actual or potential conflicts of interest with regard to this article.

Disclaimer
The opinions expressed herein are those of the authors and do not necessarily reflect those of Federal Practitioner, Frontline Medical Communications Inc., the US Government, or any of its agencies. This article may discuss unlabeled or investigational use of certain drugs. Please review the complete prescribing information for specific drugs or drug combinations—including indications, contraindications, warnings, and adverse effects—before administering pharmacologic therapy to patients.

Issue
Federal Practitioner - 36(9)a
Publications
Topics
Page Number
420-424
Sections
Author and Disclosure Information

Madeline DiLorenzo is a Resident in the Department of Internal Medicine at Boston University Medical Center in Massachusetts. Anthony Breu is a Hospitalist and the Director of Resident Education at VA Boston Healthcare System and an Assistant Professor of Medicine at Harvard University in Massachusetts. He supervises the VA Boston Medical Forum chief resident case conferences. All patients or their surrogate decision makers understand and have signed appropriate patient release forms. This article has received an abbreviated peer review.
Correspondence: Anthony Breu (anthony.breu@va.gov)

Author disclosures
The authors report no actual or potential conflicts of interest with regard to this article.

Disclaimer
The opinions expressed herein are those of the authors and do not necessarily reflect those of Federal Practitioner, Frontline Medical Communications Inc., the US Government, or any of its agencies. This article may discuss unlabeled or investigational use of certain drugs. Please review the complete prescribing information for specific drugs or drug combinations—including indications, contraindications, warnings, and adverse effects—before administering pharmacologic therapy to patients.

Author and Disclosure Information

Madeline DiLorenzo is a Resident in the Department of Internal Medicine at Boston University Medical Center in Massachusetts. Anthony Breu is a Hospitalist and the Director of Resident Education at VA Boston Healthcare System and an Assistant Professor of Medicine at Harvard University in Massachusetts. He supervises the VA Boston Medical Forum chief resident case conferences. All patients or their surrogate decision makers understand and have signed appropriate patient release forms. This article has received an abbreviated peer review.
Correspondence: Anthony Breu (anthony.breu@va.gov)

Author disclosures
The authors report no actual or potential conflicts of interest with regard to this article.

Disclaimer
The opinions expressed herein are those of the authors and do not necessarily reflect those of Federal Practitioner, Frontline Medical Communications Inc., the US Government, or any of its agencies. This article may discuss unlabeled or investigational use of certain drugs. Please review the complete prescribing information for specific drugs or drug combinations—including indications, contraindications, warnings, and adverse effects—before administering pharmacologic therapy to patients.

Article PDF
Article PDF
Related Articles

*This article has been corrected to include a missing author.

 

Case Presentation. A 63-year-old male with well-controlled HIV (CD4 count 757, undetectable viral load), epilepsy, and hypertension presented to the VA Boston Healthcare System (VABHS) emergency department with 1 week of bilateral leg swelling and exertional shortness of breath. He reported having no fever, cough, chest pain, pain with inspiration and orthopnea. There was no personal or family history of pulmonary embolism. He reported weight gain but was unable to quantify how much. He also reported flare up of chronic knee pain, without swelling for which he had taken up to 4 tablets of naproxen daily for several weeks. His physical examination was notable for a heart rate of 105 beats per minute and bilateral pitting edema to his knees. Laboratory testing revealed a creatinine level of 2.5 mg/dL, which was increased from a baseline of 1.0 mg/dL (Table 1), and a urine protein-to-creatinine ratio of 7.8 mg/mg (Table 2). A renal ultrasound showed normal-sized kidneys without hydronephrosis or obstructing renal calculi. The patient was admitted for further workup of his dyspnea and acute kidney injury.

fed03609420_t.png

Jonathan Li, MD, Chief Medical Resident, VABHS and Beth Israel Deaconess Medical Center (BIDMC). Dr. William, based on the degree of proteinuria and edema, a diagnosis of nephrotic syndrome was made. How is nephrotic syndrome defined, and how is it distinguished from glomerulonephritis?

Jeffrey William, MD, Nephrologist, BIDMC, Assistant Professor of Medicine, Harvard Medical School. The pathophysiology of nephrotic disease and glomerulonephritis are quite distinct, resulting in symptoms and systemic manifestations that only slightly overlap. Glomerulonephritis is characterized by inflammation of the endothelial cells of the trilayered glomerular capillary, with a resulting active urine sediment with red blood cells, white blood cells, and casts. Nephrotic syndrome mostly affects the visceral epithelial cells of the glomerular capillary, commonly referred to as podocytes, and hence, the urine sediment in nephrotic disease is often inactive. Patients with nephrotic syndrome have nephrotic-range proteinuria (excretion of > 3.5 g per 24 h or a spot urine protein-creatinine ratio > 3.5 g in the steady state) and both hypoalbuminemia (< 3 g/dL) and peripheral edema. Lipiduria and hyperlipidemia are common findings in nephrotic syndrome but are not required for a clinical diagnosis.1 In contrast, glomerulonephritis is defined by a constellation of findings that include renal insufficiency (often indicated by an elevation in blood urea nitrogen and creatinine), hypertension, hematuria, and subnephrotic range proteinuria. In practice, patients may fulfill criteria of both nephrotic and nephritic syndromes, but the preponderance of clinical evidence often points one way or the other. In this case, nephrotic syndrome was diagnosed based on the urine protein-to-creatinine ratio of 7.8 mg/mg, hypoalbuminemia, and edema.

Dr. Li. What would be your first-line workup for evaluation of the etiology of this patient’s nephrotic syndrome?

Dr. William. Rather than memorizing a list of etiologies of nephrotic syndrome, it is essential to consider the pathophysiology of heavy proteinuria. Though the glomerular filtration barrier is extremely complex and defects in any component can cause proteinuria, disruption of the podocyte is often involved. Common disease processes that chiefly target the podocyte include minimal change disease, primary focal and segmental glomerulosclerosis (FSGS), and membranous nephropathy, all by differing mechanisms. Minimal change disease and idiopathic/primary FSGS are increasingly thought to be at differing points on a spectrum of the same disease.2 Secondary FSGS, on the other hand, is a progressive disease, commonly resulting from longstanding hypertension, diabetes mellitus, and obesity in adults. Membranous nephropathy can also be either primary or secondary. Primary membranous nephropathy is chiefly caused by a circulating IgG4 antibody to the podocyte membrane antigen PLA2R (M-type phospholipase A2 receptor), whereas secondary membranous nephropathy can be caused by a variety of systemic etiologies, including autoimmune disease (eg, systemic lupus erythematosus), certain malignancies, chronic infections (eg, hepatitis B and C), and many medications, including nonsteroidal anti-inflammatory drugs (NSAIDs).3-5 Paraprotein deposition diseases can also cause glomerular damage leading to nephrotic-range proteinuria.

 

 

Given these potential diagnoses, a careful history should be taken to assess exposures and recent medication use. Urine sediment evaluation is essential in the evaluation of nephrotic syndrome to determine if there is an underlying nephritic process. Select serologies may be sent to look for autoimmune disease, such as systemic lupus erythematosus and common viral exposures like hepatitis B or C. Serum and urine protein electrophoreses would be appropriate initial tests of suspected paraprotein-related diseases. Other serologies, such as antineutrophil cytoplasmic antibodies or antiglomerular basement membrane antibodies, would not necessarily be indicated here given the lack of hematuria and presence of nephrotic-range proteinuria.

Dr. Li. The initial evaluation was notable for an erythrocyte sedimentation rate > 120 (mm/h) and a weakly positive antinuclear antibody (ANA) titer of 1:40. The remainder of his initial workup did not reveal an etiology for his nephrotic syndrome (Table 3).

fed03609420_t2t3.png

Dr. William, is there a role for starting urgent empiric steroids in nephrotic syndrome while workup is ongoing? If so, do the severity of proteinuria and/or symptoms play a role or is this determination based on something else?

Dr. William. Edema is a primary symptom of nephrotic syndrome and can often be managed with diuretics alone. If a clear medication-mediated cause is suspected, discontinuation of this agent may result in spontaneous improvement without steroid treatment. However,in cases where an etiology is unclear and there are serious thrombotic complications requiring anticoagulation, and a renal biopsy is deemed to be too risky, then empiric steroid therapy may be necessary. Children with new-onset nephrotic syndrome are presumed to have minimal change disease, given its prevalence in this patient population, and are often given empiric steroids without obtaining a renal biopsy. However, in the adult population, a renal biopsy can typically be performed quickly and safely, with pathology results interpreted within days. In this patient, since a diagnosis was unclear and there was no contraindication to renal biopsy, a biopsy should be obtained before consideration of steroids.

Dr. Li. Steroids were deferred in anticipation of renal biopsy, which showed stage I membranous nephropathy, suggestive of membranous lupus nephritis Class V. The deposits were strongly reactive for immunoglobuline G (IgG), IgA, and complement 1q (C1q), showed co-dominant staining for IgG1, IgG2, and IgG3, and were weakly positive for the PLA2 receptor. Focal intimal arteritis in a small interlobular vessel was seen.

Dr. William, the pathology returned suggestive of lupus nephritis. Does the overall clinical picture fit with lupus nephritis?

Dr. William. Given the history and a rather low ANA, the diagnosis of lupus nephritis seems unlikely. The lack of IgG4 and PLA2R staining in the biopsy suggests that this membranous pattern on the biopsy is likely to be secondary to a systemic etiology, but further investigation should be pursued.

Dr. Li. The patient was discharged after the biopsy with a planned outpatient nephrology follow-up to discuss results and treatment. He was prescribed an oral diuretic, and his symptoms improved. Several days after discharge, he developed blurry vision and was evaluated in the Ophthalmology clinic. On fundoscopy, he was found to have acute papillitis, a form of optic neuritis. As part of initial evaluation of infectious etiologies of papillitis, ophthalmology recommended testing for syphilis.

 

 

Dr. Strymish, when we are considering secondary syphilis, what is the recommended approach to diagnostic testing?

Judith Strymish, MD, Infectious Diseases, BIDMC, Assistant Professor of Medicine, Harvard Medical School. The diagnosis of syphilis is usually made through serologic testing of blood specimens. Methods that detect the spirochete directly like dark-field smears are not readily available. Serologic tests include treponemal tests (eg, Treponema pallidum particle agglutination assay [TPPA]) and nontreponemal tests (eg, rapid plasma reagin [RPR]). One needs a confirmatory test because either test is associated with false positives. Either test can be done first. Most laboratories, including those at VABHS are now performing treponemal tests first as these have become more cost-effective.6 The TPPA treponemal test was found to have a lower false negative rate in primary syphilis compared with that of nontreponemal tests.7 Nontreponemal tests can be followed for response to therapy. If a patient has a history of treated syphilis, a nontreponemal test should be sent, since the treponemal test will remain positive for life.

If there is clinical concern for neurosyphilis, cerebrospinal fluid fluorescent (CSF) treponemal antibody needs to be sampled and sent for the nontreponemal venereal disease research laboratory (VDRL) test. The VDRL is highly specific for neurosyphilis but not as sensitive. Cerebrospinal fluid fluorescent treponemal antibody (CSF FTA) may also be sent; it is very sensitive but not very specific for neurosyphilis.

Dr. Li. An RPR returned positive at 1:512 (was negative 14 months prior on a routine screening test), with positive reflex TPPA (Table 4). A diagnosis of secondary syphilis was made. Dr. Strymish, at this point, what additional testing and treatment is necessary?

fed03609420_t4.png

Dr. Strymish. With papillitis and a very high RPR, we need to assume that he has ophthalmic syphilis. This can occur in any stage of syphilis, but his eye findings and high RPR are consistent with secondary syphilis. Ophthalmic syphilis has been on the upswing, even more than is expected with recent increases in syphilis cases.8 Ophthalmic syphilis is considered a form of neurosyphilis. A lumbar puncture and treatment for neurosyphilis is recommended.9,10

Dr. Li. A lumbar puncture was performed, and his CSF was VDRL positive. This confirmed a diagnosis of neurosyphilis (Table 4). The patient was treated for neurosyphilis with IV penicillin. The patient shared that he had episodes of unprotected oral sexual activity within the past year and approximately 1 year ago, he came in close contact (but no sexual activity) with a person who had a rash consistent with syphilis.Dr. William, syphilis would be a potential unifying diagnosis of his renal and ophthalmologic manifestations. Is syphilis known to cause membranous nephropathy?

Dr. William. Though it is uncommon, the nephrotic syndrome is a well-described complication of secondary syphilis.11,12 Syphilis has been shown to cause nephrotic syndrome in a variety of ways. Case reports abound linking syphilis to minimal change disease and other glomerular diseases.13,14 A case report from 1993 shows a membranous pattern of glomerular disease similar to this case.15 As a form of secondary membranous nephropathy, the immunofluorescence pattern can demonstrate staining similar to the “full house” seen in lupus nephritis (IgA, IgM, and C1q, in addition to IgG and C3).16 This explains the initial interpretation of this patient’s biopsy, as lupus nephritis would be a much more common etiology of secondary membranous nephropathy than is acute syphilis with this immunofluorescence pattern. However, the data in this case are highly suggestive of a causal relationship between secondary syphilis and membranous nephropathy.

Dr. Li. Dr. Strymish, how should this patient be screened for syphilis reinfection, and at what intervals would you recommend?

Dr. Strymish. He will need follow-up testing to make sure that his syphilis is effectively treated. If CSF pleocytosis was present initially, a CSF examination should be repeated every 6 months until the cell count is normal. He will also need follow-up for normalization of his RPR. Persons with HIV infection and primary or secondary syphilis should be evaluated clinically and serologically for treatment failure at 3, 6, 9, 12, and 24 months after therapy according to US Centers for Disease Control and Prevention guidelines.9

 

 

His treponemal test for syphilis will likely stay positive for life. His RPR should decrease significantly with effective treatment. It makes sense to screen with RPR alone as long as he continues to have risk factors for acquiring syphilis. Routine syphilis testing is recommended for pregnant women, sexually active men who have sex with men, sexually active persons with HIV, and persons taking PrEP (pre-exposure prophylaxis) for HIV prevention. He should be screened at least yearly for syphilis.

fed03609420_box.png

Dr. Li. Over the next several months, the patient’s creatinine normalized and his proteinuria resolved. His vision recovered, and he has had no further ophthalmologic complications.

Dr. William, what is his long-term renal prognosis? Do you expect that his acute episode of membranous nephropathy will have permanent effects on his renal function?

Dr. William. His rapid response to therapy for neurosyphilis provides evidence for this etiology of his renal dysfunction and glomerulonephritis. His long-term prognosis is quite good if the syphilis is the only reason for him to have renal disease. The renal damage is often reversible in these cases. However, given his prior extensive NSAID exposure and history of hypertension, he may be at higher risk for chronic kidney disease than an otherwise healthy patient, especially after an episode of acute kidney injury. Therefore, his renal function should continue to be monitored as an outpatient.

Acknowledgments

The authors thank this veteran for sharing his story and allowing us to learn from this unusual case for the benefit of our future patients.

*This article has been corrected to include a missing author.

 

Case Presentation. A 63-year-old male with well-controlled HIV (CD4 count 757, undetectable viral load), epilepsy, and hypertension presented to the VA Boston Healthcare System (VABHS) emergency department with 1 week of bilateral leg swelling and exertional shortness of breath. He reported having no fever, cough, chest pain, pain with inspiration and orthopnea. There was no personal or family history of pulmonary embolism. He reported weight gain but was unable to quantify how much. He also reported flare up of chronic knee pain, without swelling for which he had taken up to 4 tablets of naproxen daily for several weeks. His physical examination was notable for a heart rate of 105 beats per minute and bilateral pitting edema to his knees. Laboratory testing revealed a creatinine level of 2.5 mg/dL, which was increased from a baseline of 1.0 mg/dL (Table 1), and a urine protein-to-creatinine ratio of 7.8 mg/mg (Table 2). A renal ultrasound showed normal-sized kidneys without hydronephrosis or obstructing renal calculi. The patient was admitted for further workup of his dyspnea and acute kidney injury.

fed03609420_t.png

Jonathan Li, MD, Chief Medical Resident, VABHS and Beth Israel Deaconess Medical Center (BIDMC). Dr. William, based on the degree of proteinuria and edema, a diagnosis of nephrotic syndrome was made. How is nephrotic syndrome defined, and how is it distinguished from glomerulonephritis?

Jeffrey William, MD, Nephrologist, BIDMC, Assistant Professor of Medicine, Harvard Medical School. The pathophysiology of nephrotic disease and glomerulonephritis are quite distinct, resulting in symptoms and systemic manifestations that only slightly overlap. Glomerulonephritis is characterized by inflammation of the endothelial cells of the trilayered glomerular capillary, with a resulting active urine sediment with red blood cells, white blood cells, and casts. Nephrotic syndrome mostly affects the visceral epithelial cells of the glomerular capillary, commonly referred to as podocytes, and hence, the urine sediment in nephrotic disease is often inactive. Patients with nephrotic syndrome have nephrotic-range proteinuria (excretion of > 3.5 g per 24 h or a spot urine protein-creatinine ratio > 3.5 g in the steady state) and both hypoalbuminemia (< 3 g/dL) and peripheral edema. Lipiduria and hyperlipidemia are common findings in nephrotic syndrome but are not required for a clinical diagnosis.1 In contrast, glomerulonephritis is defined by a constellation of findings that include renal insufficiency (often indicated by an elevation in blood urea nitrogen and creatinine), hypertension, hematuria, and subnephrotic range proteinuria. In practice, patients may fulfill criteria of both nephrotic and nephritic syndromes, but the preponderance of clinical evidence often points one way or the other. In this case, nephrotic syndrome was diagnosed based on the urine protein-to-creatinine ratio of 7.8 mg/mg, hypoalbuminemia, and edema.

Dr. Li. What would be your first-line workup for evaluation of the etiology of this patient’s nephrotic syndrome?

Dr. William. Rather than memorizing a list of etiologies of nephrotic syndrome, it is essential to consider the pathophysiology of heavy proteinuria. Though the glomerular filtration barrier is extremely complex and defects in any component can cause proteinuria, disruption of the podocyte is often involved. Common disease processes that chiefly target the podocyte include minimal change disease, primary focal and segmental glomerulosclerosis (FSGS), and membranous nephropathy, all by differing mechanisms. Minimal change disease and idiopathic/primary FSGS are increasingly thought to be at differing points on a spectrum of the same disease.2 Secondary FSGS, on the other hand, is a progressive disease, commonly resulting from longstanding hypertension, diabetes mellitus, and obesity in adults. Membranous nephropathy can also be either primary or secondary. Primary membranous nephropathy is chiefly caused by a circulating IgG4 antibody to the podocyte membrane antigen PLA2R (M-type phospholipase A2 receptor), whereas secondary membranous nephropathy can be caused by a variety of systemic etiologies, including autoimmune disease (eg, systemic lupus erythematosus), certain malignancies, chronic infections (eg, hepatitis B and C), and many medications, including nonsteroidal anti-inflammatory drugs (NSAIDs).3-5 Paraprotein deposition diseases can also cause glomerular damage leading to nephrotic-range proteinuria.

 

 

Given these potential diagnoses, a careful history should be taken to assess exposures and recent medication use. Urine sediment evaluation is essential in the evaluation of nephrotic syndrome to determine if there is an underlying nephritic process. Select serologies may be sent to look for autoimmune disease, such as systemic lupus erythematosus and common viral exposures like hepatitis B or C. Serum and urine protein electrophoreses would be appropriate initial tests of suspected paraprotein-related diseases. Other serologies, such as antineutrophil cytoplasmic antibodies or antiglomerular basement membrane antibodies, would not necessarily be indicated here given the lack of hematuria and presence of nephrotic-range proteinuria.

Dr. Li. The initial evaluation was notable for an erythrocyte sedimentation rate > 120 (mm/h) and a weakly positive antinuclear antibody (ANA) titer of 1:40. The remainder of his initial workup did not reveal an etiology for his nephrotic syndrome (Table 3).

fed03609420_t2t3.png

Dr. William, is there a role for starting urgent empiric steroids in nephrotic syndrome while workup is ongoing? If so, do the severity of proteinuria and/or symptoms play a role or is this determination based on something else?

Dr. William. Edema is a primary symptom of nephrotic syndrome and can often be managed with diuretics alone. If a clear medication-mediated cause is suspected, discontinuation of this agent may result in spontaneous improvement without steroid treatment. However,in cases where an etiology is unclear and there are serious thrombotic complications requiring anticoagulation, and a renal biopsy is deemed to be too risky, then empiric steroid therapy may be necessary. Children with new-onset nephrotic syndrome are presumed to have minimal change disease, given its prevalence in this patient population, and are often given empiric steroids without obtaining a renal biopsy. However, in the adult population, a renal biopsy can typically be performed quickly and safely, with pathology results interpreted within days. In this patient, since a diagnosis was unclear and there was no contraindication to renal biopsy, a biopsy should be obtained before consideration of steroids.

Dr. Li. Steroids were deferred in anticipation of renal biopsy, which showed stage I membranous nephropathy, suggestive of membranous lupus nephritis Class V. The deposits were strongly reactive for immunoglobuline G (IgG), IgA, and complement 1q (C1q), showed co-dominant staining for IgG1, IgG2, and IgG3, and were weakly positive for the PLA2 receptor. Focal intimal arteritis in a small interlobular vessel was seen.

Dr. William, the pathology returned suggestive of lupus nephritis. Does the overall clinical picture fit with lupus nephritis?

Dr. William. Given the history and a rather low ANA, the diagnosis of lupus nephritis seems unlikely. The lack of IgG4 and PLA2R staining in the biopsy suggests that this membranous pattern on the biopsy is likely to be secondary to a systemic etiology, but further investigation should be pursued.

Dr. Li. The patient was discharged after the biopsy with a planned outpatient nephrology follow-up to discuss results and treatment. He was prescribed an oral diuretic, and his symptoms improved. Several days after discharge, he developed blurry vision and was evaluated in the Ophthalmology clinic. On fundoscopy, he was found to have acute papillitis, a form of optic neuritis. As part of initial evaluation of infectious etiologies of papillitis, ophthalmology recommended testing for syphilis.

 

 

Dr. Strymish, when we are considering secondary syphilis, what is the recommended approach to diagnostic testing?

Judith Strymish, MD, Infectious Diseases, BIDMC, Assistant Professor of Medicine, Harvard Medical School. The diagnosis of syphilis is usually made through serologic testing of blood specimens. Methods that detect the spirochete directly like dark-field smears are not readily available. Serologic tests include treponemal tests (eg, Treponema pallidum particle agglutination assay [TPPA]) and nontreponemal tests (eg, rapid plasma reagin [RPR]). One needs a confirmatory test because either test is associated with false positives. Either test can be done first. Most laboratories, including those at VABHS are now performing treponemal tests first as these have become more cost-effective.6 The TPPA treponemal test was found to have a lower false negative rate in primary syphilis compared with that of nontreponemal tests.7 Nontreponemal tests can be followed for response to therapy. If a patient has a history of treated syphilis, a nontreponemal test should be sent, since the treponemal test will remain positive for life.

If there is clinical concern for neurosyphilis, cerebrospinal fluid fluorescent (CSF) treponemal antibody needs to be sampled and sent for the nontreponemal venereal disease research laboratory (VDRL) test. The VDRL is highly specific for neurosyphilis but not as sensitive. Cerebrospinal fluid fluorescent treponemal antibody (CSF FTA) may also be sent; it is very sensitive but not very specific for neurosyphilis.

Dr. Li. An RPR returned positive at 1:512 (was negative 14 months prior on a routine screening test), with positive reflex TPPA (Table 4). A diagnosis of secondary syphilis was made. Dr. Strymish, at this point, what additional testing and treatment is necessary?

fed03609420_t4.png

Dr. Strymish. With papillitis and a very high RPR, we need to assume that he has ophthalmic syphilis. This can occur in any stage of syphilis, but his eye findings and high RPR are consistent with secondary syphilis. Ophthalmic syphilis has been on the upswing, even more than is expected with recent increases in syphilis cases.8 Ophthalmic syphilis is considered a form of neurosyphilis. A lumbar puncture and treatment for neurosyphilis is recommended.9,10

Dr. Li. A lumbar puncture was performed, and his CSF was VDRL positive. This confirmed a diagnosis of neurosyphilis (Table 4). The patient was treated for neurosyphilis with IV penicillin. The patient shared that he had episodes of unprotected oral sexual activity within the past year and approximately 1 year ago, he came in close contact (but no sexual activity) with a person who had a rash consistent with syphilis.Dr. William, syphilis would be a potential unifying diagnosis of his renal and ophthalmologic manifestations. Is syphilis known to cause membranous nephropathy?

Dr. William. Though it is uncommon, the nephrotic syndrome is a well-described complication of secondary syphilis.11,12 Syphilis has been shown to cause nephrotic syndrome in a variety of ways. Case reports abound linking syphilis to minimal change disease and other glomerular diseases.13,14 A case report from 1993 shows a membranous pattern of glomerular disease similar to this case.15 As a form of secondary membranous nephropathy, the immunofluorescence pattern can demonstrate staining similar to the “full house” seen in lupus nephritis (IgA, IgM, and C1q, in addition to IgG and C3).16 This explains the initial interpretation of this patient’s biopsy, as lupus nephritis would be a much more common etiology of secondary membranous nephropathy than is acute syphilis with this immunofluorescence pattern. However, the data in this case are highly suggestive of a causal relationship between secondary syphilis and membranous nephropathy.

Dr. Li. Dr. Strymish, how should this patient be screened for syphilis reinfection, and at what intervals would you recommend?

Dr. Strymish. He will need follow-up testing to make sure that his syphilis is effectively treated. If CSF pleocytosis was present initially, a CSF examination should be repeated every 6 months until the cell count is normal. He will also need follow-up for normalization of his RPR. Persons with HIV infection and primary or secondary syphilis should be evaluated clinically and serologically for treatment failure at 3, 6, 9, 12, and 24 months after therapy according to US Centers for Disease Control and Prevention guidelines.9

 

 

His treponemal test for syphilis will likely stay positive for life. His RPR should decrease significantly with effective treatment. It makes sense to screen with RPR alone as long as he continues to have risk factors for acquiring syphilis. Routine syphilis testing is recommended for pregnant women, sexually active men who have sex with men, sexually active persons with HIV, and persons taking PrEP (pre-exposure prophylaxis) for HIV prevention. He should be screened at least yearly for syphilis.

fed03609420_box.png

Dr. Li. Over the next several months, the patient’s creatinine normalized and his proteinuria resolved. His vision recovered, and he has had no further ophthalmologic complications.

Dr. William, what is his long-term renal prognosis? Do you expect that his acute episode of membranous nephropathy will have permanent effects on his renal function?

Dr. William. His rapid response to therapy for neurosyphilis provides evidence for this etiology of his renal dysfunction and glomerulonephritis. His long-term prognosis is quite good if the syphilis is the only reason for him to have renal disease. The renal damage is often reversible in these cases. However, given his prior extensive NSAID exposure and history of hypertension, he may be at higher risk for chronic kidney disease than an otherwise healthy patient, especially after an episode of acute kidney injury. Therefore, his renal function should continue to be monitored as an outpatient.

Acknowledgments

The authors thank this veteran for sharing his story and allowing us to learn from this unusual case for the benefit of our future patients.

References

1. Rennke H, Denker BM. Renal Pathophysiology: The Essentials. 6th ed. Philadelphia: Lippincott Williams & Wilkins; 2014.

2. Maas RJ, Deegens JK, Smeets B, Moeller MJ, Wetzels JF. Minimal change disease and idiopathic FSGS: manifestations of the same disease. Nat Rev Nephrol. 2016;12(12):768-776.

3. Beck LH Jr, Bonegio RG, Lambeau G, et al. M-type phospholipase A2 receptor as target antigen in idiopathic membranous nephropathy. N Engl J Med. 2009;361(1):11-21.

4. Rennke HG. Secondary membranoproliferative glomerulonephritis. Kidney Int. 1995;47(2):643-656.

5. Nawaz FA, Larsen CP, Troxell ML. Membranous nephropathy and nonsteroidal anti-inflammatory agents. Am J Kidney Dis. 2013;62(5):1012-1017.

6. Pillay A. Centers for Disease Control and Prevention Syphilis Summit—Diagnostics and laboratory issues. Sex Transm Dis. 2018;45(9S)(suppl 1):S13-S16. 

7. Levett PN, Fonseca K, Tsang RS, et al. Canadian Public Health Laboratory Network laboratory guidelines for the use of serological tests (excluding point-of-care tests) for the diagnosis of syphilis in Canada. Can J Infect Dis Med Microbiol. 2015;26(suppl A):6A-12A. 

8. Oliver SE, Aubin M, Atwell L, et al. Ocular syphilis—eight jurisdictions, United States, 2014-2015. MMWR Morb Mortal Wkly Rep. 2016;65(43):1185-1188.

9. Workowski KA, Bolan GA. Sexually transmitted diseases treatment guidelines, 2015. MMWR Recommendations and Reports 2015;64(RR3):1-137. [Erratum in MMWR Recomm Rep. 2015;64(33):924.]

10. US Centers for Disease Control and Prevention. Clinical advisory: ocular syphilis in the United States. https://www.cdc.gov/std/syphilis/clinicaladvisoryos2015.htm. Updated March 24, 2016. Accessed August 12, 2019.

11. Braunstein GD, Lewis EJ, Galvanek EG, Hamilton A, Bell WR. The nephrotic syndrome associated with secondary syphilis: an immune deposit disease. Am J Med. 1970;48:643-648.1.

12. Handoko ML, Duijvestein M, Scheepstra CG, de Fijter CW. Syphilis: a reversible cause of nephrotic syndrome. BMJ Case Rep. 2013;2013:pii:bcr2012008279

13. Krane NK, Espenan P, Walker PD, Bergman SM, Wallin JD. Renal disease and syphilis: a report of nephrotic syndrome with minimal change disease. Am J Kidney Dis. 1987;9(2):176-179.

14. Bhorade MS, Carag HB, Lee HJ, Potter EV, Dunea G. Nephropathy of secondary syphilis: a clinical and pathological spectrum. JAMA. 1971;216(7):1159-1166.

15. Hunte W, al-Ghraoui F, Cohen RJ. Secondary syphilis and the nephrotic syndrome. J Am Soc Nephrol. 1993;3(7):1351-1355.

16. Gamble CN, Reardan JB. Immunopathogenesis of syphilitic glomerulonephritis. Elution of antitreponemal antibody from glomerular immune-complex deposits. N Engl J Med. 1975;292(9):449-454.

References

1. Rennke H, Denker BM. Renal Pathophysiology: The Essentials. 6th ed. Philadelphia: Lippincott Williams & Wilkins; 2014.

2. Maas RJ, Deegens JK, Smeets B, Moeller MJ, Wetzels JF. Minimal change disease and idiopathic FSGS: manifestations of the same disease. Nat Rev Nephrol. 2016;12(12):768-776.

3. Beck LH Jr, Bonegio RG, Lambeau G, et al. M-type phospholipase A2 receptor as target antigen in idiopathic membranous nephropathy. N Engl J Med. 2009;361(1):11-21.

4. Rennke HG. Secondary membranoproliferative glomerulonephritis. Kidney Int. 1995;47(2):643-656.

5. Nawaz FA, Larsen CP, Troxell ML. Membranous nephropathy and nonsteroidal anti-inflammatory agents. Am J Kidney Dis. 2013;62(5):1012-1017.

6. Pillay A. Centers for Disease Control and Prevention Syphilis Summit—Diagnostics and laboratory issues. Sex Transm Dis. 2018;45(9S)(suppl 1):S13-S16. 

7. Levett PN, Fonseca K, Tsang RS, et al. Canadian Public Health Laboratory Network laboratory guidelines for the use of serological tests (excluding point-of-care tests) for the diagnosis of syphilis in Canada. Can J Infect Dis Med Microbiol. 2015;26(suppl A):6A-12A. 

8. Oliver SE, Aubin M, Atwell L, et al. Ocular syphilis—eight jurisdictions, United States, 2014-2015. MMWR Morb Mortal Wkly Rep. 2016;65(43):1185-1188.

9. Workowski KA, Bolan GA. Sexually transmitted diseases treatment guidelines, 2015. MMWR Recommendations and Reports 2015;64(RR3):1-137. [Erratum in MMWR Recomm Rep. 2015;64(33):924.]

10. US Centers for Disease Control and Prevention. Clinical advisory: ocular syphilis in the United States. https://www.cdc.gov/std/syphilis/clinicaladvisoryos2015.htm. Updated March 24, 2016. Accessed August 12, 2019.

11. Braunstein GD, Lewis EJ, Galvanek EG, Hamilton A, Bell WR. The nephrotic syndrome associated with secondary syphilis: an immune deposit disease. Am J Med. 1970;48:643-648.1.

12. Handoko ML, Duijvestein M, Scheepstra CG, de Fijter CW. Syphilis: a reversible cause of nephrotic syndrome. BMJ Case Rep. 2013;2013:pii:bcr2012008279

13. Krane NK, Espenan P, Walker PD, Bergman SM, Wallin JD. Renal disease and syphilis: a report of nephrotic syndrome with minimal change disease. Am J Kidney Dis. 1987;9(2):176-179.

14. Bhorade MS, Carag HB, Lee HJ, Potter EV, Dunea G. Nephropathy of secondary syphilis: a clinical and pathological spectrum. JAMA. 1971;216(7):1159-1166.

15. Hunte W, al-Ghraoui F, Cohen RJ. Secondary syphilis and the nephrotic syndrome. J Am Soc Nephrol. 1993;3(7):1351-1355.

16. Gamble CN, Reardan JB. Immunopathogenesis of syphilitic glomerulonephritis. Elution of antitreponemal antibody from glomerular immune-complex deposits. N Engl J Med. 1975;292(9):449-454.

Issue
Federal Practitioner - 36(9)a
Issue
Federal Practitioner - 36(9)a
Page Number
420-424
Page Number
420-424
Publications
Publications
Topics
Article Type
Sections
Disallow All Ads
Content Gating
No Gating (article Unlocked/Free)
Alternative CME
Disqus Comments
Default
Eyebrow Default
VA Boston Medical Forum
Use ProPublica
Hide sidebar & use full width
render the right sidebar.
Article PDF Media

A Veteran With Acute Progressive Encephalopathy of Unknown Etiology

Article Type
Changed
Thu, 01/10/2019 - 16:31

Case Presentation. A 70-year-old US Marine Corps veteran of the Vietnam War with no significant past medical history was brought by ambulance to VA Boston Healthcare System (VABHS) after being found on the floor at home by his wife, awake, but with minimally coherent speech. He was moving all extremities, and there was no loss of bowel or bladder continence. He had last been seen well by his wife 30 minutes prior. When emergency medical services arrived, his finger stick blood glucose and vital signs were within normal range. In the emergency department, he was able to state his first name but then continuously repeated “7/11” to other questions. A neurologic examination revealed intact cranial nerves, full strength in all extremities, and normal reflexes. A National Institute of Health Stroke Scale (NIHSS) was 3, and a code stroke was activated. At the time of presentation, the patient was an active smoker of 15 cigarettes per day for 50 years and did not use alcohol or recreational drugs.

► Jonathan Li, MD, Chief Medical Resident, VABHS and Beth Israel Deaconess Medical Center (BIDMC). Dr. Fehnel, the patient’s medical team was most worried about a transient ischemic attack (TIA) or cerebrovascular accident (CVA). Is his presentation consistent with these diagnoses, and what else is on your differential diagnosis?

►Corey R. Fehnel, MD, Neuro-Intensivist, BIDMC, and Assistant Professor of Neurology, Harvard Medical School. This patient is presenting with what appears to be an acute encephalopathy—a sudden onset of global alteration in mental status. The most worrisome underlying etiology for this presentation would be acute stroke, but this is an uncommon cause of acute encephalopathy. The differential diagnosis at this stage remains broad, but a careful neurologic examination can help narrow the possibilities. In particular, I would aim to differentiate an apparent language deficit (ie, aphasia) from a deficit of attention. A key finding that may help is the ability to name high- or low-frequency objects. If the patient can successfully name objects, aphasia is less likely. Based on the limited examination at present, the patient produces some normal speech, but perseverates; therefore, the finding remains nonspecific. My leading diagnoses are complex partial seizure and toxic/metabolic encephalopathy.

►Dr. Li. This patient’s NIHSS score is 3. How do you use this score in your management decisions for the patient?

►Dr. Fehnel. The NIHSS is a useful tool for gauging severity of ischemic and hemorrhagic stroke. However the score is not specific for establishing the diagnosis of stroke. Many common and chronic neurologic problems will score on the NIHSS, so it can never be interpreted in isolation. If the clinical history and complete neurologic examination support the diagnosis of stroke, then the NIHSS can be used with the understanding that it is biased toward anterior circulation strokes, and posterior circulation strokes will score lower even though they are potentially more life threatening.1 In this case, even though a complex partial seizure appears more likely, it is difficult to rule out the possibility of an acute stroke affecting the thalamus or, less likely, a distal middle cerebral artery occlusion. I would consider IV thrombolysis pending further history and neuroimaging results.

►Dr. Li. Initial laboratory data include a hemoglobin of 12.8 mg/dL. The white cell count, platelet count, chemistry panel, liver function tests, thyroid-stimulating hormone, and troponin were within normal range (Table 1). 

fed00119030_t1.png
Serum and urine toxicology screening was negative. Computed tomography (CT) of the head and CT angiogram of the head and neck were negative for acute intracranial abnormality or vessel occlusion. The patient’s mental status improved shortly after, and he was able to recall going to a 7-Eleven convenience store with friends in the morning but had no recollection of subsequent events. He reported no fevers, headache, weakness, vision problems, paresthesia, dysphagia, or gait imbalance. A decision was made to admit the patient for monitoring and workup.

 

 

Do you agree with inpatient workup for this patient whose mental status has now returned to baseline? If so, what workup would you pursue next?

Dr. Fehnel. This patient requires inpatient admission to further evaluate the underlying etiology for his acute change in mental status. The improvement of his presenting deficit and largely normal neurovascular imaging make a neurovascular etiology less likely, but a careful risk factor evaluation for CVA/TIA should be performed, including continuous cardiac telemetry to detect atrial fibrillation. Magnetic resonance imaging (MRI) of the brain should be performed to rule out occult stroke and evaluate for a structural etiology given the more likely diagnosis of complex partial seizure. An electroencephalogram (EEG), preferably 24-hour continuous recording, should be performed. Without a clear toxic or metabolic etiology thus far to explain his acute global waxing-waning alteration in mental status and likely new-onset complex partial seizures, I would also pursue lumbar puncture for cerebrospinal fluid examination.

►Dr. Li. The hospital course was notable for episodes of acute combativeness and confusion. An MRI of the brain was deferred due to reports from the patient’s family of retained shrapnel in the lumbar spine. Routine EEG showed no seizure activity. This was followed by continuous video EEG monitoring, which showed subclinical seizure activity with a right temporal focus. He was started on valproic acid with improvement in his agitation, though confusion continued. He was discharged to an inpatient geriatric psychiatry nursing home with diagnosis of seizures and acute delirium.

Dr. Fehnel, seizures are often part of the workup for unexplained encephalopathy. In this case, the routine EEG was unrevealing, while the continuous video EEG proved valuable. In what situations would you pursue a continuous video EEG in addition to a routine EEG?

►Dr. Fehnel. EEG monitoring is only as good as the window of time during which the study is performed. If the suspicious clinical event is captured during a routine recording or an area of focal slowing is detected, a shorter study may be entirely sufficient. However, in cases where there is no clear alternative explanation, a patient’s mental status does not return to normal, or in the setting of mental status fluctuations without explanation, continuous video-EEG monitoring for at least 24 hours is indicated. While the prolonged study raises sensitivity, the exact duration of EEG recording required outside of the intensive care unit setting remains debated.2

►Dr. Li. If his encephalopathy were due to seizures alone, I would expect improvement in his mental status during interictal periods, which does not appear to be the case here. Do you feel the seizures alone can explain his encephalopathy?

Dr. Fehnel. Complex partial seizures and the medications used to treat them can confound the examination of patients during the interictal period. We commonly debate postictal encephalopathy vs residual effect of benzodiazepines and rapid dose escalation of antiepileptic drugs as culprit in a patient’s prolonged alteration in mental status. Serial clinical examinations, continuous EEG monitoring to rule out ongoing subclinical seizures when appropriate, and judicious use of potentially sedating medications is the most helpful approach. The key issue here is the bimodal distribution of new-onset seizures. Among children there is a higher incidence of genetically related seizure disorders; whereas among adults, “acquired” and structural etiologies are more common. For this case, a more careful evaluation of acquired/structural etiologies for new-onset seizures is indicated.

►Dr. Li. At the geriatric psychiatry nursing home, the patient continued to be combative and refused medications. He was readmitted to the VABHS with encephalopathy of unclear etiology. An expanded encephalopathy workup was unrevealing (Table 2). 

fed00119030_t23.png
A CT of the lumbar spine was obtained, which showed only small surgical clips, and he was felt to be safe to undergo an MRI. An MRI of the brain with contrast showed generalized cerebral atrophy and evidence of small vessel ischemia but no acute pathology. His mental status continued to worsen with diminished speech output and decreased interaction with his health care providers. He was no longer able to state his name when asked. At this point, lumbar puncture was pursued (Table 3).

 

 

Dr. Fehnel, this patient’s initial cerebrospinal fluid (CSF) cell count and chemistries were completely normal. Is this sufficient to rule out encephalitis? If not, what other diagnostic tests would you send?

►Dr. Fehnel. A fully normal CSF profile reduces the likelihood of a broad range of neuro-infectious etiologies but does not completely rule those out. For example, there are reports of herpes simplex virus (HSV) encephalitis producing relatively normal profiles and even negative polymerase chain reaction assays for antibodies to HSV if the specimen is obtained very early in the course of the disease.3,4 That was not the case here as the CSF was obtained several days after his initial presentation. Given this patient’s clinical syndrome, normal CSF findings, and long smoking history without regular screening examinations, I would send a CSF specimen screening for paraneoplastic and autoimmune encephalitis. Most autoimmune encephalitis syndromes are associated with CSF lymphocytic pleocytosis or slight elevation in CSF protein levels. This patient’s diagnosis is most likely an anti-Hu paraneoplastic syndrome, which can be distinguished from other autoimmune and paraneoplastic processes by the characteristically normal CSF profile. Anti-Hu antibodies are strongly associated with non-small cell lung cancer (NSCLC). I would, therefore, also obtain more advanced chest imaging.

►Dr. Li. An autoimmune and paraneoplastic encephalitis panel was sent. While this send-out panel was pending, a CT torso was obtained to evaluate for occult malignancy in light of his significant smoking history. This showed a 3-cm spiculated mass originating from the left hilum. Bronchoalviolar lavage washings returned positive for small cell lung cancer. 

fed00119030_box.png
The CSF autoimmune encephalitis panel returned positive for anti-Hu antibody. A final diagnosis of paraneoplastic encephalitis secondary to small cell lung cancer was made.

Dr. Fehnel, can you explain the mechanism by which certain neoplasms can cause encephalitis?

Dr. Fehnel. Onconeuronal antibodies Hu (NSCLC) and Ma2 (testicular seminoma), when identified, are strongly associated with the presence of an underlying malignancy. The work of Dr. Josep Dalmau and others in this area has dramatically improved our understanding of these syndromes over the past 25 years.5 The exact mechanism is not fully understood but is thought to be mediated by cytotoxic T-cell response directed at the malignancy itself with homology to intraneuronal structures, which are readily absorbed and result in neuronal cell death.6

►Dr. Li. Is there a specific treatment for paraneoplastic encephalitis, other than treating the underlying malignancy?

►Dr. Fehnel. Early treatment is associated with improved outcome and should not be delayed while waiting for laboratory confirmation in cases of high clinical suspicion. Treatment directed at the underlying tumor is the mainstay along with less specific immunosuppressive agents. Unfortunately Anti-Hu (as well as Ma2) antibodies are intraneuronal and less responsive to standard treatments relative to other paraneoplastic auto-antibodies identified on the cell surface. Immunosuppressive agents typically used in this setting include high-dose IV methylprednisolone, IV immune globulin (IVIG), rituximab, and cyclophosphamide.7

►Dr. Li. The patient was started on IVIG, methylprednisolone, cisplatin, and etoposide. His course was complicated by aspiration pneumonia, autonomic dysfunction causing tachy- and brady-arrhythmias, urosepsis, worsening somnolence, chemotherapy-induced neutropenic fevers, and ultimately septic shock. The palliative care team was closely involved throughout the final stages of his hospital course. After multiple family meetings, the patient was transitioned to comfort-focused care per family discussion and died 6 weeks after his initial presentation.

 

 

This patient had a very atypical initial presentation of small cell lung cancer. Despite the fact that a diagnosis eluded his doctors, they persisted in a thoughtful and exhaustive workup and through this perseverance were able to make the final diagnosis, which serves as an important learning case for us all.

Acknowledgments

We thank the family of this veteran for sharing his story and allowing us to learn from this case for the benefit of our future patients. We also thank Dr. Michelle Hankins, who provided oncologic expertise.

References

1. Heldner MR, Zubler C, Mattle HP, et al. National Institutes of Health stroke scale score and vessel occlusion in 2152 patients with acute ischemic stroke. Stroke. 2013;44(4):1153-1157.

2. Herman ST, Abend NS, Bleck TP, et al; Critical Care Continuous EEG Task Force of the American Clinical Neurophysiology Society. Consensus statement on continuous EEG in critically ill adults and children, part I: indications. J Clin Neurophysiol. 2015;32(2):87-95.

3. DeBiasi RL, Kleinschmidt-DeMasters BK, Weinberg A, Tyler KL. Use of PCR for the diagnosis of herpesvirus infections of the central nervous system. J Clin Virol. 2002;25(suppl 1):S5-S11.

4. Buerger KJ, Zerr K, Salazar R. An unusual presentation of herpes simplex encephalitis with negative PCR. BMJ Case Rep. 2015;2015:pii:bcr201521052.

5. Graus F, Titulaer MJ, Balu R, et al. A clinical approach to diagnosis of autoimmune encephalitis. Lancet Neurol. 2016;15(4):391-404.

6. Greenlee JE, Clawson SA, Hill KE, et al. Neuronal uptake of anti-Hu antibody, but not anti-Ri antibody, leads to cell death in brain slice cultures. J Neuroinflammation. 2014;11:160.

7. Bradshaw MJ, Linnoila JJ. An overview of autoimmune and paraneoplastic encephalitides. Semin Neurol. 2018;38(3):330-343.

Article PDF
Author and Disclosure Information

Dr. Breu is a Hospitalist and the Director of Resident Education at VA Boston Healthcare System and an Assistant Professor of Medicine at Harvard University in Massachusetts. He supervises the VA Boston Medical Forum chief resident case conferences. All patients or their surrogate decision makers understand and have signed appropriate patient release forms. This article has received an abbreviated peer review.
Correspondence: Anthony Breu (anthony. breu@va.gov)
* Co-lead authors.

Author disclosures
The authors report no actual or potential conflicts of interest with regard to this article.

Disclaimer
The opinions expressed herein are those of the authors and do not necessarily reflect those of Federal Practitioner, Frontline Medical Communications Inc., the US Government, or any of its agencies. This article may discuss unlabeled or investigational use of certain drugs. Please review the complete prescribing information for specific drugs or drug combinations—including indications, contraindications, warnings, and adverse effects—before administering pharmacologic therapy to patients.

Issue
Federal Practitioner - 36(1)
Publications
Topics
Page Number
30-34
Sections
Author and Disclosure Information

Dr. Breu is a Hospitalist and the Director of Resident Education at VA Boston Healthcare System and an Assistant Professor of Medicine at Harvard University in Massachusetts. He supervises the VA Boston Medical Forum chief resident case conferences. All patients or their surrogate decision makers understand and have signed appropriate patient release forms. This article has received an abbreviated peer review.
Correspondence: Anthony Breu (anthony. breu@va.gov)
* Co-lead authors.

Author disclosures
The authors report no actual or potential conflicts of interest with regard to this article.

Disclaimer
The opinions expressed herein are those of the authors and do not necessarily reflect those of Federal Practitioner, Frontline Medical Communications Inc., the US Government, or any of its agencies. This article may discuss unlabeled or investigational use of certain drugs. Please review the complete prescribing information for specific drugs or drug combinations—including indications, contraindications, warnings, and adverse effects—before administering pharmacologic therapy to patients.

Author and Disclosure Information

Dr. Breu is a Hospitalist and the Director of Resident Education at VA Boston Healthcare System and an Assistant Professor of Medicine at Harvard University in Massachusetts. He supervises the VA Boston Medical Forum chief resident case conferences. All patients or their surrogate decision makers understand and have signed appropriate patient release forms. This article has received an abbreviated peer review.
Correspondence: Anthony Breu (anthony. breu@va.gov)
* Co-lead authors.

Author disclosures
The authors report no actual or potential conflicts of interest with regard to this article.

Disclaimer
The opinions expressed herein are those of the authors and do not necessarily reflect those of Federal Practitioner, Frontline Medical Communications Inc., the US Government, or any of its agencies. This article may discuss unlabeled or investigational use of certain drugs. Please review the complete prescribing information for specific drugs or drug combinations—including indications, contraindications, warnings, and adverse effects—before administering pharmacologic therapy to patients.

Article PDF
Article PDF

Case Presentation. A 70-year-old US Marine Corps veteran of the Vietnam War with no significant past medical history was brought by ambulance to VA Boston Healthcare System (VABHS) after being found on the floor at home by his wife, awake, but with minimally coherent speech. He was moving all extremities, and there was no loss of bowel or bladder continence. He had last been seen well by his wife 30 minutes prior. When emergency medical services arrived, his finger stick blood glucose and vital signs were within normal range. In the emergency department, he was able to state his first name but then continuously repeated “7/11” to other questions. A neurologic examination revealed intact cranial nerves, full strength in all extremities, and normal reflexes. A National Institute of Health Stroke Scale (NIHSS) was 3, and a code stroke was activated. At the time of presentation, the patient was an active smoker of 15 cigarettes per day for 50 years and did not use alcohol or recreational drugs.

► Jonathan Li, MD, Chief Medical Resident, VABHS and Beth Israel Deaconess Medical Center (BIDMC). Dr. Fehnel, the patient’s medical team was most worried about a transient ischemic attack (TIA) or cerebrovascular accident (CVA). Is his presentation consistent with these diagnoses, and what else is on your differential diagnosis?

►Corey R. Fehnel, MD, Neuro-Intensivist, BIDMC, and Assistant Professor of Neurology, Harvard Medical School. This patient is presenting with what appears to be an acute encephalopathy—a sudden onset of global alteration in mental status. The most worrisome underlying etiology for this presentation would be acute stroke, but this is an uncommon cause of acute encephalopathy. The differential diagnosis at this stage remains broad, but a careful neurologic examination can help narrow the possibilities. In particular, I would aim to differentiate an apparent language deficit (ie, aphasia) from a deficit of attention. A key finding that may help is the ability to name high- or low-frequency objects. If the patient can successfully name objects, aphasia is less likely. Based on the limited examination at present, the patient produces some normal speech, but perseverates; therefore, the finding remains nonspecific. My leading diagnoses are complex partial seizure and toxic/metabolic encephalopathy.

►Dr. Li. This patient’s NIHSS score is 3. How do you use this score in your management decisions for the patient?

►Dr. Fehnel. The NIHSS is a useful tool for gauging severity of ischemic and hemorrhagic stroke. However the score is not specific for establishing the diagnosis of stroke. Many common and chronic neurologic problems will score on the NIHSS, so it can never be interpreted in isolation. If the clinical history and complete neurologic examination support the diagnosis of stroke, then the NIHSS can be used with the understanding that it is biased toward anterior circulation strokes, and posterior circulation strokes will score lower even though they are potentially more life threatening.1 In this case, even though a complex partial seizure appears more likely, it is difficult to rule out the possibility of an acute stroke affecting the thalamus or, less likely, a distal middle cerebral artery occlusion. I would consider IV thrombolysis pending further history and neuroimaging results.

►Dr. Li. Initial laboratory data include a hemoglobin of 12.8 mg/dL. The white cell count, platelet count, chemistry panel, liver function tests, thyroid-stimulating hormone, and troponin were within normal range (Table 1). 

fed00119030_t1.png
Serum and urine toxicology screening was negative. Computed tomography (CT) of the head and CT angiogram of the head and neck were negative for acute intracranial abnormality or vessel occlusion. The patient’s mental status improved shortly after, and he was able to recall going to a 7-Eleven convenience store with friends in the morning but had no recollection of subsequent events. He reported no fevers, headache, weakness, vision problems, paresthesia, dysphagia, or gait imbalance. A decision was made to admit the patient for monitoring and workup.

 

 

Do you agree with inpatient workup for this patient whose mental status has now returned to baseline? If so, what workup would you pursue next?

Dr. Fehnel. This patient requires inpatient admission to further evaluate the underlying etiology for his acute change in mental status. The improvement of his presenting deficit and largely normal neurovascular imaging make a neurovascular etiology less likely, but a careful risk factor evaluation for CVA/TIA should be performed, including continuous cardiac telemetry to detect atrial fibrillation. Magnetic resonance imaging (MRI) of the brain should be performed to rule out occult stroke and evaluate for a structural etiology given the more likely diagnosis of complex partial seizure. An electroencephalogram (EEG), preferably 24-hour continuous recording, should be performed. Without a clear toxic or metabolic etiology thus far to explain his acute global waxing-waning alteration in mental status and likely new-onset complex partial seizures, I would also pursue lumbar puncture for cerebrospinal fluid examination.

►Dr. Li. The hospital course was notable for episodes of acute combativeness and confusion. An MRI of the brain was deferred due to reports from the patient’s family of retained shrapnel in the lumbar spine. Routine EEG showed no seizure activity. This was followed by continuous video EEG monitoring, which showed subclinical seizure activity with a right temporal focus. He was started on valproic acid with improvement in his agitation, though confusion continued. He was discharged to an inpatient geriatric psychiatry nursing home with diagnosis of seizures and acute delirium.

Dr. Fehnel, seizures are often part of the workup for unexplained encephalopathy. In this case, the routine EEG was unrevealing, while the continuous video EEG proved valuable. In what situations would you pursue a continuous video EEG in addition to a routine EEG?

►Dr. Fehnel. EEG monitoring is only as good as the window of time during which the study is performed. If the suspicious clinical event is captured during a routine recording or an area of focal slowing is detected, a shorter study may be entirely sufficient. However, in cases where there is no clear alternative explanation, a patient’s mental status does not return to normal, or in the setting of mental status fluctuations without explanation, continuous video-EEG monitoring for at least 24 hours is indicated. While the prolonged study raises sensitivity, the exact duration of EEG recording required outside of the intensive care unit setting remains debated.2

►Dr. Li. If his encephalopathy were due to seizures alone, I would expect improvement in his mental status during interictal periods, which does not appear to be the case here. Do you feel the seizures alone can explain his encephalopathy?

Dr. Fehnel. Complex partial seizures and the medications used to treat them can confound the examination of patients during the interictal period. We commonly debate postictal encephalopathy vs residual effect of benzodiazepines and rapid dose escalation of antiepileptic drugs as culprit in a patient’s prolonged alteration in mental status. Serial clinical examinations, continuous EEG monitoring to rule out ongoing subclinical seizures when appropriate, and judicious use of potentially sedating medications is the most helpful approach. The key issue here is the bimodal distribution of new-onset seizures. Among children there is a higher incidence of genetically related seizure disorders; whereas among adults, “acquired” and structural etiologies are more common. For this case, a more careful evaluation of acquired/structural etiologies for new-onset seizures is indicated.

►Dr. Li. At the geriatric psychiatry nursing home, the patient continued to be combative and refused medications. He was readmitted to the VABHS with encephalopathy of unclear etiology. An expanded encephalopathy workup was unrevealing (Table 2). 

fed00119030_t23.png
A CT of the lumbar spine was obtained, which showed only small surgical clips, and he was felt to be safe to undergo an MRI. An MRI of the brain with contrast showed generalized cerebral atrophy and evidence of small vessel ischemia but no acute pathology. His mental status continued to worsen with diminished speech output and decreased interaction with his health care providers. He was no longer able to state his name when asked. At this point, lumbar puncture was pursued (Table 3).

 

 

Dr. Fehnel, this patient’s initial cerebrospinal fluid (CSF) cell count and chemistries were completely normal. Is this sufficient to rule out encephalitis? If not, what other diagnostic tests would you send?

►Dr. Fehnel. A fully normal CSF profile reduces the likelihood of a broad range of neuro-infectious etiologies but does not completely rule those out. For example, there are reports of herpes simplex virus (HSV) encephalitis producing relatively normal profiles and even negative polymerase chain reaction assays for antibodies to HSV if the specimen is obtained very early in the course of the disease.3,4 That was not the case here as the CSF was obtained several days after his initial presentation. Given this patient’s clinical syndrome, normal CSF findings, and long smoking history without regular screening examinations, I would send a CSF specimen screening for paraneoplastic and autoimmune encephalitis. Most autoimmune encephalitis syndromes are associated with CSF lymphocytic pleocytosis or slight elevation in CSF protein levels. This patient’s diagnosis is most likely an anti-Hu paraneoplastic syndrome, which can be distinguished from other autoimmune and paraneoplastic processes by the characteristically normal CSF profile. Anti-Hu antibodies are strongly associated with non-small cell lung cancer (NSCLC). I would, therefore, also obtain more advanced chest imaging.

►Dr. Li. An autoimmune and paraneoplastic encephalitis panel was sent. While this send-out panel was pending, a CT torso was obtained to evaluate for occult malignancy in light of his significant smoking history. This showed a 3-cm spiculated mass originating from the left hilum. Bronchoalviolar lavage washings returned positive for small cell lung cancer. 

fed00119030_box.png
The CSF autoimmune encephalitis panel returned positive for anti-Hu antibody. A final diagnosis of paraneoplastic encephalitis secondary to small cell lung cancer was made.

Dr. Fehnel, can you explain the mechanism by which certain neoplasms can cause encephalitis?

Dr. Fehnel. Onconeuronal antibodies Hu (NSCLC) and Ma2 (testicular seminoma), when identified, are strongly associated with the presence of an underlying malignancy. The work of Dr. Josep Dalmau and others in this area has dramatically improved our understanding of these syndromes over the past 25 years.5 The exact mechanism is not fully understood but is thought to be mediated by cytotoxic T-cell response directed at the malignancy itself with homology to intraneuronal structures, which are readily absorbed and result in neuronal cell death.6

►Dr. Li. Is there a specific treatment for paraneoplastic encephalitis, other than treating the underlying malignancy?

►Dr. Fehnel. Early treatment is associated with improved outcome and should not be delayed while waiting for laboratory confirmation in cases of high clinical suspicion. Treatment directed at the underlying tumor is the mainstay along with less specific immunosuppressive agents. Unfortunately Anti-Hu (as well as Ma2) antibodies are intraneuronal and less responsive to standard treatments relative to other paraneoplastic auto-antibodies identified on the cell surface. Immunosuppressive agents typically used in this setting include high-dose IV methylprednisolone, IV immune globulin (IVIG), rituximab, and cyclophosphamide.7

►Dr. Li. The patient was started on IVIG, methylprednisolone, cisplatin, and etoposide. His course was complicated by aspiration pneumonia, autonomic dysfunction causing tachy- and brady-arrhythmias, urosepsis, worsening somnolence, chemotherapy-induced neutropenic fevers, and ultimately septic shock. The palliative care team was closely involved throughout the final stages of his hospital course. After multiple family meetings, the patient was transitioned to comfort-focused care per family discussion and died 6 weeks after his initial presentation.

 

 

This patient had a very atypical initial presentation of small cell lung cancer. Despite the fact that a diagnosis eluded his doctors, they persisted in a thoughtful and exhaustive workup and through this perseverance were able to make the final diagnosis, which serves as an important learning case for us all.

Acknowledgments

We thank the family of this veteran for sharing his story and allowing us to learn from this case for the benefit of our future patients. We also thank Dr. Michelle Hankins, who provided oncologic expertise.

Case Presentation. A 70-year-old US Marine Corps veteran of the Vietnam War with no significant past medical history was brought by ambulance to VA Boston Healthcare System (VABHS) after being found on the floor at home by his wife, awake, but with minimally coherent speech. He was moving all extremities, and there was no loss of bowel or bladder continence. He had last been seen well by his wife 30 minutes prior. When emergency medical services arrived, his finger stick blood glucose and vital signs were within normal range. In the emergency department, he was able to state his first name but then continuously repeated “7/11” to other questions. A neurologic examination revealed intact cranial nerves, full strength in all extremities, and normal reflexes. A National Institute of Health Stroke Scale (NIHSS) was 3, and a code stroke was activated. At the time of presentation, the patient was an active smoker of 15 cigarettes per day for 50 years and did not use alcohol or recreational drugs.

► Jonathan Li, MD, Chief Medical Resident, VABHS and Beth Israel Deaconess Medical Center (BIDMC). Dr. Fehnel, the patient’s medical team was most worried about a transient ischemic attack (TIA) or cerebrovascular accident (CVA). Is his presentation consistent with these diagnoses, and what else is on your differential diagnosis?

►Corey R. Fehnel, MD, Neuro-Intensivist, BIDMC, and Assistant Professor of Neurology, Harvard Medical School. This patient is presenting with what appears to be an acute encephalopathy—a sudden onset of global alteration in mental status. The most worrisome underlying etiology for this presentation would be acute stroke, but this is an uncommon cause of acute encephalopathy. The differential diagnosis at this stage remains broad, but a careful neurologic examination can help narrow the possibilities. In particular, I would aim to differentiate an apparent language deficit (ie, aphasia) from a deficit of attention. A key finding that may help is the ability to name high- or low-frequency objects. If the patient can successfully name objects, aphasia is less likely. Based on the limited examination at present, the patient produces some normal speech, but perseverates; therefore, the finding remains nonspecific. My leading diagnoses are complex partial seizure and toxic/metabolic encephalopathy.

►Dr. Li. This patient’s NIHSS score is 3. How do you use this score in your management decisions for the patient?

►Dr. Fehnel. The NIHSS is a useful tool for gauging severity of ischemic and hemorrhagic stroke. However the score is not specific for establishing the diagnosis of stroke. Many common and chronic neurologic problems will score on the NIHSS, so it can never be interpreted in isolation. If the clinical history and complete neurologic examination support the diagnosis of stroke, then the NIHSS can be used with the understanding that it is biased toward anterior circulation strokes, and posterior circulation strokes will score lower even though they are potentially more life threatening.1 In this case, even though a complex partial seizure appears more likely, it is difficult to rule out the possibility of an acute stroke affecting the thalamus or, less likely, a distal middle cerebral artery occlusion. I would consider IV thrombolysis pending further history and neuroimaging results.

►Dr. Li. Initial laboratory data include a hemoglobin of 12.8 mg/dL. The white cell count, platelet count, chemistry panel, liver function tests, thyroid-stimulating hormone, and troponin were within normal range (Table 1). 

fed00119030_t1.png
Serum and urine toxicology screening was negative. Computed tomography (CT) of the head and CT angiogram of the head and neck were negative for acute intracranial abnormality or vessel occlusion. The patient’s mental status improved shortly after, and he was able to recall going to a 7-Eleven convenience store with friends in the morning but had no recollection of subsequent events. He reported no fevers, headache, weakness, vision problems, paresthesia, dysphagia, or gait imbalance. A decision was made to admit the patient for monitoring and workup.

 

 

Do you agree with inpatient workup for this patient whose mental status has now returned to baseline? If so, what workup would you pursue next?

Dr. Fehnel. This patient requires inpatient admission to further evaluate the underlying etiology for his acute change in mental status. The improvement of his presenting deficit and largely normal neurovascular imaging make a neurovascular etiology less likely, but a careful risk factor evaluation for CVA/TIA should be performed, including continuous cardiac telemetry to detect atrial fibrillation. Magnetic resonance imaging (MRI) of the brain should be performed to rule out occult stroke and evaluate for a structural etiology given the more likely diagnosis of complex partial seizure. An electroencephalogram (EEG), preferably 24-hour continuous recording, should be performed. Without a clear toxic or metabolic etiology thus far to explain his acute global waxing-waning alteration in mental status and likely new-onset complex partial seizures, I would also pursue lumbar puncture for cerebrospinal fluid examination.

►Dr. Li. The hospital course was notable for episodes of acute combativeness and confusion. An MRI of the brain was deferred due to reports from the patient’s family of retained shrapnel in the lumbar spine. Routine EEG showed no seizure activity. This was followed by continuous video EEG monitoring, which showed subclinical seizure activity with a right temporal focus. He was started on valproic acid with improvement in his agitation, though confusion continued. He was discharged to an inpatient geriatric psychiatry nursing home with diagnosis of seizures and acute delirium.

Dr. Fehnel, seizures are often part of the workup for unexplained encephalopathy. In this case, the routine EEG was unrevealing, while the continuous video EEG proved valuable. In what situations would you pursue a continuous video EEG in addition to a routine EEG?

►Dr. Fehnel. EEG monitoring is only as good as the window of time during which the study is performed. If the suspicious clinical event is captured during a routine recording or an area of focal slowing is detected, a shorter study may be entirely sufficient. However, in cases where there is no clear alternative explanation, a patient’s mental status does not return to normal, or in the setting of mental status fluctuations without explanation, continuous video-EEG monitoring for at least 24 hours is indicated. While the prolonged study raises sensitivity, the exact duration of EEG recording required outside of the intensive care unit setting remains debated.2

►Dr. Li. If his encephalopathy were due to seizures alone, I would expect improvement in his mental status during interictal periods, which does not appear to be the case here. Do you feel the seizures alone can explain his encephalopathy?

Dr. Fehnel. Complex partial seizures and the medications used to treat them can confound the examination of patients during the interictal period. We commonly debate postictal encephalopathy vs residual effect of benzodiazepines and rapid dose escalation of antiepileptic drugs as culprit in a patient’s prolonged alteration in mental status. Serial clinical examinations, continuous EEG monitoring to rule out ongoing subclinical seizures when appropriate, and judicious use of potentially sedating medications is the most helpful approach. The key issue here is the bimodal distribution of new-onset seizures. Among children there is a higher incidence of genetically related seizure disorders; whereas among adults, “acquired” and structural etiologies are more common. For this case, a more careful evaluation of acquired/structural etiologies for new-onset seizures is indicated.

►Dr. Li. At the geriatric psychiatry nursing home, the patient continued to be combative and refused medications. He was readmitted to the VABHS with encephalopathy of unclear etiology. An expanded encephalopathy workup was unrevealing (Table 2). 

fed00119030_t23.png
A CT of the lumbar spine was obtained, which showed only small surgical clips, and he was felt to be safe to undergo an MRI. An MRI of the brain with contrast showed generalized cerebral atrophy and evidence of small vessel ischemia but no acute pathology. His mental status continued to worsen with diminished speech output and decreased interaction with his health care providers. He was no longer able to state his name when asked. At this point, lumbar puncture was pursued (Table 3).

 

 

Dr. Fehnel, this patient’s initial cerebrospinal fluid (CSF) cell count and chemistries were completely normal. Is this sufficient to rule out encephalitis? If not, what other diagnostic tests would you send?

►Dr. Fehnel. A fully normal CSF profile reduces the likelihood of a broad range of neuro-infectious etiologies but does not completely rule those out. For example, there are reports of herpes simplex virus (HSV) encephalitis producing relatively normal profiles and even negative polymerase chain reaction assays for antibodies to HSV if the specimen is obtained very early in the course of the disease.3,4 That was not the case here as the CSF was obtained several days after his initial presentation. Given this patient’s clinical syndrome, normal CSF findings, and long smoking history without regular screening examinations, I would send a CSF specimen screening for paraneoplastic and autoimmune encephalitis. Most autoimmune encephalitis syndromes are associated with CSF lymphocytic pleocytosis or slight elevation in CSF protein levels. This patient’s diagnosis is most likely an anti-Hu paraneoplastic syndrome, which can be distinguished from other autoimmune and paraneoplastic processes by the characteristically normal CSF profile. Anti-Hu antibodies are strongly associated with non-small cell lung cancer (NSCLC). I would, therefore, also obtain more advanced chest imaging.

►Dr. Li. An autoimmune and paraneoplastic encephalitis panel was sent. While this send-out panel was pending, a CT torso was obtained to evaluate for occult malignancy in light of his significant smoking history. This showed a 3-cm spiculated mass originating from the left hilum. Bronchoalviolar lavage washings returned positive for small cell lung cancer. 

fed00119030_box.png
The CSF autoimmune encephalitis panel returned positive for anti-Hu antibody. A final diagnosis of paraneoplastic encephalitis secondary to small cell lung cancer was made.

Dr. Fehnel, can you explain the mechanism by which certain neoplasms can cause encephalitis?

Dr. Fehnel. Onconeuronal antibodies Hu (NSCLC) and Ma2 (testicular seminoma), when identified, are strongly associated with the presence of an underlying malignancy. The work of Dr. Josep Dalmau and others in this area has dramatically improved our understanding of these syndromes over the past 25 years.5 The exact mechanism is not fully understood but is thought to be mediated by cytotoxic T-cell response directed at the malignancy itself with homology to intraneuronal structures, which are readily absorbed and result in neuronal cell death.6

►Dr. Li. Is there a specific treatment for paraneoplastic encephalitis, other than treating the underlying malignancy?

►Dr. Fehnel. Early treatment is associated with improved outcome and should not be delayed while waiting for laboratory confirmation in cases of high clinical suspicion. Treatment directed at the underlying tumor is the mainstay along with less specific immunosuppressive agents. Unfortunately Anti-Hu (as well as Ma2) antibodies are intraneuronal and less responsive to standard treatments relative to other paraneoplastic auto-antibodies identified on the cell surface. Immunosuppressive agents typically used in this setting include high-dose IV methylprednisolone, IV immune globulin (IVIG), rituximab, and cyclophosphamide.7

►Dr. Li. The patient was started on IVIG, methylprednisolone, cisplatin, and etoposide. His course was complicated by aspiration pneumonia, autonomic dysfunction causing tachy- and brady-arrhythmias, urosepsis, worsening somnolence, chemotherapy-induced neutropenic fevers, and ultimately septic shock. The palliative care team was closely involved throughout the final stages of his hospital course. After multiple family meetings, the patient was transitioned to comfort-focused care per family discussion and died 6 weeks after his initial presentation.

 

 

This patient had a very atypical initial presentation of small cell lung cancer. Despite the fact that a diagnosis eluded his doctors, they persisted in a thoughtful and exhaustive workup and through this perseverance were able to make the final diagnosis, which serves as an important learning case for us all.

Acknowledgments

We thank the family of this veteran for sharing his story and allowing us to learn from this case for the benefit of our future patients. We also thank Dr. Michelle Hankins, who provided oncologic expertise.

References

1. Heldner MR, Zubler C, Mattle HP, et al. National Institutes of Health stroke scale score and vessel occlusion in 2152 patients with acute ischemic stroke. Stroke. 2013;44(4):1153-1157.

2. Herman ST, Abend NS, Bleck TP, et al; Critical Care Continuous EEG Task Force of the American Clinical Neurophysiology Society. Consensus statement on continuous EEG in critically ill adults and children, part I: indications. J Clin Neurophysiol. 2015;32(2):87-95.

3. DeBiasi RL, Kleinschmidt-DeMasters BK, Weinberg A, Tyler KL. Use of PCR for the diagnosis of herpesvirus infections of the central nervous system. J Clin Virol. 2002;25(suppl 1):S5-S11.

4. Buerger KJ, Zerr K, Salazar R. An unusual presentation of herpes simplex encephalitis with negative PCR. BMJ Case Rep. 2015;2015:pii:bcr201521052.

5. Graus F, Titulaer MJ, Balu R, et al. A clinical approach to diagnosis of autoimmune encephalitis. Lancet Neurol. 2016;15(4):391-404.

6. Greenlee JE, Clawson SA, Hill KE, et al. Neuronal uptake of anti-Hu antibody, but not anti-Ri antibody, leads to cell death in brain slice cultures. J Neuroinflammation. 2014;11:160.

7. Bradshaw MJ, Linnoila JJ. An overview of autoimmune and paraneoplastic encephalitides. Semin Neurol. 2018;38(3):330-343.

References

1. Heldner MR, Zubler C, Mattle HP, et al. National Institutes of Health stroke scale score and vessel occlusion in 2152 patients with acute ischemic stroke. Stroke. 2013;44(4):1153-1157.

2. Herman ST, Abend NS, Bleck TP, et al; Critical Care Continuous EEG Task Force of the American Clinical Neurophysiology Society. Consensus statement on continuous EEG in critically ill adults and children, part I: indications. J Clin Neurophysiol. 2015;32(2):87-95.

3. DeBiasi RL, Kleinschmidt-DeMasters BK, Weinberg A, Tyler KL. Use of PCR for the diagnosis of herpesvirus infections of the central nervous system. J Clin Virol. 2002;25(suppl 1):S5-S11.

4. Buerger KJ, Zerr K, Salazar R. An unusual presentation of herpes simplex encephalitis with negative PCR. BMJ Case Rep. 2015;2015:pii:bcr201521052.

5. Graus F, Titulaer MJ, Balu R, et al. A clinical approach to diagnosis of autoimmune encephalitis. Lancet Neurol. 2016;15(4):391-404.

6. Greenlee JE, Clawson SA, Hill KE, et al. Neuronal uptake of anti-Hu antibody, but not anti-Ri antibody, leads to cell death in brain slice cultures. J Neuroinflammation. 2014;11:160.

7. Bradshaw MJ, Linnoila JJ. An overview of autoimmune and paraneoplastic encephalitides. Semin Neurol. 2018;38(3):330-343.

Issue
Federal Practitioner - 36(1)
Issue
Federal Practitioner - 36(1)
Page Number
30-34
Page Number
30-34
Publications
Publications
Topics
Article Type
Sections
Disallow All Ads
Content Gating
No Gating (article Unlocked/Free)
Alternative CME
Disqus Comments
Default
Use ProPublica
Article PDF Media
Image
Disable zoom
Off
Media Folder

A Veteran With Fibromyalgia Presenting With Dyspnea

Article Type
Changed
Thu, 11/15/2018 - 12:43

Case Presentation. A 64-year-old US Army veteran with a history of colorectal cancer, melanoma, and fibrinolytic presented with dyspnea to VA Boston Healthcare System (VABHS). Seven years prior to the current presentation, at the time of her diagnosis of colorectal cancer, the patient was found to be HIV negative but to have a positive purified protein derivative (PPD) test. She was treated with isoniazid (INH) therapy for 9 months. Sputum cultures collected prior to initiation of therapy grew Mycobacterium avium complex (MAC) in 1 of 3 samples, with these results reported several months after initiation of therapy. She was a never smoker with no known travel or exposure. At the time of the current presentation, her medications included bupropion, levothyroxine, capsaicin, cyclobenzaprine, ibuprofen, and acetaminophen.

Lakshmana Swamy, MD, Chief Medical Resident, VABHS and Boston Medical Center. Dr. Monach, this patient is on a variety of pain medications and has a diagnosis of fibromyalgia. This diagnosis often frustrates doctors and patients alike. Can you tell us about fibromyalgia from the rheumatologist’s perspective and what you think of her current treatment regimen?

►Paul A. Monach, MD, PhD, Chief, Section of Rheumatology, VABHS and Associate Professor of Medicine, Boston University School of Medicine. Fibromyalgia is a syndrome of chronic widespread pain without known pathology in the musculoskeletal system. It is thought to be caused by chronic dysfunction of pain-processing pathways in the central nervous system (CNS). It is often accompanied by other somatic symptoms such as chronic fatigue, irritable bowel syndrome, and bladder pain. It is a common condition, affecting up to 5% of otherwise healthy women. It is particularly common in persons with chronic nonrestorative sleep or posttraumatic stress disorder from a wide range of causes. However, it also is more common in persons with autoimmune inflammatory diseases, such as lupus, Sjögren syndrome, or rheumatoid arthritis. Concern for one of these diseases is the main reason to consider referring a patient for evaluation by a rheumatologist. Often rheumatologists participate in the management of fibromyalgia. A patient should be given appropriate expectations by the referring physician.

Effectiveness of treatment varies widely among patients. Nonpharmacologic approaches such as aerobic exercise, cognitive behavioral therapy, and tai chi have support from clinical trials, and yoga and aquatherapy also are widely used.1,2 The classes of drugs used are the same as for neuropathic pain: tricyclics, including cyclobenzaprine; serotonin and norepinephrine reuptake inhibitors (SNRIs); and gabapentinoids. In contrast, nonsteroidal anti-inflammatory drugs and opioids are ineffective unless there is a superimposed mechanical or inflammatory cause in the periphery. The key point is that continuation of any treatment should be based entirely on the patient’s own assessment of benefit.

►Dr. Swamy. Seven years later, the patient returned to her primary care provider, reporting increased dyspnea on exertion as well as significant fatigue. She was referred to the pulmonary department and had repeat computed tomography (CT) scans of the chest, which indicated persistent right middle lobe (RML) bronchiectasis. She then underwent bronchoscopy with a subsequent bronchoalveolar lavage (BAL) culture growing MAC. Dr. Fine, please interpret the baseline and follow-up CT scans and help us understand the significance of the MAC on sputum and BAL cultures.

►Alan Fine, MD, Section of Pulmonary and Critical Care, VABHS and Professor of Medicine, Boston University School of Medicine. Prior to this presentation, the patient had a pleural-based area of fibrosis with possible associated RML bronchiectasis. This appears to be a postinflammatory process without classic features of malignant or metastatic disease. She then had a sputum, which grew MAC in only 1 of 3 samples and in liquid media only. Importantly, the sputum was not smear positive. All of this suggests a low organism burden. One possibility is that this could reflect colonization with MAC; it is not uncommon for patients with underlying chronic changes in their lung to grow MAC, and it is often difficult to tell whether it is indicative of active disease. Structural lung disease, such as bronchiectasis, predisposes a patient to MAC, but chronic MAC also may cause bronchiectasis. This chicken-and-egg scenario comes up frequently. She may have a MAC infection, but as she is HIV negative and asymptomatic, there is no urgent indication to treat, especially as the burden of therapy is not insignificant.

►Dr. Swamy. Do we need to worry about Mycobacterium tuberculosis (MTB)?

►Dr. Fine. Although she was previously PPD positive, she had already completed 1 year of isoniazid (INH) therapy, making active MTB less likely. From an infection control standpoint, it is important to distinguish MAC from MTB. The former is not contagious, and there is no need for airborne isolation.

►Dr. Swamy. Dr. Fine, where does MAC come from? Does it commonly cause disease?

►Dr. Fine. In the environment, MAC is nearly ubiquitous , especially in water and soil. In one study, 20% of showerheads were positive for MAC; when patients are infected, we may suggest changing/bleaching the showerhead, but there are no definitive recommendations.3 Because MAC is so common in the environment, it is unlikely that measures to target MAC colonization will be clinically meaningful. On the other hand, the incidence of nontuberculous mycobacterial infections is increasing across the US, and it may be a common and frequently underdiagnosed cause of chronic cough, especially in postmenopausal women.

►Dr. Swamy. Four years prior to the current presentation, the patient developed a cough after an upper respiratory tract infection that persisted for more than 2 weeks. Given her history, she underwent a repeat chest CT, which noted a slight increase in nodularity and ground-glass opacity restricted to the RML. She also reported dyspnea on exertion and was referred to the pulmonary medicine department. By the time she arrived, her dyspnea had largely resolved, but she reported persistent fatigue without other systemic symptoms, such as fevers or chills. Dr. Fine, does MAC explain this patient’s dyspnea?

►Dr. Fine. As her pulmonary symptoms resolved in a short period of time with only azithromycin, it is very unlikely that her symptoms were related to her prior disease. The MAC infection is not likely to cause dyspnea on exertion and fatigue and should be worked up more broadly before attributing it to MAC. In view of this, it would not be unreasonable to follow her clinically and see her again in 6 to 8 weeks. In this context, we also should consider the untoward impact of repeated radiation exposure derived from multiple CT scans. When a patient has an abnormality on CT scan, it often leads to further scans even if the symptoms do not match the previous findings, as in this case.

fdp03508022_box.png

►Dr. Swamy. Given her ongoing fatigue and systemic symptoms (morning stiffness of the shoulders, legs, and thighs, and leg cramps), she was referred to the rheumatology department where the physical examination revealed muscle tenderness in her proximal arms and legs with normal strength, tender points at the elbows and medial side of the bilateral knees, significant tenderness of lower legs, and no synovitis.

 

 

Dr. Monach, can you walk us through your approach to this patient? Are we seeing manifestations of fibromyalgia? What diagnoses concerns you and how would you proceed?

Dr. Monach. The history and exam are most helpful in raising or reducing suspicion for an underlying inflammatory disease. Areas of tenderness described in her case are typical of fibromyalgia, although it can be difficult to interpret symptoms in the hip girdle and shoulder girdle because objective findings are often absent on exam in patients with inflammatory arthritis or bursitis. Similarly, tenderness at sites of tendon insertion (enthuses) without objective abnormalities is common in different forms of spondyloarthritis, so tenderness at the elbow, knee, lateral hip, and low back can be difficult to interpret. What this patient is lacking is prominent subjective or objective findings in the joints most commonly affected in rheumatoid arthritis and lupus: wrists, hands, ankles, and feet.

►Dr. Swamy. Initial laboratory data include an erythrocyte sedimentation rate of 79 with a normal C-reactive protein. A tentative diagnosis of polymyalgia rheumatic is made with consideration of a trial treatment of prednisone.

Dr. Monach, this patient has an indolent infection and is about to be given glucocorticoids. Could you describe the situations in which you feel that glucocorticoids cause a relative immunosuppression?

Dr. Monach. Glucocorticoids are considered safe in a patient whose infection is not intrinsically dangerous or who has started appropriate antibiotics for that infection. Although all toxicities of glucocorticoids are dose dependent, the long-standing assertion that doses below 10 mg to 15 mg do not increase risk of infection is contradicted by data published in the past 10 to 15 years, with the caveat that these patients were on long-term treatment.

Dr. Swamy. The patient was started on prednisone 15 mg per day for 15 days. She returned to the clinic after 1 week of prednisone troutment and noted “significant improvement in fatigue, morning stiffness of shoulders, thighs, leg, back is better, leg cramps resolved, shooting pain in many joints resolved.” Further laboratory results were notable for a negative rheumatoid factor, negative antinuclear antibody, and a cyclic citrullinated peptide of 60. A presumptive diagnosis of rheumatoid arthritis (RA) was made and plaquenil 200 mg twice daily was started.

Dr. Monach, can you explain why RA comes up now on serology but was not considered initially? Why does this presentation fit RA, and was her response to treatment typical? How does this fit in with her previous diagnosis of fibromyalgia? Was that just an atypical, indolent presentation of RA?

Dr. Monach. Though her presentation is atypical for RA, in elderly patients, RA can present with symptoms resembling polymyalgia rheumatica. The question is whether she had RA all along (in which case “elderly onset” would not apply) or had fibromyalgia and developed RA more recently. The response to empiric glucocorticoid therapy is helpful, since fibromyalgia should not improve with prednisone even in a patient with RA unless treatment of RA would allow better sleep and ability to exercise. Rheumatoid arthritis typically responds very well to prednisone in the 5-mg to 15-mg range.

►Dr. Swamy. Given the new diagnosis of an inflammatory arthritis requiring immunosuppression, bronchoscopy with BAL is performed to evaluate for the presence of MAC. These cultures were positive for MAC.

 

 

Dr. Fine, does the positive BAL culture indicate an active MAC infection?

►Dr. Fine. Yes, based on these updated data, the patient has an active MAC infection. Active infection is defined as symptoms or imaging consistent with the diagnosis, supporting microbiology data (either 2 sputum or 1 BAL sample growing MAC) and the exclusion of other causes. Previously, this patient grew MAC in just one expectorated sputum; this did not meet the microbiologic criteria. Now sputum has grown in the BAL sample; along with the CT imaging, this is enough to diagnosis active MAC infection.

Treatment for MAC must consider the details of each case. First, this is not an emergency; treatment decisions should be made with the rheumatologist to consider the planned immunosuppression. For example, we must consider potential drug interactions. A specific point should be made of the use of tumor necrosis factor (TNF)-α inhibition, which data indicate can reactivate TB and may inhibit mechanisms that restrain mycobacterial disease. Serious cases of MAC infection have been reported in the literature in the setting of TNF-α inhibition.5,6 Despite these concerns, there is not a contraindication to using these therapies from the perspective of the active MAC disease. All of these decisions will impact the need to commit the patient to MAC therapy.

►Dr. Swamy. Dr. Fine, what do you consider prior to initiating MAC therapy?

Dr. Fine. The decision to pursue MAC therapy should not be taken lightly. Therapy often entails prolonged multidrug regimens, usually spanning more than a year, with frequent adverse effects. Outside of very specific cases, such as TNF-β inhibition, MAC is rarely a life-threatening disease, so the benefit may be limited. Treatment for MAC is certainly unlikely to be fruitful without a diligent and motivated patient able to handle the high and prolonged pill burden. Of note, it is also important to keep this patient up-to-date with influenza and pneumonia vaccination given her structural lung disease.

Dr. Swamy. The decision is made to treat MAC with azithromycin, rifampin, and ethambutol. The disease is noted to be nonfibrocavitary. The patient underwent monthly liver function test monitoring and visual acuity testing, which were unremarkable. Dr. Fine, can you describe the phenotypes of nontuberculous mycobacterial (NTM) disease?

►Dr. Fine. There are 3 main phenotypes of NTM.3 First, we see the elderly man with preexisting lung disease—usually chronic obstructive pulmonary disease—with fibrocavitary and/or reticulonodular appearance. Second, we see the slim, elderly woman often without any preexisting lung disease presenting with focal bronchiectasis and nodular lesions in right middle lobe and lingula—the Lady Windermere syndrome. This eponym is derived from Oscar Wilde’s play “Lady Windermere’s Fan, a Play About a Good Woman,” and was first associated with this disease in 1992.7 At the time, it was thought that the voluntary suppression of cough led to poorly draining lung regions, vulnerable to engraftment by atypical mycobacteria. Infection with atypical mycobacteria are associated with this population; however, it is no longer thought to be due to the voluntary suppression of cough.7,8 Third, we do occasionally see atypical presentations, such as focal masses and solitary nodules.

►Dr. Swamy. At 1-year follow-up she successfully completed MAC therapy and noted ongoing control of rheumatoid symptoms.

References

1. Bernardy K, Klose P, Welsch P, Häuser W. Efficacy, acceptability and safety of cognitive behavioural therapies in fibromyalgia syndrome—a systematic review and meta-analysis of randomized controlled trials. Eur J Pain. 2018;22(2):242-260.

2. Geneen LJ, Moore RA, Clarke C, Martin D, Colvin LA, Smith BH. Physical activity and exercise for chronic pain in adults: an overview of Cochrane Reviews. Cochrane Database Syst Rev. 2017;4:CD011279.

3. Aksamit TR, Philley JV, Griffith DE. Nontuberculous mycobacterial (NTM) lung disease: the top ten essentials. Respir Med. 2014;108(3):417-425.

4. Aucott JN. Glucocorticoids and infection. Endocrinol Metab Clin North Am. 1994;23(3):655-670.

5. Curtis JR, Yang S, Patkar NM, et al. Risk of hospitalized bacterial infections associated with biologic treatment among US veterans with rheumatoid arthritis. Arthritis Care Res (Hoboken). 2014;66(7):990-997.

6. Lane MA, McDonald JR, Zeringue AL, et al. TNF-α antagonist use and risk of hospitalization for infection in a national cohort of veterans with rheumatoid arthritis. Medicine (Baltimore). 2011;90(2):139-145.

7. Reich JM, Johnson RE. Mycobacterium avium complex pulmonary disease presenting as an isolated lingular or middle lobe pattern. The Lady Windermere syndrome. Chest. 1992;101(6):1605-1609.

8. Kasthoori JJ, Liam CK, Wastie ML. Lady Windermere syndrome: an inappropriate eponym for an increasingly important condition. Singapore Med J. 2008;49(2):e47-e49.

Article PDF
Author and Disclosure Information

Author disclosures
The authors report no actual or potential conflicts of interest with regard to this article.

Disclaimer
The opinions expressed herein are those of the authors and do not necessarily reflect those of Federal Practitioner, Frontline Medical Communications Inc., the US Government, or any of its agencies. This article may discuss unlabeled or investigational use of certain drugs. Please review the complete prescribing information for specific drugs or drug combinations—including indications, contraindications, warnings, and adverse effects—before administering pharmacologic therapy to patients.

Issue
Federal Practitioner - 35(8)a
Publications
Topics
Page Number
22-25
Sections
Author and Disclosure Information

Author disclosures
The authors report no actual or potential conflicts of interest with regard to this article.

Disclaimer
The opinions expressed herein are those of the authors and do not necessarily reflect those of Federal Practitioner, Frontline Medical Communications Inc., the US Government, or any of its agencies. This article may discuss unlabeled or investigational use of certain drugs. Please review the complete prescribing information for specific drugs or drug combinations—including indications, contraindications, warnings, and adverse effects—before administering pharmacologic therapy to patients.

Author and Disclosure Information

Author disclosures
The authors report no actual or potential conflicts of interest with regard to this article.

Disclaimer
The opinions expressed herein are those of the authors and do not necessarily reflect those of Federal Practitioner, Frontline Medical Communications Inc., the US Government, or any of its agencies. This article may discuss unlabeled or investigational use of certain drugs. Please review the complete prescribing information for specific drugs or drug combinations—including indications, contraindications, warnings, and adverse effects—before administering pharmacologic therapy to patients.

Article PDF
Article PDF

Case Presentation. A 64-year-old US Army veteran with a history of colorectal cancer, melanoma, and fibrinolytic presented with dyspnea to VA Boston Healthcare System (VABHS). Seven years prior to the current presentation, at the time of her diagnosis of colorectal cancer, the patient was found to be HIV negative but to have a positive purified protein derivative (PPD) test. She was treated with isoniazid (INH) therapy for 9 months. Sputum cultures collected prior to initiation of therapy grew Mycobacterium avium complex (MAC) in 1 of 3 samples, with these results reported several months after initiation of therapy. She was a never smoker with no known travel or exposure. At the time of the current presentation, her medications included bupropion, levothyroxine, capsaicin, cyclobenzaprine, ibuprofen, and acetaminophen.

Lakshmana Swamy, MD, Chief Medical Resident, VABHS and Boston Medical Center. Dr. Monach, this patient is on a variety of pain medications and has a diagnosis of fibromyalgia. This diagnosis often frustrates doctors and patients alike. Can you tell us about fibromyalgia from the rheumatologist’s perspective and what you think of her current treatment regimen?

►Paul A. Monach, MD, PhD, Chief, Section of Rheumatology, VABHS and Associate Professor of Medicine, Boston University School of Medicine. Fibromyalgia is a syndrome of chronic widespread pain without known pathology in the musculoskeletal system. It is thought to be caused by chronic dysfunction of pain-processing pathways in the central nervous system (CNS). It is often accompanied by other somatic symptoms such as chronic fatigue, irritable bowel syndrome, and bladder pain. It is a common condition, affecting up to 5% of otherwise healthy women. It is particularly common in persons with chronic nonrestorative sleep or posttraumatic stress disorder from a wide range of causes. However, it also is more common in persons with autoimmune inflammatory diseases, such as lupus, Sjögren syndrome, or rheumatoid arthritis. Concern for one of these diseases is the main reason to consider referring a patient for evaluation by a rheumatologist. Often rheumatologists participate in the management of fibromyalgia. A patient should be given appropriate expectations by the referring physician.

Effectiveness of treatment varies widely among patients. Nonpharmacologic approaches such as aerobic exercise, cognitive behavioral therapy, and tai chi have support from clinical trials, and yoga and aquatherapy also are widely used.1,2 The classes of drugs used are the same as for neuropathic pain: tricyclics, including cyclobenzaprine; serotonin and norepinephrine reuptake inhibitors (SNRIs); and gabapentinoids. In contrast, nonsteroidal anti-inflammatory drugs and opioids are ineffective unless there is a superimposed mechanical or inflammatory cause in the periphery. The key point is that continuation of any treatment should be based entirely on the patient’s own assessment of benefit.

►Dr. Swamy. Seven years later, the patient returned to her primary care provider, reporting increased dyspnea on exertion as well as significant fatigue. She was referred to the pulmonary department and had repeat computed tomography (CT) scans of the chest, which indicated persistent right middle lobe (RML) bronchiectasis. She then underwent bronchoscopy with a subsequent bronchoalveolar lavage (BAL) culture growing MAC. Dr. Fine, please interpret the baseline and follow-up CT scans and help us understand the significance of the MAC on sputum and BAL cultures.

►Alan Fine, MD, Section of Pulmonary and Critical Care, VABHS and Professor of Medicine, Boston University School of Medicine. Prior to this presentation, the patient had a pleural-based area of fibrosis with possible associated RML bronchiectasis. This appears to be a postinflammatory process without classic features of malignant or metastatic disease. She then had a sputum, which grew MAC in only 1 of 3 samples and in liquid media only. Importantly, the sputum was not smear positive. All of this suggests a low organism burden. One possibility is that this could reflect colonization with MAC; it is not uncommon for patients with underlying chronic changes in their lung to grow MAC, and it is often difficult to tell whether it is indicative of active disease. Structural lung disease, such as bronchiectasis, predisposes a patient to MAC, but chronic MAC also may cause bronchiectasis. This chicken-and-egg scenario comes up frequently. She may have a MAC infection, but as she is HIV negative and asymptomatic, there is no urgent indication to treat, especially as the burden of therapy is not insignificant.

►Dr. Swamy. Do we need to worry about Mycobacterium tuberculosis (MTB)?

►Dr. Fine. Although she was previously PPD positive, she had already completed 1 year of isoniazid (INH) therapy, making active MTB less likely. From an infection control standpoint, it is important to distinguish MAC from MTB. The former is not contagious, and there is no need for airborne isolation.

►Dr. Swamy. Dr. Fine, where does MAC come from? Does it commonly cause disease?

►Dr. Fine. In the environment, MAC is nearly ubiquitous , especially in water and soil. In one study, 20% of showerheads were positive for MAC; when patients are infected, we may suggest changing/bleaching the showerhead, but there are no definitive recommendations.3 Because MAC is so common in the environment, it is unlikely that measures to target MAC colonization will be clinically meaningful. On the other hand, the incidence of nontuberculous mycobacterial infections is increasing across the US, and it may be a common and frequently underdiagnosed cause of chronic cough, especially in postmenopausal women.

►Dr. Swamy. Four years prior to the current presentation, the patient developed a cough after an upper respiratory tract infection that persisted for more than 2 weeks. Given her history, she underwent a repeat chest CT, which noted a slight increase in nodularity and ground-glass opacity restricted to the RML. She also reported dyspnea on exertion and was referred to the pulmonary medicine department. By the time she arrived, her dyspnea had largely resolved, but she reported persistent fatigue without other systemic symptoms, such as fevers or chills. Dr. Fine, does MAC explain this patient’s dyspnea?

►Dr. Fine. As her pulmonary symptoms resolved in a short period of time with only azithromycin, it is very unlikely that her symptoms were related to her prior disease. The MAC infection is not likely to cause dyspnea on exertion and fatigue and should be worked up more broadly before attributing it to MAC. In view of this, it would not be unreasonable to follow her clinically and see her again in 6 to 8 weeks. In this context, we also should consider the untoward impact of repeated radiation exposure derived from multiple CT scans. When a patient has an abnormality on CT scan, it often leads to further scans even if the symptoms do not match the previous findings, as in this case.

fdp03508022_box.png

►Dr. Swamy. Given her ongoing fatigue and systemic symptoms (morning stiffness of the shoulders, legs, and thighs, and leg cramps), she was referred to the rheumatology department where the physical examination revealed muscle tenderness in her proximal arms and legs with normal strength, tender points at the elbows and medial side of the bilateral knees, significant tenderness of lower legs, and no synovitis.

 

 

Dr. Monach, can you walk us through your approach to this patient? Are we seeing manifestations of fibromyalgia? What diagnoses concerns you and how would you proceed?

Dr. Monach. The history and exam are most helpful in raising or reducing suspicion for an underlying inflammatory disease. Areas of tenderness described in her case are typical of fibromyalgia, although it can be difficult to interpret symptoms in the hip girdle and shoulder girdle because objective findings are often absent on exam in patients with inflammatory arthritis or bursitis. Similarly, tenderness at sites of tendon insertion (enthuses) without objective abnormalities is common in different forms of spondyloarthritis, so tenderness at the elbow, knee, lateral hip, and low back can be difficult to interpret. What this patient is lacking is prominent subjective or objective findings in the joints most commonly affected in rheumatoid arthritis and lupus: wrists, hands, ankles, and feet.

►Dr. Swamy. Initial laboratory data include an erythrocyte sedimentation rate of 79 with a normal C-reactive protein. A tentative diagnosis of polymyalgia rheumatic is made with consideration of a trial treatment of prednisone.

Dr. Monach, this patient has an indolent infection and is about to be given glucocorticoids. Could you describe the situations in which you feel that glucocorticoids cause a relative immunosuppression?

Dr. Monach. Glucocorticoids are considered safe in a patient whose infection is not intrinsically dangerous or who has started appropriate antibiotics for that infection. Although all toxicities of glucocorticoids are dose dependent, the long-standing assertion that doses below 10 mg to 15 mg do not increase risk of infection is contradicted by data published in the past 10 to 15 years, with the caveat that these patients were on long-term treatment.

Dr. Swamy. The patient was started on prednisone 15 mg per day for 15 days. She returned to the clinic after 1 week of prednisone troutment and noted “significant improvement in fatigue, morning stiffness of shoulders, thighs, leg, back is better, leg cramps resolved, shooting pain in many joints resolved.” Further laboratory results were notable for a negative rheumatoid factor, negative antinuclear antibody, and a cyclic citrullinated peptide of 60. A presumptive diagnosis of rheumatoid arthritis (RA) was made and plaquenil 200 mg twice daily was started.

Dr. Monach, can you explain why RA comes up now on serology but was not considered initially? Why does this presentation fit RA, and was her response to treatment typical? How does this fit in with her previous diagnosis of fibromyalgia? Was that just an atypical, indolent presentation of RA?

Dr. Monach. Though her presentation is atypical for RA, in elderly patients, RA can present with symptoms resembling polymyalgia rheumatica. The question is whether she had RA all along (in which case “elderly onset” would not apply) or had fibromyalgia and developed RA more recently. The response to empiric glucocorticoid therapy is helpful, since fibromyalgia should not improve with prednisone even in a patient with RA unless treatment of RA would allow better sleep and ability to exercise. Rheumatoid arthritis typically responds very well to prednisone in the 5-mg to 15-mg range.

►Dr. Swamy. Given the new diagnosis of an inflammatory arthritis requiring immunosuppression, bronchoscopy with BAL is performed to evaluate for the presence of MAC. These cultures were positive for MAC.

 

 

Dr. Fine, does the positive BAL culture indicate an active MAC infection?

►Dr. Fine. Yes, based on these updated data, the patient has an active MAC infection. Active infection is defined as symptoms or imaging consistent with the diagnosis, supporting microbiology data (either 2 sputum or 1 BAL sample growing MAC) and the exclusion of other causes. Previously, this patient grew MAC in just one expectorated sputum; this did not meet the microbiologic criteria. Now sputum has grown in the BAL sample; along with the CT imaging, this is enough to diagnosis active MAC infection.

Treatment for MAC must consider the details of each case. First, this is not an emergency; treatment decisions should be made with the rheumatologist to consider the planned immunosuppression. For example, we must consider potential drug interactions. A specific point should be made of the use of tumor necrosis factor (TNF)-α inhibition, which data indicate can reactivate TB and may inhibit mechanisms that restrain mycobacterial disease. Serious cases of MAC infection have been reported in the literature in the setting of TNF-α inhibition.5,6 Despite these concerns, there is not a contraindication to using these therapies from the perspective of the active MAC disease. All of these decisions will impact the need to commit the patient to MAC therapy.

►Dr. Swamy. Dr. Fine, what do you consider prior to initiating MAC therapy?

Dr. Fine. The decision to pursue MAC therapy should not be taken lightly. Therapy often entails prolonged multidrug regimens, usually spanning more than a year, with frequent adverse effects. Outside of very specific cases, such as TNF-β inhibition, MAC is rarely a life-threatening disease, so the benefit may be limited. Treatment for MAC is certainly unlikely to be fruitful without a diligent and motivated patient able to handle the high and prolonged pill burden. Of note, it is also important to keep this patient up-to-date with influenza and pneumonia vaccination given her structural lung disease.

Dr. Swamy. The decision is made to treat MAC with azithromycin, rifampin, and ethambutol. The disease is noted to be nonfibrocavitary. The patient underwent monthly liver function test monitoring and visual acuity testing, which were unremarkable. Dr. Fine, can you describe the phenotypes of nontuberculous mycobacterial (NTM) disease?

►Dr. Fine. There are 3 main phenotypes of NTM.3 First, we see the elderly man with preexisting lung disease—usually chronic obstructive pulmonary disease—with fibrocavitary and/or reticulonodular appearance. Second, we see the slim, elderly woman often without any preexisting lung disease presenting with focal bronchiectasis and nodular lesions in right middle lobe and lingula—the Lady Windermere syndrome. This eponym is derived from Oscar Wilde’s play “Lady Windermere’s Fan, a Play About a Good Woman,” and was first associated with this disease in 1992.7 At the time, it was thought that the voluntary suppression of cough led to poorly draining lung regions, vulnerable to engraftment by atypical mycobacteria. Infection with atypical mycobacteria are associated with this population; however, it is no longer thought to be due to the voluntary suppression of cough.7,8 Third, we do occasionally see atypical presentations, such as focal masses and solitary nodules.

►Dr. Swamy. At 1-year follow-up she successfully completed MAC therapy and noted ongoing control of rheumatoid symptoms.

Case Presentation. A 64-year-old US Army veteran with a history of colorectal cancer, melanoma, and fibrinolytic presented with dyspnea to VA Boston Healthcare System (VABHS). Seven years prior to the current presentation, at the time of her diagnosis of colorectal cancer, the patient was found to be HIV negative but to have a positive purified protein derivative (PPD) test. She was treated with isoniazid (INH) therapy for 9 months. Sputum cultures collected prior to initiation of therapy grew Mycobacterium avium complex (MAC) in 1 of 3 samples, with these results reported several months after initiation of therapy. She was a never smoker with no known travel or exposure. At the time of the current presentation, her medications included bupropion, levothyroxine, capsaicin, cyclobenzaprine, ibuprofen, and acetaminophen.

Lakshmana Swamy, MD, Chief Medical Resident, VABHS and Boston Medical Center. Dr. Monach, this patient is on a variety of pain medications and has a diagnosis of fibromyalgia. This diagnosis often frustrates doctors and patients alike. Can you tell us about fibromyalgia from the rheumatologist’s perspective and what you think of her current treatment regimen?

►Paul A. Monach, MD, PhD, Chief, Section of Rheumatology, VABHS and Associate Professor of Medicine, Boston University School of Medicine. Fibromyalgia is a syndrome of chronic widespread pain without known pathology in the musculoskeletal system. It is thought to be caused by chronic dysfunction of pain-processing pathways in the central nervous system (CNS). It is often accompanied by other somatic symptoms such as chronic fatigue, irritable bowel syndrome, and bladder pain. It is a common condition, affecting up to 5% of otherwise healthy women. It is particularly common in persons with chronic nonrestorative sleep or posttraumatic stress disorder from a wide range of causes. However, it also is more common in persons with autoimmune inflammatory diseases, such as lupus, Sjögren syndrome, or rheumatoid arthritis. Concern for one of these diseases is the main reason to consider referring a patient for evaluation by a rheumatologist. Often rheumatologists participate in the management of fibromyalgia. A patient should be given appropriate expectations by the referring physician.

Effectiveness of treatment varies widely among patients. Nonpharmacologic approaches such as aerobic exercise, cognitive behavioral therapy, and tai chi have support from clinical trials, and yoga and aquatherapy also are widely used.1,2 The classes of drugs used are the same as for neuropathic pain: tricyclics, including cyclobenzaprine; serotonin and norepinephrine reuptake inhibitors (SNRIs); and gabapentinoids. In contrast, nonsteroidal anti-inflammatory drugs and opioids are ineffective unless there is a superimposed mechanical or inflammatory cause in the periphery. The key point is that continuation of any treatment should be based entirely on the patient’s own assessment of benefit.

►Dr. Swamy. Seven years later, the patient returned to her primary care provider, reporting increased dyspnea on exertion as well as significant fatigue. She was referred to the pulmonary department and had repeat computed tomography (CT) scans of the chest, which indicated persistent right middle lobe (RML) bronchiectasis. She then underwent bronchoscopy with a subsequent bronchoalveolar lavage (BAL) culture growing MAC. Dr. Fine, please interpret the baseline and follow-up CT scans and help us understand the significance of the MAC on sputum and BAL cultures.

►Alan Fine, MD, Section of Pulmonary and Critical Care, VABHS and Professor of Medicine, Boston University School of Medicine. Prior to this presentation, the patient had a pleural-based area of fibrosis with possible associated RML bronchiectasis. This appears to be a postinflammatory process without classic features of malignant or metastatic disease. She then had a sputum, which grew MAC in only 1 of 3 samples and in liquid media only. Importantly, the sputum was not smear positive. All of this suggests a low organism burden. One possibility is that this could reflect colonization with MAC; it is not uncommon for patients with underlying chronic changes in their lung to grow MAC, and it is often difficult to tell whether it is indicative of active disease. Structural lung disease, such as bronchiectasis, predisposes a patient to MAC, but chronic MAC also may cause bronchiectasis. This chicken-and-egg scenario comes up frequently. She may have a MAC infection, but as she is HIV negative and asymptomatic, there is no urgent indication to treat, especially as the burden of therapy is not insignificant.

►Dr. Swamy. Do we need to worry about Mycobacterium tuberculosis (MTB)?

►Dr. Fine. Although she was previously PPD positive, she had already completed 1 year of isoniazid (INH) therapy, making active MTB less likely. From an infection control standpoint, it is important to distinguish MAC from MTB. The former is not contagious, and there is no need for airborne isolation.

►Dr. Swamy. Dr. Fine, where does MAC come from? Does it commonly cause disease?

►Dr. Fine. In the environment, MAC is nearly ubiquitous , especially in water and soil. In one study, 20% of showerheads were positive for MAC; when patients are infected, we may suggest changing/bleaching the showerhead, but there are no definitive recommendations.3 Because MAC is so common in the environment, it is unlikely that measures to target MAC colonization will be clinically meaningful. On the other hand, the incidence of nontuberculous mycobacterial infections is increasing across the US, and it may be a common and frequently underdiagnosed cause of chronic cough, especially in postmenopausal women.

►Dr. Swamy. Four years prior to the current presentation, the patient developed a cough after an upper respiratory tract infection that persisted for more than 2 weeks. Given her history, she underwent a repeat chest CT, which noted a slight increase in nodularity and ground-glass opacity restricted to the RML. She also reported dyspnea on exertion and was referred to the pulmonary medicine department. By the time she arrived, her dyspnea had largely resolved, but she reported persistent fatigue without other systemic symptoms, such as fevers or chills. Dr. Fine, does MAC explain this patient’s dyspnea?

►Dr. Fine. As her pulmonary symptoms resolved in a short period of time with only azithromycin, it is very unlikely that her symptoms were related to her prior disease. The MAC infection is not likely to cause dyspnea on exertion and fatigue and should be worked up more broadly before attributing it to MAC. In view of this, it would not be unreasonable to follow her clinically and see her again in 6 to 8 weeks. In this context, we also should consider the untoward impact of repeated radiation exposure derived from multiple CT scans. When a patient has an abnormality on CT scan, it often leads to further scans even if the symptoms do not match the previous findings, as in this case.

fdp03508022_box.png

►Dr. Swamy. Given her ongoing fatigue and systemic symptoms (morning stiffness of the shoulders, legs, and thighs, and leg cramps), she was referred to the rheumatology department where the physical examination revealed muscle tenderness in her proximal arms and legs with normal strength, tender points at the elbows and medial side of the bilateral knees, significant tenderness of lower legs, and no synovitis.

 

 

Dr. Monach, can you walk us through your approach to this patient? Are we seeing manifestations of fibromyalgia? What diagnoses concerns you and how would you proceed?

Dr. Monach. The history and exam are most helpful in raising or reducing suspicion for an underlying inflammatory disease. Areas of tenderness described in her case are typical of fibromyalgia, although it can be difficult to interpret symptoms in the hip girdle and shoulder girdle because objective findings are often absent on exam in patients with inflammatory arthritis or bursitis. Similarly, tenderness at sites of tendon insertion (enthuses) without objective abnormalities is common in different forms of spondyloarthritis, so tenderness at the elbow, knee, lateral hip, and low back can be difficult to interpret. What this patient is lacking is prominent subjective or objective findings in the joints most commonly affected in rheumatoid arthritis and lupus: wrists, hands, ankles, and feet.

►Dr. Swamy. Initial laboratory data include an erythrocyte sedimentation rate of 79 with a normal C-reactive protein. A tentative diagnosis of polymyalgia rheumatic is made with consideration of a trial treatment of prednisone.

Dr. Monach, this patient has an indolent infection and is about to be given glucocorticoids. Could you describe the situations in which you feel that glucocorticoids cause a relative immunosuppression?

Dr. Monach. Glucocorticoids are considered safe in a patient whose infection is not intrinsically dangerous or who has started appropriate antibiotics for that infection. Although all toxicities of glucocorticoids are dose dependent, the long-standing assertion that doses below 10 mg to 15 mg do not increase risk of infection is contradicted by data published in the past 10 to 15 years, with the caveat that these patients were on long-term treatment.

Dr. Swamy. The patient was started on prednisone 15 mg per day for 15 days. She returned to the clinic after 1 week of prednisone troutment and noted “significant improvement in fatigue, morning stiffness of shoulders, thighs, leg, back is better, leg cramps resolved, shooting pain in many joints resolved.” Further laboratory results were notable for a negative rheumatoid factor, negative antinuclear antibody, and a cyclic citrullinated peptide of 60. A presumptive diagnosis of rheumatoid arthritis (RA) was made and plaquenil 200 mg twice daily was started.

Dr. Monach, can you explain why RA comes up now on serology but was not considered initially? Why does this presentation fit RA, and was her response to treatment typical? How does this fit in with her previous diagnosis of fibromyalgia? Was that just an atypical, indolent presentation of RA?

Dr. Monach. Though her presentation is atypical for RA, in elderly patients, RA can present with symptoms resembling polymyalgia rheumatica. The question is whether she had RA all along (in which case “elderly onset” would not apply) or had fibromyalgia and developed RA more recently. The response to empiric glucocorticoid therapy is helpful, since fibromyalgia should not improve with prednisone even in a patient with RA unless treatment of RA would allow better sleep and ability to exercise. Rheumatoid arthritis typically responds very well to prednisone in the 5-mg to 15-mg range.

►Dr. Swamy. Given the new diagnosis of an inflammatory arthritis requiring immunosuppression, bronchoscopy with BAL is performed to evaluate for the presence of MAC. These cultures were positive for MAC.

 

 

Dr. Fine, does the positive BAL culture indicate an active MAC infection?

►Dr. Fine. Yes, based on these updated data, the patient has an active MAC infection. Active infection is defined as symptoms or imaging consistent with the diagnosis, supporting microbiology data (either 2 sputum or 1 BAL sample growing MAC) and the exclusion of other causes. Previously, this patient grew MAC in just one expectorated sputum; this did not meet the microbiologic criteria. Now sputum has grown in the BAL sample; along with the CT imaging, this is enough to diagnosis active MAC infection.

Treatment for MAC must consider the details of each case. First, this is not an emergency; treatment decisions should be made with the rheumatologist to consider the planned immunosuppression. For example, we must consider potential drug interactions. A specific point should be made of the use of tumor necrosis factor (TNF)-α inhibition, which data indicate can reactivate TB and may inhibit mechanisms that restrain mycobacterial disease. Serious cases of MAC infection have been reported in the literature in the setting of TNF-α inhibition.5,6 Despite these concerns, there is not a contraindication to using these therapies from the perspective of the active MAC disease. All of these decisions will impact the need to commit the patient to MAC therapy.

►Dr. Swamy. Dr. Fine, what do you consider prior to initiating MAC therapy?

Dr. Fine. The decision to pursue MAC therapy should not be taken lightly. Therapy often entails prolonged multidrug regimens, usually spanning more than a year, with frequent adverse effects. Outside of very specific cases, such as TNF-β inhibition, MAC is rarely a life-threatening disease, so the benefit may be limited. Treatment for MAC is certainly unlikely to be fruitful without a diligent and motivated patient able to handle the high and prolonged pill burden. Of note, it is also important to keep this patient up-to-date with influenza and pneumonia vaccination given her structural lung disease.

Dr. Swamy. The decision is made to treat MAC with azithromycin, rifampin, and ethambutol. The disease is noted to be nonfibrocavitary. The patient underwent monthly liver function test monitoring and visual acuity testing, which were unremarkable. Dr. Fine, can you describe the phenotypes of nontuberculous mycobacterial (NTM) disease?

►Dr. Fine. There are 3 main phenotypes of NTM.3 First, we see the elderly man with preexisting lung disease—usually chronic obstructive pulmonary disease—with fibrocavitary and/or reticulonodular appearance. Second, we see the slim, elderly woman often without any preexisting lung disease presenting with focal bronchiectasis and nodular lesions in right middle lobe and lingula—the Lady Windermere syndrome. This eponym is derived from Oscar Wilde’s play “Lady Windermere’s Fan, a Play About a Good Woman,” and was first associated with this disease in 1992.7 At the time, it was thought that the voluntary suppression of cough led to poorly draining lung regions, vulnerable to engraftment by atypical mycobacteria. Infection with atypical mycobacteria are associated with this population; however, it is no longer thought to be due to the voluntary suppression of cough.7,8 Third, we do occasionally see atypical presentations, such as focal masses and solitary nodules.

►Dr. Swamy. At 1-year follow-up she successfully completed MAC therapy and noted ongoing control of rheumatoid symptoms.

References

1. Bernardy K, Klose P, Welsch P, Häuser W. Efficacy, acceptability and safety of cognitive behavioural therapies in fibromyalgia syndrome—a systematic review and meta-analysis of randomized controlled trials. Eur J Pain. 2018;22(2):242-260.

2. Geneen LJ, Moore RA, Clarke C, Martin D, Colvin LA, Smith BH. Physical activity and exercise for chronic pain in adults: an overview of Cochrane Reviews. Cochrane Database Syst Rev. 2017;4:CD011279.

3. Aksamit TR, Philley JV, Griffith DE. Nontuberculous mycobacterial (NTM) lung disease: the top ten essentials. Respir Med. 2014;108(3):417-425.

4. Aucott JN. Glucocorticoids and infection. Endocrinol Metab Clin North Am. 1994;23(3):655-670.

5. Curtis JR, Yang S, Patkar NM, et al. Risk of hospitalized bacterial infections associated with biologic treatment among US veterans with rheumatoid arthritis. Arthritis Care Res (Hoboken). 2014;66(7):990-997.

6. Lane MA, McDonald JR, Zeringue AL, et al. TNF-α antagonist use and risk of hospitalization for infection in a national cohort of veterans with rheumatoid arthritis. Medicine (Baltimore). 2011;90(2):139-145.

7. Reich JM, Johnson RE. Mycobacterium avium complex pulmonary disease presenting as an isolated lingular or middle lobe pattern. The Lady Windermere syndrome. Chest. 1992;101(6):1605-1609.

8. Kasthoori JJ, Liam CK, Wastie ML. Lady Windermere syndrome: an inappropriate eponym for an increasingly important condition. Singapore Med J. 2008;49(2):e47-e49.

References

1. Bernardy K, Klose P, Welsch P, Häuser W. Efficacy, acceptability and safety of cognitive behavioural therapies in fibromyalgia syndrome—a systematic review and meta-analysis of randomized controlled trials. Eur J Pain. 2018;22(2):242-260.

2. Geneen LJ, Moore RA, Clarke C, Martin D, Colvin LA, Smith BH. Physical activity and exercise for chronic pain in adults: an overview of Cochrane Reviews. Cochrane Database Syst Rev. 2017;4:CD011279.

3. Aksamit TR, Philley JV, Griffith DE. Nontuberculous mycobacterial (NTM) lung disease: the top ten essentials. Respir Med. 2014;108(3):417-425.

4. Aucott JN. Glucocorticoids and infection. Endocrinol Metab Clin North Am. 1994;23(3):655-670.

5. Curtis JR, Yang S, Patkar NM, et al. Risk of hospitalized bacterial infections associated with biologic treatment among US veterans with rheumatoid arthritis. Arthritis Care Res (Hoboken). 2014;66(7):990-997.

6. Lane MA, McDonald JR, Zeringue AL, et al. TNF-α antagonist use and risk of hospitalization for infection in a national cohort of veterans with rheumatoid arthritis. Medicine (Baltimore). 2011;90(2):139-145.

7. Reich JM, Johnson RE. Mycobacterium avium complex pulmonary disease presenting as an isolated lingular or middle lobe pattern. The Lady Windermere syndrome. Chest. 1992;101(6):1605-1609.

8. Kasthoori JJ, Liam CK, Wastie ML. Lady Windermere syndrome: an inappropriate eponym for an increasingly important condition. Singapore Med J. 2008;49(2):e47-e49.

Issue
Federal Practitioner - 35(8)a
Issue
Federal Practitioner - 35(8)a
Page Number
22-25
Page Number
22-25
Publications
Publications
Topics
Article Type
Sections
Disallow All Ads
Content Gating
No Gating (article Unlocked/Free)
Alternative CME
Disqus Comments
Default
Use ProPublica
Article PDF Media
Image
Disable zoom
Off
Media Folder

A Veteran With Alcohol Use Disorder and Acute Pancreatitis

Article Type
Changed
Thu, 11/15/2018 - 12:43
Discussion of treatment for a young patient with untreated posttraumatic stress disorder and acute abdominal pain.

Case Presentation. A 23-year-old male U.S. Army veteran with a history of alcohol use disorder and posttraumatic stress disorder (PTSD) presented to the VA Boston Healthcare System (VABHS) West Roxbury campus emergency department (ED) with epigastric abdominal pain in the setting of consuming alcohol. The patient had served in the infantry in Afghanistan during Operation Enduring Freedom. He consumed up to 12 alcoholic drinks per day (both beer and hard liquor) for the past 3 years and had been hospitalized 3 times previously; twice for alcohol detoxification and once for PTSD. He is a former tobacco smoker with fewer than 5 pack-years, he uses marijuana often and does not use IV drugs. In the ED, his physical examination was notable for a heart rate of 130 beats per minute and blood pressure of 161/111 mm Hg. He was alert and oriented and had a mild tremor. The patient was diaphoretic with dry mucous membranes, tenderness to palpation in the epigastrium, and abdominal guarding. A computed tomography (CT) scan of the abdomen revealed acute pancreatitis without necrosis. The patient received 1 L of normal saline and was admitted to the medical ward for presumed alcoholic pancreatitis.

► Rahul Ganatra, MD, MPH, Chief Medical Resident, VABHS and Beth Israel Deaconess Medical Center. Dr. Weber, we care for many young people who drink more than they should and almost none of them end up with alcoholic pancreatitis. What are the relevant risk factors that make individuals like this patient more susceptible to alcoholic pancreatitis?

Horst Christian Weber, MD, Gastroenterology Service, VABHS, and Assistant Professor of Medicine, Boston University School of Medicine. While we don’t have a good understanding of the precise mechanism of alcoholic pancreatitis, we do know that in the U.S., alcohol consumption is responsible for about one-third of all cases.1 Acute pancreatitis in general may present with a wide range of disease severity. It is the most common cause of gastrointestinal-related hospitalization,2 and the mortality of hospital inpatients with pancreatitis is about 5%.3,4 Therefore, acute pancreatitis represents a prevalent condition with a critical impact on morbidity and mortality. Alcoholic pancreatitis typically occurs after many years of heavy alcohol use, not after a single drinking

FED03502034_t.PNG
binge in alcohol-naïve individuals.1 Regarding this patient, we also know that smoking is an independent risk factor for pancreatitis and may have a synergistic effect with alcohol.1,5 In addition, there are genetic mutations that can increase a person’s risk for pancreatitis, which is of consideration given this patient’s young age. Apart from those 2 possible contributors, there does not currently seem to be another more likely explanation than alcohol.

Dr. Ganatra. At this point, the chemistry laboratory paged the admitting resident with the notification that the patient’s blood was grossly lipemic. Ultracentrifugation was performed to separate the lipid layer and his laboratory values result (Table). Notable abnormalities included polycythemia with a hemoglobin of 17.4 g/dL, hyponatremia with a sodium of 129 mmol/L, normal renal function, elevated aspartate aminotransferase (AST) and alanine aminotransferase (ALT) (AST 258 IU/L and ALT 153 IU/L, respectively), hyperbilirubinemia with a total bilirubin of 2.7 mg/dL, and a serum alcohol level of 147 mg/dL. Due to anticipated requirement for a higher level of care, the patient was transferred to the Medical Intensive Care Unit (MICU).

Dr. Breu, can you help us interpret this patient’s numerous laboratory abnormalities? Without yet having the triglyceride level available, how does the fact that the patient’s blood was lipemic affect our interpretation of his labs? What further workup is warranted?

[embed:render:related:node:152200]► Anthony Breu, MD, Medical Service, VABHS, Assistant Professor of Medicine, Harvard Medical School. First, the positive alcohol level confirms a recent ingestion. Second, he has elevated transaminases with the AST greater than the ALT, which is consistent with alcoholic liver disease. While the initial assumption is that this patient has alcohol-induced pancreatitis, the elevations in bilirubin and alkaline phosphatase may suggest gallstone pancreatitis, and the lipemic appearing serum could suggest triglyceride-mediated pancreatitis. If the patient does have elevated triglyceride levels, the sodium level may indicate pseudohyponatremia, a laboratory artifact seen if a dilution step is used. To further evaluate the patient, I would obtain a triglyceride level and a right upper quadrant ultrasound. Direct ion-selective electrode analysis of the sodium level can be done with a device used to measure blood gases to exclude pseudohyponatremia.

► Dr. Ganatra. A right upper quadrant ultrasound was obtained in the MICU, which showed hepatic steatosis and hepatomegaly to 19 cm, but no evidence of biliary obstruction by stones or sludge. The common bile duct measured 3.2 mm in diameter. A triglyceride level returned above assay at > 3,392 mg/dL. A review of the medical record revealed a triglyceride level of 105 mg/dL 16 months prior. The Gastroenterology Department was consulted.

Dr. Weber, we now have 2 etiologies for pancreatitis in this patient: alcohol and hypertriglyceridemia. How do each cause pancreatitis? Is it possible to determine in this case which one is the more likely driver?

Dr. Weber. The mechanism for alcohol-induced pancreatitis is not fully known, but there are several hypotheses. One is that alcohol may increase the synthesis or activation of pancreatic digestive enzymes.6 Another is that metabolites of alcohol are directly toxic to the pancreas.6 Based on the epidemiologic observation that alcoholic pancreatitis usually happens in long-standing users, all we can say is that it is not very likely to be the effect of an acute insult. For hypertriglyceridemic pancreatitis, we believe the injury is due to the toxic effect of free fatty acids in the pancreas liberated by lipolysis of triglycerides by pancreatic lipases. Higher triglycerides are associated with higher risk, suggesting a dose-response relationship: This risk is not greatly increased until triglycerides exceed 500 mg/dL; above 1,000 mg/dL, the risk is about 5%, and above 2,000 mg/dL, the risk is between 10% and 20%.7 In summary, we cannot really determine whether the alcohol or the triglycerides are the main cause of his pancreatitis, but given his markedly elevated triglycerides, he should be treated for hypertriglyceridemic pancreatitis.

Dr. Ganatra. Dr. Breu, regardless of the underlying etiology, this patient requires treatment. What does the literature suggest as the best course of action regarding crystalloid administration in patients with acute pancreatitis?

►Dr. Breu. There are 2 issues to discuss regarding IV fluids in acute pancreatitis: choice of crystalloid and rate of administration. For the choice of IV fluid, lactated Ringer solution (LR) may be preferred over normal saline (NS). There are both pathophysiologic and evidence-based rationales for this choice. As Dr. Weber alluded to, trypsinogen activation is an important step in the pathogenesis of acute pancreatitis and requires a low pH compartment. As most clinicians have experienced, NS may cause a metabolic acidosis; however, the use of LR may mitigate this. A 2011 randomized clinical trial showed that patients who received LR had less systemic inflammatory response syndrome (SIRS) and lower C-reactive protein (CRP) levels at 24 hours compared with patients who received NS.8 While these are surrogate outcomes, they, along with the theoretical basis, suggest LR is preferred.

Regarding rate, the key is fast and early.9 In my experience, internists often underdose IV rehydration within the first 12 to 24 hours, fail to change the rate based on clinical response, and leave patients on high rates too long. In a patient like this, a rate of 350 cc/h is a reasonable place to start. But, one must reassess response (ie, ensure there is a decrease in hematocrit and/or blood urea nitrogen) every 6 hours and increase the rate as needed. After the first 24 to 48 hours have passed, the rate should be lowered.

[embed:render:related:node:143663]►Dr. Ganatra. The patient received 2 mg of IV hydromorphone and a 2 L bolus of LR. This was followed by a continuous infusion of LR at 200 cc/h. Dr. Weber, apart from the standard therapies for pancreatitis, what are our treatment options in hypertriglyceridemic pancreatitis?

Dr. Weber. In the acute setting, IV insulin with or without dextrose is the most extensively studied therapy. Insulin rapidly decreases triglyceride levels by activating lipoprotein lipase and inhibiting hormone- sensitive lipase. The net effect is reduction in serum triglycerides available to be hydrolyzed to free fatty acids in the pancreas.7 For severe cases (ie, where acute pancreatitis is accompanied by hypocalcemia, lactic acidosis or a markedly elevated lipase), apheresis with therapeutic plasma exchange to more rapidly reduce triglyceride concentration is the preferred therapy. The goal is to reduce triglycerides to levels

FED03502034_box.PNG
below 500 mg/dL, and this often can be accomplished after a single apheresis session. While there is no universally accepted maintenance strategy, there are good data to support the use of statins and fibrates to lower lipids and prevent future episodes in patients with markedly elevated triglycerides.10

Dr. Ganatra. Due to the possibility that the patient would require apheresis, which was not available at the VABHS West Roxbury campus, the patient was transferred to an affiliate hospital. The patient was started on 10% dextrose at 300 cc/h and an IV insulin infusion. His triglycerides fell to < 500 mg/dL over the subsequent 48 hours, and ultimately, apheresis was not required. Enteral nutrition by nasogastric (NG) tube was initiated on hospital day 6. The patient’s hospital course was notable for acute respiratory distress syndrome that required intubation for 7 days, hyperbilirubinemia (with a peak bilirubin of 10.5 mg/dL), acute kidney injury (with a peak creatinine 4.7 mg/dL), fever without an identified infectious source, alcohol withdrawal syndrome that required phenobarbital, and delirium. Nine days later, he was transferred back to the VABHS West Roxbury campus. His condition stabilized, and he was transferred to the medical floor. On hospital day 14, the patient’s mental status improved, and he began tolerating oral nutrition.

Dr. Breu, over the years, the standard of care regarding when to start enteral nutrition in pancreatitis has changed considerably. This patient received enteral nutrition via NG tube but also had periods of being NPO (nothing by mouth) for up to 6 days. What is the current best practice for timing of initiating enteral nutrition in acute pancreatitis?

Dr. Breu. It is true that the standard of care has changed and continues to evolve. Many decades ago, patients with acute pancreatitis would routinely undergo NG tube suction to reduce delivery of gastric contents to the duodenum, thereby decreasing pancreas activation, allowing it to rest.11 The NG tube also allowed for decompression of any ileus that had formed. Beginning in the 1970s, several clinical trials were performed, showing that NG tube suction was no better than simply making the patient NPO.12,13 More recently, we have begun to move toward earlier feeding. Again, there is a pathophysiologic rationale (bowel rest is associated with intestinal atrophy, predisposing to bacterial translocation and resulting infectious complications) and increasing evidence supporting this practice.9 Even in severe pancreatitis, hunger may be used to initiate oral intake.14

 

 

Dr. Ganatra. On hospital day 16, the patient developed sudden-onset right-sided back and flank pain, and his hemoglobin dropped to 6.1 mg/dL, which required transfusion of packed red blood cells. He remained afebrile and hemodynamically stable. Dr. Weber, what are the major complications of acute pancreatitis, and when should we suspect them? Should we be worried about complications of pancreatitis in this patient?

[embed:render:related:node:133708]

Dr. Weber. Organ failure in the acute setting can occur due to activation of cytokine cascades and the systemic inflammatory response syndrome and is described by clinical and radiologic criteria called the Atlanta Classification.15 Apart from organ failure, the most serious complications of acute pancreatitis are necrosis of pancreatic tissue leading to walled-off pancreatic necrosis and the formation of peripancreatic fluid collections and pseudocysts, which occur in about 15% of patients with acute pancreatitis. These complications are serious because they can become infected, which portends a higher mortality and in some cases require surgical resection.

Other complications of acute pancreatitis include pseudoaneurysm formation, which is when a vessel bleeds into a pancreatic pseudocyst, and thromboses of the splenic, portal, or mesenteric veins. Thrombotic complications may occur in up to half of patients with pancreatic necrosis but are uncommon without some degree of necrosis.16 No necrosis was noted on this patient’s initial CT scan, so the probability of thrombosis is low. Also, as it takes several weeks for pseudocyst formation to occur, a bleeding pseudoaneurysm is unlikely at this early stage. Therefore, a complication of pancreatitis is unlikely in this patient, and evaluation for other causes of abdominal pain should be considered.

Dr. Ganatra. A noncontrast CT of the abdomen and pelvis was obtained and revealed no evidence of complications or other acute pathology. His pain was managed conservatively, and hemoglobin remained stable. Over the next 5 days, the patient’s symptoms gradually resolved, his oral intake improved, and he was discharged home on gemfibrozil 600 mg twice daily 19 days after admission. He declined psychiatry follow-up for his PTSD, and after discharge he did not keep his scheduled gastroenterology (GI) follow-up appointment. Four months later, the patient presented again with epigastric abdominal pain similar to his initial presentation. The patient had resumed drinking, stating that “alcohol is the only thing that helps [with the PTSD].” He had not been taking the gemfibrozil. He was admitted with a recurrent episode of pancreatitis; however, his triglycerides on admission were 119 mg/dL.

Dr. Weber, this patient’s triglycerides declined rapidly over a period of just 4 months with questionable adherence to gemfibrozil. However, he was admitted again with another episode of pancreatitis, this time in the setting of alcohol use alone without markedly elevated triglycerides. What do we know about recurrence risk for pancreatitis? Are some etiologies of pancreatitis more likely to present with recurrent attacks than are others?

Dr. Weber. The rate of recurrence following an episode of acute pancreatitis varies according to the cause, but in general, about 20% to 30% of patients will experience a recurrence, and 5% to 10% will go on to develop chronic pancreatitis.17 Alcoholic pancreatitis does carry a higher risk of recurrence than pancreatitis due to other causes; the risk is as high as 50%. Not surprisingly, recurrence of acute pancreatitis increases risk for development of chronic pancreatitis. As this patient is a smoker, it is worth noting that smoking potentiates pancreatic damage from alcohol and increases the risk for both recurrent and chronic pancreatitis.5

Dr. Ganatra. The patient was treated with IV hydromorphone and IV LR at 350 cc/h. Oral nutrition was begun immediately. He manifested no organ dysfunction, and his symptoms improved over the course of 48 hours. He was discharged home with psychiatry and GI follow-up scheduled. Dr. Breu and Dr. Weber, how should we counsel this patient to reduce his risk of recurrent attacks of pancreatitis in the future, and what options do we have for pharmacotherapy to decrease his risk?

Dr. Breu. I’ll let Dr. Weber comment on mitigating the risk of hypertriglyceride-induced pancreatitis and reserve my comments to pharmacotherapy in alcohol use disorder. This patient may be a candidate for naltrexone therapy, either in oral or intramuscular formulations. Both have been showed to reduce the risk of returning to heavy drinking and may be particularly beneficial in those with a family history.18,19 Acamprosate is also an option.

Dr. Weber. Data on recurrence risk in hypertriglyceridemic pancreatitis are limited, but there are case reports suggesting that a fatty diet and alcohol use are implicated in recurrence.20 I would counsel the patient on lifestyle modifications that are known to reduce this risk. I agree with Dr. Breu that devoting our efforts to helping him reduce or eliminate his alcohol consumption is the single most important thing we can do to reduce his risk for recurrent attacks. Since the patient reports that he drinks alcohol in order to cope with his PTSD, establishing care with a mental health provider to address this is of the utmost importance. In addition, smoking cessation and promoting medication adherence with gemfibrozil will also reduce risk for future episodes, but continued alcohol use is his strongest risk factor.

 

►Dr. Ganatra. After discharge, the patient engaged with outpatient psychiatry and GI. He still reports feeling that alcohol is the only thing that alleviates his PTSD and anxiety symptoms. He is not currently interested in pharmacotherapy for cessation of alcohol use.

Acknowledgments
The authors thank Ivana Jankovic, MD, Matthew Lewis Chase, MD, Christopher Worsham, MD, Lakshmana Swamy, MD.

References

1. Dufour MC, Adamson MD. The epidemiology of alcohol-induced pancreatitis. Pancreas. 2003;27(4):286-290.

2. Peery AF, Dellon ES, Lund J, et al. Burden of gastrointestinal disease in the United States: 2012 update. Gastroenterology. 2012;143(5):1179-1187.e1-e3.

3. Cavallini G, Frulloni L, Bassi C, et al; ProInf-AISP Study Group. Prospective multicentre survey on acute pancreatitis in Italy (ProInf-AISP): results on 1005 patients. Dig Liver Dis. 2004;36(3):205-211.

4. Banks PA, Freeman ML; Practice Parameters Committee of the American College of Gastroenterology. Practice guidelines in acute pancreatitis. Am J Gastroenterol. 2006;101(10):2379-2400.

5. Hartwig W, Werner J, Ryschich E, et al. Cigarette smoke enhances ethanol-induced pancreatic injury. Pancreas. 2000;21(3):272-278.

6. Chowdhury P, Gupta P. Pathophysiology of alcoholic pancreatitis: an overview. World J Gastroenterol. 2006;12(46):7421-7427.

7. Scherer J, Singh VP, Pitchumoni CS, Yadav D. Issues in hypertriglyceridemic pancreatitis: an update. J Clin Gastroenterol. 2014;48(3):195-203.

8. Wu BU, Hwang JQ, Gardner TH, et al. Lactated Ringer’s solution reduces systemic inflammation compared with saline in patients with acute pancreatitis. Clin Gastroenterol Hepatol. 2011;9(8):710-717.e1.

9. Tenner S, Baillie J, DeWitt J, Vege SS; American College of Gastroenterology. American College of Gastroenterology guideline: management of acute pancreatitis. Am J Gastroenterol. 2013;108(9):1400-1415; 1416.

10. Preiss D, Tikkanen MJ, Welsh P, et al. Lipid-modifying therapies and risk of pancreatitis: a meta-analysis. JAMA. 2012;308(8):804-811.

11. Nardi GL. Pancreatitis. N Engl J Med. 1963;268(19):1065-1067.

12. Naeije R, Salingret E, Clumeck N, De Troyer A, Devis G. Is nasogastric suction necessary in acute pancreatitis? Br Med J. 1978;2(6138):659-660.

13. Levant JA, Secrist DM, Resin H, Sturdevant RA, Guth PH. Nasogastric suction in the treatment of alcoholic pancreatitis: a controlled study. JAMA. 1974;229(1):51-52.

14. Zhao XL, Zhu SF, Xue GJ, et al. Early oral refeeding based on hunger in moderate and severe acute pancreatitis: a prospective controlled, randomized clinical trial. Nutrition. 2015;31(1):171-175.

15. Banks PA, Bollen TL, Dervenis C, et al; Acute Pancreatitis Classification Working Group. Classification of acute pancreatitis—2012: revision of the Atlanta classification and definitions by international consensus. Gut. 2013;62(1):102-111.

16. Easler J, Muddana V, Furlan A, et al. Portosplenomesenteric venous thrombosis in patients with acute pancreatitis is associated with pancreatic necrosis and usually has a benign course. Clin Gastroenterol Hepatol. 2014;12(5):854-862.

17. Yadav D, O’Connell M, Papachristou GI. Natural history following the first attack of acute pancreatitis. Am J Gastroenterol. 2012;107(7):1096-1103.

18. Jonas DE, Amick HR, Feltner C, et al. Pharmacotherapy for adults with alcohol use disorders in outpatient settings: a systematic review and meta-analysis. JAMA. 2014;311(18):1889-1900.

19. Garbutt JC, Kranzler HR, O’Malley SS, et al. Efficacy and tolerability of long-acting injectable naltrexone for alcohol dependence: a randomized controlled trial. JAMA. 2005;293(13):1617-1625.

20. Piolot A, Nadler F, Cavallero E, Coquard JL, Jacotot B. Prevention of recurrent acute pancreatitis in patients with severe hypertriglyceridemia: value of regular plasmapheresis. Pancreas. 1996;13(1):96-99.

Article PDF
Author and Disclosure Information

Dr. Breu is a Hospitalist and the Director of Resident Education at VA Boston Healthcare
System and an Assistant Professor of Medicine at Harvard University in Massachusetts and
supervises the VA Boston Medical Forum Chief Resident case conferences. All patients or their
surrogate decision makers understand and have signed appropriate patient release forms. This
article has received an abbreviated peer review.
Correspondence: Dr. Breu (anthony. breu@va.gov)

Author disclosures
The authors report no actual or potential conflicts of interest with regard to this article.

Disclaimer
The opinions expressed herein are those of the authors and do not necessarily reflect those of
Federal Practitioner, Frontline Medical Communications Inc., the U.S. Government, or any of its agencies. This article may discuss unlabeled or investigational use of certain drugs. Please review the complete prescribing information for specific drugs or drug combinations—including indications, contraindications, warnings, and adverse effects—before administering pharmacologic therapy to patients.

Issue
Federal Practitioner - 35(2)a
Publications
Topics
Page Number
34-39
Sections
Author and Disclosure Information

Dr. Breu is a Hospitalist and the Director of Resident Education at VA Boston Healthcare
System and an Assistant Professor of Medicine at Harvard University in Massachusetts and
supervises the VA Boston Medical Forum Chief Resident case conferences. All patients or their
surrogate decision makers understand and have signed appropriate patient release forms. This
article has received an abbreviated peer review.
Correspondence: Dr. Breu (anthony. breu@va.gov)

Author disclosures
The authors report no actual or potential conflicts of interest with regard to this article.

Disclaimer
The opinions expressed herein are those of the authors and do not necessarily reflect those of
Federal Practitioner, Frontline Medical Communications Inc., the U.S. Government, or any of its agencies. This article may discuss unlabeled or investigational use of certain drugs. Please review the complete prescribing information for specific drugs or drug combinations—including indications, contraindications, warnings, and adverse effects—before administering pharmacologic therapy to patients.

Author and Disclosure Information

Dr. Breu is a Hospitalist and the Director of Resident Education at VA Boston Healthcare
System and an Assistant Professor of Medicine at Harvard University in Massachusetts and
supervises the VA Boston Medical Forum Chief Resident case conferences. All patients or their
surrogate decision makers understand and have signed appropriate patient release forms. This
article has received an abbreviated peer review.
Correspondence: Dr. Breu (anthony. breu@va.gov)

Author disclosures
The authors report no actual or potential conflicts of interest with regard to this article.

Disclaimer
The opinions expressed herein are those of the authors and do not necessarily reflect those of
Federal Practitioner, Frontline Medical Communications Inc., the U.S. Government, or any of its agencies. This article may discuss unlabeled or investigational use of certain drugs. Please review the complete prescribing information for specific drugs or drug combinations—including indications, contraindications, warnings, and adverse effects—before administering pharmacologic therapy to patients.

Article PDF
Article PDF
Discussion of treatment for a young patient with untreated posttraumatic stress disorder and acute abdominal pain.
Discussion of treatment for a young patient with untreated posttraumatic stress disorder and acute abdominal pain.

Case Presentation. A 23-year-old male U.S. Army veteran with a history of alcohol use disorder and posttraumatic stress disorder (PTSD) presented to the VA Boston Healthcare System (VABHS) West Roxbury campus emergency department (ED) with epigastric abdominal pain in the setting of consuming alcohol. The patient had served in the infantry in Afghanistan during Operation Enduring Freedom. He consumed up to 12 alcoholic drinks per day (both beer and hard liquor) for the past 3 years and had been hospitalized 3 times previously; twice for alcohol detoxification and once for PTSD. He is a former tobacco smoker with fewer than 5 pack-years, he uses marijuana often and does not use IV drugs. In the ED, his physical examination was notable for a heart rate of 130 beats per minute and blood pressure of 161/111 mm Hg. He was alert and oriented and had a mild tremor. The patient was diaphoretic with dry mucous membranes, tenderness to palpation in the epigastrium, and abdominal guarding. A computed tomography (CT) scan of the abdomen revealed acute pancreatitis without necrosis. The patient received 1 L of normal saline and was admitted to the medical ward for presumed alcoholic pancreatitis.

► Rahul Ganatra, MD, MPH, Chief Medical Resident, VABHS and Beth Israel Deaconess Medical Center. Dr. Weber, we care for many young people who drink more than they should and almost none of them end up with alcoholic pancreatitis. What are the relevant risk factors that make individuals like this patient more susceptible to alcoholic pancreatitis?

Horst Christian Weber, MD, Gastroenterology Service, VABHS, and Assistant Professor of Medicine, Boston University School of Medicine. While we don’t have a good understanding of the precise mechanism of alcoholic pancreatitis, we do know that in the U.S., alcohol consumption is responsible for about one-third of all cases.1 Acute pancreatitis in general may present with a wide range of disease severity. It is the most common cause of gastrointestinal-related hospitalization,2 and the mortality of hospital inpatients with pancreatitis is about 5%.3,4 Therefore, acute pancreatitis represents a prevalent condition with a critical impact on morbidity and mortality. Alcoholic pancreatitis typically occurs after many years of heavy alcohol use, not after a single drinking

FED03502034_t.PNG
binge in alcohol-naïve individuals.1 Regarding this patient, we also know that smoking is an independent risk factor for pancreatitis and may have a synergistic effect with alcohol.1,5 In addition, there are genetic mutations that can increase a person’s risk for pancreatitis, which is of consideration given this patient’s young age. Apart from those 2 possible contributors, there does not currently seem to be another more likely explanation than alcohol.

Dr. Ganatra. At this point, the chemistry laboratory paged the admitting resident with the notification that the patient’s blood was grossly lipemic. Ultracentrifugation was performed to separate the lipid layer and his laboratory values result (Table). Notable abnormalities included polycythemia with a hemoglobin of 17.4 g/dL, hyponatremia with a sodium of 129 mmol/L, normal renal function, elevated aspartate aminotransferase (AST) and alanine aminotransferase (ALT) (AST 258 IU/L and ALT 153 IU/L, respectively), hyperbilirubinemia with a total bilirubin of 2.7 mg/dL, and a serum alcohol level of 147 mg/dL. Due to anticipated requirement for a higher level of care, the patient was transferred to the Medical Intensive Care Unit (MICU).

Dr. Breu, can you help us interpret this patient’s numerous laboratory abnormalities? Without yet having the triglyceride level available, how does the fact that the patient’s blood was lipemic affect our interpretation of his labs? What further workup is warranted?

[embed:render:related:node:152200]► Anthony Breu, MD, Medical Service, VABHS, Assistant Professor of Medicine, Harvard Medical School. First, the positive alcohol level confirms a recent ingestion. Second, he has elevated transaminases with the AST greater than the ALT, which is consistent with alcoholic liver disease. While the initial assumption is that this patient has alcohol-induced pancreatitis, the elevations in bilirubin and alkaline phosphatase may suggest gallstone pancreatitis, and the lipemic appearing serum could suggest triglyceride-mediated pancreatitis. If the patient does have elevated triglyceride levels, the sodium level may indicate pseudohyponatremia, a laboratory artifact seen if a dilution step is used. To further evaluate the patient, I would obtain a triglyceride level and a right upper quadrant ultrasound. Direct ion-selective electrode analysis of the sodium level can be done with a device used to measure blood gases to exclude pseudohyponatremia.

► Dr. Ganatra. A right upper quadrant ultrasound was obtained in the MICU, which showed hepatic steatosis and hepatomegaly to 19 cm, but no evidence of biliary obstruction by stones or sludge. The common bile duct measured 3.2 mm in diameter. A triglyceride level returned above assay at > 3,392 mg/dL. A review of the medical record revealed a triglyceride level of 105 mg/dL 16 months prior. The Gastroenterology Department was consulted.

Dr. Weber, we now have 2 etiologies for pancreatitis in this patient: alcohol and hypertriglyceridemia. How do each cause pancreatitis? Is it possible to determine in this case which one is the more likely driver?

Dr. Weber. The mechanism for alcohol-induced pancreatitis is not fully known, but there are several hypotheses. One is that alcohol may increase the synthesis or activation of pancreatic digestive enzymes.6 Another is that metabolites of alcohol are directly toxic to the pancreas.6 Based on the epidemiologic observation that alcoholic pancreatitis usually happens in long-standing users, all we can say is that it is not very likely to be the effect of an acute insult. For hypertriglyceridemic pancreatitis, we believe the injury is due to the toxic effect of free fatty acids in the pancreas liberated by lipolysis of triglycerides by pancreatic lipases. Higher triglycerides are associated with higher risk, suggesting a dose-response relationship: This risk is not greatly increased until triglycerides exceed 500 mg/dL; above 1,000 mg/dL, the risk is about 5%, and above 2,000 mg/dL, the risk is between 10% and 20%.7 In summary, we cannot really determine whether the alcohol or the triglycerides are the main cause of his pancreatitis, but given his markedly elevated triglycerides, he should be treated for hypertriglyceridemic pancreatitis.

Dr. Ganatra. Dr. Breu, regardless of the underlying etiology, this patient requires treatment. What does the literature suggest as the best course of action regarding crystalloid administration in patients with acute pancreatitis?

►Dr. Breu. There are 2 issues to discuss regarding IV fluids in acute pancreatitis: choice of crystalloid and rate of administration. For the choice of IV fluid, lactated Ringer solution (LR) may be preferred over normal saline (NS). There are both pathophysiologic and evidence-based rationales for this choice. As Dr. Weber alluded to, trypsinogen activation is an important step in the pathogenesis of acute pancreatitis and requires a low pH compartment. As most clinicians have experienced, NS may cause a metabolic acidosis; however, the use of LR may mitigate this. A 2011 randomized clinical trial showed that patients who received LR had less systemic inflammatory response syndrome (SIRS) and lower C-reactive protein (CRP) levels at 24 hours compared with patients who received NS.8 While these are surrogate outcomes, they, along with the theoretical basis, suggest LR is preferred.

Regarding rate, the key is fast and early.9 In my experience, internists often underdose IV rehydration within the first 12 to 24 hours, fail to change the rate based on clinical response, and leave patients on high rates too long. In a patient like this, a rate of 350 cc/h is a reasonable place to start. But, one must reassess response (ie, ensure there is a decrease in hematocrit and/or blood urea nitrogen) every 6 hours and increase the rate as needed. After the first 24 to 48 hours have passed, the rate should be lowered.

[embed:render:related:node:143663]►Dr. Ganatra. The patient received 2 mg of IV hydromorphone and a 2 L bolus of LR. This was followed by a continuous infusion of LR at 200 cc/h. Dr. Weber, apart from the standard therapies for pancreatitis, what are our treatment options in hypertriglyceridemic pancreatitis?

Dr. Weber. In the acute setting, IV insulin with or without dextrose is the most extensively studied therapy. Insulin rapidly decreases triglyceride levels by activating lipoprotein lipase and inhibiting hormone- sensitive lipase. The net effect is reduction in serum triglycerides available to be hydrolyzed to free fatty acids in the pancreas.7 For severe cases (ie, where acute pancreatitis is accompanied by hypocalcemia, lactic acidosis or a markedly elevated lipase), apheresis with therapeutic plasma exchange to more rapidly reduce triglyceride concentration is the preferred therapy. The goal is to reduce triglycerides to levels

FED03502034_box.PNG
below 500 mg/dL, and this often can be accomplished after a single apheresis session. While there is no universally accepted maintenance strategy, there are good data to support the use of statins and fibrates to lower lipids and prevent future episodes in patients with markedly elevated triglycerides.10

Dr. Ganatra. Due to the possibility that the patient would require apheresis, which was not available at the VABHS West Roxbury campus, the patient was transferred to an affiliate hospital. The patient was started on 10% dextrose at 300 cc/h and an IV insulin infusion. His triglycerides fell to < 500 mg/dL over the subsequent 48 hours, and ultimately, apheresis was not required. Enteral nutrition by nasogastric (NG) tube was initiated on hospital day 6. The patient’s hospital course was notable for acute respiratory distress syndrome that required intubation for 7 days, hyperbilirubinemia (with a peak bilirubin of 10.5 mg/dL), acute kidney injury (with a peak creatinine 4.7 mg/dL), fever without an identified infectious source, alcohol withdrawal syndrome that required phenobarbital, and delirium. Nine days later, he was transferred back to the VABHS West Roxbury campus. His condition stabilized, and he was transferred to the medical floor. On hospital day 14, the patient’s mental status improved, and he began tolerating oral nutrition.

Dr. Breu, over the years, the standard of care regarding when to start enteral nutrition in pancreatitis has changed considerably. This patient received enteral nutrition via NG tube but also had periods of being NPO (nothing by mouth) for up to 6 days. What is the current best practice for timing of initiating enteral nutrition in acute pancreatitis?

Dr. Breu. It is true that the standard of care has changed and continues to evolve. Many decades ago, patients with acute pancreatitis would routinely undergo NG tube suction to reduce delivery of gastric contents to the duodenum, thereby decreasing pancreas activation, allowing it to rest.11 The NG tube also allowed for decompression of any ileus that had formed. Beginning in the 1970s, several clinical trials were performed, showing that NG tube suction was no better than simply making the patient NPO.12,13 More recently, we have begun to move toward earlier feeding. Again, there is a pathophysiologic rationale (bowel rest is associated with intestinal atrophy, predisposing to bacterial translocation and resulting infectious complications) and increasing evidence supporting this practice.9 Even in severe pancreatitis, hunger may be used to initiate oral intake.14

 

 

Dr. Ganatra. On hospital day 16, the patient developed sudden-onset right-sided back and flank pain, and his hemoglobin dropped to 6.1 mg/dL, which required transfusion of packed red blood cells. He remained afebrile and hemodynamically stable. Dr. Weber, what are the major complications of acute pancreatitis, and when should we suspect them? Should we be worried about complications of pancreatitis in this patient?

[embed:render:related:node:133708]

Dr. Weber. Organ failure in the acute setting can occur due to activation of cytokine cascades and the systemic inflammatory response syndrome and is described by clinical and radiologic criteria called the Atlanta Classification.15 Apart from organ failure, the most serious complications of acute pancreatitis are necrosis of pancreatic tissue leading to walled-off pancreatic necrosis and the formation of peripancreatic fluid collections and pseudocysts, which occur in about 15% of patients with acute pancreatitis. These complications are serious because they can become infected, which portends a higher mortality and in some cases require surgical resection.

Other complications of acute pancreatitis include pseudoaneurysm formation, which is when a vessel bleeds into a pancreatic pseudocyst, and thromboses of the splenic, portal, or mesenteric veins. Thrombotic complications may occur in up to half of patients with pancreatic necrosis but are uncommon without some degree of necrosis.16 No necrosis was noted on this patient’s initial CT scan, so the probability of thrombosis is low. Also, as it takes several weeks for pseudocyst formation to occur, a bleeding pseudoaneurysm is unlikely at this early stage. Therefore, a complication of pancreatitis is unlikely in this patient, and evaluation for other causes of abdominal pain should be considered.

Dr. Ganatra. A noncontrast CT of the abdomen and pelvis was obtained and revealed no evidence of complications or other acute pathology. His pain was managed conservatively, and hemoglobin remained stable. Over the next 5 days, the patient’s symptoms gradually resolved, his oral intake improved, and he was discharged home on gemfibrozil 600 mg twice daily 19 days after admission. He declined psychiatry follow-up for his PTSD, and after discharge he did not keep his scheduled gastroenterology (GI) follow-up appointment. Four months later, the patient presented again with epigastric abdominal pain similar to his initial presentation. The patient had resumed drinking, stating that “alcohol is the only thing that helps [with the PTSD].” He had not been taking the gemfibrozil. He was admitted with a recurrent episode of pancreatitis; however, his triglycerides on admission were 119 mg/dL.

Dr. Weber, this patient’s triglycerides declined rapidly over a period of just 4 months with questionable adherence to gemfibrozil. However, he was admitted again with another episode of pancreatitis, this time in the setting of alcohol use alone without markedly elevated triglycerides. What do we know about recurrence risk for pancreatitis? Are some etiologies of pancreatitis more likely to present with recurrent attacks than are others?

Dr. Weber. The rate of recurrence following an episode of acute pancreatitis varies according to the cause, but in general, about 20% to 30% of patients will experience a recurrence, and 5% to 10% will go on to develop chronic pancreatitis.17 Alcoholic pancreatitis does carry a higher risk of recurrence than pancreatitis due to other causes; the risk is as high as 50%. Not surprisingly, recurrence of acute pancreatitis increases risk for development of chronic pancreatitis. As this patient is a smoker, it is worth noting that smoking potentiates pancreatic damage from alcohol and increases the risk for both recurrent and chronic pancreatitis.5

Dr. Ganatra. The patient was treated with IV hydromorphone and IV LR at 350 cc/h. Oral nutrition was begun immediately. He manifested no organ dysfunction, and his symptoms improved over the course of 48 hours. He was discharged home with psychiatry and GI follow-up scheduled. Dr. Breu and Dr. Weber, how should we counsel this patient to reduce his risk of recurrent attacks of pancreatitis in the future, and what options do we have for pharmacotherapy to decrease his risk?

Dr. Breu. I’ll let Dr. Weber comment on mitigating the risk of hypertriglyceride-induced pancreatitis and reserve my comments to pharmacotherapy in alcohol use disorder. This patient may be a candidate for naltrexone therapy, either in oral or intramuscular formulations. Both have been showed to reduce the risk of returning to heavy drinking and may be particularly beneficial in those with a family history.18,19 Acamprosate is also an option.

Dr. Weber. Data on recurrence risk in hypertriglyceridemic pancreatitis are limited, but there are case reports suggesting that a fatty diet and alcohol use are implicated in recurrence.20 I would counsel the patient on lifestyle modifications that are known to reduce this risk. I agree with Dr. Breu that devoting our efforts to helping him reduce or eliminate his alcohol consumption is the single most important thing we can do to reduce his risk for recurrent attacks. Since the patient reports that he drinks alcohol in order to cope with his PTSD, establishing care with a mental health provider to address this is of the utmost importance. In addition, smoking cessation and promoting medication adherence with gemfibrozil will also reduce risk for future episodes, but continued alcohol use is his strongest risk factor.

 

►Dr. Ganatra. After discharge, the patient engaged with outpatient psychiatry and GI. He still reports feeling that alcohol is the only thing that alleviates his PTSD and anxiety symptoms. He is not currently interested in pharmacotherapy for cessation of alcohol use.

Acknowledgments
The authors thank Ivana Jankovic, MD, Matthew Lewis Chase, MD, Christopher Worsham, MD, Lakshmana Swamy, MD.

Case Presentation. A 23-year-old male U.S. Army veteran with a history of alcohol use disorder and posttraumatic stress disorder (PTSD) presented to the VA Boston Healthcare System (VABHS) West Roxbury campus emergency department (ED) with epigastric abdominal pain in the setting of consuming alcohol. The patient had served in the infantry in Afghanistan during Operation Enduring Freedom. He consumed up to 12 alcoholic drinks per day (both beer and hard liquor) for the past 3 years and had been hospitalized 3 times previously; twice for alcohol detoxification and once for PTSD. He is a former tobacco smoker with fewer than 5 pack-years, he uses marijuana often and does not use IV drugs. In the ED, his physical examination was notable for a heart rate of 130 beats per minute and blood pressure of 161/111 mm Hg. He was alert and oriented and had a mild tremor. The patient was diaphoretic with dry mucous membranes, tenderness to palpation in the epigastrium, and abdominal guarding. A computed tomography (CT) scan of the abdomen revealed acute pancreatitis without necrosis. The patient received 1 L of normal saline and was admitted to the medical ward for presumed alcoholic pancreatitis.

► Rahul Ganatra, MD, MPH, Chief Medical Resident, VABHS and Beth Israel Deaconess Medical Center. Dr. Weber, we care for many young people who drink more than they should and almost none of them end up with alcoholic pancreatitis. What are the relevant risk factors that make individuals like this patient more susceptible to alcoholic pancreatitis?

Horst Christian Weber, MD, Gastroenterology Service, VABHS, and Assistant Professor of Medicine, Boston University School of Medicine. While we don’t have a good understanding of the precise mechanism of alcoholic pancreatitis, we do know that in the U.S., alcohol consumption is responsible for about one-third of all cases.1 Acute pancreatitis in general may present with a wide range of disease severity. It is the most common cause of gastrointestinal-related hospitalization,2 and the mortality of hospital inpatients with pancreatitis is about 5%.3,4 Therefore, acute pancreatitis represents a prevalent condition with a critical impact on morbidity and mortality. Alcoholic pancreatitis typically occurs after many years of heavy alcohol use, not after a single drinking

FED03502034_t.PNG
binge in alcohol-naïve individuals.1 Regarding this patient, we also know that smoking is an independent risk factor for pancreatitis and may have a synergistic effect with alcohol.1,5 In addition, there are genetic mutations that can increase a person’s risk for pancreatitis, which is of consideration given this patient’s young age. Apart from those 2 possible contributors, there does not currently seem to be another more likely explanation than alcohol.

Dr. Ganatra. At this point, the chemistry laboratory paged the admitting resident with the notification that the patient’s blood was grossly lipemic. Ultracentrifugation was performed to separate the lipid layer and his laboratory values result (Table). Notable abnormalities included polycythemia with a hemoglobin of 17.4 g/dL, hyponatremia with a sodium of 129 mmol/L, normal renal function, elevated aspartate aminotransferase (AST) and alanine aminotransferase (ALT) (AST 258 IU/L and ALT 153 IU/L, respectively), hyperbilirubinemia with a total bilirubin of 2.7 mg/dL, and a serum alcohol level of 147 mg/dL. Due to anticipated requirement for a higher level of care, the patient was transferred to the Medical Intensive Care Unit (MICU).

Dr. Breu, can you help us interpret this patient’s numerous laboratory abnormalities? Without yet having the triglyceride level available, how does the fact that the patient’s blood was lipemic affect our interpretation of his labs? What further workup is warranted?

[embed:render:related:node:152200]► Anthony Breu, MD, Medical Service, VABHS, Assistant Professor of Medicine, Harvard Medical School. First, the positive alcohol level confirms a recent ingestion. Second, he has elevated transaminases with the AST greater than the ALT, which is consistent with alcoholic liver disease. While the initial assumption is that this patient has alcohol-induced pancreatitis, the elevations in bilirubin and alkaline phosphatase may suggest gallstone pancreatitis, and the lipemic appearing serum could suggest triglyceride-mediated pancreatitis. If the patient does have elevated triglyceride levels, the sodium level may indicate pseudohyponatremia, a laboratory artifact seen if a dilution step is used. To further evaluate the patient, I would obtain a triglyceride level and a right upper quadrant ultrasound. Direct ion-selective electrode analysis of the sodium level can be done with a device used to measure blood gases to exclude pseudohyponatremia.

► Dr. Ganatra. A right upper quadrant ultrasound was obtained in the MICU, which showed hepatic steatosis and hepatomegaly to 19 cm, but no evidence of biliary obstruction by stones or sludge. The common bile duct measured 3.2 mm in diameter. A triglyceride level returned above assay at > 3,392 mg/dL. A review of the medical record revealed a triglyceride level of 105 mg/dL 16 months prior. The Gastroenterology Department was consulted.

Dr. Weber, we now have 2 etiologies for pancreatitis in this patient: alcohol and hypertriglyceridemia. How do each cause pancreatitis? Is it possible to determine in this case which one is the more likely driver?

Dr. Weber. The mechanism for alcohol-induced pancreatitis is not fully known, but there are several hypotheses. One is that alcohol may increase the synthesis or activation of pancreatic digestive enzymes.6 Another is that metabolites of alcohol are directly toxic to the pancreas.6 Based on the epidemiologic observation that alcoholic pancreatitis usually happens in long-standing users, all we can say is that it is not very likely to be the effect of an acute insult. For hypertriglyceridemic pancreatitis, we believe the injury is due to the toxic effect of free fatty acids in the pancreas liberated by lipolysis of triglycerides by pancreatic lipases. Higher triglycerides are associated with higher risk, suggesting a dose-response relationship: This risk is not greatly increased until triglycerides exceed 500 mg/dL; above 1,000 mg/dL, the risk is about 5%, and above 2,000 mg/dL, the risk is between 10% and 20%.7 In summary, we cannot really determine whether the alcohol or the triglycerides are the main cause of his pancreatitis, but given his markedly elevated triglycerides, he should be treated for hypertriglyceridemic pancreatitis.

Dr. Ganatra. Dr. Breu, regardless of the underlying etiology, this patient requires treatment. What does the literature suggest as the best course of action regarding crystalloid administration in patients with acute pancreatitis?

►Dr. Breu. There are 2 issues to discuss regarding IV fluids in acute pancreatitis: choice of crystalloid and rate of administration. For the choice of IV fluid, lactated Ringer solution (LR) may be preferred over normal saline (NS). There are both pathophysiologic and evidence-based rationales for this choice. As Dr. Weber alluded to, trypsinogen activation is an important step in the pathogenesis of acute pancreatitis and requires a low pH compartment. As most clinicians have experienced, NS may cause a metabolic acidosis; however, the use of LR may mitigate this. A 2011 randomized clinical trial showed that patients who received LR had less systemic inflammatory response syndrome (SIRS) and lower C-reactive protein (CRP) levels at 24 hours compared with patients who received NS.8 While these are surrogate outcomes, they, along with the theoretical basis, suggest LR is preferred.

Regarding rate, the key is fast and early.9 In my experience, internists often underdose IV rehydration within the first 12 to 24 hours, fail to change the rate based on clinical response, and leave patients on high rates too long. In a patient like this, a rate of 350 cc/h is a reasonable place to start. But, one must reassess response (ie, ensure there is a decrease in hematocrit and/or blood urea nitrogen) every 6 hours and increase the rate as needed. After the first 24 to 48 hours have passed, the rate should be lowered.

[embed:render:related:node:143663]►Dr. Ganatra. The patient received 2 mg of IV hydromorphone and a 2 L bolus of LR. This was followed by a continuous infusion of LR at 200 cc/h. Dr. Weber, apart from the standard therapies for pancreatitis, what are our treatment options in hypertriglyceridemic pancreatitis?

Dr. Weber. In the acute setting, IV insulin with or without dextrose is the most extensively studied therapy. Insulin rapidly decreases triglyceride levels by activating lipoprotein lipase and inhibiting hormone- sensitive lipase. The net effect is reduction in serum triglycerides available to be hydrolyzed to free fatty acids in the pancreas.7 For severe cases (ie, where acute pancreatitis is accompanied by hypocalcemia, lactic acidosis or a markedly elevated lipase), apheresis with therapeutic plasma exchange to more rapidly reduce triglyceride concentration is the preferred therapy. The goal is to reduce triglycerides to levels

FED03502034_box.PNG
below 500 mg/dL, and this often can be accomplished after a single apheresis session. While there is no universally accepted maintenance strategy, there are good data to support the use of statins and fibrates to lower lipids and prevent future episodes in patients with markedly elevated triglycerides.10

Dr. Ganatra. Due to the possibility that the patient would require apheresis, which was not available at the VABHS West Roxbury campus, the patient was transferred to an affiliate hospital. The patient was started on 10% dextrose at 300 cc/h and an IV insulin infusion. His triglycerides fell to < 500 mg/dL over the subsequent 48 hours, and ultimately, apheresis was not required. Enteral nutrition by nasogastric (NG) tube was initiated on hospital day 6. The patient’s hospital course was notable for acute respiratory distress syndrome that required intubation for 7 days, hyperbilirubinemia (with a peak bilirubin of 10.5 mg/dL), acute kidney injury (with a peak creatinine 4.7 mg/dL), fever without an identified infectious source, alcohol withdrawal syndrome that required phenobarbital, and delirium. Nine days later, he was transferred back to the VABHS West Roxbury campus. His condition stabilized, and he was transferred to the medical floor. On hospital day 14, the patient’s mental status improved, and he began tolerating oral nutrition.

Dr. Breu, over the years, the standard of care regarding when to start enteral nutrition in pancreatitis has changed considerably. This patient received enteral nutrition via NG tube but also had periods of being NPO (nothing by mouth) for up to 6 days. What is the current best practice for timing of initiating enteral nutrition in acute pancreatitis?

Dr. Breu. It is true that the standard of care has changed and continues to evolve. Many decades ago, patients with acute pancreatitis would routinely undergo NG tube suction to reduce delivery of gastric contents to the duodenum, thereby decreasing pancreas activation, allowing it to rest.11 The NG tube also allowed for decompression of any ileus that had formed. Beginning in the 1970s, several clinical trials were performed, showing that NG tube suction was no better than simply making the patient NPO.12,13 More recently, we have begun to move toward earlier feeding. Again, there is a pathophysiologic rationale (bowel rest is associated with intestinal atrophy, predisposing to bacterial translocation and resulting infectious complications) and increasing evidence supporting this practice.9 Even in severe pancreatitis, hunger may be used to initiate oral intake.14

 

 

Dr. Ganatra. On hospital day 16, the patient developed sudden-onset right-sided back and flank pain, and his hemoglobin dropped to 6.1 mg/dL, which required transfusion of packed red blood cells. He remained afebrile and hemodynamically stable. Dr. Weber, what are the major complications of acute pancreatitis, and when should we suspect them? Should we be worried about complications of pancreatitis in this patient?

[embed:render:related:node:133708]

Dr. Weber. Organ failure in the acute setting can occur due to activation of cytokine cascades and the systemic inflammatory response syndrome and is described by clinical and radiologic criteria called the Atlanta Classification.15 Apart from organ failure, the most serious complications of acute pancreatitis are necrosis of pancreatic tissue leading to walled-off pancreatic necrosis and the formation of peripancreatic fluid collections and pseudocysts, which occur in about 15% of patients with acute pancreatitis. These complications are serious because they can become infected, which portends a higher mortality and in some cases require surgical resection.

Other complications of acute pancreatitis include pseudoaneurysm formation, which is when a vessel bleeds into a pancreatic pseudocyst, and thromboses of the splenic, portal, or mesenteric veins. Thrombotic complications may occur in up to half of patients with pancreatic necrosis but are uncommon without some degree of necrosis.16 No necrosis was noted on this patient’s initial CT scan, so the probability of thrombosis is low. Also, as it takes several weeks for pseudocyst formation to occur, a bleeding pseudoaneurysm is unlikely at this early stage. Therefore, a complication of pancreatitis is unlikely in this patient, and evaluation for other causes of abdominal pain should be considered.

Dr. Ganatra. A noncontrast CT of the abdomen and pelvis was obtained and revealed no evidence of complications or other acute pathology. His pain was managed conservatively, and hemoglobin remained stable. Over the next 5 days, the patient’s symptoms gradually resolved, his oral intake improved, and he was discharged home on gemfibrozil 600 mg twice daily 19 days after admission. He declined psychiatry follow-up for his PTSD, and after discharge he did not keep his scheduled gastroenterology (GI) follow-up appointment. Four months later, the patient presented again with epigastric abdominal pain similar to his initial presentation. The patient had resumed drinking, stating that “alcohol is the only thing that helps [with the PTSD].” He had not been taking the gemfibrozil. He was admitted with a recurrent episode of pancreatitis; however, his triglycerides on admission were 119 mg/dL.

Dr. Weber, this patient’s triglycerides declined rapidly over a period of just 4 months with questionable adherence to gemfibrozil. However, he was admitted again with another episode of pancreatitis, this time in the setting of alcohol use alone without markedly elevated triglycerides. What do we know about recurrence risk for pancreatitis? Are some etiologies of pancreatitis more likely to present with recurrent attacks than are others?

Dr. Weber. The rate of recurrence following an episode of acute pancreatitis varies according to the cause, but in general, about 20% to 30% of patients will experience a recurrence, and 5% to 10% will go on to develop chronic pancreatitis.17 Alcoholic pancreatitis does carry a higher risk of recurrence than pancreatitis due to other causes; the risk is as high as 50%. Not surprisingly, recurrence of acute pancreatitis increases risk for development of chronic pancreatitis. As this patient is a smoker, it is worth noting that smoking potentiates pancreatic damage from alcohol and increases the risk for both recurrent and chronic pancreatitis.5

Dr. Ganatra. The patient was treated with IV hydromorphone and IV LR at 350 cc/h. Oral nutrition was begun immediately. He manifested no organ dysfunction, and his symptoms improved over the course of 48 hours. He was discharged home with psychiatry and GI follow-up scheduled. Dr. Breu and Dr. Weber, how should we counsel this patient to reduce his risk of recurrent attacks of pancreatitis in the future, and what options do we have for pharmacotherapy to decrease his risk?

Dr. Breu. I’ll let Dr. Weber comment on mitigating the risk of hypertriglyceride-induced pancreatitis and reserve my comments to pharmacotherapy in alcohol use disorder. This patient may be a candidate for naltrexone therapy, either in oral or intramuscular formulations. Both have been showed to reduce the risk of returning to heavy drinking and may be particularly beneficial in those with a family history.18,19 Acamprosate is also an option.

Dr. Weber. Data on recurrence risk in hypertriglyceridemic pancreatitis are limited, but there are case reports suggesting that a fatty diet and alcohol use are implicated in recurrence.20 I would counsel the patient on lifestyle modifications that are known to reduce this risk. I agree with Dr. Breu that devoting our efforts to helping him reduce or eliminate his alcohol consumption is the single most important thing we can do to reduce his risk for recurrent attacks. Since the patient reports that he drinks alcohol in order to cope with his PTSD, establishing care with a mental health provider to address this is of the utmost importance. In addition, smoking cessation and promoting medication adherence with gemfibrozil will also reduce risk for future episodes, but continued alcohol use is his strongest risk factor.

 

►Dr. Ganatra. After discharge, the patient engaged with outpatient psychiatry and GI. He still reports feeling that alcohol is the only thing that alleviates his PTSD and anxiety symptoms. He is not currently interested in pharmacotherapy for cessation of alcohol use.

Acknowledgments
The authors thank Ivana Jankovic, MD, Matthew Lewis Chase, MD, Christopher Worsham, MD, Lakshmana Swamy, MD.

References

1. Dufour MC, Adamson MD. The epidemiology of alcohol-induced pancreatitis. Pancreas. 2003;27(4):286-290.

2. Peery AF, Dellon ES, Lund J, et al. Burden of gastrointestinal disease in the United States: 2012 update. Gastroenterology. 2012;143(5):1179-1187.e1-e3.

3. Cavallini G, Frulloni L, Bassi C, et al; ProInf-AISP Study Group. Prospective multicentre survey on acute pancreatitis in Italy (ProInf-AISP): results on 1005 patients. Dig Liver Dis. 2004;36(3):205-211.

4. Banks PA, Freeman ML; Practice Parameters Committee of the American College of Gastroenterology. Practice guidelines in acute pancreatitis. Am J Gastroenterol. 2006;101(10):2379-2400.

5. Hartwig W, Werner J, Ryschich E, et al. Cigarette smoke enhances ethanol-induced pancreatic injury. Pancreas. 2000;21(3):272-278.

6. Chowdhury P, Gupta P. Pathophysiology of alcoholic pancreatitis: an overview. World J Gastroenterol. 2006;12(46):7421-7427.

7. Scherer J, Singh VP, Pitchumoni CS, Yadav D. Issues in hypertriglyceridemic pancreatitis: an update. J Clin Gastroenterol. 2014;48(3):195-203.

8. Wu BU, Hwang JQ, Gardner TH, et al. Lactated Ringer’s solution reduces systemic inflammation compared with saline in patients with acute pancreatitis. Clin Gastroenterol Hepatol. 2011;9(8):710-717.e1.

9. Tenner S, Baillie J, DeWitt J, Vege SS; American College of Gastroenterology. American College of Gastroenterology guideline: management of acute pancreatitis. Am J Gastroenterol. 2013;108(9):1400-1415; 1416.

10. Preiss D, Tikkanen MJ, Welsh P, et al. Lipid-modifying therapies and risk of pancreatitis: a meta-analysis. JAMA. 2012;308(8):804-811.

11. Nardi GL. Pancreatitis. N Engl J Med. 1963;268(19):1065-1067.

12. Naeije R, Salingret E, Clumeck N, De Troyer A, Devis G. Is nasogastric suction necessary in acute pancreatitis? Br Med J. 1978;2(6138):659-660.

13. Levant JA, Secrist DM, Resin H, Sturdevant RA, Guth PH. Nasogastric suction in the treatment of alcoholic pancreatitis: a controlled study. JAMA. 1974;229(1):51-52.

14. Zhao XL, Zhu SF, Xue GJ, et al. Early oral refeeding based on hunger in moderate and severe acute pancreatitis: a prospective controlled, randomized clinical trial. Nutrition. 2015;31(1):171-175.

15. Banks PA, Bollen TL, Dervenis C, et al; Acute Pancreatitis Classification Working Group. Classification of acute pancreatitis—2012: revision of the Atlanta classification and definitions by international consensus. Gut. 2013;62(1):102-111.

16. Easler J, Muddana V, Furlan A, et al. Portosplenomesenteric venous thrombosis in patients with acute pancreatitis is associated with pancreatic necrosis and usually has a benign course. Clin Gastroenterol Hepatol. 2014;12(5):854-862.

17. Yadav D, O’Connell M, Papachristou GI. Natural history following the first attack of acute pancreatitis. Am J Gastroenterol. 2012;107(7):1096-1103.

18. Jonas DE, Amick HR, Feltner C, et al. Pharmacotherapy for adults with alcohol use disorders in outpatient settings: a systematic review and meta-analysis. JAMA. 2014;311(18):1889-1900.

19. Garbutt JC, Kranzler HR, O’Malley SS, et al. Efficacy and tolerability of long-acting injectable naltrexone for alcohol dependence: a randomized controlled trial. JAMA. 2005;293(13):1617-1625.

20. Piolot A, Nadler F, Cavallero E, Coquard JL, Jacotot B. Prevention of recurrent acute pancreatitis in patients with severe hypertriglyceridemia: value of regular plasmapheresis. Pancreas. 1996;13(1):96-99.

References

1. Dufour MC, Adamson MD. The epidemiology of alcohol-induced pancreatitis. Pancreas. 2003;27(4):286-290.

2. Peery AF, Dellon ES, Lund J, et al. Burden of gastrointestinal disease in the United States: 2012 update. Gastroenterology. 2012;143(5):1179-1187.e1-e3.

3. Cavallini G, Frulloni L, Bassi C, et al; ProInf-AISP Study Group. Prospective multicentre survey on acute pancreatitis in Italy (ProInf-AISP): results on 1005 patients. Dig Liver Dis. 2004;36(3):205-211.

4. Banks PA, Freeman ML; Practice Parameters Committee of the American College of Gastroenterology. Practice guidelines in acute pancreatitis. Am J Gastroenterol. 2006;101(10):2379-2400.

5. Hartwig W, Werner J, Ryschich E, et al. Cigarette smoke enhances ethanol-induced pancreatic injury. Pancreas. 2000;21(3):272-278.

6. Chowdhury P, Gupta P. Pathophysiology of alcoholic pancreatitis: an overview. World J Gastroenterol. 2006;12(46):7421-7427.

7. Scherer J, Singh VP, Pitchumoni CS, Yadav D. Issues in hypertriglyceridemic pancreatitis: an update. J Clin Gastroenterol. 2014;48(3):195-203.

8. Wu BU, Hwang JQ, Gardner TH, et al. Lactated Ringer’s solution reduces systemic inflammation compared with saline in patients with acute pancreatitis. Clin Gastroenterol Hepatol. 2011;9(8):710-717.e1.

9. Tenner S, Baillie J, DeWitt J, Vege SS; American College of Gastroenterology. American College of Gastroenterology guideline: management of acute pancreatitis. Am J Gastroenterol. 2013;108(9):1400-1415; 1416.

10. Preiss D, Tikkanen MJ, Welsh P, et al. Lipid-modifying therapies and risk of pancreatitis: a meta-analysis. JAMA. 2012;308(8):804-811.

11. Nardi GL. Pancreatitis. N Engl J Med. 1963;268(19):1065-1067.

12. Naeije R, Salingret E, Clumeck N, De Troyer A, Devis G. Is nasogastric suction necessary in acute pancreatitis? Br Med J. 1978;2(6138):659-660.

13. Levant JA, Secrist DM, Resin H, Sturdevant RA, Guth PH. Nasogastric suction in the treatment of alcoholic pancreatitis: a controlled study. JAMA. 1974;229(1):51-52.

14. Zhao XL, Zhu SF, Xue GJ, et al. Early oral refeeding based on hunger in moderate and severe acute pancreatitis: a prospective controlled, randomized clinical trial. Nutrition. 2015;31(1):171-175.

15. Banks PA, Bollen TL, Dervenis C, et al; Acute Pancreatitis Classification Working Group. Classification of acute pancreatitis—2012: revision of the Atlanta classification and definitions by international consensus. Gut. 2013;62(1):102-111.

16. Easler J, Muddana V, Furlan A, et al. Portosplenomesenteric venous thrombosis in patients with acute pancreatitis is associated with pancreatic necrosis and usually has a benign course. Clin Gastroenterol Hepatol. 2014;12(5):854-862.

17. Yadav D, O’Connell M, Papachristou GI. Natural history following the first attack of acute pancreatitis. Am J Gastroenterol. 2012;107(7):1096-1103.

18. Jonas DE, Amick HR, Feltner C, et al. Pharmacotherapy for adults with alcohol use disorders in outpatient settings: a systematic review and meta-analysis. JAMA. 2014;311(18):1889-1900.

19. Garbutt JC, Kranzler HR, O’Malley SS, et al. Efficacy and tolerability of long-acting injectable naltrexone for alcohol dependence: a randomized controlled trial. JAMA. 2005;293(13):1617-1625.

20. Piolot A, Nadler F, Cavallero E, Coquard JL, Jacotot B. Prevention of recurrent acute pancreatitis in patients with severe hypertriglyceridemia: value of regular plasmapheresis. Pancreas. 1996;13(1):96-99.

Issue
Federal Practitioner - 35(2)a
Issue
Federal Practitioner - 35(2)a
Page Number
34-39
Page Number
34-39
Publications
Publications
Topics
Article Type
Sections
Disallow All Ads
Content Gating
No Gating (article Unlocked/Free)
Alternative CME
Disqus Comments
Default
Eyebrow Default
VA Boston Medical Forum
Use ProPublica
Article PDF Media
Image
Teambase ID
18000D9E.SIG
Disable zoom
Off